Текст
                    Г. И. ИВЧЕНКО, Ю. И. МЕДВЕДЕВ, А. В. ЧИСТЯКОВ
СБОРНИК
ЗАДАЧ
ПО
МАТЕМАТИЧЕСКОЙ
СТАТИСТИКЕ
Допущено
Государственным комитетом СССР
по народному образованию
в качестве учебного пособия
для студентов высших
технических учебных заведений
Москва «Высшая школа» 1989


ББК 22.172 И 25 УДК 519.2 Рецензенты: кафедра прикладной математики Московского инженерно-строительного института (зав. кафедрой проф. В. В. Кучеренко) и проф. Э. А. Надарая (Тбилисский государственный университет) Ивченко Г. И. и др. И25 Сборник задач по математической статистике: Учеб. пособие для втузов/Г. И. Ивченко, Ю. И. Медведев, А. В. Чистяков — М.: Высш. шк., 1989. — 255 с: ил. ISBN 5-06-000049-4 Задачник дополняет учебное пособие «Математическая статистика» (1984). Он охватывает все основные разделы современной статистической теории и предназначен для проведения практических занятий и лабораторных работ по курсу математической статистики, предусмотренному новой программой по высшей математике для втузов. Большинство задач снабжены решениями или методическими указаниями, ко всем задачам даны ответы. 1602090000(43090000000) — 524 И - - 89—89 ББК 22.172 001 (01) —89 5(78 ISBN 5-06-000049-4 © Г. И. Ивченко, 10. И. Медведев, А. В. Чистяков, 1989
ОГЛАВЛЕНИЕ Предисловие 4 ТЕОРЕТИЧЕСКОЕ ВВЕДЕНИЕ И ЗАДАЧИ. Глава 1 Основы статистического описания, выборочные характеристики и их распределения 6 Глава 2 Оценивание параметров распределений 37 § 1. Оценки и их общие свойства 46 § 2. Оптимальные оценки 57 § 3. Оценки максимального правдоподобия (о.м.п.) .... 67 § 4. Доверительное оценивание 74 Глава 3 Проверка статистических гипотез 80 § 1. Критерии согласия 89 § 2. Выбор из двух простых гипотез 99 § 3. Сложные гипотезы 102 § 4. Проверка гипотез и доверительное оценивание .... 104 § 5. Критерий отношения правдоподобия (к.о.п.) 106 § 6. Разные задачи 107 Глава 4 Линейная регрессия и метод наименьших квадратов 110 Глава 5 Решающие функции 121 Глава 6 Статистика стационарных .„2 последовательностей ОТВЕТЫ И РЕШЕНИЯ 137 Приложения 240 Указатель распределений 251 Литература 253
ПРЕДИСЛОВИЕ Настоящий сборник задач охватывает все традиционные разделы современной статистической теории. Его содержание соответствует курсу математической статистики, предусмотренному новой программой по высшей математике для инженерных специальностей высших технических учебных заведений. Часть задач повышенной сложности может быть использована в качестве заданий для учебно-исследовательских и курсовых работ. Задачи в основном носят аналитический характер: в них требуется показать справедливость того или иного утверждения или провести исследование. Они непосредственно дополняют или раскрывают принципиальные положения математической статистики. В сборник включены задачи, связанные с моделированием случайных величин на ЭВМ и получением исходного для статистической обработки материала. Фактически на основе любой «теоретической» задачи, в которой речь идет о статистическом алгоритме анализа данных, можно поставить, задавая конкретные значения параметров модели (причем возможно неограниченное число вариантов), соответствующую «практическую» задачу, формулируя в качестве предварительного этапа задание смоделировать исходные данные, используя или готовые таблицы случайных чисел, или получаемые с помощью специально составленных программ. В дальнейшем, при обработке этих «экспериментальных» данных с помощью соответствующего теоретического алгоритма, имеется возможность сравнить предсказание теории с известными исходными параметрами, при которых моделировалась выборка. По степени трудности задачи, помещенные в сборнике, не одинаковы. Для решения некоторых из них могут потребоваться значительные усилия, такие задачи отмечены звездочкой. Большинство задач, решение которых не сводится к применению стандартных алгоритмов, снабжено подробными решениями, даны методические указания. 4
В начале каждой главы приведены основные понятия, теоретические положения и формулы из соответствующего раздела теории, которые непосредственно используются при решении помещенных в данную главу задач. В конце книги имеются статистические таблицы, необходимые для получения числовых результатов. Указатель распределений облегчит поиск задач, в которых рассматриваются различные аспекты исследования одной и той же модели. При составлении задачника использованы отечественные и зарубежные источники (учебники, задачники, журнальные статьи и др.). Авторы будут признательны всем, кто поделится своими соображениями по улучшению содержания книги. Замечания можно направлять по адресу: Москва, Ж-28, Б. Вузовский пер., 3/12, МИЭМ, кафедра теории вероятностей и математической статистики. Авторы
ТЕОРЕТИЧЕСКОЕ ВВЕДЕНИЕ И ЗАДАЧИ Глава 1 ОСНОВЫ СТАТИСТИЧЕСКОГО ОПИСАНИЯ, ВЫБОРОЧНЫЕ ХАРАКТЕРИСТИКИ И ИХ РАСПРЕДЕЛЕНИЯ 1. Статистические данные, являющиеся исходным «материалом» в задачах математической статистики, обычно являются результатом наблюдения некоторой конечной совокупности случайных величин Χ=(ΛΊ, ..., Хп), характеризующей исход изучаемого эксперимента. В таких случаях говорят, что эксперимент состоит в проведении η испытаний, в которых результат i-ro испытания описывается случайной величиной Χι, ί= Ι, ..., п. Совокупность наблюдаемых случайных величин X = (X , Хп) называется выборкой, сами величины Xit ί=1, ..., η, — элементами выборки, а их число η — ее объемом. Множество X = {x = (xi, ..., хп)) всех возможных реализаций выборки X = (X Хп) называется выборочным пространством. Когда истинное распределение случайной величины X (функция распределения Fx{x\, ..., хп) = = P(X\^Lx А"я^л:я)) неизвестно (полностью или хотя бы частично) и указан лишь класс (семейство) допустимых распределений ¥={F(x\ хп)\, которому принадлежит распределение Fx выборки X, то говорят о статистической модели (X, F) (или, короче, о модели F). Математическая статистика решает (в рамках заданной модели F) задачи уточнения (выявления) различных свойств истинного распределения Fx по результатам проводимых наблюдений (по выборке X). Часто рассматриваются эксперименты, в которых проводятся повторные независимые наблюдения над некоторой случайной величиной £, (ее распределение обозначается символом L(£)). В таких случаях выборка Х = = (Χι, ..., Хп) представляет собой совокупность независимых одинаково распределенных случайных величин, причем L (Xi) = L(£), t=l, ..., η; говорят также для краткости, что Х = (Xt, ..., Х„) есть выборка из распределения L(|). Статистическая модель для повторных независимых наблюдений обозначается кратко в виде F = б
= {/■ξ), т. е. указывается лишь класс допустимых функций распределения исходной случайной величины ξ. Если F = {F{x; θ), θεθ), т.е. допустимые функции распределения задаются с точностью до значений некоторого параметра Θ, то такая модель называется параметрической, а множество Θ возможных значений параметра θ — параметрическим множеством. В дальнейшем рассматриваются только модели абсолютно непрерывного или дискретного типа и для единообразия используется обозначение fi{x) = f{x) (для параметрических моделей f(x; θ)) как для плотности распределения случайной величины ξ в случае, когда распределение Fi, абсолютно непрерывно, так и для вероятности Ρ(ξ = *) в дискретном случае. В случае параметрической модели распределение вероятностей на выборочном пространстве X , отвечающее параметру 0, обозначается символом Ρθ. Аналогично, Е0Т(Х), DqT{X), ... — обозначения соответствующих моментов заданной функции Т(Х) от выборки X в случае, когда Fx(x; θ)— функция распределения выборки. 2. Во многих задачах математической статистики рассматриваются последовательности случайных величин |η„), сходящиеся в том или ином смысле к некоторому пределу η (случайной величине или константе), когда л->-оо. В дальнейшем используются два вида сходимости: сходимость по вероятности (η,ι-^-ηο Ρ(Ιηη— ηΙ> >ε)->-0 -Ve>0) и сходимость по распределению, или слабая сходимость ( ί(η„)->- ί(η) или г\п-*-г\оРЛ11(х) ->- -+F,(i)VieC(F,), где C{F) — множество точек непрерывности функции F(x). При этом из Р-сходимости следует L-сходимость. Многие результаты о Р-сходимости различных выборочных характеристик являются следствием следующего общего утверждения о сходимости функций от случайных величин [1, с. 20]: ρ если η„; ->- d = const, i = 1 г, и φ(*ι, ..., χ,) — произвольная непрерывная в некоторой окрестности точки {с сг) функция, то φ(η„, tw)-^"<p(c с). 3. Если Х= (Χι Хп) — выборка из некоторого распределения L (£)· т0 F\{x) = F(x) называют теоретической функцией распределения, а ВД = -^ = -£-£ *(*-*■) (ΐ·ΐ) i = I 7
— эмпирической функцией распределения (здесь μ„(χ4 — число элементов выборки, удовлетворяющих условию Xj^x, а е(лг) = |. Ρ 5л' — функция Хевисайда). По теореме Бернулли, Fn(x)£*-F(x)tyx, когда л->-оо, т. е. при больших η значение Fn(x) может служить оценкой для F(x). Более глубокое обоснование для оценивания теоретической функции распределения с помощью эмпирической функции распределения дают теоремы Гли- венко и Колмогорова об асимптотических свойствах Fa(x) при больших η [1, с. 15]. Если случайная величина ξ дискретна и принимает значения аи а2, ···, то более наглядное представление о законе распределения ξ дадут частоты h,/n, где я, — число членов выборки, равных а,: в этом случае А,/лА-Р(| = а,), г=1, 2, ..., когда л-»-оо. Для величин |, имеющих плотность \%{х) = f(x), можно рассмотреть частоты hk/n событий (|еД*}, где (Δ*} — система непересекающихся интервалов, образующих разбиение области возможных значений |. В этом случае при п-+оо ^-^Р(|еД*)= J f(x)dx, и если Δ* малы, то по частотам hk/n можно построить два «графика», напоминающие график функции f(x) и называемые гистограммой и полигоном частот соответственно [1, с. 16], которые могут дать некоторое предварительное представление о законе распределения |. Каждой теоретической характеристике g = \g(x)dF(x) соответствует ее статистический аналог G=G(X) = \ g{x)dFn{x) = 4- Σ g(Xi), ί= I называемый эмпирической или выборочной характеристикой. В частности, статистическими аналогами для теоретических моментов являются выборочные моменты. Выборочным моментом k-го порядка называют величину Anh = Anh(X) = -L· £#. п .■= ι При k= 1 величину Ап\ называют выборочным средним и обозначают X: 8
Выборочным центральным моментом й-го порядка называют величину Мпк = Мпк{Х) = -i-Σ {Х:-Х)к. 1=1 При й = 2 величину Мп2 называют выборочной дисперсией и обозначают символом S2 = S2(X): " i= I используется также обозначение S'2 = —^-j—S2. Аналогично вводят выборочные абсолютные моменты, выборочные семиинварианты и т. д. Другим примером выборочных характеристик являются выборочные квантили. При этом р-квантиль для любой функции распределения F{x) определяется как ςρ = = m\{x: F{x)~^ ρ), 0<р<1, а выборочная р-квантиль Ζη,ρ есть р-квантиль эмпирической функции распределения Fn(x). Если расположить элементы выборки X = = (Хи ..., Хп) в порядке возрастания их величин, то получится последовательность новых случайных величин Х{\) ^ -Х(2) ^ ... ^ Х(п) , называемая вариационным рядом выборки; при этом Xw — k-я порядковая статистика, k = 1, ..., η, а ΛΌ) и Х(П) — экстремальные (соответственно минимальное и максимальное) значения выборки. Тогда Ζη,Ρ выражается через порядковые статистики: 2 _{*(М+о ПРИ пР дробном, л,р | Хш при пр целом. В частности, Ζη,\/ϊ — выборочная медиана. Любая выборочная характеристика, имеющая вид непрерывной функции от конечного числа величин Апк (в частности, сами выборочные моменты, а также выборочные центральные моменты Mnk), сходится по вероятности при п-+оо к соответствующей теоретической характеристике и может служить оценкой последней, когда число наблюдений η достаточно велико. Аналогич- 9
но, Ζη,ρ-?*-ζΡ, если только распределение L(£) обладает гладкой плотностью. 4. В выборочной теории изучаются различные свойства распределений выборочных характеристик как в точной, так и асимптотической (т. е. при большом объеме выборки) постановках. При исследовании асимптотического (при п-+оо) поведения соответствующих распределений широко используются предельные теоремы теории вероятностей и прежде всего две основные из них: закон больших чисел и центральная предельная теорема. Напомним их простейшие формулировки [2, с. 150—154]. Закон больших чисел. Если случайные величины η,, η2, ... независимы, одинаково распределены и имеют математическое ожидание Εη, = а, то — (ηι + ... + η„) -»- а при η -»- оо. Центральная предельная теорема: если дополнительно к предыдущему существует ϋη, = σ2 > 0, то при η -*■ оо £((Л. + ... + η« - ηα)/{-φία))-+ N(0, I). Многомерный вариант центральной предельной теоремы имеет следующий вид: пусть r-мерные случайные векторы х\п = (г\п\, ..·, ппг), η = 1, 2, .... независимы, одинаково распределены и имеют конечные моменты a-t = Εη,,·, Ьц = сои (η υ, η,,), i, j = 1 г. Тогда £(ς«., ..-, ς-,)—- Ν(0,Β= ||Μί). где ςη; = (ηι,- + ··· + Цш — па^/л/п, i = 1, ..., г. (Определение многомерного нормального распределения см. ниже в п. 6). Приведем некоторые утверждения о сходимости функций от случайных величин, которые понадобятся в дальнейшем при решении задач. Р PL 1°. Если ηη->-η " функция ψ непрерывна, то φ(ηη)->-φ(η) 2°. Пусть (η„, ς„}, η = 1,2,..., — последовательность пар случайных величин. Тогда а) η* — ς,ΑΟ, ςΛ^ς=>η„^ς; б) L(r\a)-*L(r\), ς„-^0=^η„ς„-^0; 10
в) L (η,,)-»- ί,(η), ς„ A-c = const=> Ι(η„ + ςη)-»- Ι(η + + с), Ι(η„ς„)->- L(ct)), ί.(ηη/ς,,)-»- Цх\/с) при с Φ 0; Γ) Ήι — ζπ-^0, L(qn)-*- L(q), функция φ непрерывная =^φ(ηπ)— фЫ-^О. 3°. Пусть Тп = Тп(Х), X = [Х\, ..., Хп), — оценка скалярного параметра θ в модели F = {F(x; θ), θεθ)« такая, что LabJn (Тп — θ)) -»- JV(0, σ2(θ)) при η -»- оо ы всех θεθ. Пусть, далее, функция φ дифференцируема и φ' =£0. Тогда Ы-фЫТп)- φ(θ)])-> ЛГ(0, [φ'(θ)ίσ2(θ)). Кроме того, если функции ср' и σ непрерывны, то 4v^fe£rb "(»·'>■ Обобщение утверждения 3° на случай векторного параметра θ = (θι, ..., θ,) имеет следующий вид. 4°. Пусть Тп = {Тщ, ..., Тп,)—_оценка параметра Θ, удовлетворяющая условию £о(У" (Тп — Θ)) ->■ N(0, Σ(Θ)) гср« «-+00 и всех θ £Ξ θ. Тогда для любой дифференцируемой функции φ or r переменных UbRtfF «)- <ρ(θ)))- МО, ^2(θ)) при условии, что v(Q) ф 0, где υ2(θ) = ή'(θ)Σ(θ)δ(θ), 6(θ) = = (-=£-, ···> "^δ/ ' ^Суг"' кРоме того, функция φ непрерывно дифференцируема и все элементы матрицы вторых элементов Σ(θ) непрерывны по Θ, то lu(-^mt „)-№]/»(? -))- МО, О- На основании центральной предельной теоремы выборочный момент Ank асимптотически нормален с параметрами a.k = Εξ* и —D£* = (ct2* — al)/n, что кратко записывается так: L (Ank) ~ iV (α*, (аг* — α2)/"). Асимптотически нормальным является и совместное распределение любого конечного числа выборочных моментов Ank, а также (при некоторых дополнительных условиях) распределение любой дифференцируемой функции от конечного числа моментов Апь В частности, асимптотически нормальными являются и центральные выборочные моменты Mnk- Исследование асимптотического поведения распреде- 11
лений порядковых статистик Х^) при η -*■ оо проводится методом прямого анализа точных распределений величин X(k). При этом средние члены вариационного ряда (т. е. когда номер k = k(n) удовлетворяет условию *-р, 0< ρ < 1) для распределений L (ξ), обладающих гладкой плотностью, оказываются асимптотически нормальными; для крайних же порядковых статистик (т, е. для Х^), X(n-s + l) при фиксированных r,s^\) класс предельных распределений исчерпывается распределениями трех типов, отличных от нормального [1, с, 26]. 5. Напомним некоторые формулы из теории вероятностей, которые часто используются для нахождения явного вида распределений при преобразованиях случайных величин. Пусть вектор X = (Хи ..., Xk) имеет абсолютно непрерывное распределение с плотностью f(x), x = (xh ... , xk)eS^Rk, и А = (Л hk):S-+R" — произвольное взаимно однозначное и гладкое (т.е. все частные производные dhi(x)/dxj непрерывны) преобразование, якобиан которого a/ii(x) dh„(x) /(ж) = det дх\ дХ[ θΛ,(χ) θ/ι*(χ) дХк дх„ не обращается на 5 в нуль, Тогда плотность распределения случайного вектора Υ = h(X) = (hi(X) hu{X)) имеет следующий вид: φ(ί/) = f(h "'(у ))/1ДЛ~'(</)) I. У = (У , £/*) € h(S),(1.2) где Л-1—обратное к Л преобразование: h~\h(x)) = х. Выделим два часто встречающихся частных случая. Если k = 1, то речь идет о преобразовании случайной величины: Υ = h(X), где h(x) — взаимно однозначная гладкая функция с не обращающейся в нуль производной. В этом случае плотность распределения Υ имеет вид φΟ/) = «Λ-,(ί/))/ΙΛ'(Λ-,(ί/))Ι. (ΐ·3) Если речь идет о линейном преобразовании: Υ = АХ + + b det Α ξ= α Φ О, то плотность распределения Υ равна <f{y) = f{A-\y-b))/\a\. В ряде статистических задач приходится иметь дело с частным ς = ξ/η двух независимых случайных величин, 12
плотности распределения которых Д и £, известны. Плотность распределения величины ς может быть вычислена по формуле оо Ш)= S k(xyMx)Mdx. (1.4) — оо 6. Для удобства дальнейшего изложения приведем наиболее часто встречающиеся в приложениях распределения Ζ,(ξ) и некоторые их свойства. 1) Нормальное распределением^, σ2), — оо < μ < оо, σ > О, имеет плотность ———е 2а' , — оо < * < оо; при этом μ = Εξ, σ2 = D£, а центральные моменты μ* = = Ε(ξ — μ)* равны соответственно μ2/- + ι = 0, μ2, = = ηρ-σ2Λ = 1·3...(2γ- 1)σ2'. Распределение N(0, 1) называют стандартным нормальным, его функцию рас- χ пределения обозначают Ф(х) = —— \ e~''/2dt; уравне- ние Ф(«р) = ρ, ρ 6(0, 1), однозначно определяет р-кван- тиль ир, при этом и\ - ρ = —Up\ используется также обозначение cY = «(ι+ν)/2. Случайный вектор ξ = (ξι, .,,, ξ*) имеет (й-мерное) нормальное распределение Ν(μ = = (μι, ..., μ*), Σ = IIσ;/II*). если его характеристическая функция имеет вид1 Ее"'1 =βχρ{ίί'μ-·5-ί'Σ<}, t = (tlt ·-. /*): при этом Ε(ξ) = (Εξι Εξ*) = μ, D({) = Ε(ξ - μ) (ξ - μ)' = II cov (ξ,-, ξ,) || ί = || σ,,-1| * = Σ · Если |Σ| Φ 0, то распределение Λ^μ, 2) называется невырожденным {собственным) и имеет плотность Я*) = (2л)-|"|2| 2εχΡ{-1-(χ-μ)'Σ",^-μ)}. χ = (χ,, .... Xk)£R\ Важнейшим свойством нормального распределения явля- ' При матричных преобразованиях векторы понимаются как вектор столбцы, означает транспонирование. 13
ется то, что при линейном преобразовании η = А\{А — заданная матрица) снова получается нормальный случайный вектор, при этом L(r\) = Ν(Α\ι, ΑΣΑ'). В частности, если η = Ι/'ξ, где U — ортогональная матрица, приводящая 2 к диагональному-виду [/'"£ U = D = II J·, II J J -" II U Aft Μ (λ;, /= 1, ..., k — собственные числа 2), το L(r\) = =Λ^(1/'μ, £>), т. е. компоненты вектора η некоррелирова- ны, следовательно, и независимы. Полагая Ζ = = D-|/2i/'(g — μ) (если все λ,>0), получим Ι (Ζ) = =N(0, Ek), где Ek — единичная матрица размера k. Таким образом, всегда можно указать линейное преобразование, переводящее невырожденный нормальный вектор в вектор с независимыми стандартными нормальными компонентами. В статистических приложениях, где имеют место выборки из нормального распределения, важную роль играют следующие утверждения [1, с. 29—31]: 1°. Если X = (Хи ..., Хп) — выборка из распределения Ν(μ, σ2) и t = BX,Q -, = X'AiX, i = 1,2 — соответственно линейная и квадратичные функции от X, то для независимости t и Qi достаточно выполнения условия ВА; = О, а для независимости Q\ и Q2 — условия А\А2 = А^А\ = = 0. 2°. Пусть μ = 0, σ2 = 1 и А? = А\ (матрица Л, — идемпотентная); тогда L(Q\) = χ2(0, где г = rang Л, = = trAi — след матрицы А{. 3°. Теорема Фишера. Выборочные среднее X и дисперсия S2 независимы и при этом L(-\fn(X — μ)/σ) = = N(0, 1), aL(nS2/o2) = χ2(π — 1) (определение хи-квад- рат распределения см. ниже). 2) Гамма-распределение Г(а, λ), α, λ > 0, задается плотностью * * —, л:>0 (здесь Γ(λ) =\ tl~le~'dt, Γ(λ)α 0 λ>0 — гамма-функция), и имеет моменты Εξ6 = = α6Γ(λ + 6)/Γ(λ), b>— λ. В частности, Εξ = αλ, Όξ = α2λ. Частный случай Γ (α, 1) называют экспоненциальным или показательным распределением. Другой частный случай Г(2, /г/2) называют распределением хи-квадрат с η степенями свободы и обозначают χ2(η); при этом χ2(/ι) =L(£i + ··· + ύ), где слагаемые независимы и L(ti) = N(0, 1), i = 1, ..., п. Для р-квантили распределения χ (η) используют обозначение χ2,,,,. 14
3) Распределение Вейбулла Ща, а, Ь) зависит в общем случае от трех параметров: параметра положения (сдвига) a£R\ параметра формы а>0 и параметра масштаба b > О и задается функцией распределения Здесь ед=1-ехр(-(^-)а),^а. Εξ = Ω + ^Γ(ι + 1-),οξ = ^[Γ(ι+-|-)-Γ2(ι+1-)]. Частный случай W(a, 1, b) известен как двухпарамет- рическое экспоненциальное распределение, a W(a, 2, b) — как распределение Релея. 4) Бета-распределение β(α, Ь), a, b > 0, задается плотностью ха~\\ — х)"-1/Ща, Ь), 0<*<1, где В(а, Ь) = Г, _}.{ — бета-функция. Здесь Е*" — а (а + bf(a + b + \) · 5) Равномерное распределение R(a, b), — оо < а < < 6 < оо, имеет постоянную плотность fix) =-. , о — а о ^ * ^ Ь. Здесь *(т=т) = ^°· ') = К1· 0. Εξ = -ψ- , ϋξ =^^ · 6) Распределение Коши K(a), — <x> <; a < оо, задается плотностью —, —оо < χ <с оо . Для этого π 1 -+- (χ — ay распределения не существуют моменты, в том числе и математическое ожидание, постоянная а совпадает с медианой д/2. Распределение Коши обладает следующим интересным свойством: если случайные величины ξι, ..., ξ„ независимы и Ζ,(ξ,) = Κ(α,), ι = 1, ..., η, το Ζ,(ξ) = Κ(α), где черта означает среднее арифметическое. 7) Распределение Стыодента с η степенями свободы S(n) =L(tn ξ η/Vxn/")' гДе случайные величины η и χ„ независимы и при этом Ι(η) =#(0, 1), Ι(χϊ) = χ2(«). Это распределение имеет плотность S„(x) = — —Г— +| , - оо < χ < оо ; 15
для его р-квантили используется обозначение tp,n. 8) Распределение Снедекора с п\ и щ степенями свободы В(п\, п.2)=Ц F„,,„, = _1-:-^-) , где случайные величины χ£ и χ£ независимы и при этом ЦуХ) = χ2(η;), i = 1, 2. Плотность этого распределения имеет вид его р-квантиль обозначается />,„,,„,, при этом Fi _р, „,,„, = 9) Биномиальное распределение Βί(η, ρ) — это распределение числа «успехов» в η независимых испытаниях с двумя исходами («успех» — «неуспех») и неизменной вероятностью «успеха» ρ ζ (0, 1) (схема Бернулли). Здесь Ρ(ξ = k) = CknpУ - \ k = 0, 1 n(q= \-p), Εξ = пр, Щ = npq. При η = 1 имеем распределение Бернулли Bi(\, p). 10) Полиномиальное распределение М(п; р\, ..., ри), р\ + ··· -\- ри = 1 — это распределение случайного вектора ν = (vi vN) с целочисленными неотрицательными компонентами, удовлетворяющими условию vi + ··· + -|- vn = η, которое имеет вид Ρ(ν = Λ) = ft"^, p'\...phN, h = (A , hN), Α ι +... + hN = n. Здесь _ . . ί ηρ{\ —pi) при i = j, Ev,- = np„ cov (v«, v,-) = { ... ' \-npip; при ι φ ι. Если произведено п независимых испытаний с N возможными исходами, вероятности которых неизменны и равны Pi, ..., рц соответственно, то, обозначив через ν, число реализаций i-ro исхода, i = 1 Ν, будем иметь, что L(v) = М(п; р\, ..., ры). Если N = 2, то М(п; р, 1 — р) = = Bi(n, ρ), т. е. полиномиальное распределение сводится к биномиальному. 11) Распределение Пуассона Π(λ), λ > 0, задается вероятностями Ρ(ξ = k) = е -тр. k = 0, 1, 2, ... ; при 16
этом λ = Εξ = ϋξ и вообще Ε(ξ),- = λ', где (α),- = = а(а - 1)...(α - / + 1), / > 1, (а)а = 1. _ 12) Отрицательное биномиальное распределение Bi(r, ρ), ρ 6 (0, 1), г = 1, 2, ..., задается вероятностями Ρ(ξ = й) = С? + *-1рУ, ft = 0, 1, 2, ... (9 = 1 -р). Это распределение числа «успехов», предшествующих г-му «неуспеху» в бесконечной последовательности испытаний Бернулли. Здесь Εξ = rp/q, Dl = rp/q2. В частном случае г = 1 распределение Ш(\, р) называется геометрическим. 13) Гипергеометрическое распределение H(r, N, п) задается вероятностями Ρ(ξ = k) = CtCr-r/CnH, max(0, η + r - Ν) < k < mm{n, r). Если в урне содержатся N шаров, г из которых красные и N — r — черные, и из урны извлекается случайная выборка без возвращения объема п, то случайная величина ξ — число красных шаров в выборке — имеет указанное распределение. Здесь Εξ = Ц-, Οξ = ^(1 — -'/*/) тйг и вообще Ε®'· = 7^· Дальнейшие свойства этих распределений рассмотрены в задачах 1.39—1.55. Если статистическая модель F= {/^} задается распределением какого-нибудь стандартного типа при неизвестных определяющих его параметрах θ (или некоторых из них, если параметров несколько), то модель имеет такое же название. Например, говорят о нормальной модели #(θ, σ2), когда среднее неизвестно, о модели Λ/(μ, θ2), когда неизвестна лишь дисперсия, об общей нормальной модели Λ^(θι, Θ2), когда оба параметра неизвестны, о пуассоновской модели Π(θ) и т. д. 7. Для изучения и иллюстрации эффективности различных статистических процедур удобно использовать статистическое моделирование, реализуемое с помощью последовательности псевдослучайных чисел. Псевдослучайными числами называют последовательности чисел, получаемые по некоторому алгоритму и обладающие свойствами последовательности случайных чисел. Методы получения псевдослучайных чисел рассматриваются в учебной и монографической литературе [9, 10]. Обычно для получения реализации последовательности независимых случайных чисел с произвольным 17
распределением используют реализацию последовательности независимых случайных чисел, равномерно распределенных на отрезке [0, 1]. Реализацию равномерно распределенных случайных чисел ί/ο, ί/ι, ί/2, ... (1.5) наиболее часто получают линейным конгруэнтным методом [9] : UH = zn/m, (1.6) где Zn — последовательность, определяемая рекуррентным соотношением ζη + ι = azn + с (mod пг), где z0 — начальное значение, а, с, т — положительные целые числа. Последовательность (1.5), определяемую формулой (1.6), строго говоря, нельзя рассматривать как реализацию независимой последовательности равномерно распределенных чисел, так как она является либо периодической, либо содержит период с подходом. При этом длина периода Τ не превышает т, так как не превышает т число различных значений ζ„, η = О, 1, 2 Очевидно, что бессмысленно использование таких последовательностей длиной, превосходящей длину подхода и периода. Однако при специальном выборе констант а, с, т и Zo последовательность (1.5) имеет максимально возможный период т. Условия, при которых последовательность имеет максимальный период, приводятся в следующей теореме [9]. Теорема. Длина периода линейной конгруэнтной последовательности (1.5) равна пг тогда и только тогда, когда: 1) cum — взаимно простые числа; 2) Ь = а — 1 кратно ρ для любого простого р, являющегося делителем гп\ 3) b кратно 4, если ш кратно 4. В приложении приведены две программы датчиков равномерных псевдослучайных чисел на автокодах машин ЕС и БЭСМ-6. В датчике D\ были использованы постоянные а = 843314861, с = 453816693, пг = 231, а в D2 — постоянные а = 431777206549, с = 232354146751, пг = 2й. Обе программы вызываются оператором Y = = RAN(K), где К — произвольное целое число из отрезков [1, 2—1] и [1, 241 — 1] соответственно. 18
Наличие полного периода, однако, еще не обеспечивает хороших свойств псевдослучайных чисел. Даже в датчиках, рекомендованных для широкого использования, нередко обнаруживаются существенные недостатки. Проверка «качества» последовательностей, вырабатываемых датчиками, проводится с помощью различных статистических критериев [8]. При этом обычно ограничиваются проверкой равномерной распределенности s-цепочек (s = 1, 2, ...) последовательности (1.5) и далее используют эту последовательность для решения модельных задач. Приведем несколько примеров моделирования выборок. Для получения η равномерно распределенных чисел Х\, Хо, ...,'Хп можно воспользоваться программой DIMENSION A(\ 0 0) К = S DOM = 1, 1 0 0 1 A(I) = RAN(K) STOP END, где оператор Υ = RAN(K) вызывает подпрограмму датчика случайных чисел. Результаты вычислений при η = 100 приведены в табл. 1.1. Таблица 1.1 (100 равномерно распределенных случайных чисел) 0,168 718 549 099 550 100 571 121 012 562 0,273 823 754 304 855 405 976 526 517 067 0,878 428 459 009 660 210 881 431 522 072 0,983 533 664 214 965 515 286 836 027 577 0,588 138 369 919 770 320 191 741 032 562 0,693 243 574 124 075 625 596 146 537 087 0,298 848 279 829 880 430 501 051 542 092 0,403 953 484 034 185 735 906 456 047 597 tf,008 558 189 739 990 540 811 361 052 602 0,113 663 394 944 29'5 845 216 766 557 107 Воспользовавшись программой «ВР» (см. Приложение), получаем вариационный ряд выборки, приведенной в табл. 1.1. На рис. 1 приведен график эмпирической функции распределения Fn(x), вычисленной по этому вариационному ряду. Воспользуемся теперь программой «НЧ» (см. Приложение) для получения η нормально распределенных 19
Рис. 1 случайных чисел ΛΊ, Хг Хп с параметрами μ= ΕΧι, σ2 = DA",·. На рис. 2—4 приведены три гистограммы при μ = 1, σ2 = 4 и η = 10, 100, 1000. Разобьем ось Ох на интервалы длины /г, равной при η = 10, 100, 1000 соответственно 3; 1,5; 0,75. Точка χ = 1 при любом п. являлась граничной точкой интервалов. В табл. 1.2 приведены значения оценок л л х = -\- Σ χι. s'2= —r-Σ № - *f " f = I ™ ' f = I параметров μ = 1 и σ2 = 4. Рис. 2 20
Таблица 1.2. η Χ S'2 10 0,676 3,901 . 100 1,016 4,315 1000 0,988 4,306 1.1. Предложить способ моделирования последовательности испытаний Бернулли ΛΊ, ^2, ···, Хп, ···, где Р(Хп= 1)= \-Р(Хп =0) = р. Указание. Использовать последовательность псевдослучайных чисел, равномерно распределенных на отрезке [0, 1]. „ 2—„ 2d1 У' ήπό е 7 2 3 4 5 6 7 χ Рис. 3 η--woo К'1 -5-4 -3-2 Рис. 4 21
1.2. Смоделировать последовательность испытаний Бернулли, указанную в задаче 1.1, с ρ = 0,4 и η = 1000. Вычислить частоты μ*/£, где μ* = Х\ + ··· + Хк, при k = 100, 200, ..., 900, 1000. В системе координат хОу построить график, соединив прямыми соседние точки (*, \ik/k), k = 100, 200, ..., 1000. 1.3. Указать способ моделирования независимых испытаний в полиномиальной схеме с исходами 1, 2, ..., N и вероятностями исходов соответственно р\, р2, ..., ры- 1.4. Указать способ моделирования симметричного блуждания с дискретным временем по целым точкам прямой с началом в точке 0 (вероятности перехода в соседние точки за один шаг предполагаются одинаковыми). 1.5. Пусть случайная величина ξ равномерно распределена в интервале [0, 1], F(x)— непрерывная функция распределения. Найти функцию распределения случайной величины η = F~\l·,), где χ = F~\y) — функция, обратная функции у = F(x). 1.6. Предложить способ моделирования случайной последовательности ΛΊ, Х2, ..., Хп, ··· , где P(Xn^t) = = 1 — е~l/a, t^0 (а > 0 — постоянная). | Указание. Воспользоваться предыдущей задачей. 1.7. Смоделировать независимые показательно распределенные величины ΛΊ, ^2, ..., Ал с α = 1, л = 100. Построить график эмпирической функции распределения и гистограмму; вычислить первый и второй выборочные моменты Ani, Ап2. Указание. Использовать предыдущую задачу и программу «ВР». 1.8. Указать способ моделирования эрланговской случайной последовательности {Х,} с параметрами (а, т.) (т. е. ЦХ,) = Г(а, т), / = 1, 2, ...). 1.9. Используя центральную предельную теорему, указать способ моделирования приближенно нормально распределенных случайных чисел Хп, η = 1, 2, .... 1.10. Пусть Хци ..., Хмп — реализация последовательности приближенно нормально распределенных чисел, каждое из которых получено суммированием N равномерно распределенных слагаемых (см. предыдущую задачу). Получить три реализации (при N = 2, 4, 12) выборок с η = 100, α = 0, σ2 = 1. Для каждой выборки построить эмпирические функции распределения и гистограммы; получить оценки а и σ2. 1.11. По выборкам предыдущей задачи вычислить 22
3-й и 4-й выборочные центральные моменты и сравнить их с истинными значениями теоретических моментов. 1.12. Указать способ моделирования выборки из биномиального распределения Bi(k, p). 1.13. Пусть ν„ — число успехов в η испытаниях Бер- нулли с вероятностью успеха ρ 6 (0, 1). При больших η вычислить границу δν такую, чтобы событие | — — — р\ ^ δν имело вероятность «-у. Укладываются ли r эти границы при у = 0,98 результаты следующего эксперимента (эксперимент Бюффона): при η = 4040 бросаниях монеты наблюдалось h = 2048 выпадений «герба»? Указание. Применить теорему Муавра—Лапласа; монету считать симметричной. 1.14. Используя такой же подход, как в предыдущей задаче, проверить соответствие теории следующих данных: среди η = 10000 «случайных чисел» 0, 1, ..., 9 числа, не превосходящие 4, встретились h = 5089 раз. 1.15. Смоделировать выборку объема η =■ 1000 из распределения Бернулли Bi(\, 3/5) и аналогично задаче 1.13 проверить соответствие экспериментальных данных предсказанию теории. I Указание. Воспользоваться задачей 1.1. 1.16. Проводились опыты с бросанием одновременно 12 игральных костей. Наблюдаемую случайную величину ξ считали равной числу костей, на которых выпало 4, 5 или 6 очков. Пусть hi — число опытов, в которых наблюдалось значение ξ = i, i = 0, 1, ..., 12. Данные для п = 4096 опытов приведены [3, с. 38] в следующей таблице: i hi 0 0 1 7 2 60 3 198 4 430 5 731 6 948 7 847 8 536 9 257 10 71 11 11 12 0 Всего η = 4096 а) Построить график частот hi/n и сравнить его с графиком функции се~х'/2. б) Вычислить выборочные среднее и дисперсию, а также выборочные коэффициенты асимметрии и эксцесса. в) Принимая L(|) = Bif\2, —), найти δ из условия Р(\Х — а\ Ι ^ δ) = 0,998 и сравнить с δ вычисленное по указанным данным отклонение выборочного среднего от теоретического сц. 23
Указание. При оценке указанной в п. в) вероятности использовать теорему об асимптотической нормальности выборочного среднего. 1.17 (продолжение задачи 1.16). В предыдущем эксперименте наблюдалась также случайная величина ξ, равная числу костей с 6 очками. Таблица наблюдавшихся данных в этом случае имеет [3, с. 45] вид I ы 0 447 1 1145 2 1181 3 796 4 380 5 115 6 24 >7 8 Всего л=4096 Ответить на вопросы задачи 1.16, считая при этом L(l) = Bi(\2, {-). 1.18. Смоделировать выборку объема п. = 1000 из распределения L (ξ) = fit Η, —J и проанализировать соответствующие данные аналогично задаче 1.16. | У к а з а н и е. Воспользоваться задачей 1.12. 1.19. Наблюдались показания 500 наугад выбранных часов, выставленных в витринах. Пусть i — номер промежутка от i-ro часа до (i + 1)-го, i = 0, 1, ..., 11, а Ы — число часов, показания которых принадлежат ί-му промежутку. Результаты таким образом сгруппированных наблюдений оказались следующими [4, с. 459]: I h. 0 41 1 34 2 54 3 39 4 49 5 45 6 41 7 33 8 37 9 41 10 47 11 39 Всего л=500 а) Построить полигон частот и сравнить его с графиком функции f(x) = с, 0 ^ χ < 12. б) Рассматривая эти данные как независимые наблюдения над дискретной случайной величиной ξ, принимающей значения, совпадающие с серединами соответствующих интервалов (т.е. значения 0,5; 1,5; ...; 11,5), вычислить выборочные среднее и дисперсию. в) Принимая, что в п. б) случайная величина ξ имеет равномерное распределение, найти δ из условия Р(\Х — αϊ | ^ δ) = 0,98 и сравнить с ним наблюдавшееся значение отклонения \Х — ot11. 1.20. Смоделировать выборку из полиномиального распределения Μ (500; 1/5, 1/5, 1/5, 1/5, 1/5) и, рассмат- 24
ривая эти данные как наблюдения над случайной величиной |, принимающей значения — 2, — 1, 0, 1, 2, проанализировать соответствующие данные аналогично задаче 1.19. |Указание. Воспользоваться задачей 1.3. 1.21. В опытах наблюдалась неотрицательная непрерывная случайная величина ξ. Ее значения (упорядоченные по величине и округленные с точностью до 0,01) для η = 50 опытов оказались равными: 0,01 0,01 0,04 0,17 0,18 0,22 0,22 0,25 0,25 0,29 0,42 0,46 0,47 0,47 0,56 0,59 0,67 0,68 0,70 0,72 0,76 0,78 0,83 0,85 0,87 0,93 1,00 1,01 1,01 1,02 1,03 1,05 1,32 1,34 1,37 1,47 1,50 1,52 1,54 1,59 1,71 1,90 2,10 2,35 2,46 2,46 2,50 3,73 4,07 6,03. Построить эмпирическую функцию распределения и гистограмму, сравнить гистограмму с графиком функции се~х/а, χ > 0. Вычислить выборочные среднее и дисперсию. 1.22. Получена выборка объема η = 100: 0,144 0,937 1,787 —1,052 —0,192 0,169 2,623 2,135 1,759 0,811 0,724 — 0,110 1,752 —0,378 0,417 1,360 1,365 2,587 1,621 2,344 1,379 0,560 1,858 2,453 —0,356 1,503 —0,134 2,950 — 0,816 0,717 2,468 1,131 1,047 1,355 1,162 —0,491 0,261 — 0,183 0,467 0,502 -0,805 0,228 2,286 0,364 —0,312 — 0,045 2,559 0,129 0,898 0,877 3,285 1,554 1,418 0,423 — 0,489—0,255 1,092 0,402—0,051 0,020,0,398 1,399 2,121 — 0,026 1,087 2,018 —0,437 1,661 1,091 0,363 1,229 0,416 1,705 1,124 1,341 2,320 0,176 —0,541 0,837 3,329 2,382 — 0,454 2,537 —0,299 1,363 0,644 0,975 1,294 3,194 0,605 1,978 1,109 2,434 —0,094 0,735 0,143 —0,421 —0,773 1,570 0,947. Построить эмпирическую функцию распределения и гистограмму. Вычислить выборочные среднее и дисперсию, а также выборочные коэффициенты асимметрии и эксцесса. 1.23. При проведении η = 2608 опытов по наблюдению числа α-частиц (ξ), излучаемых радиоактивным веществом за определенный период времени (7,5 с), получены следующие данные (Л; — число опытов, для которых число частиц \ = i, i = 0, 1, ...): I hi 0 57 1 203 2 383 3 525 4 532 5 408 6 273 7 139 8 45 9 27 10 10 11 4 >12 2 Всего =2608 Построить график частот /ц/п и вычислить выборочные среднее и дисперсию [7, с. 177]. 25
В следующих ниже задачах X = (X , Хп) — выборка из некоторого распределения L (ξ), F (χ) и Fn(x) — соответствующие теоретическая и эмпирическая функции распределения (см. (1.1)). 1.24. Для заданной точки х0 такой, что 0 < F(xo) < 1, и заданного числа t оценить при больших η вероятность события \Fn{x) - F(x0)\ <С //Vn. Указание. Воспользоваться теоремой Муавра — Лапласа. 1.25. Пусть Х\ < хг — дое заданные точки на числовой прямой такие, что 0 < F (х\) < F (х2) < 1. Доказать формулу cov (Fn (*ι), Fn (х2)) = 1 F (*,) (1 - F (х2)). Указание. Представить случайные величины μ„ (xt) и Δ„(*ι, х2) = μη(χ2) — μη(χι) в виде сумм независимых индикаторов: , . . . (\ при Χι^,χι, i = l,...,n, μ-(") = 4.+..· + η.. гдеЛ1 = {0 при Xt>xu 1 ПрИ Х\ < X; < Х2, &п(х\, Χ2) = ζ\ +■■■ +ζη, где ς,- = ] /' = 1, -,η, О в остальных случаях. 1.26. Пусть Х\ < χι < ... < Xjv-i — заданные точки на числовой прямой такие, что 0 < F (х\) < F (х2) < ... ... < F{xn-\) < 1. Рассмотрев случайные величины ν,- = μη(Χι) — μη(*ι-ι), i = 1, .... N (здесь μ„(Λ!ο) = О, μη(χΝ) = η), убедиться в том, что случайный вектор ν = (ν,, ..., vjv) имеет полиномиальное распределение М(п; pi, ..., ры), где р; = F (χι) — F (χι-\), i = 1, ..., Ν, F (χ0) = 0, F (χν) = 1. Получить отсюда результат задачи 1.25. 1.27. Вывести следующие формулы для моментов выборочных моментов: £к „ „„,. ι л А \ a*+s ~ α>α' ЕАпк = Ε ξ* = a*, cov (Ant, Ans) ES2=·" * η»ο2 (" - 1) / л-3 i\ μ* = Ε (ξ - a,)*, cov (X, S2) = 2^-L μ3. 26
Вычислить значения этих моментов для случая Щ) = Λί(μ,σ2). 1.28. Доказать, что для любых фиксированных г ^ 2, 1 ^ k\ < ... < k, совместное распределение выборочных моментов АПк,, ·.·, Апк, при η -*■ оо асимптотически нормально Ν{ο. = (α*,, ..., α*,).— Σ), гДе Σ = ΙΙσ</ = а*. + */ _ - α,,α*,||ί, т. е. L (Vn (Л*, - а*,), i = 1, ..., /■)->· Ν (Ο, Σ) (предполагается, что все указанные теоретические моменты существуют). Кроме того, если φ (ж), ж = (Ж|, ..., ж,), — любая дифференцируемая функция, то L(V" (φ(Α>*,,... ..., Α·*,) — φ (α)))->-Ν (0, у2) при условии, что υ Φ О, где »'=b'ibj=(» *£%_.· Указание. Применить центральную предельную теорему для векторных случайных величин и утверждение 4°, п. 4, гл. 1. 1.29*. Доказать, что при η -*■ оо выборочная дисперсия Sn асимптотически нормальна Λί(μ2, (μι — μί)/η) и при этом Ε S2n ~ μ2, D 52, ~ (μ4 — μί)/η (предполагается, что μ4 < оо). Указание. Воспользоваться утверждением 2°а), п. 4 гл. 1. 1.30. Доказать, что совместная функция распределения двух порядковых статистик Х(Г) и Хщ (1 ^ г < s ^ п) имеет вид η it — πι Fr,s (χ ι, ж2) = Σ Σ ,-,,"' ^Fm (*ι) [Ζ7 (*2) - л! = /- / —-max (0. s — m) J ч J' - F(*,)f(l - F(x2)f-m-\ если Ж| < Χι, и /v.s (ж,, ж2) = Ρ (X(s) < ж2) = №) = = Σ OnF>(x2){l - /4*2))"-', если λι > ж2. Вывести отсюда формулу для F, (ж) = = Ρ {ХА < х)- 1.31. Пусть распределение L (ξ) абсолютно непрерывно и его плотность F'(x) = f(x). Вывести следующую формулу для плотности совместного распределения порядковых статистик Х(к,), ..., Λ^,) (1 < k\ < ... < k, < η): 27
£*'-*'(* *'' = (ft, - l)!(ft2-ft,-l)!...(ft,-ft,_l-l)!(n-ftf)!X Xi*,_l(ii)(f(jti) - F(Xi)f'-kl-1 ...(F(x,)- - Fixr-if-"-'-^ - F{xr))n-k'f{x,)...f{xr), X\ < X2 < ... < */■· В частности, совместная плотность всех η порядковых статистик Λ^ΐ),..., А(П) равна gi...„(*ι, .... *„) = п\ f (*,)... f(xn), Х\ < хг < ... < хп. 1.32*. Доказать, что если в некоторых окрестностях квантилей с^, и ςρ,(0 < р\ < pi < 1) плотность / (х) непрерывна вместе с производной и f (&,,) > О, i = 1,2, то при η -*■ оо выборочные квантили Z„.Pi = А([яр,] + i), i = 1,2, асимптотически нормальны Л/((&>,> &>,), — ||σ,;||?), где σ,,- = = fl ,T,P > . i ^ /· Обобщить на случай л квантилей. , /Ы/Ы 1.33*. Доказать, что для выборки из абсолютно непрерывного распределения крайние порядковые статистики Х(Г) и А(л_5+|) при п-+оо и фиксированных г, s ^ 1 асимптотически независимы. Указание. Перейти к случайным величинам у.п = nF (А(,)) и η„ = n[l — F(A(„_s + i))] и воспользоваться результатом задачи 1.31. 1.34. Пусть L(l·,) = Г (1, 1). Доказать, что случайные величины Y, = {η — г + 1)(А(,) — Х(г-ц),г — 1,..., п{Х{0) = = 0) независимы и одинаково распределены с плотностью f (х) = е~', χ > 0. Основываясь на этом, вычислить ЕАда, DA(t) и исследовать асимптотическое поведение EA(rt) и DA(„) при п. -у оо, Указание. Воспользоваться представлением η η Σ Xr = Σ (" — r + l)(Xr — Xr-\), xa = 0, и резуль- татом задачи. 1.31. 1.35. Убедиться в том, что для случая L(E) = = R (0, 1) распределения порядковых статистик имеют вид L (Ада) = p(k,n-k+ 1), L(X(I) - Ада) = = β(/ - *, η - I + k + 1), 1 ^ k < / ^ п. Вычислить средние и дисперсии этих распределений, а также cov (Ада, Ащ). 28
1.36. Пусть Ζ,(ξ) = R(a,b). Показать, что плотность совместного распределения экстремальных значений выборки Ail, и А(„) имеет вид "ft(" ~ V (*2 — х\)"~2, а<х,< Получить следующие формулы: cv па + b _ v a + nb ЬЛ0) = „ ^ ι . сЛ(„) = - „ + 1 ' '"> η + 1 ' DA(„ = DA(n, = {пп±\-{па12), cov(A(l), А(л)) = _ (t> - а)2 (η + 1)2(η + 2) 1.37. Пусть L (ξ) есть распределение Вейбулла W (a, a, b). Найти распределение минимального значения выборки А(|) и вычислить EA(i) и DA(i). 1.38. Пусть Xi = (A/i, Αί2), ί= 1,...,л, — независимые наблюдения над двумерной случайной величиной 6= (ЕьЫ с функцией распределения F{x\,Xi). Эмпирическая функция распределения определяется в данном случае формулой п. Fn {χι, χ2) = — Σ e(xt — % и) е(х* — Xi*) / = ι (ср. с (1.1)). Вычислить EF„(xi,X2) и DF,, (χ,, х2) и показать, что Fn(Xi, Xi) -*■ F (х\, х2), когда η ->- оо. Построить выборочный коэффициент корреляции р„ и. показать, что р„ -»■ ρ = когг (ξ,, ξ2), если Ε(ξ?ξ2) < оо и Ώξ, > О, / = 1,2. 1.39. Говорят, что распределение La, зависящее от параметра а, является воспроизводящим по этому параметру, если для независимых случайных величин ξι и ξ2 с распределениями соответственно La, и La, выполняется свойство ί,(ξι + Ъ) = Lfl.+fl.". это записывается также в виде La, %■ La, = La,+a„ где %. означает операцию свертки. Убедиться в справедливости следующих утверждений: 1) Ν(μ,,σ?) * Ν(μ2, σΐ) = Ν (μ, +μ2, σ? +σΙ); 2) Γ (α, λ,) * Γ (α, λ2) = Γ (α, λ, + λ2); 3) Λί (πι;ρι, ..., ρ„)* Μ(λ2; ρι, ..., ρΝ) = Μ (ηι + ηχ Ρ\, .... Ρν)\ в частности, ΒΊ (πι, ρ) ^< ΒΊ (η2, ρ) = ΒΊ(η.\ + η2, ρ); 4) Π (λ,) *Π (λ2) = Π (λ, + λ2); 29
5) Bi(ritp)*Bl(r2,p) = Ъ1(п + r2,p). Указание. Использовать тот факт, что характеристическая функция суммы независимых случайных величин равна произведению характеристических функций слагаемых; при этом для дискретных случайных величин вместо характеристических функций Έ-ё'1 удобно использовать производящие функции ЕД 1.40*. Пусть случайный вектор Υ имеет невырожденное нормальное распределение Ν(μ,Σ) и Q = (Υ — — μ)Ά(Υ — μ), где матрица А удовлетворяет условию А = ΑΣΑ.ι Доказать, что L(Q) = χ2(/η), где пг = = tr(i4Z). В частности, при А = Σ-' число степеней свободы т совпадает с размерностью вектора Y. 1.41. Совместное распределение двух случайных величин X и Υ описывается следующим образом: условное распределение X при условии К = у нормально N(y, σ?), a L(Y) = Ν{μ, al). Доказать, что L(X) = Ν {μ, σ? + σ22). Указание. Вычислить плотность распределения X по формуле оо fx(x) = \ hiy(x,y)fY(y)dy, — оо где }хп(х,у) — плотность условного распределения L(X\Y = y). 1.42. Случайные величины Х\ и Х3 независимы, причем L(Xi) = Г (α, λ), ЦХ, + Х2) = Г (α, λ + μ), μ > 0. Как распределена случайная величина Л"2? I Указание. Вычислить характеристическую функ- I цию для Χι (см. решение задачи 1.39 п. 2)). 1.43. Пусть ξι и ξ2— независимые равномерно распределенные величины на отрезке [0, 11, Показать, что величины ηι =V— 21n£2cos (2πξι), η2=-\/ — 21n£2sin (2πξι) независимы и нормально распределены с параметрами (0,1). | Указание. Воспользоваться формулой (1.2), 1.44. Пусть случайные величины Х\ и Х2 независимы и L(Xi) = Г (α, λ,-), ί=1,2. Доказать, что случайные величины Κι = ΛΊ + Лг и Kj = Х\/{Х\ + А"2) независимы и при этом L (К,) = Г (α, λ| + λ2) — воспроизводимость по параметру λ (см. задачу 1.39 п. 2), L(Y2) = β(λι,λ2). 30
1.45. Доказать, что L ( х" _ " ) ->- iV(0, 1) при п -»■ оо. Установить формулу Ε (Χ2,)* = η {η + 2)... (η + 2{k — 1)), * = 1,2,.... Указание, Воспользоваться свойством воспроизводимости по параметру λ распределения Г (α, λ) и применить центральную предельную теорему. 1.46. Убедиться в том, что L.(a + —) = К (а), где случайные величины ξ и η независимы и нормальны N (0, σ2), а также L(a + tgy = K(a), где Ζ,(ξ) = *(--£,-£.). 1.47. Проверить, что если L(t„) = S(n), то моменты Е/я существуют лишь при k < n и равны Ε/2' = '·w^^(2Vί", 9W 2Г < "' Е/«Л+ ' = θ' (я — 2) (л — 4) ... (я — 2/·) 2л + I < п. Доказать, что S (и)->-JV (О, 1) при η ->■ оо и, более того, плотность s„(x) -► -г£-е~х'12- Установить, что // '. ) = = β(!4). Указание. Использовать формулу Стирлинга для гамма-функции: Г (ζ) ~ -\/2ηζ гг~ ]е~г, ζ -*■ оо, и применить закон больших чисел к случайной величине %п/п (см. решение задачи 1.45). При вычислении моментов учесть, что /„ = η "γ-ι-, а также независи- мость сомножителей. Использовать задачу 1.44. 1.48*. Пусть F (х; пь п2) — функция распределения закона Снедекора S (πι, η2), а В (г, a, b) — функция распределения закона β (a, b). Установить равенство F(x\ щ, п2) = В{ -1 ;-!- ,-М, * > 0. Получить отсюда выражение для плотности /„,,„,(х) распределения 5 (πι, η2). Найти моменты этого распределения. 1.49* (продолжение задачи 1.48). Доказать, что при любых фиксированных *<= (0, 1) и а > 0 lim -i- In [1 - В {χ; a, b)] = In (1 - χ). Получить отсюда, что при любых фиксированных / > 0 и т ^ 1 31
lim — In [1 — F (ln\ m, n)] = - -L In (1 + mt). Указание. Воспользоваться формулой Стирлинга (см. указание к задаче 1.47) и теоремой о среднем значении: I I \иа~\\ -u)b-ldu =C°-l\(\-u)b-,du = χ χ = c4' (i-*)\ ce[x, i]. 1.50*. Показать, что плотность sn(x) распределения Стьюдента S(n) выражается через плотность fi,n{x) распределения Снедекора 5(1, п) следующим образом: sn{x) = \x\fi.nix2). Доказать соотношение lim— In Р(/я > dV") = —4-1п (1 + d2) V d > 0. 1 -> со 1 -ώ I Указание. Использовать тот факт, что L{t2n) = S(l, n). 1.51. Пусть Х=(Х[, ..., Χι, Χι + \, ..., Χι + т) — выборка из экспоненциального распределения Ζ.(ξ) = Т(а, 1). Рассмотрев случайную величину У = Ц—-— , '" , J , ι Χι + ι + ... + Λι+ m доказать, что L(Y) = 5(2/, 2m). Ι Указание. Использовать тот факт, что L{tl) = I =5(1, я). 1.52. Пусть целочисленный случайный вектор ν = = (ν \Ν) имеет полиномиальное распределение М(п; ρ,, ... , ρΝ). а) Показать, что производящая функция для (vb ..., ν*), к ^ N, имеет вид Ε(χ]'...χΐ)=[ΐ+ Σ Ρ.{*«-1)]", в частности, L(vi) = Bi(n, р\). б) Вывести следующую общую формулу для смешанных факториальных моментов: E{\\)kl..{\N\N = (n)fe, + ... + kNp\'.-pff. в) Пусть η'' = Σ Civ,, С> = Σ Ctp,, /= 1, 2, cW= l = I i - I 32
Ν _ = Σ C'Cfpi. Показать, что Εη' = пО, cov (η1, η2) = ί = Ι =n(CTC1-Cl С2). Указание. Использовать результат, приведенный в решении задачи 1.39 п. 3). 1.53* (продолжение задачи 1.52). Доказать, что для любого k < N при η -*■ оо и фиксированных р* 6 (0, 1), »= 1, ..·, N, £((v,-- п/>/)/лЯ / = 1, ..., k)-+N(0, Xk = = ΙΙσ,-illi), где σ;/=/P-(l - Р«) при / = / I —РФ) при ί =£ /· ^ |Σ,| *=0. I Указание. Воспользоваться теоремой непрерыв- | ности для характеристических функций. 1.54. Доказать, что если случайные величины ξ,, ..., \Ν независимы и £(!,■)= Π(λ;), /' = 1, ..., Ν, то условное распределение L{b, ·■·, &νΙξι +·.· + £ν = η) = М(л; pi, ..., ρΝ), где ρ,· = -—;—'———, / = 1 Ν. Отсюда, в частности λ| + ... + λΝ следует, что L(|il£i + ·.· + In = η) = Bi{n, p,). 1.55. Пусть имеются две случайные величины ξ и Д. причем L(/\) = Г(а, г) при некотором α > 0 и целом г~^\, а условное распределение L(£| Л = λ) = ΙΊ(λ). Показать, что безусловное распределение £(ξ) = В/(г, р) 1.56. Пусть X = (Л , Ля) — выборка из Ν(μ, σ2). Доказать, что Л и (Л,—Л, ..., Л„ — Л) независимы. Получить отсюда независимость выборочных среднего Л и дисперсии 52. Указание. Воспользоваться тем, что для нормальных случайных величин из их некоррелированности следует независимость. 1.57*. Пусть Х = (Я[, ..., Лп) — выборка из распределения JV(0, 1) и квадратичная форма Q = Х'Х разложена на сумму двух квадратичных форм Q = Q\-\-Qi, где Q, = ΧΆιΧ и rang A, = m, i = 1, 2. Доказать, что если гц + п.2 = п, то Q, и Q2 независимы и L(Qi) = %2(ш), i=l,2. 2-190 33
Указание. Проверить, что матрицы А\ и А2 идем- потентны и А\А2 = 0; далее результат следует из утверждений 1° и 2°, п. 6, 1) гл. 1. Замечание. Справедливо более сильное утверждение, именно, если Q = Qi + ■■■ + Q*. где Q; = X'AJi, rang Αι = ni} i = 1, ..., k, το η = ti\ -j- ··· + "* **=>- o- Qi. ···. Qk независимы и L(Qi) = χ2(η<), / = 1, .... k. 1.58*. Пусть Χ = (ΑΊ, ..., Χ„)— выборка из нормального Λ^μ, σ2) распределения. Найти распределение случай- у у ной величины η = - ' - IS 1.59*. Использовать обозначения, введенные в задаче 1.38, и ее решении, и пусть Щ)=Ц». ^).Σ=||ίσ9Ρ3σ1>-ι<Ρ<ι. а) Доказать, что (Хи Х2) и (5ь S\2, Si) независимы; б) ПуСТЬ Q = n(Xl-^, Χί-μίΥΣ~\Χί-\4, *2 - — μ2), доказать, что L(Q) = χ2(2); в) пусть о2и Τ = -Jn — 2ря/д/1 — pi где ря = = S12/S1S2 — выборочный коэффициент корреляции. Доказать, что при ρ = 0 L(T) = S(n-2), получить отсюда распределение р„. Указание, а) См. задачу 1.56 и ее решение; б) воспользоваться задачей 1.40; в) использовать следующий факт [4, с. 435]: плотность совместного распределения случайных величин (5ь 5i2, Si) имеет вид (при ρ = 0) л-1 / 2 \ 2 я ( Х\ ■ X t \ /(*„ *,,, х2) = \ я "V-? ' ~ ' Ισ? "''} 4πΓ(η — 2)(σισ2) Х\, Χι < 0, х\2 < Х\Х2- Далее рассмотреть новые случайные величины Υ\ = -fn S V« Si; γ n I C2 5ΪΛ V n c2 σι S2 of V Si J a% и учесть, что Т = Y\/^Y2/{rt — 2). 1.60*. Пусть Χ и S2— выборочные среднее и дисперсия для выборки объема η из распределения Π(λ). Доказать, что при η -*■ оо и любом λ > 0 34
L^Tn^-^^(X-1^-rS2)/x)^N(0, 1). Указание. Воспользоваться задачами 127—1.29 и установить сначала асимптотическую нормальность Л/[0, 1) случайной величины ς„=-ι/^—\^~ ZT^2)/^· Далее использовать тот факт, что Χ/λ·Ζ>- 1 при η -*■ со и утверждение 2° в) п. 4 гл. 1. При вычислении моментов использовать следующие формулы для центральных моментов пуассоновского распределения Π(λ): μ2 = Цз = λ, μ4 = λ + 3λ2. 1.61. Пусть Χ\, ..., Χη — независимые наблюдения над случайной величиной ξ с Εξ = μ, ϋξ = σ2 > О, Εξ4 < οο и Χ, S2 — соответствующие выборочные среднее и дисперсия. Доказать, что при η -*■ со L(Tn = -fn(X-v)/S)-+N(0, 1). Указание. Воспользоваться центральной предельной теоремой, сходимостью S/σ-^-Ι при η со и утверждением 2° в) п. 4 гл. 1. ..62. пусть ^„ы=4^.μ2), || ^;σ2Ρχ2Ρ|Ι> где -1<Р = korr(S,, h)< 1- Доказать, что условное распределение ί<(ξ2ΙΕι = х) = = N(m(x), с2), где условное среднее т(х)= Ε(ξ2|ξ, = *) = μ2 + ^-ρ(*-μι) — функция регрессии ξ2 на ξ,, являющаяся в данном случае линейной по х, а условная дисперсия σ2 = = D(|2lii = x) = σ2(1 — ρ2) не зависит от х. Установить, существуют ли другие распределения £(ξι, Ъ)< отличные от нормального, обладающие такими же свойствами условного распределения £(ξ2| Ει = *)■ Указания. 1. Вычислить условную плотность ξ2 при условии ξ, = χ по формуле h,\i,(y\x) = k&Xx> У)/к1х) и убедиться в том, что она равна -J=expi —(ί/~т^хЩ с указанными т(х) и σ . 2. Рассмотреть совместную плотность кь(х, у) = = к>(х)}ыь(У\х), где hMi{y\x) — указанная выше условная, а к(х) — произвольная плотности распределения. 2* 35
1.63* (обобщение задачи 1.62). Пусть Х°\ ЛЯ'— два случайных вектора произвольной размерности, Е(Х(,)) = = μ« D(X«)=2,, i=l, 2, cov(tf". Α<5') = Σ,2. COv(tf2>, *<") = Σ2, (=Σ,Α "/.(*<", Χ<2)) = Λί((μ<", μ(2)). ZJ21 ^122 Доказать, что условное распределение L (Д^2)|Х(|) = = х<" = #(М(ж(|)), В), где Λί(*<")=μ<2> + Λ(*0>-μ<"), Α=Σ2,Ση'. о = 2j22 2j2i2jh 2j|2' Убедиться в том, что в случае dim Xм = 1, dimX(1) = = р-1, Л= -i-(alp,a2p,...,ap-'.p), В = ± где Σ"'= ||σ"'||ϊ. Указания. 1. Рассмотреть линейное преобразование У(|> = Х(|>, У(2> = Хт - АХ(]) и убедиться в том, что У(1> и У(2> независимы; отсюда следует, что L(XW\X[I) = *(,)) = L(K(2) +АУ(|)|К(|) =*(|)) = = L (У|2) + Ах(1)). 2. Записав совместную плотность f#'Wv(x Хр-\, Хр) = ρ = С ехр { - 1 Σ (* - μ.) (■*/ - μ,) σ''} = = Cexp{ -γ[(χΡ ~ μΡ)2σρρ + ρ- Ι + 2{χ„ - μρ) Σ (* - μί)σ'ρ] + ·.) , получить, что переменная хр войдет в выражение условной плотности в виде ехр{ - J σ"(*ρ-μρ + Σ (*-μι) οίρ/σή) . 1.64. В основе алгоритма моделирования значений случайной величины с пуассоновским распределением 36
Π (λ) лежит следующий факт (доказать!): пусть ί/,, / = 0, 1, 2, ... — независимые случайные величины с рав- номерным распределением /?(0, 1) и £ = maxi k;Y[Ui ^ > в'1} , тогда L (ξ) =Π (λ). Указание. Используя соотношениеl( — Σ 1πίΛ) = = Г (1, k), вычислить вероятность события (ξ = k) = k к+ I = { — Σ In ίΛ- < λ, — Σ In Ui > 4 . 1.65. Пусть на отрезке [α, 6] задана ограниченная плотность распределения / (.*), с = max / (χ). Определим случайную величину <щ*<ь ν = min {: > 1 : с U2i- ι < / (α + (ft - α) Uv)}, где {ί/,} те же, что и в предыдущей задаче. Доказать, что случайная величина ξ = а + (ft — я) U2v имеет плотность распределения / (х). Замечание. Этот результат дает способ моделирования распределения с произвольной плотностью, удовлетворяющей указанным ограничениям. Глава 2 ОЦЕНИВАНИЕ ПАРАМЕТРОВ РАСПРЕДЕЛЕНИЙ 1. Пусть задана статистическая модель F= {F} для схемы повторных независимых наблюдений над некоторой случайной величиной ξ и X = (Х\, ..., Хп) — выборка из L (ξ)· Всякая случайная величина Τ = Τ (Χ), являющаяся функцией лишь от выборки, называется статистикой. Часто требуется по выборке X оценить истинное значение некоторой неизвестной теоретической характеристики g = g(F), т. е. построить такую статистику Т(Х), значение которой можно было бы считать разумным приближением для истинного значения характеристики g. В этом случае статистику Τ (X) называют оценкой g (для g). Для оценивания g можно использовать различные оценки, и их качество сравнивают, исходя из той или иной меры точности оценок (меры близости оценки к истинному значению оцениваемой характеристики). Если определен некоторый класс оценок Те и 37
выбрана мера точности, то оценку, оптимизирующую эту меру, называют оптимальной (в классе Те). Наиболее распространенной мерой точности является среднеквадратическая ошибка Е(Т(Х) — gf. Эта мера порождает и соответствующий критерий оптимальности оценок — критерий минимума среднеквадратической ошибки. Часто ограничиваются рассмотрением лишь класса Те несмещенных оценок: Τ = Τ (Χ) ζ Ур^ -ο· Ε Τ = gV F 6 F. Для несмещенных оценок Е(Т — gf = = D7", т. е. мерой точности таких оценок является величина их дисперсии, а критерием оптимальности для несмещенных оценок является критерий минимума дисперсии. Если модель F параметрическая: F = [F(x;Q), Οε £Ξθ}, то любая теоретическая характеристика является функцией от параметра Θ. Таким образом, в данном случае речь идет об оценивании параметрических функций, для которых будем использовать обозначение τ (θ). Статистика Т= Τ (X) является несмещенной оценкой для τ (θ), если выполняется соотношение Е>Т = τ(0)νδε=Θ. Оптимальной в классе 7\ несмещенных оценок функции τ = τ (θ) является статистика Τ*, для которой D07"* ^ ^ D7-V Τ £Ξ 7"t и νΟ^Ξθ. Для оптимальной оценки Т* иногда используют обозначение τ*, чтобы подчеркнуть, что она относится к функции τ (0). Оптимальная оценка Т* (в заданной модели F и для заданной параметрической функции τ (θ)) существует не всегда, но в тех случаях, когда она существует, она единственна [1, с. 42]. Важным гвляется линейность свойства оптимальности: если T'f — оптимальная оценка τ; = τ;(0), /' = 1,2 то оптимальной оценкой линейной комбинации 2 с/т/ является ста- тистика 2 ciTf [1, с 44]. / Обязательным для любого правила оценивания является свойство состоятельности, которое означает, что при неограниченном возрастании объема выборки η оценка должна сходиться по вероятности к оцениваемой характеристике каким бы ни было истинное распределение наблюдений. Таким образом, состоятельность — это асимптотическое свойство оценок (в отличие от свойств несмещенности и оптимальности). Когда хотят подчеркнуть зависимость рассматриваемых статистик от объема выборки, их отмечают индексом п. При установлении свойства состоятельности полезен следующий простой критерий [1, с. 72]: если ЕаТп = τ(θ) -4- ε„, D07"„ = δ„ иг„ = 38
= ε„(θ)->-0 (т. е. 7\,— асимптотически несмещенная оценка τ (θ)), δ„ = δ„(θ) -► О при п -> оо для всех Οεθ, то Τ η — состоятельная оценка τ (θ). 2. Рассмотрим кратко общие критерии существования оптимальных оценок и способы их нахождения в рамках общей параметрической модели F= {F(x, θ), θ εΞ Θ). Пусть f (x; 0) — плотность распределения наблюдаемой случайной величины ξ (или вероятность события {ζ = χ} в дискретном случае) и χ = (х\, ..., х„) — реализация выборки X = [X Хп). Функция L (χ; θ) = f (*,; θ)Χ ...Xf(xn\Q), рассматриваемая при фиксированном х^Х как функция параметра Οεθ, называется функцией правдоподобия. В дальнейшем предполагается, что L (χ; θ) > 0 при всех дс εΞ Χ и θ εΞ θ и дифференцируема по 0. Более того, справедливо следующее правило перемены порядка дифференцирования и интегрирования (в случае скалярного параметра Θ): для любой статистики Τ (в частности, для Τ = const) ±\ Τ (χ) L (χ; θ) dx = \ Τ (x)L· L (χ; θ) dx (dx = dx\ ... dx„) (интегрирование ведется по всему выборочному пространству X, указанные интегралы, по предположению, абсолютно сходятся при всех θ εΞ 0, везде для дискретных моделей интегрирование заменяется соответствующим сум мированием). Наконец, введем случайную величину irfY.a\ — d\nL(X;0) ν д \п f (Xr. 0) υ (Λ· Q> до Λ до · ι = 1 называемую вкладом выборки, и будем предполагать, что 0 < E0U2{X; 0) < оо V θεθ', Модели, для которых выполняются все перечисленные условия, называют регулярными. Для регулярной модели EoU (Χ; θ) = О V θ ε θ и определена функция /„(θ) = Do U (X; θ) = EeU2(X; θ), называемая функцией информации (функцией Фишера). Величину /(0) = «в) = E0(^4f^)2 = - Ео(^^) называют также количеством (фишеровской) информации, содержащейся в одном наблюдении (последнее выражение используется в тех случаях, когда функция 39
f (χ; θ) дважды дифференцируема по θ). Для схемы повторных независимых наблюдений ί„(θ) = ηί(θ). Введенные понятия непосредственно обобщаются на случай векторного параметра θ = (θι, ..., 0,). В этом случае под вкладом выборки понимается случайный вектор U = (Ui(X;Q) U,(X;Q)), где U,(X; θ) = -±-\nL(X; θ), /= 1 г, а аналогом функции информации является информационная матрица выборки Ιη = /„(Θ) = De(i/) = = Ε§(ί/ί/'). Информационную матрицу одного наблюдений Ζ, = / = ||g//||i можно вычислить по формулам а _ а ((л _ F /AlniHiiiL a in/(*■; β) \ _ Я«/ - &/(в) - ^ ^ ^ j - (последнее равенство справедливо для дважды дифференцируемых функций / (χ; Θ). Для схемы повторных независимых наблюдений /„(θ) = ηΙ(θ). В данном случае в определении регулярной модели предполагается, что матрица /(Θ) невырождена при всех Οεθ. Для регулярных моделей можно установить нижнюю границу для дисперсий несмещенных оценок заданной дифференцируемой параметрической функции τ(θ). Именно (неравенство Рао — Крамера): для любой оценки Τ = Τ(Χ)^Ττ и всех Οεθ имеет место неравенство ~ т ^ ft'(θ)|2 η 8 ^ ■ /о ■ если параметр θ скалярный, и неравенство ΌαΤ^ &'(θ)/-'(θ) 6(θ), Ь (θ) = (-gi- ,..,-^-) . если θ = (0 , θ,). Оценка 7"* εΓ„ для которой достигается указанная нижняя граница, называется эффективной. Если такая оценка существует, то она является, следовательно, оптимальный (в классе Ττ) и единственной. Критерием эффективности является следующее представление [1, с. 47, 52]: Τ {X) - τ (θ) = a(Q)U (X; θ), если θ — скаляр, Τ {Χ) — τ (θ) = α'(θ) U(X; θ), если θ — вектор, где α (θ) (соответственно α (θ)) — некоторая функция (вектор-функция) θ. В заданной модели F эффективная оценка может су- 40
ществовать только для какой-то одной параметрической функции τ (θ) (с точностью до преобразования ατ(θ) + 6, где а и b — константы). В тех случаях, когда эффективной оценки не существует, для отыскания оптимальной оценки Τ* = τ* (в классе несмещенных оценок Γτ) можно использовать следующий алгоритм {критерий Бхаттачария) [1, с. 50, 53]: учитывая старшие производные функции правдоподобия L — L (Χ; Θ), подбирают такую их линейную комбинацию, чтобы получить представление вида -■ + . 2 <*,....-. ав,,...ав, J ' I ίι ' ' J при этом последовательно полагают s = 2, 3 Если при некотором значении s ^ 2 и коэффициентах а. = α. (Θ) это удается сделать, то статистика Τ = Τ (Χ) является оптимальной оценкой функции τ = τ (θ). 3. Наиболее эффективным способом построения оптимальных оценок является использование так называемых достаточных статистик. Статистика Τ = Τ (Χ) (вообще говоря, векторная) называется достаточной для модели F= [F [х\ θ), θ ε Θ] (или достаточной для параметра Θ), если условная плотность (или вероятность — в дискретном случае) L {x\t\ θ) случайного вектора (выборки) X = (X ,Хп) при условии Τ (Χ) = t не зависит от параметра Θ. Эквивалентным определением достаточности является следующее: для любого события A cz X условная вероятность Р0 (X <= А | Τ (X) = t) не зависит от Θ. Это свойство статистики Τ означает, что она содержит всю информацию о параметре Θ, имеющуюся в выборке. Действительно, вероятность любого события, которое может произойти при фиксированном Т, не зависит от Θ, и, следовательно, оно не может нести дополнительной информации о Θ. Сама выборка X, очевидно, является достаточной статистикой, но обычно стремятся найти достаточную статистику наименьшей размерности, представляющую исходные данные в наиболее сжатом виде, в этом смысле говорят о минимальной достаточной статистике. Минимальная достаточная статистика является функцией любых других достаточных статистик. Практически достаточные статистики обычно находят на основании следующего критерия факторизации [1, с. 55]: статистика Τ (X) 41
достаточна для параметра θ тогда и только тогда, когда функция правдоподобия L (χ; Θ) представима в виде L(x;Q) = g(T(x);Q)h(x), где g и h — неотрицательные функции и h не зависит от Θ. Если Τ—достаточная статистика, то таковой же является и любая взаимно однозначная функция от Т. Роль достаточных статистик в теории оценивания определяется теоремой Рао — Блекуэлла — Колмогорова [1, с. 58], согласно которой для любой несмещенной оценки Г, заданной функции τ (θ) можно построить новую несмещенную оценку Τ* = Εο(7Ί|7"), зависящую от достаточной статистики Т, для которой D07* ^ D07|. Следовательно, оптимальную оценку надо искать среди функций от достаточной статистики. При отыскании явного вида оптимальных оценок важную роль играет свойство полноты достаточной статистики. Статистика Τ называется (ограниченно) полной, если для всякой (ограниченной) функции φ (Τ), из того, что Ео<р(7") = Ονδ. следует, что φ(ί) = О на множестве значений Т. Для полной достаточной статистики всякая функция от нее является оптимальной оценкой своего среднего. Следовательно, если оценивается заданная параметрическая функция τ (θ), то оптимальная несмещенная оценка τ* — такая функция τ* = Н(Т) от полной достаточной статистики Т, которая удовлетворяет уравнению несмещенности fU)H(T) = τ (θ). Это уравнение либо имеет единственное решение, либо решений нет. В последнем случае класс Ττ несмещенных оценок τ (θ) пуст. Многие модели математической статистики укладываются в схему r-параметрического экспоненциального семейства, т. е. когда функция f (χ; θ), θ = (θ θ,) <= e θ cz R' представима в виде г f (χ; θ) = expi 2 QiBj(x) + С (θ) + D (χ)} (или приводится к такому виду заменой параметров). В этом случае Т = {Т ТГ), Τι = Τι{Χ)= Σ Β£Χί), j = ί= 1 = 1 г, — минимальная достаточная статистика, и она полная, если dim θ = г. 4. Одним из наиболее универсальных методов оценивания неизвестных параметров распределений являет- 42
ся метод максимального правдоподобия. По этому методу оценкой максимального правдоподобия (о.м.п.) θη параметра θ по выборке Х = (Х\,...,Х„) является такая точка параметрического множества Θ, в которой функция правдоподобия L (χ; θ)Λ при каждом X = χ достигает максимума, т. е. L(x; б„) > L(x; 0)V δ, или L(x; б„) = = sup L (χ; θ). Если для каждого χ <= X максимум L (χ; θ) Οεθ достигается во внутренней точке θ и L (χ; Θ) дифференцируема по Θ, то о.м.п. θ„ удовлетворяет уравнению „ Л з с d\nL(x;Q) п . d\nL(x; θ) „ . правдоподобия }■ ' = 0 (или —*■ ' = О, / = = 1,...,г, если θ = (0 ΘΓ)). Если оценивается некоторая параметрическая функция τ (θ), то ее о.м.п. τ„ = τ (θ„) — это так называемое свойство инвариантности оценок максимального правдоподобия. В тех случаях, когда уравнение правдоподобия не удается решить точно, прибегают к различным приближенным методам решения. Одним из них является рекуррентный метод накопления Фишера, согласно которому (k + l)-e приближение для о.м.п. вычисляется по формуле θ* + ι = θ* + U(x;Qk)/ni(Qk), k = О, 1,2,... ; при этом в качестве начального приближения θα берут значение какой-нибудь легко вычисляемой состоятельной оценки Θ. Для регулярных моделей оценки максимального правдоподобия обладают рядом важных асимптотических свойств. Именно [1, с. 73—75J, если (Г„ существует, единственна и лежит внутри Θ, то она является состоятельной оценкой Θ, причем ее распределение являетсл асимптотически нормальным: Lo(V«(6« - θ)) — ΛΤ(0,/-'(θ)). Л-»-оо если дополнительно предположить, что функция f (χ; θ) I <32f (χ- 01 I трижды дифференцируема по θ и при этом ' V ' <; I dui dOj двь ' ^ Μ (χ), где функция Μ (χ) не зависит от θ и интегрируема: ΕοΑί(|) < оо. Если элементы матрицы / (Θ) непрерывны по 0, то также Мл/" (& - 0)) ~ N(0,1-%)). 43
Более того, если τΑ(θ) — непрерывно дифференцируемая функция ит„ = т (θ„) — ее о.м.п., то при η -*■ оо h (д/« (τ» - τ(θ))) — N(0, σ2τ(0)), а также Ч^^*Ь»<0.1). где σ?(θ) = 6'(в)/-Чв)Ь(в), 6(θ) =(■» «), Вели- чина σ?(θ)/η называется асимптотической дисперсией статистики τ„ и совпадает с границей Рао — Крамера для дисперсий несмещенных оценок функции τ (θ). Такое свойство оценок максимального правдоподобия называется асимптотической эффективностью. Если имеется некоторая другая состоятельная и асимптотически нормальная оценка Т„ для функции τ(θ): Lq (V" {Тп — τ (θ))) ->■ Ν (0, σ? (θ)), то ее «качество» можно λι-»-οο измерять величиной eff (Тп; θ) = σ?(θ)/σ£(θ), называемой асимптотической эффективностью оценки Тп: оценка тем асимптотически «лучше» (точнее), чем больше ее асимптотическая эффективность; для о.м.п. же эта величина равна 1. 5. Наряду с точечным оцениванием неизвестных параметров распределений в математической статистике используется оценивание с помощью доверительных интервалов или (в случае векторного параметра) доверительных множеств. Пусть θ — скаляр. При интервальном оценивании ищут две такие статистики Τι = = Τι(Χ), ί = 1,2, что 7Ί < Ϊ2, для которых при заданном доверительном уровне γε(0, 1) выполняется условие PQ (7, (Х)< θ < ЩХ)) > γ V θ ^ Θ. (*) Такой (случайный) интервал (7,,72)с:в называют у-до- верительным интервалом для 0. Длина этого интервала Τι — Т\ характеризует точность локализации неизвестного параметра, а доверительный уровень у — его «надежность»: вероятность ошибиться, утверждая, что eetiiJi), не превышает 1 —у. Поэтому на практике величину у выбирают обычно близкой к 1 {у = 0,95; 0,99 и т. д.), и при выбранном у стремятся построить кратчайший (в том или ином классе) интервал. Иногда рассматривают односторонние доверительные интервалы: верхний (вида 0 < Т2 (X)) или нижний (вида Т\ (X) < θ), определяемые условиями, аналогич- 44
ными (>Ц), в которых опускают соответствующую вторую границу. Аналогично определяют доверительный интервал для отдельной компоненты (например, θι) в случае векторного параметра: Ро(7-, (X) < Θ, < Т2 (X)) >vV6e6, а также доверительный интервал для параметрической функции τ(θ): Ρθ(7Ί (Χ) < τ (θ) < Τ2 (Χ)) > γ V θ е Θ. Если оценивается векторный параметр θ = (θ|, ..., θ,), то у-доверительная область для него есть такое случайное подмножество Gy(X) cz&, которое удовлетворяет условию Ρθ(θ е Gy(X)) >νΥθεθ. Такое подмножество строят обычно с помощью некоторой статистики Τ (Χ), распределение которой известно. Если требуется оценить скалярный параметр θ и существует такая случайная величина G (Χ; Θ), зависящая от наблюдений X = (Хи .... Хп) и оцениваемого параметра, что: 1) распределение G{X; θ) не зависит от θ и 2) при каждом χ е X функция G (χ; Θ) непрерывна и строго монотонна по θ (в этом случае С (Χ; Θ) называют центральной статистикой), то γ-доверительный интервал для θ строят следующим образом. Определим числа g, < g2 из условия Pe(gi < G (Χ; θ) < g3) = γ и решим уравнения G (Χ; Θ) = g\,g2 относительно Θ. Обозначая через Τι = Ti(X), i = 1,2, 7Ί < Ϊ2, эти решения, получаем искомый интервал вида (Т\,Тъ). Методику, основанную на использовании центральных статистик, можно применять и для оценивания отдельных компонент параметрического вектора θ = (θι θ,), а также для оценивания скалярных параметрических функций τ = τ (θ). Если уже имеется некоторая точечная оценка Τ = = Τ (X) параметра θ и ее функция распределения F (t; Q) непрерывна и монотонна по Θ, то, определив из уравнений (относительно Θ) F(T-Q) = (\ -γ)/2, 1 - F(T - 0;θ) = (1 - γ)/2 два случайных числа Τι = Ti(X), i = 1, 2, 7Ί < 7*2, получим центральный γ-доверительный интервал (7Ί, Τ а) для Θ. Для больших выборок в ряде случаев удается построить приближенные доверительные интервалы, осно- 45
ванные на оценках максимального правдоподобия. Так, если τ (θ), θ = (θι, ...,θ,), — непрерывно дифференцируемая функция и τ„ = τ (θ„) — ее оценка максимального правдоподобия, то в случае регулярной модели асимптотическим -^-доверительным интервалом для τ (θ) является интервал (τ„ ± εΊστφ„)/^/η), где σ?(θ) = b'(Q)I~l(Q)b(Q), *(fl) =(^--^f-)' c-< = ф-'(-4^) ■ В частности, асимптотическим γ-доверительным интервалом для ска- лярного параметра θ является интервал (θ"„ ± cy/^Jni(Qn)). Подобные интервалы являются асимптотически наикратчайшими и они основаны на стандартной нормальной аппроксимации для о.м.п.: £,(τη)~ΛΤ(τ(β),σ?(θ„)/η). § 1. Оценки и их общие свойства 2.1. Убедиться в несмещенности и состоятельности следующих статистик: а) Тп(Х) = Fn(x) как оценки теоретической функции распределения F (х) в заданной точке х; б) Т„(Х) = Ank как оценки теоретического момента α* = Εξ*; в) Тп(Х) = — 2 № — αι)2 как оценки дисперсии μ2 = = Щ в случае, когда среднее αϊ = Εξ известно; г) Тп(Х) = —зт ^2 Ξ ^'2 как оценки М-2 в общем случае. Является ли 52 состоятельной оценкой μ2? Указание. Воспользоваться задачей 1.27 и неравенством Чебышева (предполагается, что соответствующие теоретические моменты существуют). 2.2. В каких случаях статистика Т„(Х) = л]А„2/2 является состоятельной оценкой теоретического среднего а,? 2.3. По выборке X = {Х\, ..., Х„) из распределения L (ξ) построить несмещенную оценку его характеристической функции (х. ф.). | Указание. Рассмотреть эмпирическую х. ф. 2.4. Пусть X = ΡΉ, Хп), .·., (Xnt, Xn2)) — выборка из распределения двумерной случайной величины ξ = = (ξι, ξ2). Доказать, что несмещенной оценкой для μι ι = = ΰον(ξι,ξ2) является статистика Т(Х) = r-5i2, где 5|2 — выборочная ковариация (см. решение задачи 1.38). 46
У к а з а и ii е. Рассмотреть случайную величину ξ, -\- ξ2 и воспользоваться решением задачи 2.1 п. г). 2.5. Пусть X = (Х\, ..., Хп) — выборка из распределения fit(l,0). Описать класс параметрических функций τ (0), для которых существуют несмещенные оценки Τ (X). Убедиться, что в этот класс не входят, в частности, функции τ (θ) = 1 /θ" при а > 0 и τ (θ) = О4 при b > п. 2.6*. По результатам η испытаний оценивается неизвестная вероятность «успеха» θ в схеме Бернулли Bi (1, θ). Обозначая через гп число успехов в этих испытаниях и рассматривая класс оценок вида Τ = '" ° , вы- " + ρ числить ср^дпекзадратическую ошибку оценки Τ и сравнить ее с ошибкой «обычной» оценки гп/п. 2.7. Пусть L (ξ) = Bi (k, 0) и η = 1. Рассмотрим функции вида τ,5(θ) = θ'(1 — 0)s при целых г, s ^ 0. Показать, что несмещенная оценка для τ,5 (0) существует лишь при г + s ^ k и в этом случае она имеет вид T(X) = (X),(k - X)s/(k),+s, где (а), = а (а — 1)... (а — г + 1), г > 1, (а)0 = 1. 2.8. Пусть X = (ΛΊ, ...,Х„) — выборка из распределения Bi(k, θ) и Τ = Х\ + ... + Хп. Описать класс параметрических функций τ (0), для которых существуют несмещенные оценки вида Н(Т). Построить несмещенную оценку такого вида для τ/(θ) = Θ'. Указание. Воспользоваться свойством воспроизводимости распределения Bi(k,0) (см. задачу 1.39 п. 3) и задачу 1.52 п.б)). 2.9. Пусть Щ) = Π(θ) и η = 1. Проверить, что Т(Х) = = (X)j — несмещенная оценка τ(θ) = θ; (/'=1, 2, ...), а для функций τ(0) = 0-α при а > 0 несмещенных оценок не существует. Построить несмещенную оценку для (1+Θ)-'. 2.10. Пусть по одному наблюдению над дискретной случайной величиной ξ с распределением /(χ; Θ) = = е~° — /(\ — е~°), χ = 1,2, ... (урезанное в нуле пуас- соновское распределение), требуется оценить функцию τ(θ) = 1 — е~°. Убедиться в том, что здесь имеется единственная несмещенная оценка, по она практически бесполезна. 2.11. Пусть L(|) = Bi(r, 0) и η = 1. Построить несмещенную оценку функции τ(θ) = 0s (s ^ 1 — целое) и 47
убедиться в том, что при г = 1 эта оценка практически бесполезна. Указание. Воспользоваться формулой (1 — Θ)-' = оо = Σ ci+ ,_,θ'. / = о х , 2.12. Показать, что Т(Х) = 2 -г- — единственная < = ι ' несмещенная оценка функции τ(θ) = 1п(1 — θ) в модели 5/(1, Θ) при η = 1. — 2 2.13. Убедиться в том, что Т* = X2 — — — несме- п щенная оценка функции τ(θ) = θ2 в модели N(Q, σ2). 2.14. В модели Λ(μ, θ2) требуется оценить τ(θ) = θ2 по выборке объема п. Показать, что выборочная дисперсия S2 имеет меньшую среднеквадратическую ошибку, чем несмещенная оценка τ* = — Σ №— μ)2· Какая из двух несмещенных оценок τ* и 5 (см. задачу 2.1 п. г)) точнее? 2.15. Доказать, что Т„(Х) = д/^-· — Σ Ι*< —μΙ — не- смещенная и состоятельная оценка параметра θ в модели Λ^μ, θ2). 2.16*. Путгь X = (X , Х„) — выборка из распреде- η ления Ν(μ, θ2) и Τ2 = Σ № — μ)2· Доказать, что несме- < = ι щенной оценкой для функции τ*(θ) = θ* при любом целом г(т) k^ 1 является статистика τ* = —s—)—ttv^*· Срав- 2Ti^i+A) нить оценку τΐ с оценкой, указанной в предыдущей задаче. Указание. Использовать тот факт, что Lo(7"2/02) = = %\п). 2.17. Дана выборка X = (Х\, А"2, Х3) из распределения Щ, Θ2) и пусть Τ = Т(Х) = -л/Χ'ί + X'i + XI Рассмотрим статистику рт(х) = -^=rl{\x\ ^ 7"), где !{■) — индикатор, которая как функция переменного χ представляет собой плотность равномерного распределения на отрезке 48
[— Τ, Τ]. Убедиться в том, что рт(х) при любом χ является несмещенной оценкой для плотности исходного распределения N{0, Θ2). Указание. Воспользоваться тем, что L0(T2/62) = = χ2(3). 2.18. Рассмотрим задачу оценивания неизвестной дисперсии θ! в общей нормальной модели N(Qi, θ|). Пусть X = (X Хп)— соответствующая выборка и 5' — несмещенная оценка 02 (см. задачу 2.1 п. г)). Рассмотрим класс оценок вида 7\ = λ5'2. Убедиться в том, что при ——— < λ < 1 статистика 7\ имеет меньшую средне- квадратическую ошибку, чем 5'. При каких целых k этому подклассу принадлежат статистики —ττΣ № — — X)2? Найти оптимальную (по критерию минимума среднеквадратической ошибки) оценку в классе (7\). 2.19* (продолжение задачи 2.18). Построить оптимальные оценки вида 7\ = XS'2, минимизирующие меры Ео(7\ — 02)4 и Ео|7\ — 011 соответственно. I Указание. Воспользоваться теоремой Фишера и I задачей 1.45. 2.20. Доказать, что в модели задачи 2.18 несмещенной оценкой для функции τ*(θ) = 02 при любом целом k ~^t 1 является статистика »-(т) T(H|zq-s' где 52 — выборочная дисперсия. Рассмотреть случай η = 2 и вычислить смещение статистики \Х]—Хч\ как оценки 02. 1 Указание. Учесть, что La(iiS2/Ql) = χ\η — 1). 2.21. Пусть X = (X , Хп) — выборка из распределения Γ(θ, λ) и Τ = Х\ -\-... -\- Хп. Убедиться в том, что статистика τ'α = ,Л„ п' , Т~а — несмещенная оценка 1 (λη — α) функции τα(θ) = θ~α при любом а < λ/ζ. Указание. Воспользоваться свойством воспроизводимости гамма-распределения (см. задачу 1.39 п. 2)). 2.22*. Продолжительность горения электрических ламп имеет распределение Γ(θ, 1). Чтобы оценить 0, берут выборку из η ламп и наблюдают «времена жизни» пер- 49
вых г перегоревших ламп Х(]) < Х(2) < ... < Х{Г). Построить оптимальную несмещенную оценку вида Т(Х) = г * = 1 Указание. Перейти к величинам Υ, =" ~~q + (Χι г) — — Х(, - η), г = 1 η (Л"(о) = 0), и воспользоваться задачей 1.34. 2.23. По выборке X = (X , Хп) из распределения R(Q, 2Θ) требуется оценить параметр Θ. Рассмотреть класс оценок вида Т= Т(Х) = аХ(„) + βΧ(]), α, β ^ 0, и найти в этом классе оптимальную несмещенную оценку. | У к а з а н и е. Воспользоваться задачей 1.36. 2.24. Оценивается параметр 0 равномерного распределения R(0, θ) по выборке X = (X Хп). Убедиться в том, что обе статистики 7", = -^-i—Х{„) и Т2 = (η + 1)ΛΌ) несмещенные. Какая из них предпочтительнее? Указание. Воспользоваться задачей 1.36; установить, что оценка 7"2 не является состоятельной. 2.25. Пусть X = (X , Хп) — выборка из /?(θι, 02). Доказать, что статистики 7", = (X(i) + A"(„))/2 и 7"2 = = " (X (rl) — Λ"(ΐ)) — несмещенные и состоятельные оценки функций π(0) = (θι + 02)/2 и τ2(θ) = 02 — 0, соответственно. (Указание, Воспользоваться задачей 1.36. 2.26. Убедиться в том, что если X = (Х\, ..., Х„) — выборка из распределения Вейбулла с неизвестным параметром сдвига θ — W( θ, α, b), то статистика Т(Х) = = Х(\)— ЬГ( 1 -\ Jn_l/a — несмещенная и состоятельная оценка параметра Θ. | Указание. Воспользоваться решением зада- | чи 1.37. 2.27. Показать, что для логистического распределения с плотностью /(*; Θ) = е~х + и(\ + е~х + и)~2, — оо <х< сю, 6е(- оо, оо), несмещенной и состоятельной оценкой параметра θ является выборочное средне'е X. 2.28. Показать, что выборочное среднее X для модели Коши Κ(θ) не является состоятельной оценкой параметра 0. Указание. Воспользоваться свойством среднего арифметического для распределения Кошн. 50
2.29. Оценивание для полиномиального распределения. Пусть случайная величина ξ принимает конечное число значений αϊ, ..., an с неизвестными вероятностями Р\,—, Pn {р\ + ··· + Ры = 1)· Чтобы оценить параметры О = (рь ..., ры-\) (параметр ры = 1 — pt — ... — рл/— ι), произведено η независимых наблюдений над |. Пусть νΛ — число членов выборки, равных аг, г = 1, ..., N. а) Показать, что статистики Тг=—!—, г = 1 N — η несмещенные и состоятельные оценки параметров р\, ..., ры соответственно. б) Описать класс параметрических функций τ(θ), для которых существуют несмещенные оценки вида ЩТи...,Ты). в) Построить несмещенную и состоятельную оценку N функции τ(θ) = 2 dpi ■ ί = I Указание. Учесть, что L(vi, ... , vn) = Μ (η; ρ , ρΝ), и воспользоваться решением задачи 1.52. 2.30. Оценивание по методу моментов. Пусть X = = (X , Х„)—выборка из распределения L(|) e {F(x; θ), θ = (θ , θ,)εθ) и моменты α*(θ) = Eelk, k = \ г, существуют. Тогда, решая относительно θι, ..., θ, уравнения α*(θ) = АПк, k = 1, ..., г, где А„к = А,,к(Х) — выборочный момент k-ro порядка, получаем значения оценок параметров, найденных методом моментов. Найти по методу моментов оценки параметров гамма- распределения Γ(θ|,θ2) и убедиться в их состоятельности. 2.31. Найти методом моментов оценки параметров «двойного» распределения Пуассона, задаваемого вероятностями Ρβ(ξ = ,) = 4{^4-^4-). х = 0,1,2,..., θ = (θι,02), 0<θ,<θ2. Такое распределение описывает, например, число столкновений с молекулами газа в камере Вильсона частиц, получающихся при распадении ядра урана в результате бомбардировки его нейтронами. Вычислить значения полученных оценок для следующих данных, полученных при η = 327 наблюдениях над случайной величиной ξ (через пх обозначено число наблюдений, в которых ξ = х): 51
χ η, 0 28 1 47 2 81 3 67 4 53 5 24 6 н'З 7 8 9 3 9 2 10 1 2.32. Смоделировать выборки, объемы которых η = = 10,100,1000 из равномерного распределения /?(0.0) при 0 = 1, и оценить методом моментов параметр Θ. 2.33.* Выборочный контроль. В некоторых системах статистического контроля качества продукции поступают следующим образом. Из партии, содержащей TV изделий, случайным образом без возвращения отбирают на контроль η изделий, каждое из которых проверяют на доброкачественность. Если число k обнаруженных в выборке дефектных изделий удовлетворяет неравенству k^.k0, где ko — задаваемый заранее некоторый уровень (#0<"), то их заменяют на исправные, после чего вся партия принимается. Если же k~>ko, то контролю подвергают все /V изделий и все дефектные изделия заменяют исправными. Обозначим через D неизвестное число дефектных изделий в партии (D = 0, 1, ..., Ν) и пусть случайная величина ξ — число обнаруженных при описанном способе действия дефектных изделий. Тогда Ρ0(ξ = k) = f(k; D, η) = cHCi,-"D/CS, k = 0, 1, .... k0, η PD(l=D)= Σ }{k;D,n){nPHD>k0). к = *o + I Предположим, что требуется оценить заданную функцию x(D) от числа дефектных изделий в партии. Доказать, что всегда существует и притом единственная статистика 7"(ξ), являющаяся несмещенной оценкой t(D), т.е. условиями Ed Щ) = 2 7Ш*; D, п) + T(D) £ f(k; D, η) = x(D), к = 0 * = *а + I D = 0, 1,..., Ν , функция T(k) однозначно определена. Рассмотреть случай x(D) = D. Указание. Использовать тот факт, что при D^k0 гипергеометрические вероятности j(k; D; п.) = 0 для k>k0. 2.34*. Оценивание для конечной совокупности. Пусть имеется конечная совокупность U = [ии ..., иц) из N объектов, каждый из которых характеризуется некоторой ве- 52
личиной х(и), « е U. Значения х-, = х(щ), i = 1, ..., Ν, неизвестны, и требуется оценить их сумму Τ = Т(х) = = 2 χι- Предположим, что можно наблюдать каждое < = 1 подмножество (выборку) s = (ы,·,, ..., щф)) элементов из II с некоторой вероятностью p(s). Таким образом, если 5 = [s] — совокупность всех выборок, то 2 p(s) = 1- В этом случае говорят, что задан выборочный план А = = (U, S, Р). В качестве оценок для Τ рассматриваются статистики e(s, χ), которые зависят от χ только через те Χι, для которых щ е s (т.е. оценка есть функция выбранных объектов и их наблюдавшихся х-зиачений). Оценкой Горвица — Томпсона называется статистика где п(и) = 2 p(s)—вероятность включения в выборку s : s з и объекта и. а) Доказать, что e(s, х) — несмещенная оценка Т(х\ т. е. 2/>(s$s.*)= T(x) A/*etf", S и что других несмещенных оценок вида ^а(и)х(и) не су- и е s ществует. б) Вывести формулу для дисперсии оценки Горвица — Томпсона: оф, х) = Σ^Μ<^ - О + ΣΛ«^- ι). где п(и, υ) = 2 P(s) — вероятность включения в вы- борку объектов и и υ. в) Проверить, что несмещенной оценкой для De(s, x) является статистика Ms х)= Υ J^Hf-l \\ + V *ИФ) / Ф. ») Л и Φ υ 53
г) Показать, что среднее и дисперсия объема n(s) выборки s для выборочного плана A = (U, S, Ρ) выражаются через вероятности включения я(и) и п(и, ν) следующим образом: t En(s) = 2"("). Dn(s) = Σ (π(ιι,υ) — π(ιι)π(υ)) + и и φ υ + Σ"("Χ1 -п(и)). и Указание. Ввести индикаторные случайные величи- ι \ П. если «ε s ι \ ί \ ны ν(") = л ^ и записать n(s) и e(s, х) в ,N ' (0, если u^s v ' \ ■ / виде n(s) = Σϊ("). Φ. *) = Σν("Μ")/π(") · U U 2.35* (продолжение задачи 2.34). Рассмотрим выборочный план А* = (U, S, Р), порождающий равновероятные выборки без повторения объема п. В этом случае множество 5 состоит из (Ν)α всех упорядоченных комбинаций длины η из различных элементов U и p(s)= l/(W)n-Vse=S. а) Показать, что оценка Горвица — Томпсона имеет в данном случае вид e(s,x) =—Σ *(")■ liES ι N б) Обозначим через μ= Τ(χ)/Ν, σ2 =- Σ (*< — — μ)2 соответственно среднее и дисперсию совокупности U. Тогда -дг-e(s, χ) =ξ λ: (выборочное среднее наблюденных х-значений) является несмещенной оценкой для μ. Проверить, что Όχ = ( ы)а2' в) Доказать, что статистика лвляется несмещенной оценкой σ2. Замечание. Справедлив более сильный результат: S2(s, χ) является оптимальной оценкой σ2 в классе всех несмещенных квадратичных оценок, т. е. оценок вида Σ а(и, v)(x{u) — λ-χχ(υ) — χ). U, UG S 54
2.36.* Оценивание размера конечной совокупности. Пусть имеется конечная совокупность U, число элементов которой N неизвестно. Из этой совокупности η раз извлекается простая бесповторная выборка объема m (каждый раз любая из Сы возможных комбинаций элементов 11 может быть извлечена с равной вероятностью). Обозначим через μ, = μ,{η, m, N) число наблюдавшихся элементов, каждый из которых повторился ровно г раз (г = 1, 2, ..., п). Рассматривается задача оценивания параметрических функций χ(Ν) по выборочным данным (μι, ..., μ„). Доказать, что в классе линейных статистик L = {/ = η = Υ, lr\ir) несмещенная оценка для τ(Ν) существует лишь г = I в случае, когда τ(Ν)— полином от — степени k^Ln — 1. /ζ В этом случае, если τ(Ν) = 2 C]/N', то единственной / = ι несмещенной оценкой для τ(Ν) является статистика , = |Ц = | m'+(n)/+lJ η В частности, —- У г(г — 1)μΛ — единственная m*n(n- i)rf ι линейная несмещенная оценка для r(N) = \/N. Указание. Представить μ, в виде суммы индикаторов: μΛ = ξ(ί' + ... + ξ$, где $r) = ', если ί'-й элемент U повторился г раз, и ξ(;° = 0 в противном случае, »= \,...,Ν. 2.37.* (продолжение задачи 2.36). Пусть η = μ, + + ... + μ„ — общее число наблюдавшихся элементов и Η — класс статистик вида Н{х\). Доказать, что: а) если N^.mn, то для любой функции τ(Ν) несмещенной оценкой в классе Η является статистика τ*= 2 (-1)п_,С||(С7)ят(/)/2(-1)',~/^(СГГ; I =т ; = О б) если же априори N может быть любым целым числом (NZ^m), то указанная статистика является несме- 55
щенной оценкой функции τ(Ν) при дополнительном условии, что τ{Ν) = f(N)(C")~n, где f(N)— многочлен степени не выше тп, удовлетворяющий условиям /(0) = f(\) = = ... =f(m-\) = 0. 2.38. Метод Монте-Карло. При отыскании значений различных величин (определяемых, например, некоторыми уравнениями или интегралами) часто используют вычислительный метод, основанный на вероятностной интерпретации искомых величин и использовании реализаций случайных испытаний, — так называемый метод Монте-Карло, или метод статистических испытаний. Сущность этого метода состоит в следующем: исходя из смысла вычисляемой величины а, подбирают такую случайную величину ξ, чтобы а = Εξ; далее моделируют выборку Х = (Х\, ..., Хп) из распределения Д|) и_в качестве оценки а используют выборочное среднее X. Тогда (см. задачу 1.61) при η-+<χ> P(V^I*-al/S'<cv)-*2(D(cY)-l = у; таким образом, ошибка в определении а этим методом с вероятностью γ не превышает CyS'/л/п, когда η велико. Пусть, например, требуется вычислить интеграл a =\...\f(tl,...,tr)dtl...dtr, ν, где ν, = {(/,, ..., tr):0^.ti^. 1, i = I,..., r). Здесь, очевидно, можно положить ξ = f(r\\, ..., η,), где ηι, ..., ηΓ—независимые равномерно распределенные на [0, 1] случайные величины, и смоделировать выборку X возможно, следовательно, с помощью последовательности (1.5). ι Оценить указанным методом интеграл а =\ exdx , о используя 100 чисел последовательности (1.5), и сравнить полученное значение а* с точным значением а. При каком δ будет выполняться соотношение Р(| α — -α*|<δ)^0,99? 2.39. Вычислить методом Монте-Карло значение интеграла p(r;Oi,o2)=-^- \\ expl ±-(х\ + xUdx\dx2 при г = 3, ΰ\ = 1, 02 = 2, моделируя соответствующую выборку объема η = 100. 56
Указание. Если |ь |2 — независимые случайные величины nL(lj) =N(0,of), / = 1, 2, то р(г, σ,,σ2) = Ρ(ξ? + ξ^<Γ2). Далее воспользоваться задачей 1.61. 2.40.* Случайное блуждание. Частица, «стартуя» в момент / = 0 из точки k (0<Ck<.N), блуждает по целым точкам отрезка [0, N]. Если в момент / частица находилась в точке /, то в момент / -\- 1 она находится в точке / + 1 с вероятностью ρ или в точке / — 1с вероятностью q — 1 — р(1 ^ /^ N — 1). В точках 0 и N частица поглощается и случайное блуждание прекращается ([71, гл. XIV). 1) Найти вероятность πω поглощения частицы в точке N. 2) Вычислить trik = Ετ«, где τ* — время до поглощения частицы. 3) Смоделировать 100 реализаций описанного случайного блуждания при N = 7, k = 3, ρ = 0,6 и ρ = 0,5 и найти оценки величин пш и т*. Указание. 1) Составить для f(k) = π*.ν уравнение в конечных разностях: f(k)=pf(k + \) + qf(k-\), k= 1,..., Ν-Ι, /(0) = 0, f(N)=\, и убедиться в том, что единственным его решением является щы = (1 — λ*)/(1 — λΝ), λ = q/p, если ρ φ Φ q, и щы = k/N, если ρ = q = —. 2) Составить для m* уравнение в конечных разностях: trik = ptrik + i + qrrik-\ + 1, k = 1, ..., Ν — 1, m0 — = щц = 0, и убедиться в том, что единственным его k N решением является т* = щ,ы, если я-г я—Ρ ρ φ q, и mk = k(N — k), если р = q =—. 3) Поступать как и при решении задачи 1.4. § 2. Оптимальные оценки 2.41.* Доказать, что оптимальная несмещенная оценка всегда является симметричной функцией наблюдений. | Указание. Если Т= Т(Х) — несмещенная оценка 57
τ(0), то рассмотреть симметрическую статистику Τ* = —'ΣΤ(πΧ), где π =(. '". j — перестановка из η элементов, пХ = (Х1и ..., Х1п) и суммирование производится по всем п\ перестановкам. Показать, что А, 7* < Do 7". 2.42. Доказать следующие свойства оптимальных оценок: если Т* = Т*(Х) — оптимальная несмещенная оценка некоторой функции τ = τ(0), то: 1) для любой статистики ψ = ψ(Λ) с Ε0ψ = 0 νδ^θ выполняется равенство covo (Τ*, ψ) = ΘΥ0; 2) для любой другой несмещенной оценки Т = (X) cov0(7"*, T)= Do7"*. Указание. В первом случае рассмотреть несмещенные оценки вида 7\ = Τ* + λψ, λε/?1. Во втором случае положить ψ = Τ* — Т. 2.43. Проверить, что количество информации i(Q) для соответствующих моделей имеет указанный в таблице вид. Модель ί(0) Ν( Ο,σ2) Ι/σ2 Ν(μ,θ2) 2/02 Γ(0.λ) λ/02 Κ(0) 1/2 Bi(k, θ) */[0(Ι- -0)] Π(0) ι/ο Bl(r.O) /[0(ΐ- -0)2] 2.44. Показать, что информационная матрица для общей нормальной моделиЛДбь θ|) имеет вид Ι(θ) = 1/Θ? ι2 О О 2/θ22 2.45.* Показать, что информационная матрица для N — I где модели задачи 2.29 имеет вид /(Θ) = \\gij(Q||ί θ = (ρ,, ..., ρν-\), = Ι/ρ, + 1/ρ.ν при i = l ,_ при г φ ι, μΝ μχ Вычислить /_'(θ). Указание. Записать вероятности f(a,; θ) = Ρο(ξ = = a,) = pr, r = 1, ..., yV, в виде ^<θ> - \',%~ ■ρΝ-\· Ν π f(a,; θ) = Π ρ^' = (1 -ρ, -...-Ρν-ιΓ'-^'Χ Ν- Ι Χ Πρ*""'1, ; = ι где δ(α;, α;) = 1 при ί = / и δ(α;-, α,) = 0 при i Φ }. 58
2.46.* Модель ^= [F(x; θ},θεθ) называется экспоненциальной, если функция f(x; 0) имеет вид {(х; 0) = exp[A(Q)B(x) + С(0) + D(x)). Доказать, что эффективная оценка τ* для некоторой параметрической функции τ(θ) существует тогда и только тогда, когда модель F— экспоненциальная; при этом τ(θ) = τ(θ) -^^- , τ* = — У B(Xi), если θ — скаляр , до. -I дС(0) I дА(0) , τ* =-f Σ ВЩ, если θ = (θι, ..., θ,). Вывести следующие формулы для дисперсии Όατ* τ'(0) D„T* = Dot* ;= ι nA'(0) <?τ(0) <5Θ( , если θ ал(0) скаляр, / θθί , если θ = (θι, ..., θ,). Указание. Воспользоваться критерием эффективности. 2.47. Доказать, что для экспоненциальной модели со скалярным параметром функция ί'(θ) = (C'(Q)A"(Q)— - С'ЩАЩ/АЩ и ΕοΒ(ξ) = - С'(9)/Л'(0). Указание. Сравнить выражение для Dut* в предыдущей задаче с границей Рао — Крамера. 2.48. Проверить, что для заданных регулярных моделей функция τ(0), допускающая эффективную оценку τ*, эта оценка и ее дисперсия Dot* имеют указанный в таблице вид: Модель Ν(0,σ·) Λ/(μ,θ2) Γ(0,λ) Γ(α,0) fl(0,1) τ(ο) 0 0* 0 Г'(0)/Г(0) ι/ο τ* ϋ,(λ'-μ)2 Χ/λ I 71 — У In Χ— Ιπα "/= 1 - — Σ ΐπΛ " .-I Α,τ* σ2/;ι 207" θ2/λη τ'(0)/« Ι π!)2 59
Продолжение Модель Bi(k,0) П(0) Щг,0) т(о) 0 О г0/(1-в) τ* X/k Я X ΰ0τ' 0(1 — 0) /А/г θ/η γ0/[ιι(1-0)Ί Ι Указание. Воспользоваться задачей 2.46. 2.49. Проверить непосредственно, что выборочное среднее X в логистической модели (см. задачу 2.27) не является эффективной оценкой Θ. Указание. Воспользоваться результатом задачи 2.27. 2.50. Доказать, что оценка 7"* в задаче 2.13 является оптимальной. Указание. Рассмотреть линейные комбинации вида -j- α(θ) -^г-f b(Q) —- и воспользоваться крите- рием Бхаттачария. 2.51. Рассматривается задача оценивания функции τ(θ) = θ2 в модели Γ(θ,λ) по выборке X = {X Хп). Доказать, что 7"* = Т*{Х) = λ(λ^+1) *' ~ оптимальная несмещенная оценка τ(θ); вычислив D07"*, ^убедиться в том, что эта оценка не является эффективной. I Указание. Воспользоваться задачами 2.21, 2.43 и указанием к задаче 2.50. 2.52. Пусть X = (Х\, ..., Хп) — выборка из распределения JV(0i, Θ|). Применив критерий Бхаттачария, доказать, что X и S (см. задачу 2.1 п. г)) являются оптимальными несмещенными оценками соответственно для θι и θ2. Сравнить дисперсии этих оценок с соответствующими границами Рао — Крамера. Указание. В первом случае достаточно рассмотреть а!п^-| во втором — рассмотреть линейные ком- бинации вида -j-j α(θ) —^ ВД та* использовать задачу 2.44. 60
2.53.* Пусть Х\, 5Ί и A"2, 52 —оптимальные несмещенные оценки для среднего и дисперсии одного и того же нормального распределения, вычисленные по двум независимым выборкам объемов п\ и пг соответственно. Какие функции от этих статистик являются наилучшими оценками тех же параметров, учитывающими всю исходную информацию? Сравнить точность новых оценок с исходными. I Указание. Использовать задачи 2.52 и 2.14. 2.54*. Пусть X = (X], ..., Хп) — выборка из обратного гауссовского распределения, задаваемого плотностью Я*; К μ) = (^г-)"гехр{ -^Я . * >0. λ>0, μ=*0. 1) Убедиться в том, что X — оптимальная несмещенная оценка параметра μ в любом случае, известен или нет параметр λ. Получить отсюда, что ΕΛΊ=μ, ΌΧι = μ3/λ. 2) Найти эффективную оценку λ-1 в случае известного μ. I Указание. Воспользоваться задачей 2.46 и крите- I рием Бхаттачария. 2.55. Предположим, что ищется оценка для дифференцируемой вектор-функции τ(θ) = (τι(θ), ..., тт(9)), θ = = (θι, ..., θ,). Показать, что в случае регулярной модели для произвольной несмещенной оценки Τ = (7\(Д(), ..., Тт(Х))) справедливо неравенство информации Da(T)= \\сомв(Т,, Γ,)||Γ>β'(θ)/7'(θ)β(θ), где Ζ?(θ) = || —^—1| , и неравенство AC^-Ai между матрицами одинаковой размерности означает, что матрица А\ — А2 является неотрицательно определенной. В частности, для τ(θ) = Q Du(7)>/7'(0) . Указание. Рассмотреть произвольную линейную комбинацию С|Т|(9)+ ...+ с,пТт(в) = с'т(9), несмещенной оценкой которой является с'Т, и применить неравенство Рао — Крамера для скалярных оценок. 2.56. Показать, что если для некоторой функции τ(θ) существует эффективная оценка, то она является достаточной статистикой. Таким образом, для регулярных экспоненциальных моделей (см. задачу 2.46) достаточная статистика всегда существует и имеет вид Т(Х) = л =: 2 = B(Xi) (что следует из критерия факторизации). 61
2.57. Доказать полноту достаточной статистики η Τη = Σ -^ί Для биномиальной модели Bi(k, θ) (см. зада- i = I чу 2.48). Получить отсюда, что в данном случае несмещенные оценки существуют лишь для полиномов τ(θ) = τ = Σ α;θ' степени r^.kn, и при этом оптимальная оценка i = о τ* = Σ а,{Тп)1/(кп)!. i -о Сравните этот результат с задачами 2.5, 2.7 и 2.8. I Указание. Воспользоваться свойством воспроизво- I димости распределения Bi(k, θ) (см. задачу 1.39 п. 3). 2.58. Доказать полноту достаточной статистики η Тп = Σ -^ί Для пуассоновской модели Π(θ) (см. задачу 1 = I 2.48). Показать, что оптимальной оценкой для сходящегося при всех θ>0 степенного ряда τ(θ) = £ α,θ'явля- i>° ется статистика τ* = Σαι{Τη),/η'. Ί Указание. Воспользоваться свойством воспроизводимости распределения Π(θ) (см. задачу 1.39 п. 4)и задачу 2.9). 2.59. (продолжение задачи 2.58). Построить оптимальные оценки для функций τ(θ) = е0^-'1 , π*(θ) = e~°Qk/kl , * = 0, 1 и τ,(θ) = Pb(£>r), r= 1,2, ... 2.60*. Пусть X = (Х1} ..., Х„) — выборка из распределения степенного ряда, задаваемого вероятностями оо f(x; θ) = α(χ)θ7Ρ), * = /, /+ 1,..., №) = Σ α(#',θεθ, где Θ = (0, R) и /?>0 — радиус сходимости ряда f(Q). 1) Показать, что в данной модели эффективная оценка существует лишь для функции τ(θ) = θ/'(θ)//(θ), и она имеет вид τ* = X. η 2) Доказать, что Тп= Σ -К/—полная достаточная i= I статистика и ее распределение имеет вид: 62
pu(7„ = /) = 0'Ья(/)//я(6), l^nl, где bn{t) = coef ,7"(2). 3) Убедиться в том, что статистика т* _ \b,,(Tn — s)/b„(Tn) при Тп^п1 +s, Ts ~ \ О при Tn<nl + s — оптимальная оценка функции τ4(0) = 0s для любого s = 1, 2, .... 4) Построить оптимальную оценку функции τ(θ) = оо = 2 а/9' > гДе степенной ряд предполагается сходящимся i =' на Θ; получить отсюда, в частности, что оптимальная оценка функции /(0) имеет вид /* = Ьп+\(Т„)/Ьп{Т„), если ТпХп + \)1, и /* = 0 при Та<(п + 1)1. Указание. 1) Применить критерий эффективности для экспоненциальной модели (см. задачу 2.46). 2) Использовать производящую функцию оо φ; θ) == 2 2х!(х; θ) = /(20)//(θ). χ = I k-nl 3) Учесть соотношение £ a(j)bn(k — j) = bn + \{k) при k^(n+\)l. 2.61. Показать, что оптимальной оценкой для τ(0) = G урезанного в нуле пуассоновского распределения (см. задачу 2.10) по выборке Λ!=(ΛΊ, ..., Х„) является статистика τ* = 7"ЛЯ0Г~'/ДЯ0Г "Ρ" Τ = Х{ + ... + Х„^п + 1 и τ* = О при Τ = η, где Δ"0* = Σ (- \f~rCrark. г -О I Указание. Применить задачу 2.60. 2.62. По выборке X = (X ,Хп) из распределения Bi(r,Q) построить оптимальные оценки для тι(θ) = 0s при целом s> 1 и τ2(θ) = Ρ0(ξ = 0) = (1 — Θ)Γ. Указание. Воспользоваться решением задачи 2.60; см. также указание к задаче 2.11. 2.63*. Рассмотрим модель с конечным числом N возможных исходов и неизвестными вероятностями исходов р\, ..., ры (см. задачу 2.29). Показать, что Τ = (\\, ..., ...,νΛ'_ι) — минимальная полная достаточная статистика. Получить отсюда, что несмещенные оценки в этой модели существуют лишь для полиномов от pi, ...,ры степени, меньшей или равной п, и найти явный вид этих оценок. 63
Указание. Воспользоваться критерием для /--параметрического экспоненциального семейства и задачами 2.29 и 2.45, а также 1.52 п. б.). 2.64. Доказать оптимальность оценок, указанных в задачах 2.13 и 2.16. Указание. Воспользоваться свойством полных достаточных статистик. 2.65. Пусть Х= (Χι, ..., Хп) — выборка из распределения Ζ.(ξ) = N(Q, σ2). Построить оптимальную оценку для τ(θ) = Ρο(ξ^^ο), где хо — заданное число. Указание. Рассмотреть несмещенную оценку Г, = 1(Х]^хй), где 1(A) — индикатор события А, и воспользоваться теоремой Рао — Блекуэлла — Колмогорова (см. при этом решение задачи 2.64 и 1.56). 2.66. Проверить, воспользовавшись критерием для т- параметрического экспоненциального семейства, что в случае модели #(01, θ2) минимальной полной достаточной η статистикой является пара (X, 2 %Ь> а также (X, S2). Ус- i= 1 тановить оптимальность оценок, указанных в задаче 2.20 (ср. с задачей 2.52). Указание. Перейти к новым параметрам θ{ = _ "θΓ· °2 _ ~W 2.67. Показать, что в модели Μθ, γ2θ2) достаточной статистикой является пара Г= (X, S ), но эта статистика не полная. Указание. Рассмотреть функцию φ(Γ) = (η -\- γ2) χ XS2[(" — \)y2]-X2 и вычислить ее среднее. 2.68. По результатам п~^2 независимых измерений диаметра Οι круга построить оптимальную несмещенную оценку его площади. Указание. Погрешности измерений считать нормальными N(0, Θ2) случайными величинами; использовать задачу 2.66. 2.69*. Доказать следующее утверждение (теорема Басу): если для модели F = {F(x; θ),θ^θ) существует полная достаточная статистика Τ и если статистика 7Ί имеет распределение, не зависящее от параметра Θ, то 7", и Τ независимы. Указание. Установить, что для любого события А условная Рй(Т\^А\Т) и безусловная Р0(7,еЛ) вероятности совпадают. 64
2.70*. Пусть (Хи Х2, Хз) — выборка из распределения Щ) = М{0, Θ2). Построить оптимальную оценку для τ(θ) = = Ρο(ξίζχο). I Указание. См. указание к задаче 2.65, задачу 2.69 | и решение задачи 1.58. 2.71*. Пусть X = (Хи ..., Хп) — выборка из_распределе- ния Ν{β\, θ!). Доказать, что статистики T = {X,S2) и (/ = = ( — , ί = 1, ..., п\ независимы. Указание. Установить, что распределение U не I зависит от θ = (θι, 0г), и применить теорему Басу (см. | задачу 2.69). 2.72*. По выборке Х=(Хи ..., Хп) из распределения Ν(θι, 0г) построить оптимальную несмещенную оценку для функции τ(θ)=Ρο(ξ<*ο) = φ(-^-=^-). Указание. Рассмотреть несмещенную оценку 7Ί = = /(ΛΊ^Χο), где 1(A)— индикатор события А, и вычислить Н(Т) = Ео(7| \Т), где Т=(Х, S2); использовать задачи 2.71 и 1.58. 2.73. Убедиться в оптимальности оценок, указанных в задаче 2.21; показать, что при а — ХпТ^О—целом несмещенных оценок для ти(Э) = θ_α не существует. I Указание. Воспользоваться полнотой достаточной | статистики Т. 2.74*. Пусть Х= (Х\, ..., Хп) — выборка из распределения Γ(θ, λ), Τ = Χλ + ... + Χη и φ(χ) — заданная функция, для которой τ(θ) = Εοφ(ξ) существует. Доказать, что оптимальна» оценка τ(θ) имеет вид τ*=Γ(λ)Γ^:)-1))ύ(^λ"ι(ι-^"ι)λ"1^· Получить отсюда результаты задач 2.48, 2.51 и 2.73. | Указание. Установить равенство Еот* = τ(θ). 2.75* (продолжение задачи 2.74). Проверить, что оптимальной оценкой функции надежности τ(θ; t) = Ρθ(ξ^ί) является статистика τ* = [1 -B(t/T; λ, λ(η - l))]e (Τ - ί), гдеВ(х; a, b) — функция бета-распределения β(α,6) и е(х) — функция Хевисайда. В частности, для распределения Γ(θ,1) функция τ(θ; t) = е~'/0, а τ* = (1 —t/T)n~ le{T — t). I Указание. Положить в задаче 2.74 <р(*) = е(х — t). 3-190 65
2.76*. Доказать, что для распределения Вейбулла с неизвестным параметром масштаба θ — №(0, λ, θ) полной η достаточной статистикой является Τ = Т(Х) = 2 Χϊ, а оптимальная оценка τ(θ) = Εοφ(ξ), где ψ(χ) — заданная функция, имеет вид T* = [n-\)^(tT)^X\-tT-2dt. о В частности, Εοξλ = θλ и потому Т/п — оптимальная оценка θ\ 2.77. Показать, что для двухпараметрического экспоненциального распределения W(Qlt 1,θ2) достаточной статистикой является пара Т= (Ха>Х). Построить несмещенные оценки вида αΧμ)+βΧ для неизвестных параметров модели. Указание. Применить критерий факторизации и воспользоваться решением задачи 1.34, приняв во внимание, что La( ~ ' \ =Г(1,1). 2.78. Пусть наблюдаемая случайная величина ξ имеет область изменения [α(θ),6], где α(θ) — заданная монотонная функция Θ. Показать, что минимальное значение выборки Af(i) является достаточной статистикой для θ тогда и только тогда, когда плотность fi(x; θ) имеет вид fa(x; θ) = g{x)/h(Q), a(Q)^.x^b. Этот же результат справедлив и для статистики Х(„) в случае области [α,6(θ)], где 6(0) — заданная монотонная функция Θ. | Указание. Применить критерий факторизации. 2.79. Пусть Х= (Хи ..·, Хп) — выборка из распределения /?(0,θ). Доказать, что Xin\ = max Xt — полная дос- 1 <!i^ η таточная статистика для Θ. Получить отсюда, что Т* = = — Х(П) — оптимальная несмещенная оценка Θ. Рассмотреть класс статистик 7\ = XT* и убедиться в том, что в нем имеются оценки с меньшей среднеквадратической ошибкой, чем у оценки Т*. I Указание. Использовать задачу 2.24. 2.80. Доказать полноту достаточной статистики Τ = = (X(i), X(n)) для модели /?(θι, θ2). Убедиться в оптимальности оценок, указанных в задаче 2.25. Построить оптимальные оценки для θ| и θ2. I Указание. Воспользоваться задачей 1.36. 66
2.81*. Показать, что статистика Τ = (Χμ), Χ^)) — достаточная для модели /?(α(θ), Ь(0)), где α(θ)<6(θ)νθ — заданные непрерывные функции скалярного параметра Θ. Определить, в каких случаях существует одномерная достаточная статистика и установить ее вид. Убедиться, в частности, что для модели R( — θ, θ) достаточной статистикой является тах(|Х(1)|, ΙΛ^Ι), а для моделей R(d, θ+l) и /?(θ, 2Θ) минимальной достаточной статистикой является Т. 2.82*. Пусть произведено одно наблюдение X над дискретной случайной величиной с распределением ^Ηθ'Ο-θ)2 πρΡΗΗ;Γθ7ΐ!'2,..., ^(Ο'1)· Показать, что X — не полная, но ограниченно полная достаточная статистика. Указание. Решить уравнение несмещенности Εοφ(Α') = 0-V9 в классе всех функций и в подклассе ограниченных функций. 2.83*. Оценивание размера конечной совокупности. Пусть в задачах 2.36, 2.37 величина m = 1. Доказать, что случайная величина η является полной достаточной статистикой, и, следовательно, указанные в задаче 2.37 оценки — оптимальные. Замечание. Этот результат справедлив и для произвольного значения т. § 3. Оценки максимального правдоподобия (о.м.п.) 2.84*. Показать, что если в случае регулярной модели для диффренцируемой параметрической функции τ(θ) существует эффективная оценка τ*, то о. м. п. θ„ параметра θ однозначно определяется уравнением τ(θ) = τ*. Применив этот результат, найти θ„ для моделей, приведенных в задаче 2.48. Указание. Воспользоваться критерием эффективно- сти и показать, что —— <0 (предполагается2 I o = 0„ ся, что функция правдоподобия L = L(x; θ) дважды дифференцируема по параметру Θ). 2.85. Вычислить асимптотическую эффективность выборочной медианы Тп = Х([пу2]+1) как оценки среднего θ модели Ν(θ, σ2). Указание. Воспользоваться задачей 1.32 об асимптотической нормальности выборочных квантилей. 3* 67
2.86. Доказать, что для общей нормальной модели N(QU θ22) о. м. п.б = (θ,„, Q2a) = (X, 5). I Указание. Составить и решить уравнения правдо- I подобия. 2.87 (продолжение задачи 2.86). Показать, что τ„ = = Ф(—~с ) — о. м. п. функции τ(θ) = Ф( —-——) (см. задачу 2.72). Найти асимптотическое распределение Хп При П—у оо . Указание. Воспользоваться свойством инвариантности о, м. п. и утверждением об их асимптотической нормальности. 2.88. Доказать асимптотическую несмещенность и состоятельность о. м. п. θ„ параметра θ модели#(ц, Θ2) (ср. с задачей 2.16) и исследовать ее предельный при л-»-ао закон распределения. Вычислить асимптотическую эффективность оценки, приведенной в задаче 2.15. | Указание. Использовать задачи 2.43 и 2.84. 2.89. Пусть Х=(Хи ..., Хп) —выборка из распреде- ления#(0, 20). Найти о. м. п. θ„ и доказать ее состоятельность. 2.90. По выборке ((Χι, V,), ..., (Χ„, Υ„)) из двумерного нормального распределениям (0, 0), 2 г ) с неизвестными σ2>0 ире(— 1,1) построить о. м. п. σ3 и р. Указание. Перейти к новым параметрам q=(q\, qz), положив <7, = <7,(θ) = - 2j2(1 _ р2 , <72 = <72(θ) = — — — (здесь θ = (σ2, ρ)\ и воспользоваться свой- о\1 - ρ2; ством инвариантности о. м. п. 2.91*. Имеется выборка ((ΑΊ, У,), ..., (Х„, Уя)) из двумер- (0, 0),|L ,|П, 0е е(— 1,1). Составить уравнение правдоподобия для отыскания о. м. п. θ„ и вычислить ее асимптотическую дисперсию. 2.92 (продолжение задачи 2.91). Рассмотреть в качестве оценки θ выборочный коэффициент корреляции Тп = = —2 Χ'Υί и вычислить его асимптотическую эффективность. Указание. При вычислении моментов использовать | характеристическую функцию. 68
2.93*. Пусть Χ = (Хи ..., Х„) — выборка из ^-мерного нормального распределения^ (μ, Σ) с неизвестными μ = = (μ,, ..., μ*) и Σ = ΙΙσ,,Ιΐΐ, |Σ| φ О, т. е. Χ, = (Χη, .... Χ№), / = 1, ..., η, — независимые случайные величины с плотностью «'·θ) = шаге **{-т<*-№-1(*-м χ = (хи ..., хк), θ = (μ, Σ). Обозначим Χ = (Χι, ..., Xk), где Χι = -7-Σ *«. Σ = ||S4||f, где S4 = — 2 №/ - ЩХп - _ '= I " /= I A"j)—выборочная ковариацня, соответствующая теоретической ковариацпи σ,,, так что χ=—Σ χι. ς = ς(Α) = -!-2 (я-вд-*)'. η ί = ι " ί= ι 1) Доказать,_ что о. м. п. параметров μ и Σ равны соответственно X и Σ. 2) Убедиться в том, что 2 — несмещенная оценка Σ. "-' 3) Получить следующее выражение для максимума функции правдоподобия: тахЦж; Θ) = Цх; х, £(*)) = (2πί?Γ*"/2 |Σ(*)Γ"/2 . Указание. Привести функцию правдоподобия к виду Цх;в) = [(2π)*|Σ|Γ^ρ{--^χ-μ)'Σ-,(χ-μ)- -^-1γ(Σ-'Σ(*))); использовать задачу 2.4. 2.94*. Пусть X = (ΑΊ,..., Х„) — выборка из логнормаль- ного распределения, т.е. Xi=eYl, raeL(Yi) = jV(0i, θ|>). Построить о. м. п. для функций τ,(θ) = ΕοΑΊ и τ2(θ) = ЦДь Вычислить Εοΐΐη и убедиться в асимптотической несмещенности оценки τι„. У к а з а н и е. Воспользоваться свойством инвариантности о. м. п. 2.95. Распределение Кептайна. Это распределение задается плотностью '<*0) = iSrexpi —wis(x)-0ι)1 · ° =(0"°2)' где g(x) — некоторая дифференцируемая монотонно возрастающая функция. Убедиться в том, что справедливо 69
следующее обобщение результата задачи 2.86: о. м. п. θ„ = =(g,т), wbfigTO, T? = -!-i(g(x.)-gf- яв- ляется ли g эффективной оценкой Οι? Показать, что при известном значении Οι = а эффективной оценкой θί> является статистика 7-? = ^-2(g№)-a)2 " i = ι [ср. с соответствующими результатами для нормальной модели (задача 2.48)]. I Указание. Воспользоваться задачей 2.46. 2.96. Пусть случайная величина ξ имеет распределение типа степенного ряда (см. задачу 2.60). Показать, что уравнение правдоподобия для_нахождения о. м. п. б„ в данном случае имеет вид μ(θ) = X, где μ(θ) = Εοξ. Вычислить асимптотическую дисперсию оценки 0„. Применить эти результаты для оценивания параметра θ модели Bi(r, 0). 2.97 Записать уравнения метода накопления для приближенного вычисления о. м. п. 0„ параметра 0 урезанного в нуле пуассоновского распределения (см. задачу 2.10). I Указание. Использовать решение задачи 2.96. 2.98. Пусть в полиномиальном распределении М(п; р\, ρ ν) вероятности исходов pt = ρ,(θ), i = 1, ..., Ν, где 0 — неизвестный скалярный параметр. Записать уравнения метода накопления для приближенного вычисления о. м. п. θ„. 2.99. Рассматривается задача оценивания параметра 0 модели Коши/С(0) по соответствующей выборке Χ= (Χι, . , Х„). Записать уравнения метода накопления для приближенного вычисления о. м. п. 0„. Рассмотреть в качестве оценки θ выборочную медиану Т„ = Х,гп ι л и вычислить ее асимптотическую эффективность. I Указание. Использовать задачи 2.43 и 1.32. 2.100. Пусть X = (Χι, ..., Хп) — выборка из равномерного распределения /?(0, 0). Показать, что в данном случае о. м. п. θ„ = -Υ(„), убедиться в ее состоятельности и найти ее предельный закон распределения (п->-оо). I Указание. Воспользоваться задачами 2.24 и 2.79. 2.101. Показать, что в случае модели /?(0——, 0 -+- + —J любое значение 0е Х{п) —, X{i) + -у- является 70
о. м. п. θ„. Какая точка этого интервала является несмещенной оценкой Θ? Указание. Использовать решение задачи 2.80 и задачу 1.36. 2.102. Показать, что для параметра сдвига θ распределения Вейбулла W(Q,a,b) при 0<а^1 о. м. п. θ„ = = Х(]), убедиться в ее состоятельности и найти ее предельный при п-+оо закон распределения. I Указание. Использовать решение задач 1.37 и 2.26. 2.103. Случайная величина ξ, характеризующая срок службы элементов электронной аппаратуры, имеет распределение Релея W(0, 2, ι/θ), плотность которого f(x; θ) = = (2x/Q)e~*/e, дс^О. Построить по соответствующей выборке Х= (Х\, ..., Хп) о. м. п. θ„ (ср. с задачей 2.76). 2.104. По выборке X = (Х\, ..., Хп) из распределения Γ(θ, λ) требуется оценить функцию τ(θ) = —. Показать, что о. м. п. τη = λ/Χ. Убедиться в состоятельности этой оценки и найти ее предельный при п-*- оо закон распределения. Указание. Воспользоваться задачами 2.21, 2.43 и 2.84. 2.105*. Доказать, что для распределения Лапласа, задаваемого плотностью f[x; θ) = — е~[х~й\ x^R, о. м. п. θ„ совпадает с выборочной медианой. Можно ли здесь воспользоваться теоремой об асимптотической нормальности о. м. п.? 2.106*. Пусть X = (ΑΊ, ..., Хп) — выборка из распределения^^, I). Тогда (см. задачу 2.84) о. м. п. θ„ = X и La{X) = N(Q, \/п). Рассмотреть в качестве оценки θ статистику оХ при \Х\ >ая, ЬХ при \Х\ <а„, где константа а(!->-0, но -\/ηα„—>- оо при п-*- оо, и вычислить ее асимптотическую эффективность. л2.107. Привести примеры о. м. п. θ„, для которых D0G„ = о(п~1). Указание. Рассмотреть модель R(0,Q) (см. задачу 2.100) и модель Вейбулла (см. задачи 2.102 и 1.37). 2.108. Рассмотрев задачу оценивания функции τ(θ) = = θ-1 в модели П(0), убедиться в том, что о. м. п. τ„ ни при каком η не имеет конечных моментов, но ее симптотическая дисперсия существует и равна (63я)"'. Тп = 71
I Указание. Использовать задачи 2.84, 1.39 и 2.43. 2.109*. Преобразования, стабилизирующие дисперсию. Для моделей Bi{k,Q), Π(θ), Ν(μ, θ2) и Γ(θ, λ) найти такие параметрические функции τ(θ), чтобы асимптотические дисперсии соответствующих о. м. п. τ„ не зависели от параметра Θ. | Указание. Использовать задачу 2.43. 2.110. Смоделировать выработки объемами η = 10, 100, 1000 и получить о. м. п. параметров следующих распределений: 1) iV(0b θ2), при моделировании положить Θ, = 1, θ?=4; 2) Bi(\, θ), при моделировании положить θ = 0,7; 3) R(0, θ), при моделировании положить 0 = 1. Указание. Использовать задачи 2.86, 2.84 и 2.100 соответственно. 2.111*. Оценивание размера конечной совокупности. В условиях задачи 2.83 установить, что о. м. п. N неизвестного размера совокупности N при η>1 однозначно находится из условия S(#,4)<n<S(#- Ι,η), где S[N, k) = 1 n NN++ l__k /1 π η^1 при ЛГ>Л>1, S[k - — 1, k) = 00. Если же η = 1, то S= 1. Определить, при каких значениях η оценка $ = η. Предполагая, что η, Λ/->-οο , 0<α0 5^ а =—-г ^αι < °° , где ао, αϊ — некоторые константы, получить приближенное выражение для о. м. п. а = п/(Я + 1)· Обобщить этот результат на случай произвольного значения ш. 2.112 (продолжение задачи 2.111). Для оценки неизвестного числа Л/ рыб в озере проводят следующий эксперимент. На первом этапе по схеме случайной выборки без возвращения вылавливают тх рыб, метят их и выпускают обратно в озеро. На втором этапе по аналогичной схеме вылавливают еще т2 рыб и подсчитывают число μ2 оказавшихся среди них меченых рыб (так что число различных пойманных за оба улова рыб η = ml-\-ni2 — μ2). Показать, что о. м. п. # по данным μ2 определяется равенством # = —■—— I. Сравнить этот результат при т, = т2 с результатом, полученным в задаче 2.36. I Указание. Учесть, что статистика μ2 имеет гипер- | геометрическое распределение Н{ти N, m2). 72
2.113*. Выборочный контроль. Имеется партия из Л/ из делий, содержащая некоторое (неизвестное) число D дефектных изделий. Чтобы оценить параметр D или некоторую заданную функцию от него t(D), случайным образом без возвращения из всей партии извлекается n(n<.N) изделий, каждое из которых проверяется на доброкачественность. Пусть Х-, = 1, если С-е проверяемое изделие дефектно, и Xi = О в противном случае, i = 1, ..., п. 1) Показать, что dn = Х\ + ··· + %п (общее число обнаруженных в выборке X = (Х{, ..., Хп) дефектных изделий) есть полная достаточная статистика для D, имеющая гипергеометрическое распределение H(D,N,n), и, основываясь на этом, убедиться в том, что несмещенные оценки существуют лишь в случаях, когда t(D) — многочлен степени не выше п. В этом случае, если t(D) = = Σ ai(D)it(D)i = D(D - \)...(D - j + 1), (D)0 = 1, то on l = o тимальной несмещенной оценкой t(D) является статистика л τ* = T{dn) = 2 aidMNyin),. / = o 2) Получить явный вид оптимальных оценок для функций ti(D) = D и t2{D) = D(N — D), которые с точностью до множителей являются соответственно средним и дисперсией статистики dn (см. п. 6J гл. 1). 3) Убедиться в том, что о. м. п. 6п = [{N + \)dn/n]. У к>а з а н и е. Использовать решение задачи 2.33 и формулы для моментов распределения ^(Ο,Ν,η). 2.14. Объединение статистической информации. Пусть Xj = (Xju ..., А";Я/), /' = 1, ..., k, — независимые выборки из распределений N(Qj\y θ!), /= \,...,k, соответственно и Xh S] = S2[Xj) — соответствующие выборочные средние и дисперсии. Доказать, что θ = (ΑΊ, ..., Xk, Θ2) — о. м. п. для θ = (θ,,, ..., θ*,, θ2), где θ| = —-—-—2 niSl> несме- Я 1 ~T~ -■- I ftk : __ ι щенной же оценкой для общей дисперсии θ| является статистика Θ2 = "■ + - + "'§» = ! ν „/S,2. η, + ... + nt-k n,+... + nk-k.^l > ' I Указание. Использовать решение задачи 2.86. 73
§ 4. Доверительное оценивание 2.115. Показать, что γ-доверительный интервал для параметра θ модели_#(0, θ2), Θ>0, по выборке X = (Xlf ..., .., Х.п) имеет вид {Х/{\ + cv/V"). */0 - с,/лШ>. Получить соответствующее решение для моделиУУ(0, θ ), θ<0. Указание. Использовать тот факт, что£о((^— -6)V^/e)=jv(o, ι). 2.116. Пусть X = (Хи ..., Хп) — выборка из распределения N{6, а ). 1) Убедиться в том, что любой интервал вида Δν(.Υ) = = {Х — ^-g2, X— l^gi)' гДе gi<g2 —любые числа, удовлетворяющие условию CD(g2) — Φ(^ι) = Υ, является γ-доверительным интервалом для параметра Θ. Доказать, что наикратчайшим среди этих интервалов является интервал А'У[Х) = (х ±-?=- c-λ . 2) Сколько необходимо произвести наблюдений η = = п(1,у), чтобы точность локализации параметра при доверительном уровне ν была равна заданной величине I? Вычислить n(l,y) при γ = 0,99, I = 0,5 и I = 0,1 (величина σ = 1). Как изменяется доверительный уровень γ в зависимости от / и п? Указание. Воспользоваться центральной статистикой G{X; θ) = -^-{Х - θ). 2.117. Доказать, что γ-доверительным интервалом для среднеквадратического отклонения θ модели Λ^(μ,θ2) является любой интервал δν(Χ) = (Т/й2, Т/а\), где Т2 = η = "Σι {Χ· — μ)2, а числа α^<.α2 выбираются из условия i= 1 \ xkn(x2)dx = γ/2, где kn(i) — плотность распределения πι Х2(гс). Определить наикратчайший в этом классе интервал δ*(*)· , , Указание. Воспользоваться тем, что Ц>(Т /Θ ) = = х2И- 2.118 (продолжение задачи 2.117). Показать, что центральный γ-доверительный интервал для дисперсии Θ2 имеет вид 11 Напомним, что ст=Ы(|+Т)/2=Ф_/—5^)· 74
Δγ(Χ) = (T2/g2, 72/Ы g, = Χ2ι^„ , g2 = *\+i „ , в то время как наикратчайшим среди интервалов такого вида, где числа gi < g2 удовлетворяют условию \ kn(t)dt= ν, является интервал Δ'Ί(Χ) = {Т\2, Tl2) (см. ре- шение предыдущей задачи). 2.119. По выборке X = {Х\, ..., Хп) из распределения N(Q], θ|) построить односторонние и двусторонний γ-дове- рительные интервалы для среднего θ|. Указание. Использовать утверждение L ^V^=T^-)=5(rt-l). 2.120. По выборке X = {Хи ..., Хп) из распределения W(Q], ΘΙ) построить односторонние и двусторонний γ-дове- рительные интервалы для дисперсии τ = 02. I Указание. Воспользоваться теоремой Фишера. 2.121. По реализации (2,96 3,07 3,02 2,98 3,06) выборки объема 5 из нормального распределения с неизвестными параметрами рассчитать 0,95-доверительные интервалы для среднего и дисперсии. 2.122. Пусть X = {Χι,.... Ха) и Υ = (К,, ..., Ym) - две независимые выборки, причем первая из распределения Ц0"\ σΤ), а вторая из распределения Λ^θ*2', σί). Построить γ-доверительный интервал для разности средних τ = = θ^ - 0ί2). Указание. Установить, что L({X— Υ — τ)/σ) = 2.123 (продолжение задачи 2.122). Пусть в отличие от предыдущего случая все наблюдения имеют одинаковую, но неизвестную дисперсию Θ2-, т. e.L(Xi) =N(Qfp, Ql),L(Yj) = = Af($2), θ|). По-прежнему требуется оценить разность средних τ = θίι1' — $?. Рассмотреть более общую ситуацию, когда дисперсии неизвестны, но различаются лишь известным множителем, т. е.ВД) = N{UI\ cQl),L{Yf) =N{$V, θ2), с — известно. Указание. Установить, что случайная величина /,+,_, =Л/тП(тт++72) <*~ y-^)NnS\X) + mS\Y) имеет распределение Стьюдента S{m + η — 2). 2.124. В результате двух равноточных измеренной угла получены следующие результаты (в градусах): 20,76 и 75
20,98. Еще шесть независимых и равноточных измерений того же угла выполнены с помощью другого прибора и были получены такие результаты: 21,64; 21,54; 22,32; 20,56; 21,43 и 21,07. Предполагается, что случайные ошибки результатов измерений распределены нормально, причем известно, что первый прибор менее точен (ему соответствует дисперсия, превышающая в четыре раза дисперсию, соответствующую второму приб'ору). Рассчитать 0,95-до- верительный интервал для разности систематических ошибок, отвечающих этим приборам. | Указание. Воспользоваться решением задачи 2.123. 2.125 (продолжение задачи 2.124). Пусть, наконец, выборки имеют разные дисперсии, т.е. L(A",) = Af(#i'', θφ2), ЦУ1) =Ν{#\\ θφ2). Построить доверительный интервал для отношения τ = ОД'2/^' . Указание. Установить, что центральной статистикой является в данном случае Ρ _ /i(m-l) S\X)/ /-„_,.„_,- т(л_() ■_/ τ. 2.126. В двух лабораториях определялась концентрация серы (в %) в стандартном образце дизельного топлива. Шесть независимых равноточных измерений в первой лаборатории дали следующие результаты: 0,869, 0,874, 0,867, 0,875, 0,870, 0,869. В результате аналогичных пяти равноточных измерений во второй лаборатории были получены такие значения: 0,865, 0,870, 0,866, 0,871, 0,868. Предполагая справедливым нормальный закон ошибок измерений, построить 0,95-доверительный интервал для отношения дисперсий измерений в 1-й и 2-й лабораториях. Если есть основания считать эти дисперсии одинаковыми, то рассчитать аналогичный интервал для разности систематических ошибок, допускаемых в обеих лабораториях. 2.127. Пусть Х = (Х , Хп) и Υ={Υι, ..., Km) — две независимые выборки из распределений Γ(θ|, 1) и Г(02, 1) соответственно. Построить центральный γ-доверительный интервал для отношения τ = θ2/θ,. I Указание. Использовать задачу 1.51. · 2.128. Убедиться в том, что (х0) + 1π('η~γ) , Χ(Λ , где Х{\) ■= min Xit есть γ-доверительный интервал для пара- 1 < i sg η метра θ экспоненциального распределения с плотностью 76
Указание. Найти распределение статистики Х,\) и учесть, что событие (Χ(ΐ)Ξ^Θ) является достоверным. 2.129. Убедиться в том, что (Х[п), Х[П)/^\ — у) есть у- доверительный интервал для параметра 0 модели R[0, θ) по выборке объема п. I Указание. Установить, что Lo((Xln)'/Q)") = R(0, l) I (см. задачу 1.35). 2.130. Рассмотрим модель №(0, λ, θ) (см. задачу 2.76). Убедиться в том, что центральным γ-доверительным интервалом для функции τ(θ) = 0λ является интервал (2T/lf+j 2n , 27/Xi^_j, 2л). В частности, при λ = 1 имеем соответствующее решение для экспоненциальной модели Г(0, 1). I Указание. Использовать решение задачи 2.76. 2.131*. Убедиться в том, что γ-доверительная область для параметров (θ,,τ = θ'|) общей нормальной модели Ν(0|, О?) по выборке X = (Xt, ..., Х„) имеет вид Gy(X) = {(θ,,τ):τ>«(Χ-θ,)2/^„ nS2/!^ < 2 ■" ' <τ<η52/Χ?-„π ,), где γι V2 = у. I Указание. Воспользоваться теоремой Фишера 2.132*. Пусть [Xi = (Хп, Χί2), ί = 1, ..., η) — выборка из двумерного нормального распределения ^((θ,.θ,). 2 = ιι;;σ2(, ;ррц).-кр<1, с известной матрицей Σ. Используя задачу 1.59, построить 7-ДОверительную область для θ = (0,, θ2). 2.133. Пусть X ={Х Х„) — выборка из распределения Βί(1, θ). Основываясь на точечной оценке Τ = X параметра Θ, показать, что центральный γ-доверительный интервал для него (7Ί, 7"г) определяется условиями Σ СТ(1 - 7,)"-г = Σ СгпЩ\ - 72)"-г = -Ц1^. г = пТ г = й Построить приближенный доверительный интервал для θ при больших п. Указание. Использовать задачи 1.39 п. 3), 2.43 и 2.84. 2.134. По выборке X = (Х1у ..., Хп) для бернуллиевской модели βί(1, θ) построить асимптотический (при п->-оо) γ-доверительный интервал для Θ, основываясь на нормальной аппроксимации L^[n{X — Ъ)/лЩ\ — θ)) ~ jV(0,1) 77
(теорема Муавра—Лапласа). Сравнить полученное решение с решением, основанным на асимптотических свойствах оценок максимального правдоподобия (см. задачу 2.133). 2.135 (продолжение задачи 2.134). Убедиться в том, что ( arcsin-«y.Y ±-4^) —асимптотический γ-доверитель- ный интервал для функции τ(θ) = airsin^/i^ I Указание. Использовать задачу 2.109. 2.136. При 540 испытаниях в схеме Бернулли положительный результат наблюдался 216 раз. Рассчитать 0,95- доверительный интервал для дисперсии числа положительных исходов. 2.137. Пусть X = (ΛΊ, ..., Хп) — выборка из распределения Π(θ). Основываясь на точечной оценке Τ = X, показать, что центральный γ-доверительный интервал (Τι, Τ2) для θ определяется условиями rha " ~,ΐΌ Μ - 2 ■ Приближенный же •у-доверительный интервал (при больших п) есть (X ± Су^/Х/п). 2.138. Построить асимптотический γ-доверительный интервал для параметра θ пуассоновской модели _П((Э), воспользовавшись нормальной аппроксимацией U(2^nbJX—\jQ))~N(0, 1) (см. задачу 2.109) или аппроксимацией f/ v"( — ) λ —JV(0, 1) (центральная предельная теорема). Сравнить с соответствующим решением предыдущей задачи. 2.139. Независимые случайные величины Х\ и Х2 имеют распределения Пуассона с параметрами λι и λ2 соответственно. Пусть известно значение их суммы Х\-\-Х2 = = п. Построить при этом условии доверительный интервал для θ = λι/(λ| + λ2) по наблюдению над ΑΊ. Указание. Найти условное распределение ЦХ\\Х\ + -f- Х2 = п) (см. задачу 1.54) и воспользоваться решением задачи 2.133. 2.140. Построить асимптотический γ-доверительный интервал для параметра θ распределения степенного ряда (см. задачу 2.60). Применить полученный результат для оценивания параметра θ модели Bi(r, θ). I Указание. Использовать задачу 2.96 и ее решение. 78
2.141. Построить асимптотический γ-довернтельный интервал для параметра θ модели Γ(θ, λ). Указание. Использовать задачи 2.43 и 2.48, а также аппроксимацию, полученную в задаче 2.109. 2.142. Построить асимптотический γ-доверительный интервал для параметра θ модели Ν(μ, θ2). Указание. Воспользоваться аппроксимацией /.0У2п(1пЭл — Ιπθ) ~Ν(0, 1), полученной в задаче 2.109. 2.143. Построить асимптотический γ-доверительный интервал для функции τ(θ) = ф(-^-——J в модели #(θ|, θ§) (см. задачу 2.72). I Указание. Использовать задачу 2.87. 2.144*. Для полиномиальной модели М(п; ри ■■·, Ρν) с неизвестными параметрами р\,...,ры (см. задачу 2.29) построить асимптотическую (при п-+ао) γ-доверитель- ную область для р\ pNy основанную на соответствующих оценках максимального правдоподобия. Указание. Использование задачи 2.63, 2.45 и асимптотический вариант задачи 1.40: если L(Yn) ~ ~#(μ„, Σ„) при η -+ оо и IXJ φ 0, то ЩУа - μ„)'Σ,Γ ' X Χ (Υп — μ„)) -► x2(m), где т — размерность вектора Yn. 2.145*. Пусть п,Х и 52 — соответственно объем, выборочные среднее и дисперсия выборки из распределения Λ^Ο,,ΘΙ). Показать, что с вероятностью у результат следующего, (п + 1)-го испытания находится в интервале (Я ± /l+x п _ , SV(n + l)/(n-l)) · I Указание. Воспользоваться теоремой Фишера. 2.146 (продолжение задачи 2.145). В результате пяти независимых взвешиваний одного и того же тела получены следующие результаты (в граммах): 4,12; 3,92; 4,55; 4,04; 4,35. Считая погрешности измерений нормальными #(0, Θ2) случайными величинами, указать 0,95-дове- рительный интервал для результата предстоящего шестого взвешивания. 2.147. Пусть Щ) = Х2(п), где число степеней свободы η неизвестно. Рассчитать приближенный 0,9-доверитель- ный интервал для п, соответствующий реализации ξ = = 157,4. | Указание. Воспользоваться нормальной аппрокси- | мацией для распределения хи-квадрат (задача 1.45). 2.148. По выборкам, полученным в задаче 2.110, по- 79
строить доверительные интервалы для соответствующих параметров. Указание. Воспользоваться задачами 2.119, 2.120, 2.133 и 2.129 соответственно. Глава 3 ПРОВЕРКА СТАТИСТИЧЕСКИХ ГИПОТЕЗ 1. Статистической гипотезой (или просто гипотезой) называют любое предположение о виде или свойствах распределения наблюдаемых в эксперименте случайных величин. Пусть для исследуемого процесса сформулирована некоторая гипотеза #о (ее называют основной или нулевой гипотезой), тогда задача проверки этой гипотезы заключается в конструировании такого правила (алгоритма), которое позволяло бы по результатам соответствующих наблюдений (по имеющимся статистическим данным) принять или отклонить Но- Любое такое правило называют статистическим критерием (или просто критерием) согласия для гипотезы Но- Если гипотеза Н0 однозначно фиксирует распределение наблюдений, то ее называют простой, в противном случае — сложной. Пусть результат эксперимента описывается некоторой случайной величиной X = {Х\ Хп) и Н0 — некоторая гипотеза о ее распределении. Пусть, далее, Τ = Т(Х) — некоторая статистика, характеризующая отклонение эмпирических данных от соответствующих (гипотезе Но) гипотетических значений, распределение которой в случае справедливости Но известно (точно или хотя бы приближенно). Тогда для каждого достаточно малого числа а>0 можно определить подмножество 7,,а = = [t\t = Т(х), дсеХ), удовлетворяющее (точно или хотя бы приближенно) условию Р(Ге=Г,«1ЯоХа. (3.1) Любое такое подмножество Т1а порождает следующий критерий согласия для гипотезы Н0: если t = Т{х) — наблюдавшееся значение статистики Т[х), то при t e Т\а гипотеза Я0 отвергается; в противном случае считается, что данные не противоречат Я0 (согласуются с Но); другими словами, если t<£T\a., то гипотеза На принимается (подчеркнем, что факт t φ. 7Ία не является доказательством истинности Но)· Если гипотеза Но истинна, то согласно указанному правилу мы можем ее отвергнуть 80
(т. е. принять неправильное решение) с вероятностью, меньшей или равной а. Число а называют уровнем значимости критерия, а множество 7Ία — критическим множеством (областью) для гипотезы Н0. Статистику Τ в описанной методике называют статистикой критерия, а сам критерий — критерием Т\а- Итак, согласно описанной методике, критерий определяется заданием соответствующей критической области 7Ία в множестве значений статистики Τ при выбранном уровне значимости а. Для того чтобы иметь возможность сравнивать различные критерии (порождаемые разными статистиками Т), надо ввести понятия альтернативного распределения (альтернативной гипотезы) и мощности критерия. Любое допустимое распределение Fx = F выборки X, отличающееся от гипотетического (т. е. распределения при гипотезе Но), называют альтернативным распределением, или альтернативой. Совокупность всех альтернатив называют альтернативной гипотезой и обозначают И\. Функцией мощности критерия Г,а называют следующий функционал на множестве всех допустимых распределений {F\: W{F)=W{Tia;F) = P(T^T](i\F). (3.2) Таким образом, W(F) — это вероятность попадания значения статистики критерия в критическую область, когда истинным распределением наблюдений является распределение F. Если Fe Ни то значение W(F) называют мощностью критерия при альтернативе F; оно характеризует вероятность принятия правильного решения (отклонение Но) в ситуации, когда Но ложна. Из двух критериев с одним и тем же уровнем значимости α лучшим считается тот, мощность которого при альтернативах больше. Желательным свойством критерия Г,а является свойство несмещенности, которое означает, что одновременно с условием W(Tia-F)^a Vfe H0 (3.3) должно выполняться условие W(7V, F)>a λ/F^H, (3.4) (т. е. при альтернативе вероятность попадания в критическую область должна быть больше, чем при основной гипотезе). 81
Функцию мощности удается вычислить далеко не всегда (для этого надо знать распределение статистики критерия при всех альтернативах), однако часто можно исследовать ее асимптотическое поведение при объеме выборки п-+оо (чтобы подчеркнуть зависимость функции мощности от объема выборки, пишут Wn(F)). Исследуя асимптотические свойства критериев, прежде всего рассматривают вопрос, является ли критерий состоятельным. При этом состоятельность критерия означает, что я1што Wn[F) =1 VFeWi. (3.5) Таким образом, состоятельный критерий при большом числе наблюдений «улавливает» любые отклонения от основной гипотезы с вероятностью, близкой к 1: если истинной является любая фиксированная альтернатива, то при больших η попадают в критическую область с вероятностью, близкой к 1, и, следовательно, отвергают основную гипотезу, которая является ложной (т. е. принимается правильное решение). Более детальные свойства состоятельного критерия можно исследовать, рассматривая асимптотическое поведение мощности W„(F„) при «близких» альтернативах Fn, т. е. когда последовательность альтернатив {Fn\ сближается (в том или ином смысле) при п->-оо с основной гипотезой Но. Основной интерес при этом представляет «пороговый» случай, т. е. определение такой последовательности {Fn}, для которой lim W„(Fn) = у, α<γ<1, (3.6) П —»· оо и вычисление этого предела у. 2. Наиболее известными критериями проверки простой гипотезы Но- /ч(*) = F(x) являются критерий Колмогорова и критерий X2. Критерий Колмогорова применяют, когда F(x) непрерывна. Статистикой критерия является величина Dn = = Dn(A")= sup \F„(x)— F(x)\ —максимальное от- — оо <с х <;-оо клонение эмпирической функции распределения Fn(x) от гипотетической F(x). При фиксированном χ величина F„(x) является оптимальной оценкой для F(x) и с ростом η Fn(x)-+F(x), поэтому по крайней мере при больших п, в тех случаях, когда гипотеза Н0 истинна, значение Dn не должно существенно отклоняться от нуля. Точное распределение 9{^fnDn<Li) уже при η^20 хорошо при- 82
ближается предельным распределением Колмогорова оо K(t) = 2 (—■ 1)'ехр{— 2/2/2}, для которого составлены / = - оо таблицы. Критическая область критерия определяется неравенством ^fnDrl^ta, где K(ta) = 1 — α. Часто исходные статистические данные предварительно «группируют», что осуществляется следующим образом. Пусть X = (Хи .... Хп) — -повторные независимые наблюдения под некоторой случайной величиной ξ с множеством возможных значений Е. Рассмотрим некоторое разбиение Ε = Е\\]...\]Еы, Ei(]Ej=^0, ίφ'ι, и пусть ν/ — число элементов выборки X, попавших в подмножество Е„ a pj = pj(F) = \dF(x) — вероятность попадания в Ej при заданном распределении F величины ξ, / = 1 ..., Ν (ν, + ■■■ + vn = /ζ, ρ, + ... + рн = 1). Тогда вектор частот ν = (ν,, ..., vn) имеет полиномиальное распределение М(п; ρ = (ρι, ..., ρΝ)), и каждая гипотеза о распределении Ζ-(ξ) трансформируется d соответствующую гипотезу о оекторе ρ распределения М(п; р). Таким образом, в данной методике переходят от исходных наблюдений X = {Х\, ..., Х„) к частотам ν = (vb ..., ν,ν) попадания элементов выборки в соответствующие подмножества Е\, ...,Εν. Такой «частотный» способ представления статистических данных называют методом группировки наблюдений, а подмножества Et En — интервалами группировки. Относительная частота ν,/η попадания в интервал Ej является состоятельной оценкой вероятности Pi, поэтому в качестве меры отклонения эмпирических данных от гипотетических значений р° можно выбирать различные функции от разностей | —' р°\ , /= 1,..., ..., Л'. Наиболее употребительной является мера /V , οι γ _ χ2 _ у К - "Ρι)' _ " / = 1 "/''/ предложенная К. Пирсоном. Если Но — простая гипотеза, однозначно фиксирующая вероятности р° = (р,0, ..., ..., р°), то при 0<р°<1, /=1,..., N, и n-юо соответствующий критерий согласия, называемый критерием хи-квадрат, асимптотически задается критической областью (Хл^Хг'-а.л'-И, где X2,,—р-квантиль распределе- 83
ния l'\r). Другие применения подобной методики см о [1, гл. III]. 3. Важной является задача проверки однородности статистического материала. Пусть имеются две независимые выборки X = (X],..., Х„) и Υ = (Υ , Υ„), описывающие один и тот же процесс, явление и т. д., но полученные, вообще говоря, о разных условиях. Требуется установить, являются ли они выборками из одного и того же распределения или же закон распределения наблюдений от выборки к выборке менялся, т. е. требуется проверить гипотезу однородности Но о том, что F\(x) = F2(x), где F\(x) и F2(x) — функции распределения выборок X и Υ соответственно. Одним из распространенных критериев однородности является критерий Смирнова, применяемый в случае непрерывных распределений. Критерий основан на статистике Dnm = D„m(X,Y) = = sup \Fln(x) — F2m(x)\, где Fi„(j;) и ^„(j;) - эмпири- ческие функции распределения, построенные по выборкам X и Υ соответственно. В случаях, когда справедлива гипотеза Но, функции Fin(x) и F2m(x) с увеличением объемов выборок пит «сближаются» и поэтому статистика Dnm не должна сильно отличаться от 0. Точное распределение Ρ(γ—37—Dnm^t) приближается предельным распределением Колмогорова K(t). Критическая область критерия определяется неравенством γ пт Dnm^ta, где K(ta) = 1 — α. Другим часто применяемым критерием является критерий однородности X2. Его используют для проверки однородности данных, имеющих дискретную структуру или сводимых к этому группировкой. Кроме того, он применим для сравнения любого числа выборок. Пусть осуществлено k серий независимых наблюдений, объемы которых Л|, ..../и, и в каждой серии наблюдался некий переменный признак, принимающий одно из s возможных значений (исходов). Пусть ν,,· — число реализаций i-ro исхода в /-Й серии (£ ν,·; = nit /=1,..., / = 1 ..., k). Требуется проверить гипотезу Но о том, что все наблюдения производились над одной и той же случайной величиной. Статистикой критерия X2 в данном случае является величина 84
χΐ=η(Σ 2-^--ι), ГДе V,- = 2 Vi/, i = 1 S, Λ = Π| + ... + ЛА . /= ι Критическую область задают в виде .#j^Xi2-a.(s-ix*-i), где границу критерия определяют из таблиц квантилей распределения X2. Вероятность ошибочно отклонить при этом истинную гипотезу приблизительно равна а, если η достаточно велико. 4. Если X = (Х\ Х„) — выборка из распределения Ц1) и множество F всех допустимых распределений наблюдаемых случайной величины ξ задано в параметрической форме: F=[F(x\B), 0 = (0 0,)εθ), то гипотезы о распределении L (ξ) формулируются в терминах неизвестного параметра θ и называются параметрическими. В общем случае (основная) параметрическая гипотеза задается в виде ho'-Q^Qo при некотором подмножестве θο^ζθ. В этом случае альтернативная гипотеза имеет вид Я:: 0 εθι = Θ\θο. Таким образом, в рамках параметрической модел-и альтернативная гипотеза конкретизируется и имеет такую же форму, как и основная гипотеза; в данном случае отклонение основной гипотезы эквивалентно принятию конкретной альтернативной гипотезы. В общей теории проверки параметрических гипотез критерии принято задавать указанием соответствующих критических областей непосредственно в выборочном пространстве Х= {х= (*ι, ..., χ„)}. Таким образом, при уровне значимости α критерий проверки гипотезы Но задается выбором такого подмножества XiaczX, для которого выполняется условие Ρο(Λ:εΞΛ:Ι(1)<α Λ/Θ^Θο, (3.7) являющееся аналогом условия (3.1). В этом случае сам критерий (называемый критерием Х\а) формируется следующим образом: если χ — наблюдавшаяся реализация выборки X, то при х^Х\л гипотезу Я0 отвергают (принимают альтернативную гипотезу Я,), если же х^Хо«=Х\а, то гипотезу /?о принимают. Для функции мощности в данном случае используются обозначения (ср. с (3.2)): W(Q)= W(Yle;0)= Р0(Х^Хы), ΟξξΘ. 85
Вероятности ошибочных решений для критерия Х\а выражаются через его функцию мощности следующим образом: вероятность ошибки первого рода (отклонить Но, когда она верна) равна W(Q), Θ^Θο (в символическом виде Ρ(/?ι Ι //о)), а вероятность ошибки второго рода (принять Но, когда она ложна) равна 1 — Щ0), θεθ, (в символическом виде Р(Яо1Я,)). Рациональный принцип выбора критической области формулируется в терминах вероятностей ошибок следующим образом: при заданном числе испытаний устанавливается граница для вероятности ошибки первого рода и при этом выбирается та критическая область, для которой вероятность ошибки второго рода минимальна. Пусть X и иХ*а — два критерия одного и того же уровня значимости α для гипотезы Но. Если U?(X*a; θ)^ <№(ΧΙα;θ) при θεΞθο и Ψ(\'Λ; θ)^ W(X,«; θ) при θεΞθ, (со строгим неравенством по крайней мере при одном Эе0|), то говорят, что критерий \'а равномерно мощнее критериях ία и ему, очевидно, следует отдать предпочтение, поскольку он приводит к меньшим ошибкам. Если указанные неравенства выполняются для любого критерия ΧΙα, то Χ,α называют равномерно наиболее мощным (р. н. м.) критерием. В случае простой альтернативы Н\ (множество 0, состоит из одной точки) вместо р. н. м. говорят о наиболее мощном критерии. В некоторых случаях указанный принцип сравнения критериев позволяет определить оптимальный (в рассматриваемой задаче) критерий. Иногда задачу построения оптимального критерия удается решить в классе несмещенных критериев, т. е. когда одновременно с (3.7) выполняется условие Щ6)>а -νβ6=θ|. В теории часто удобно рассматривать так называемые рандомизированные критерии, когда при наблюдении χ гипотезу Й0 отвергают с некоторой вероятностью φ(χ) и принимают с дополнительной вероятностью 1 — ц>(х). Функция φ(χ) (Ο^φ(χ)^Ι, хеХ) называется критической функцией; описанная выше конструкция нерандомизированного критерия Χία соответствует выбору в качестве φ(χ) индикаторной функции множества Χια:φ(χ) = 1 при xeXia и φ(χ) = 0 при х^Х|а. Функция мощности рандомизированного критерия определяется соотношением ΒΡ(θ)=Α7(φ;θ) = £0φ(Χ). 5. В основе большинства способов построения оптимальных критериев лежит фундаментальный результат, полученный Ю. Нейманом и Э. Пирсоном о существова- 86
нии наиболее мощного критерия в задаче проверки простой гипотезы при простой альтернативе. Именно, если Θ = (θο, θι}, то при любом уровне значимости α наиболее мощный критерий проверки гипотезы Но- θ = θο при альтернативе Н\\ θ = θι существует и задается критической областью x''={*-l^^-WW>ci' ■ {3-8) а где L(x; θ) = Π /(*,; θ) — функция правдоподобия (о не- 1= 1 которых особенностях, связанных с дискретностью наблюдений, см. подробнее в [1, § 4.2]). Если проверяется простая гипотеза Я0: θ = θο против сложной альтернативы Н\\ θ^θ\{θο), то р. н. м. критерий существует, если критическая область Х'а =Χ*α(θο', θ,), определенная в (3.8), не зависит от конкретного θι <= θ\[θ0}; в этом случаеХ\а и есть р. н. м. критерий. Такое обстоятельство имеет место для важного класса моделей F, обладающих монотонным отношением правдоподобия (т. е. таких, которые обладают достаточной статистикой Т(Х), и при этом функция l(x) = g(T(x); Q\)/g(T(x); θ0) монотонна по Τ (см. критерий факторизации в п. 3 гл. 2), и в случае односторонних альтернатив Hf: θ 3£ θ0 (θ — скаляр) [1, с. 155]. Более того, для таких моделей р. н. м. критерий проверки простой гипотезы Я0: θ = θο против правосторонней альтернативы Ht'. θ>θο является одновременно р. н. м. критерием проверки сложной гипотезы Но'- θ^θο против Я|+ того же уровня значимости (аналогичное утверждение справедливо и для двойственной проблемы проверки Но: θ^θο против Η f: θ<θο [5, с. 101]). В частности, для экспоненциальной модели, задаваемой плотностью f(x; θ) =.exp{A(Q)B(x) + С(в) + D(x)}, η статистика Т(Х) = Σ Β(Χι) достаточна, и если функция i = 1 Α(θ) строго монотонна, то вид р. и. м. критериев Х\л указан в следующей таблице: .4(0 И нЫ>оа {7Хж)«гД ЯП0<0о [Цх1>сЗ 87
При проверке простой гипотезы Н0: θ = θο против двусторонней альтернативы Η\: θ Φ θο в ряде случаев также удается построить р. н. м. несмещенный критерий [1, с. 159]. Удовлетворительное решение этой задачи в ряде случаев получают с помощью следующего приема: если для рассматриваемой модели существуют р. н. м. критерии против односторонних альтернатив #Г и Θ + (соответственно Xfi, и Xt<L2)> то используют критерий вида Х1а = = Х^ЦХич при αϊ +α2 = α. Особый интерес представляют малые отклонения от нулевой гипотезы Я0: θ = θο- В этом случае при исследовании свойств критерия можно ограничиться анализом локального поведения функции мощности критерия ЩЭ) в окрестности точки θο. При таком подходе часто удается построить локальный наиболее мощный критерий, даже тогда, когда р. н. м. критерия не существует [1, с. 161]. 6. Часто весьма полезным бывает тот факт, что задача проверки простой гипотезы относительно θ является обратной по отношению к задаче построения доверительного множества для Θ. Именно, еслиО^(^) есть γ-довери- тельное множество для Θ, то Аоа= {x:Qo^Gy(x)) определяет область принятия гипотезы Но'- θ = θο с уровнем значимости α = 1—у. Верно и обратное, т.е. если для каждого θο^Θ имеется какой-либо критерий Х\а =Α,α(θο) проверки гипотезы Н0: θ = θο, то, определив для каждого дсеХ подмножество Gy(x) = {Q:x^Xw(Q)}, у = 1—α, получим, что Gy(X) — γ-доверительное множество для Θ. Таким образом, если для некоторой модели известно решение одной из этих задач, то по описанному алгоритму можно получить решение другой. При этом р. н. м. критерии соответствуют наикратчайшим доверительным множествам и наоборот. 7. Одним из наиболее универсальных методов построения критериев проверки сложных параметрических гипотез является метод отношения правдоподобия. Общий вид критерия отношения правдоподобия (к. о. п.) для проверки гипотезы Но: θ^θο таков: Хы =Χ]α(Θο, Θ) = {χ: λη(χ) = suoL{x; 0)/supL(*; 0)^ca}, OeBa О ев где граница сЛ выбирается из условия W(Q) = PoCUX)<Ca)<a Α/θеΘο. Во многих практических задачах такой подход приводит к удовлетворительным решениям. Кроме того, при неко- 88
торых условиях к. о. п, обладает свойством асимптотической оптимальности для больших выборок. Если выполняются условия регулярности, обеспечивающие существование, единственность и асимптотическую нормальность оценки максимального правдоподобия 6„ = ф\п, ..., Θ™) параметра θ = (θι, ..., θ,) (см. главу 2, п. 4), то при простой гипотезе Н0: θ = θο для больших объемов выборки η к.о. п. задается асимптотически критической областью Хы = {χ: -2Ιηλ„(*)>Χ?_«.,), где X2., — р-квантиль распределения Х2(г). При этом он является состоятельным (№„(£))->-1 при п->-оо -уб=^Э0) и его мощность при близких альтернативах вида θΊπ) = = θο + β/V". β = (βι. ···. β>·) Φ 0, удовлетворяет при п-+са соотношению Wn(e<">-l-f,(x?_e.,; λ2), где λ2 = β'/(θο)β, /(θ) — информационная матрица модели, a Fr{t;X2)—функция нецентрального распределения χ2 с числом степеней свободы г и параметром нецентральности λ2 [1, с. 173]. Аналогичными асимптотическими свойствами к. о. п. обладает и при сложных гипотезах Я0 [1, с. 174—176]. § 1. Критерии согласия 3.1. Для данных задачи 1.13 проверить, согласуются ли они с гипотезой Н0 о том, что монета была симметричной. Уровень значимости положить равным: а) 0,05; б) 0,1. 3.2. По данным задачи 1.14 проверить гипотезу Но о случайности чисел. При каком уровне значимости гипотеза #о отвергается? 3.3. При η = 4000 независимых испытаний события А\, Ai% Л3, составляющие полную группу, осуществились соответственно 1905, 1015 и 1080 раз. Проверить, согласуются ли эти данные при уровне значимости 0,05 с гипотезой На: р\ = 1/2, р2 = р3 = 1/4, где р; = Р(Л,). 3.4. В десятичной записи числа π среди первых 10002 знаков после запятой цифры 0, 1, ..., 9 встречаются соответственно 968, 1026, 1021, 974, 1014, 1046, 1021, 970, 948, 1014 раз [6, с. 96]. Можно ли при уровне значимости 0,05 считать эти цифры случайными? При каком \ не значимости эта гипотеза отвергается? 89
3.5. Согласуются ли данные, приведенные в задачах 1.16 и 1.17, с гипотезой о симметричности костей? 3.6. Крупная партия товаров может содержать долю дефектных изделий. Поставщик полагает, что эта доля составляет 3 %, а покупатель — 10%. Условия поставки: если при проверке 20 случайным образом отобранных товаров обнаружено не более одного дефектного, то партия принимается на условиях поставщика, в противном случае — на условиях покупателя. Требуется определить: 1) каковы статистические гипотезы, статистика критерия, область ее значений, критическая область; 2) какое распределение имеет статистика критерия, в чем состоят ошибки первого и второго рода и каковы их вероятности. 3.7. Согласуются ли данные задачи 1.19 при уровне значимости 0, 1 с гипотезой Но о том, что показания часов равномерно распределены на интервале (0,12)? При каких значениях уровня значимости гипотеза Н0 не отклоняется? 3.8. В экспериментах с селекцией гороха Мендель наблюдал частоты различных видов семян, полученных при скрещивании растений с круглыми желтыми семенами и растений с морщинистыми зелеными семенами. Эти данные и значения теоретических вероятностей по теории наследственности приведены в следующей таблице: Семена Круглые и желтые Морщинистые и желтые Круглые и зеленые Морщинистые и зеленые Σ Частота 315 101 108 32 ;ι=556 Вероятность 9/16 3/16 3/16 1/16 1 Следует проверить гипотезу HQ о согласовании частотных данных с теоретическими вероятностями (на уровне значимости α^0,9). 3.9. Используя таблицу значений какой-либо функции (cosx, е", \пх и т.д.), записать 100 цифр, выбирая из каждого значения функции второй знак справа. Проверить для такой выборки гипотезу о случайности цифр 0, 1, ..., 9. Уровень значимости положить равным: а) 0,05; б) 0,01. 3.10. Группируя данные задачи 1.21 по N = 4 равновероятным (при гипотезе Н0) интервалам, проверить ги- 90
потезу На: F\{x) = 1 — е~х, х^О (уровень значимости принять равным 0, 1). 3.11. Пусть по выборке X = (Х\, ..., Хп) требуется проверить гипотезу об экспоненциальное™ распределения наблюдаемой случайной величины ξ, т. е. Но: F%{x) = = \ — е~'/0, х^О (параметр θ>0 неизвестен). Применяя метод группировки с интервалами Е,-= [(/— l)a,ja), j = 1, ..., Ν—\, Εν = [(Ν — 1)α,οο) , где α>0 — заданное число, построить критерий согласия I2 для гипотезы Н0. Проанализировать данные задачи 1.21 с этих позиций, принимая N = 3, а = 1. 3.12. В генетической модели Фишера [6, с. 79] принимается, что вероятности появления потомства, классифицируемого по четырем типам, имеют вид Р.(0) = -Ш-, Р2(0) = ρ3(θ) = -Ц^-, ρ4(θ) = -f- · 1Де Эе(0, 1) — неизвестный параметр. Как выглядит критерий X для проверки соответствия этой модели реальным данным? 3.13. При 8000 независимых испытаний события А, В и С, составляющие полную группу, осуществились 2014, 5012 и 974 раз соответственно. Верна ли при уровне значимости 0,05 гипотеза: р(А) = 0,5 — 2Θ, р(В) = 0,5 + + Θ, р(С) = θ (0<θ<0,25)? I Указание. См. решение задачи 3.12. 3.14. Для данных задачи 1.23 проверить гипотезу Но: Щ) = Π(θ), где θ — неизвестный параметр. Указание. В качестве оценки неизвестного параметра θ принять выборочное среднее [1,с. 117]. 3.15. Через равные промежутки времени в тонком слое раствора золота регистрировалось число частиц ξ золота, попавших в поле зрения микроскопа. По данным наблюдений, приведенных в следующей таблице: Число частиц т, 0 112 [ 168 2 130 3 68 4 32 5 5 6 1 7 1 Итого £т; = 518 проверить гипотезу Н0: Ζ,(ξ) = П(0), где θ — неизвестный параметр. 3.16. В таблице приведены числа т, участков равной площади 0,25 км2 южной части Лондона, на каждый из которых приходилось по i попаданий самолетов-снаря- 91
дов во время второй мировой войны. Проверить согласие опытных данных с законом распределения Пуассона, приняв за уровень значимости α = 0,05: i 111, 0 229 1 211 2 93 3 35 4 7 5 и более 1 Итого 2т,=576 3.17. Среди 2020 семей, имеющих двух детей, 527 семей, в которых два мальчика, и 476 — две девочки (в остальных 1017 семьях дети разного пола). Можно ли с уровнем значимости 0,05 считать, что количество мальчиков в семье с двумя детьми — биномиальная случайная величина? 3.18. Во время эпидемии гриппа среди 2000 контролируемых индивидуумов одно заболевание наблюдалось у 181 человека, дважды болели гриппом лишь 9 человек. У остальных 1810 человек заболевания не было. Согласуются ли при уровне значимости 0,05 эти данные с гипотезой, согласно которой число заболеваний отдельного индивидуума в течение эпидемии — биномиальная случайная величина? I Указание. См. решение задачи 3.17. 3.19*. Исследовать асимптотическое (при п-+оо) поведение среднего и^дисперсии статистики Х\ критерия согласия X2 при «близких» альтернативах вида НУ:р, = tf = Р? + Т-· / = 1. ···. Ν, Σ β/ = 0 Vn / = I I Указание. Использовать формулы для Е(у2\Р)н I D(Xl\p), приведенные в [1, с. 113]. 3.20*. Пусть μο = μο("ι Ν) — число пустых интервалов при группировке η наблюдений по N равновероятным (при гипотезе Но) интервалам. Рассмотрим гипотезы вида ^:p, = //f)=^{l+^)./=l.....^. где max |Ь,|<с<оо, £ Ь, = 0, b2(N) = ±Σ Ь]-^Ъ> > 0. 92
Доказать, что при п, Л/->-оо, —- = р>0 £(ц0|Мл)) = Ne-> +4^-bXN)p3/2e-> + 0{N"*), ΰ(μο|//ί") = Ne~\\ -e-p(l +ρ))(1 + 0(ЛГ"2)). Указание. Использовать формулы (3.16), приведенные в [1, с. 120]. 3.21. Поступающие в институт абитуриенты разбиты на два потока по 300 человек в каждом. Итоги экзамена по одному и тому же предмету на каждом потоке оказались следующими: на 1-м потоке баллы 2, 3, 4 и 5 получили соответственно 33, 43, 80 и 144 человека; соответствующие же данные для 2-го потока таковы: 39, 35, 72 и 154. Можно ли при уровне значимости 0,05 считать оба потока однородными? 3.22. Следующая таблица содержит данные о смертности среди матерей, родивших первого ребенка, в четыре различные периода времени [6, с. 102] (гс;- — число матерей, Vj — число смертных исходов) л/ v/ 1072 22 1133 23 2455 49 1995 33 Проверить гипотезу Н0 о том, что в уровнях смертности между этими периодами не существует различия. Указание. Применить критерий однородности χ2 для испытаний с двумя исходными. 3.23* Пусть произведены две серии из п{ и п2 независимых испытаний, в каждом из которых наблюдается либо исход А, либо исход А. Результаты сведены в следующую таблицу, в столбцах которой указано число реализаций соответствующих исходов для каждой серии: А А V (1) V|| «л V | = Л| (2) V|2 V,2 V 2=Л2 Σ V| v2 Л=Л|+Л2 93
1) Убедиться в том, что статистика Х\ для проверки гипотезы Но об однородности испытаний представима в виде Х\ = Z2n, где статистика zn={— —W nntn2 . \ "Ι «2 / V V| V2 2) Доказать, что L(Z„ | tf 01-hV(0, 1) при пи «2->-°о и, основываясь на этом, построить критерий проверки гипотезы //ο.'Ρι=ρ>2 против односторонней альтернативы Я, : р\>рг (здесь pL — вероятность реализации А в испытаниях ί'-й серии, i = 1, 2). 3.24. Пусть vi,...,vjv — независимые случайные величины, причем Ну) = Π(θ;), i = 1, ..., Ν, где параметры θ, неизвестны. Пусть дополнительно известно, что νι -\- ... + -\- vN = п. Построить при этом условии критерий для проверки гипотезы однородности Н0: θι = ... = θ^,. |У каза ние, Воспользоваться задачей 1,54, 3.25* (критерий пустых блоков.) Пусть Х = (Х\, ,.,, ,,,, Ха) — выборка из распределения L(Q = R(0,l), 0 = = А"(о,<^(1,<^(2)<,.,^А'(„,^А'(Л + 1, = 1 — ее вариационный ряд и Д = Cfy-u, Х(/)], i = 1, .,,, η + 1, — порождаемые ею выборочные блоки. Пусть, далее, У = (Υ\, ,,,, ,,,, Υ,„) — независимая от X выборка из некоторого другого распределения L(v) на отрезке [0,1], функция распределения которого F(x) имеет плотность f(x) = F (χ). Обозначим через κ,- = κ{η,ηϊ) число элементов выборки У, попавших в блок Д, i = 1, ..., η + 1. 1) Доказать, что при гипотезе однородности Но: £(ξ) =ϊ(η) вектор блоковых частот κ = (κι, ...,κη + ι) принимает все возможные значения с одинаковой вероятностью (Сп + т)~[; убедиться в том, что такой же вид имеет условное распределение Щ ξ„ + ι Ι ξι + ... + ξ„ + ι = tn), где случайные величины ξι £,l±j независимы и имеют геометрическое распределение Bl(\,p), где ре(0, 1) — произвольно. 2) Рассмотрев статистику So{n,m) — число пустых блоков: п + 1 s0(n,m) = 2 !(?« = °)· где К')—индикатор, i = I и используя вытекающие из п. 1 представление L(so(n,m)) =ί.("Σ'/(&-■ = 0)1 ξ, + ... + ξπ + , = m) , 4 i= I ' 94
доказать, что so(n,m) имеет гипергеометрическое распределение Н(п-\- 1, п-{-т, п)\ получить отсюда выражения для среднего и дисперсии статистики so(n,m) при гипотезе Но- 3) Доказать, что если /г,т->-оо так, что т/п = р>0, то L(s0(n,m)\Ho)~N(n/(\ + ρ), ηΡ2/(\ + ρ)3). 4) Доказать, что в предыдущих условиях для любой альтернативы #,, задаваемой плотностью f(x) φ Ι, хе[0,1) •■№«Н ах 1 1 + р/(*) 1 + Ρ Основываясь на этих результатах, обосновать критерий пустых блоков для проверки гипотезы однородности Я0 [1, с. 127]. Указания. 1) Воспользоваться тем, что условное распределение вектора κ = (у.ь ..., κη + ι) при фиксированных значениях (Х(1), .... Х(п)) = {хь ..., χ„) есть полиномиальное распределение М(т\ хи х2 — хи ..., ..., Хп Хп — 1, 1 Хп). Далее использовать задачи 1.39 п. 5)и 1.31. 2) Рассмотреть независимые ^случайные величины ξ. имеющие распределение Р(|,- = г) = Ρ(ξ, = г\Ь> >0), г = 1, 2, ..., i = 1, 2 и убедиться в том, что Р(1,- + ... + Is = т) = СпГ-\ qspm~s, q=\-p. 3) Воспользоваться нормальной аппроксимацией для биномиального распределения: при /г->-оо и О < ρ < 1, k = np + t^Jnp, q, \t\ < с < oo, ^J2nnpq Записать вероятность P(so(n,m) = k) в виде P(s0{n,m) =k) = b(k;n + \,p)X Xb{n — k;m^\,p)/b(n;n + m,p), p= 1/(1+p). 4) Вычислить Ε[/(κ, = 0)|Я,] = Р(х, = 0|Я,), используя при этом задачу 1.31. При оценке интеграла применить неравенство Коши — Буняковского: ι ι ι ( \ gi(x)g2(x)dx)2 ^ \ gKx)dx\gl(x)dx при g,(x) = Vl+P/W, ga(jf) = £i~'(4 95
3.26. Проверить гипотезу независимости для следующей таблицы сопряженности двух признаков (уровень значимости принять равным 0, 05): ξι ί!| а? аз Σ и 6| 3009 3047 2974 9030 ь. 2832 3051 3038 8921 6з 3008 2997 3018 9023 Σ 8849 9095 9030 26974 3.27. Из 300 абитуриентов, поступивших в институт 97 человек имели балл 5 в школе и 48 получили 5 на вступительных экзаменах по тому же предмету, причем только 18 человек имели 5 и в школе и на экзамене. С уровнем значимости 0,1 проверить гипотезу о независимости оценок 5 в школе и на экзамене. 3.28*. Рассмотреть таблицу 2 χ 2 сопряженности двух признаков Si 1 0 Σ и I Vii V2| V.| 0 V|2 V22 V.2 V V|. v2. IX 1) Убедиться в том, что статистика JCi [1, с. 131] для проверки гипотезы На о независимости признаков |, и |г в данном случае допускает представление %% = Z„, где Zn = "3/2(v|| Vfi^ ' )/ VV1 - V2. V. | V.2 = / V|| V,2 \ / >IV.|V.2 \ V.| V.2 J V V|.V2. 96
Ρ = 2) Показать, что выборочный коэффициент корреляции р„ = Zn/л/п и, следовательно, Zn/л/п^р = ко г г (ξι, £г) при п-+оо (см. задачу 1.38); установить равенства = Р(Лб)-Р(Л)Р(б) _ г А, _ (. . =,Г Р(б)Р(б) -11/2 где события Λ = {ξ, = Ι}, Α = {ξ,=0(, β = {ξ2=1}, В = {Ъ = 0}. 3) Доказать, что L{Zn | Я0)->-Л/(0, 1) при п->-оо, и, основываясь на этом, построить критерий проверки гипотезы Но против альтернативы Н\: Р{А\В)>Р(А\В), означающей положительную сопряженность событий А и В (А в паре с В встречается с большей вероятностью, чем в паре с В). 3.29. Имеются две группы данных о приеме в вуз, классифицированные_ по двум признакам: «принято (А) — не принято· (А)» и пол «мужчины (В)—женщины (В)». А А Σ В 97 263 360 В 40 42 82 Σ 137 305 п=442 А А Σ В 235 35 270 В 38 7 45 Σ 273 42 n=3l5 Для каждой таблицы проверить гипотезу Но о независимости признаков Л и β против альтернативы Н\\ Ρ(Λ|β)>Ρ(Λ|β). 3.30. В следующей таблице [3, с. 722] приведены 818 случаев, классифицированных по двум признакам: наличию прививки против холеры (признак А) и отсутствию заболевания (признак β): А А Σ В 276 473 749 В 3 66 69 Σ 279 539 818 Построить критерий проверки гипотезы Но о независимости признаков А и В против альтернативы Н\ о положи- 1—190 97
тельной сопряженности А к В (т. е. об эффективности вакцинации). 3.31. Можно ли с уровнем значимости 0,001 считать, что последовательность чисел 1,05; 1,12; 1,37; 1,50; 1,51; 1,73; 1,85; 1,98 является реализацией случайного вектора, все 8 компонент которого независимые одинаково распределенные случайные величины? 3.32*. Основываясь на следующем представлении производящей функции статистики Тп — числа инверсий в повторной случайной выборке объема η [I.e. 135]: ф„(2) = Ε/" =iff(l+H ... + Zr) , г = I доказать, mo L{Tn)~N( ~ -т^А при п-+<х>. Указание. Перейти к характеристической функции и убедиться в том, что при п->-оо и И^с<оо Eexp{/,(7„--4^iL) J_}^exp(-^/2(. 3.33. Проверить гипотезу случайности для данных задачи 1.22. Указание. Воспользоваться асимптотическим вариантом критерия, основанного на статистике Тп (см. предыдущую задачу). 3.34. Получить выборки, объемы которых п. = 20, 50, 100, равномерно распределенных случайных чисел. Проверить по этим выборкам гипотезу о равномерности распределения по критериям χ2 и Колмогорова. 3.35. Получить выборки объема п. = 100 приближенно нормально распределенных чисел, используя суммирование N равномерно распределенных слагаемых (Л/ = 4, 8, 12). Проверить гипотезу о нормальной распределенности при помощи критериев χ2 и Колмогорова. 3.36. Получить выборку (Χι) объема η = 200 равномерно распределенных случайных чисел. С помощью критерия Смирнова проверить, что подвыборки (Х%, t = 1, 2, ..., 100) и (Хц + \, Ί = 0, 1, .... 99) являются выборками из одного и того же распределения. 3.37. Смоделировать последовательность (А",} полиномиальных псевдослучайных величин, принимающих значения 1 N. Образовать из этой последовательности две выборки (X2i, i= \,...,п) и (A"2l + i, i = 0, ..., η—I). С помощью критерия χ2 проверить гипотезу независимо- 98
сти величин, соответствующих этим выборкам. Расчеты провести для Л/= 2, 4, 10, η = 100. 3.38. Получить выборки, объемы которых η = 10, 20, 40, 100, равномерных псевдослучайных чисел. С помощью статистик Тп (числа инверсий в вариационном ряду выборки) проверить гипотезу случайности. § 2. Выбор из двух простых гипотез 3.39. Пусть X = (ΑΊ, .... А"„) — выборка из биномиального распределения Bi(k; θ). Построить критерий Неймана — Пирсона для проверки гипотезы #о'. θ = θο против альтернативы Н\\ θ = θ, (0<θ0<θι<1) и вычислить его мощность. 3.40 (продолжение задачи 3.39). Показать, что при п->-оо этот критерий асимптотически задается критической областью η {Т^гкпво — илЛ/кпво(\-во)}, Τ == Σ */. Ф(«») = α , ί = Ι и его мощность №„(θ,) при θ, = #,"> = θ0 + -jl- , β>0, удовлетворяет соотношению Указание. Воспользоваться теоремой Муавра — Лапласа. 3.41. По выборке X = [Х\, .... Хп) из пуассоновского распределения Π(θ) построить критерий Неймана — Пирсона для проверки гипотезы Но: θ = θ0 против альтернативы Н\: θ = θ|(0<θο<θι) и вычислить его мощность. Исследовать асимптотическое поведение характеристик этого критерия при больших объемах выборки. Указание. Использовать задачу 1.39 п. 4) и нормальную аппроксимацию для пуассоновского распределения с растущим параметром. Рассмотреть «близкую» альтернативу такого вида, как и в задаче 3.40. 3.42. Чтобы проверить гипотезу Н0: θ = θο против альтернативы И\\ θ = θι (0<θο<θι<1) в схеме Бер- нулли с неизвестной вероятностью «успеха» Θ, осуществлен эксперимент, в котором наблюдается число «успехов», предшествующих первому «неуспеху». Построить наиболее мощный критерий при уровне значимости 4* 99
α = Θ6, где s^ 1 — заданное целое число, и убедиться в том, что вероятность ошибки второго ряда этого критерия β = 1 — θί. 3.43. Пусть X = (ΑΊ, .... Х„) — выборка из экспоненциального распределения Γ(θ, 1). Построить наиболее мощный критерий для проверки гипотезы Но: θ = θ0 против альтернативы Η\·. θ = θι и вычислить его функцию мощности. Указание. Воспользоваться тем, что Ζ.ο(2Χ,-/θ) = = lb) (см. задачу 1.51), и решением задачи 1.39 п. 2). 3.44. Пусть для распределения Коши, К(&) проверяется гипотеза Но: 0 = 0 против альтернативы Н\: 0 = 1. Показать, что при уровне значимости а = —— — arctg—«0,352 наиболее мощный критерий по одному наблюдению имеет вид χ'\α = [Х^ 1/2} и его мощность равна — Η arctg—«0,648. Если же а = = —(arctg3 — arctg l) «0,148, то критерий имеет вид Х*\а = {1^А"^3}, а мощность равна —arctg2«0,352. 3.45*. Как выглядит критерий проверки гипотезы Но'- Ц%) = R{ — ο,,α) против альтернативы Н\\ Ζ.(ξ) = N(0, о2) (параметры а и σ заданы), если известно, что наблюдаемая непрерывная случайная величина ξ имеет распределение, симметричное относительно нуля? Рассмотреть случай большой выборки. Провести с этих позиций анализ следующих данных: —0,460 —0,114 —0,325 +0,196 -0,174 при а = 1/2, σ2 = 0,09. Указание. Применить центральную предельную теорему 3.46. В последовательности независимых испытаний вероятности положительных исходов одинаковы и равны р. Построить критерий проверки гипотезы Но: ρ = 0 против альтернативы Н\\ ρ = 0,01 и определить наименьший объем выборки, при котором вероятности ошибок первого и второго рода не превышают 0,01. 3.47. Дана выборка X = (ΑΊ, ..., Л"„) из распределения N(0 σ2). Как выглядит наиболее мощный критерий для различения двух простых гипотез #о:0 = 0о и Н\ :θ = θι? Вычислить его мощность и убедиться в том, что он не- смещен. 100
3.48 (продолжение задачи 3.47). Определить минимальный объем выборки η* = η*(α, β), при котором вероятности ошибок первого и второго рода не превышают соответственно оси β. 3.49. Пусть X и Υ— выборочные средние двух выборок, объемы которых п. и т, из распределений N(Q, σ?) и JV(02, σ|) соответственно. Основываясь на статистике Τ ={Χ — Υ)/σ, где σ2 = σ?/η + ol/m, построить критерий проверки гипотезы Я0: Δ = θι — 02 = 0 против альтернативы Η\·. Δ > 0. Пусть заданы вероятности ошибок первого и второго рода α и β и объем η первой выборки. Определить минимальный объем т* второй выборки, необходимый для того, чтобы ошибочные заключения могли быть сделаны с вероятностями, не превосходящими α и β. Указание. Воспользоваться решениями задач 3.47 и 3.48. 3.50. По выборке объема η построить наиболее мощный критерий для различения двух простых гипотез относительно неизвестной дисперсии нормального распределения (среднее известно). Найти мощность критерия. 3.51.* Пусть по наблюдению X требуется различить два распределения с плотностями fdx) (гипотеза Но) и /ι(.ΐ) (гипотеза Н\). Рассмотрим критерий вида ВД = {х: /,(*) ^ cfQ(x)}, c>0, и пусть а(с) и β(ί) — соответствующие вероятности ошибок первого и второго рода. Показать, что: ' I — а(с) ^ ^ а(с) 2) а(с) + β(ί) ^ 1 — несмещенность; 3) min(a(c) + β(ί)) = α(1) + β(1), т. к. критерий, минимизирующий сумму вероятностей ошибок, есть ^ι(Ι); 4) пусть А" — повторная выборка объема п, т. е. X ={Х , Хп), fix) = Ylfiixt). / = 0, 1. Вероятности ;= ι ошибок критерия Х\{\) обозначим в этом случае ап и β„. Доказать, что если \{o(x)\n({\(x)/{o(x))dx = δ < 0, то σ,η, βπ->-0 при η ->- оо (таким образом, при бесконечно большой выборке возможно полное разделение гипотез Я0 и Я,). 101
У к а з а и и е. Записать Х\{\) = [х: Тп(х) = = — У In ('; > 0) и применить к статистике Тп(Х) закон больших чисел. Заметим также, что согласно неравенству Йспсена всегда 6^0 (1, с. 121]. 3.52.* Пусть ξ = (ξ , ξ,) — нормальный случайный вектор, имеющий при гипотезе Я, распределение Λ/(μ(,), A), i = 0, 1 (общая ковариационная матрица А предполагается невырожденной). Построить критерий Неймана — Пирсона для различения гипотезы Я0 при альтернативе Н\ по одному наблюдению над ξ, а также критерий, минимизирующий сумму вероятностей ошибок. § 3. Сложные гипотезы 3.53. Для биномиальной модели Bi(k, θ) построить р.п.м. критерий по выборке объема η для проверки гипотезы На: О ^ 00 против альтернативы Я,: θ > θ0. Указание. Воспользоваться свойством модели с монотонным отношением правдоподобия и решением задачи 3.39. 3.54. Убедиться в том, что построенный в задаче 3.41 критерий Неймана — Пирсона (для пуассоновской модели П(0)) является одновременно р.н.м. критерием для проверки гипотезы На'- 0 ^ 00 против альтернативы Яг. 0>00. | У к а з а и и е. См. решение задачи 3.53. 3.55. Пусть в схеме Бернулли с неизвестной вероятностью «успеха» 0 испытания продолжаются до получения г-го «неуспеха» и Тг — наблюдаемое число «успехов». Построить р.п.м. критерий проверки гипотезы Но: 0 ^ Оо против альтернативы Н\: 0 > Оо и показать, что при г —>- оо соответствующая критическая граница при уровне значимости α имеет вид ia = (г00 — ua-v/rOo)/( 1 — Оо), Ф(«*) = ос. Указание. Воспользоваться свойством модели с монотонным отношением правдоподобия, представлением Тг = Х\ +... + Хг, где Χι, .... Хг — независимые одинаково распределенные случайные величины и L(Xt) = βί'(Ι,Ο), и применить центральную предельную теорему. 3.56. Показать, что построенные в задаче 3.43 критерии являются р.н.м. критериями в задачах проверки сложных односторонних гипотез соответственно Но: θ < Оо против //,: 0 > Оо и Н0: 0 ^ 00 против Я,·. 0 < Оо. 102
3.57 (выборочный контроль). Пусть партия из N изделий содержит неизвестное число θ дефектных изделий, θε{0, 1, ..., Μ). Чтобы проверить гипотезу #0'. θ ^ θο против альтернативы Н\\ θ > θο, берут на контроль η изделий и каждое из них проверяют. Основываясь на статистике Τ — обнаруженное в выборке число дефектных изделий, построить р.н.м. критерий. Указание. Убедиться в том, что распределение статистики Τ (гипергеометрическое распределение #(θ, Ν, η)) имеет монотонное отношение правдоподобия. 3.58. Для нормальной модели Ν(θ, σ2) с неизвестным средним построить р.н.м. критерии для проверки гипотез Hq: θ ^ θο против Я,: θ > θο и На: θ ^ Θο против Н\: 0<θ0. Указание. Использовать решение задачи 3.47 и свойства экспоненциальной модели. 3.59. Убедиться в том, что построенный в задаче 3.50 критерий для случая θο > θι одновременно является р.н.м. критерием проверки сложной гипотезы Но: θ ^ θο при левосторонней альтернативе Ηι: θ < θο", аналогично, критерий для случая θο<θ| является р.н.м. критерием проверки гипотезы Но: θ^ θο при правосторонней альтернативе Н\: θ > θο. Указание. Воспользоваться свойствами экспоненциальной модели (см. п. 5 введения к гл. 3). 3.60.* Основываясь на задачах 3.47 и 3.58 и применяя прием объединения двух односторонних критических областей, построить несмещенный критерий для проверки гипотезы о среднем Но'- θ = θο против двусторонней альтернативы Н\\ θ Φ θο. Является ли этот критерий р.н.м. критерием? 3.61.* Пусть X = (ΑΊ, ..., Кп)— выборка из нормального распределения Ν(μ, θ2). Построить р.н.м. несмещенный критерий проверки простой гипотезы Но', θ = θο при двусторонней альтернативе Н\\ θ Φ θο. Указание. Применить теорему 4.5 [1, с. 159] об общем виде р.н.м. несмещенного критерия и использовать решение задачи 3.50. 3.62. По выборке объема η из распределения Γ(θ, 1) построить р.н.м. несмещенный критерий для проверки гипотезы Но: θ = θο против альтернативы Н\: θ Φ θο. Указание. Использовать решение задач 3.43 и 3.61. 3.63. По выборке большого объема η построить ло- 103
кальный наиболее мощный критерий проверки гипотезы Но: θ = θο против общей альтернативы Н\\ θ Φ θο для модели Bi(k, θ). Показать, что его функция мощности Wn(Q) при уровне значимости а и локальных альтернативах вида θ = θ(η) = θο + β/V" удовлетворяет предельному соотношению "-» V νθο(Ι-θο) W Λ V6o(l - θο) / Указание. Воспользоваться общим видом асимптотического (при больших п) двустороннего критерия для регулярных моделей; Х1а = {\Щх; θο)| ^-ιια/2Λ/ηί(β0)\, где U(x; θ) — функция вклада выборки X = = (Х\, .... Хп) и ϊ(θ) — функция информации Фишера [1, с. 162]. Использовать решения задач 3.39 и 3.40. 3.64. По выборке большого объема η построить локальный наиболее мощный критерий проверки гипотезы #о: θ = θο против альтернативы Н\\ θ φ θο для модели Π(θ). Показать, что его функция мощности Wn(B) при уровне значимости а и локальных альтернативах вида θ = θ(η) = θο + β/V" удовлетврряет предельному соотношению lim №„(θ<">) = ф(- JL· + цЛ + ф(-£-+ „Λ. I Указание. Использовать указание к задаче 3.63 I и решение задачи 3.41. § 4. Проверка гипотез и доверительное оценивание В задачах 3.65—3.72 воспользоваться принципом соответствия между задачами доверительного оценивания и проверки гипотез (см. п. 6 введения к гл. 3). 3.65. Используя построенные в задачах 2.119 — 2.120 доверительные интервалы для параметров θ| и θ2 нормальной модели N(Q[,Ql), построить критерии проверки нулевой гипотезы Я0 против альтернативы Н\ в следующих задачах: У) Но: θι = θю, Hi: Qf > 0ю; 2) Ho'- θι = θ ίο, Η\\ θι < θιο; 3) Я0: θ, = θ,ο, Η ι: θ, φ θ,0; 4) Я0: θ2 = 020, Η,: θ2 > θ20; 5) Но: 02 = θ20, Я,: θ2 < θ20; 6) Η0: θ2 = θ20, Яг. θ2 Φ θ20. 104
3.66. Используя решения задач 2.122—2.123, убедиться в том, что критерий уровня значимости а для гипотезы о равенстве средних двух нормальных моделей в случае известных дисперсий имеет вид *ΐα = {(*, у): \х — у\ ^ "ι -α/2Λ/σι/" + °l/rn), а если дисперсии неизвестны — следующий вид: Хи = {(х,уУ- \х — у\ ^ ^ /, _а/гп+т_0-у—П-±-Т: („52(х) + mS2(y))}. ν пт(п -+- т — 2) ν ν / ' ^"' 3.67. Основываясь на решении задачи 2.125, показать, что для проверки гипотезы о равенстве дисперсий двух нормальных моделей можно использовать критерий γ, _ [(г ,л . "("» - ') s\x) <· ρ либо i(n-i) S2(ji) ^- f.„-i.»,-i, н(т - 1) S*(x) F 1 ^ 1 _ « ,,,-i.m-lJ· m(n-l) S2(y) 3.68. В условиях задачи 2.127 построить критерий проверки гипотезы однородности Н0: τ = Θ2/Θ1 = 1 (т. е. θι = Θ2) и вычислить его функцию мощности. 3.69. Основываясь на доверительном интервале задачи 2.128, построить критерий проверки гипотезы Но: θ = θο для соответствующей модели. Вычислить функцию мощности этого критерия и убедиться в том, что он несмещенный. 3.70. Используя результаты задачи 2.129, построить критерий проверки гипотезы Но', θ = θο для равномерного распределения R(0,Q)l вычислить его функцию мощности и убедиться в том, что он несмещенный. 3.71. Основываясь на результате задачи 2.130, убедиться в том, что критерий проверки гипотезы Н0: θ = θο для модели Вейбулла №(0, λ, Θ) имеет вид Х\а = [Τ ^ γ χϋ,.ί«} []{τ^ у χ?-α,.ί«}, α, +«2 = ос Чтобы получить несмещенный критерий, величины χ|,,ίη и χ?-ο,.2η выбираются так же, как в задаче 3.62. 3.72. Используя условие задачи 2.131 и ее результат, построить критерий проверки гипотезы Hq: (0i,Gj) = = (θιο, θκο). 105
§ 5. Критерий отношения правдоподобия (к.о.п.) 3.73.* Получить форму к.о.п. для гипотезы о среднем Но: θι = θ,ο нормальний модели N(QU θ2.) и показать, что для больших выборок он имеет вид Χία = [χ: л/п— 1 \х — θ|0Ι/5(χ)^ —«=/21, а его мощность при альтернативе θ^η) = θιο + β/V" ПРИ η ->- оо равна 1 — F^ut/?, β2/θ!Ϊ)(ΰΜ. π. 7 введения к гл. 3). Указание. Использовать задачи 1.47 и 2.44 и асимптотическую теорию к.о.п. [1, с. 175]. 3.74.* Убедиться в том, что к.о.п. для гипотезы о дисперсии #о: Θ2 = 020 нормальной модели N(QU θ?>) имеет вид Х1а = {η52(χ)/Θ22ο^χ«2,.η-,} U И2(х)/в220 > χ?-α,„-,}, где αϊ + ос2 = α и S2 — выборочная дисперсия для выборки объема п. Вычислить функцию мощности этого критерия и определить, при каких значениях αϊ и аг он будет несмещенным. Указание. Воспользоваться тем, что Lo(nS°(X)/Q2) = = Х2(" — 1). и решением задачи 3.61. 3.75. Построить к.о.п. для гипотезы Но: θ = θο в модели βί(Ι,θ) и убедиться в том, что его асимптотический (при больших объемах выборки) вариант совпадает с локальным наиболее мощным критерием, построенным в задаче 3.63 (при значении параметра k = 1). Указание. Воспользоваться общей теорией к.о.п. для полиномиального распределения [1, с. 170— 171]. 3.76. Построить к.о.п. для гипотезы Но'- θ = θο в модели Π(θ) и убедиться в том, что его асимптотический вариант для больших выборок совпадает с локальным наиболее мощным критерием, построенным в задаче 3.64. Указание. Воспользоваться тем, что при η ->- оо предельные распределения при гипотезе Но статистик— 21πλ„ и Q(„2) = Un(Qo)/ni(Qo) совпадают [1, с. 169. J.77.* Пусть Χ,, ..., Xk — выборочные средние для независимых выборок объемов л,, ..., л* из совокупностей βί(Ι,θ,), ..., βί'(1,θ*) соответственно. Построить и рассчитать асимптотический (при /г,, ..., rik-*-оо) вариант к.о.п. для гипотезы однородности Я0: θι = ... = θ*. Убедиться 106
в том, что этот критерий имеет такой же вид, как и критерий однородности χ2 [1, с. 124—126]. | У к а з а н и е. Использовать решение задачи 3.75. 3.78.* Пусть X; = (Хц, .... ;Ц), 7 = 1. .... к, — независимые выборки из совокупностей Π(θι), ..., П(0*) соответственно. Построить и рассчитать асимптотический (при πι, ..., л*->-оо) вариант к.о.п. для гипотезы однородности Но', θι = ... = θ*. Проанализировать с этих позиций следующие данные: выборочные суммы для четырех выборок, объемы которых 120, 100, 100 и 125, из пуассо- новских совокупностей оказались равными соответственно 251, 323, 180 и 426. Одинаковы ли генеральные средние? 3.79.* Пусть л/, X; и S/ — соответственно объем, выборочные среднее и дисперсия для выборки из совокуп ности JV(Qi/, θ!), J = 1, ..., k (выборки предполагаются независимыми). Построить к.о.п. для гипотезы однородности Но: θιι = ... = θι*. Убедиться в том, что в случае двух выборок (k = 2) этот критерий имеет вид (ср. с за дачей 3.66) Х\а = {\Т[ ^ /|-0/г.л,+л>-г}, где Т /у νη / Я|Яа(Я| +"г — 2) 1 - (А, - Хг) γ (n|+„2Xn|S?.+ n,s5) · Указание. Использовать задачи 2.86 и 2.114, а также утверждение, что L(T\Hq) = S(n — 2) [1, теорема 1.12]. 3.80.* Пусть S}, .... Si—выборочные дисперсии, построенные по независимым выборкам объемов п{, ..., я* из совокупностей ΛΛ(Θ|;, ΘΙ;), /=1. ■··. k, соответственно. Построить к.о.п. для гипотезы о равенстве дисперсий Но: θ2ι = ... = Θςα. Убедиться в том, что в случае двух выборок (k = 2) этот критерий имеет (ср. с задачей 3.67) вид Х\а = {F ^ Ρα,,η, - 1.л2 - I) U [F ^ F\ - сц.л, - 1.Я, - I ]. где ос, + «2 = a, F = [п\(щ — l)S?]/[n2(«! — l)Sj]. Указание. Использовать решение задачи 3.79, а также утверждение L(F\Hos = S(n( — 1, л2 — 1) [1, теорема 1.13]. § 6. Разные задачи 3.8J. Наблюдаемые случайные величины X , Хп независимы и нормальны, но, вообще говоря, имеют разное распределение. Требуется проверить гипотезу Но о том, что они одинаково распределены. Используя задачу 1.58, убедиться в том, что соответствующая критическая об- 107
ласть при уровне значимости а может быть задана в виде Χίο. = {ΙηΙ > ^α), где иа определяется через функцию бета-распределения с помощью соотношения В ( 1 — vl; η — 2 1 \ , ^ —5—, -s-j = ос (для нахождения να могут быть использованы таблицы бета-распределения β(—-j—. "о-))· 3.82. Пусть Χι = (Хц, Xi2), г = 1, ..., п, — независимые наблюдения над двумерной случайной величиной ξ = = (h,L·), имеющей нормальное распределение с неизвестными параметрами, и р„ — построенный по этим данным выборочный коэффициент корреляции. Основываясь на результатах задачи 1.59, убедиться в том, что критическая область х.. =[|р«|>-7=<|:-/'Г ) задает критерий уровня значимости а для гипотезы Но о независимости компонент ξ, и |г. 3.83. Пусть наблюдаемые случайные величины ΛΊ, ..., Хп независимы и L(Xi) = П(0;), i = 1, ..., п. Требуется проверить гипотезу однородности Hq: Θι = ... = θ„. Основываясь на результате задачи 1.60, убедиться в том, что при больших η можно использовать критерий Х\л = = [\Тп\> -Ua/ϋ). 3.84. Какой критерий согласия может быть построен на основе результата задачи 1.61? 3.85.* (Асимптотическая эффективность критериев). Рассмотрим задачу проверки простой гипотезы Hq: 0 = Θο против альтернативы ΙΙλ: 0 > Go для некоторой модели со скалярным параметром θεθ, где Θ — некоторый интервал действительной оси. Предположим, что в данной задаче используются критерии вида Х\ = [Тп ^ у,,}, где Тп — некоторая статистика для выборки объема п, обладающая следующими свойствами: а) существуют функции μ(θ) и σ(0) > 0 такие, что равномерно по θ в интервале θο ^ 0 <: θο + Ά> гДе η > 0 — любое число, при η -*- оо h{T„) ~ ΛΤ(μ(θ), а2(0)/гс); б) μ(0) дифференцируема в точке θ0 и μ'(θ0) > 0, а σ(0) непрерывна в θο. Доказать, что: 1) критическая граница γ„ при уровне значимости а асимптотически имеет вид Υ« = μ(θο)— и^воУл/п; 108
2) при близких альтернативах вида θ(η) = θο + β/V". Ρ > 0, мощность Wn($n)) критерия удовлетворяет предельному соотношению β(β, а)= lfm ΙΡ„(θ(η)) = Φ(βμ'(θο)/σ(θ0) + "*)■ Замечание. Величина е = еф, ос) называется эффективностью Питмена критерия ΧίΛ = {Τη ^ 32= μ(θ0) — «ασ(θ0)/^/π} и ее часто используют в качестве меры сравнения различных критериев: мера е дает представление о локальном поведении кривой мощности критерия в окрестности точки θ0 для больших выборок. 3.86* (продолжение задачи 3.85). Пусть Т^, ) = = 1, 2,— две статистики, удовлетворяющие сформулированным условиям; соответствующие им характеристики будем отмечать индексом ]. Предположим, что для каждого η существует целое Νη такое, что Я№о + β/лАО = ^Лео + β/лЯ т. е. мощности соответствующих критериев при альтернативе θ(η) равны, если η — объем выборки в первом случае и Νη — во втором. Пусть также Νη^>-οο при п-*оо. Доказать, что »-.»«/, V σ2(00) / ' \ σ,(Οο) / e\ где е' = (μ'(θο)/σ(θ0))2. Замечание. Величина е', являющаяся возрастающей функцией эффективности Питмена еф, а) при фиксированных β и ос, также может служить мерой асимптотической эффективности критерия. Сформулированное утверждение означает, что относительная эффективность второго критерия по отношению к первому равна пределу обратного отношения выборочных объемов, необходимых для того, чтобы достичь одинаковой мощности при указанных альтернативах θ(η). 3.87. Пусть требуется проверить гипотезу Я0: θ = θ0 против альтернативы Ηι: θ> θο для нормальной модели Ν(θ, σ2). Построить критерии вида Х\ = {Τη ^ γ„}, основываясь на статистиках V-P = X — выборочное среднее и 7l2) = 2„. |/2 — выборочная медиана, и показать, что относительная эффективность второго критерия по отношению к первому λ = 2/π = 0,637... 109
Указание. Воспользоваться решениями задач 3.86, 3.85 и 1.32. 3.88.* Пусть наблюдается случайный вектор X = = (Χι, ..., Х„), имеющий распределение Lo(X) = = N(0t, Σ = ||σ,;||"), где 0— неизвестный скалярный параметр, t = (t , /„), 0 < t\ < /2 < ·■· < tn — известные константы и σ1;- = //, i^j. (Если η(/), / ^ О,— винеров- ский процесс [2, с. 221], т. е. однородный случайный процесс с независимыми приращениями, причем L(r\(t)) = = N(Qt, t), то Xi = η(/;), i = 1, .., п, т. е. X — наблюдения над η(/) в моменты / /„.) Убедиться в том, что достаточной статистикой для θ является последнее наблюдение Хп и, основываясь на этом, построить критерии проверки гипотезы На: θ = О, т. е. гипотезы об отсутствии систематического тренда (смещения, сноса) (рассмотреть альтернативы #Г: θ>0, ЯГ: Θ<0η//,:Θ^0). Указания. 1. Воспользоваться критерием факторизации, установив при этом равенство ίΣ = = (0...01). 2. Использовать решения задач 3.47, 3.58 и 3.60. Глава 4 ЛИНЕЙНАЯ РЕГРЕССИЯ И МЕТОД НАИМЕНЬШИХ КВАДРАТОВ 1. Под линейной регрессионной моделью понимают такую ситуацию, когда наблюдаемые случайные величины X,, ..., Хп «в среднем» линейно зависят от некоторых неслучайных факторов Ζ\, ..., 2* (k<n), значения которых могут меняться от опыта к опыту. В этом случае исходные статистические данные состоят из множества наблюдавшихся значений «откликов» Хь ..., Хп и соответствующих значений факторов, т. е. имеют вид (χ,; ζν, ..., 2^), i = 1, ..., п, и считается, что ЕХ; = Σ#β/= ζ^'β,^ = (ζΡ, .... z<i\ где β = (βι, ..., β4) — совокупность неизвестных параметров, называемых коэффициентами регрессии. Если ввести случайные величины ε, = X,- — ζ(,)β, называемые «ошибками» измерений, и матрицу плана Ζ — ||2(1)...2(л)|| размером kX.n, то в матричных обозначениях модель записывается в виде X = Ζ'β + ε, X = (X,, .... Х„), ε = (ε,, ..., ε„). (4.1) НО
Здесь Ε (ε) = О и обычно предполагается, что наблюдаемые случайные величины некоррелированы и имеют одинаковые дисперсии, т. е. матрица вторых моментов вектора наблюдений X имеет вид Ό(Χ) = D(e) = Εεε' = σ2Εη. (4.2) Величина σ2 называется остаточной дисперсией и обычно также неизвестна. Если неслучайные переменные имеют вид ζ, = α//), где а//) — полином, то говорят о параболической регрессии. В приложениях часто параметр k = 2, а векторы z(i) имеют вид z(,) = (1, /,-), т. е. в данном случае ЕХ| = β, + + β2^ί, ί = 1, ···, п. (среднее значение наблюдений является линейной функцией одного фактора /). Этот случай называют простой регрессией, прямую φ(/) = βι + βί/ — линией регрессии, а коэффициент β2 — ее наклоном. В ряде задач регрессионного анализа делаются дополнительные предположения о виде распределения «ошибок» ε и часто считается, что они нормально распределены, т. е. Ζ,(ε) = N(0, σ2Εη). В этом случае модель имеет вид ЦХ) = Ν(Ζ'β, σ2Εη) (4.3) и ее называют нормальной регрессией. К. модели линейной регрессии сводятся многие задачи прикладных исследований, в которых речь идет об опре делении вектора неизвестных параметров β = (β , β*), причем обычно можно измерить лишь некоторые функции от этих параметров, а прямое их измерение невозможно. К задачам такого типа относятся, в частности, задачи восстановления функциональной зависимости. В этом случае неизвестными параметрами являются коэффици енты разложения восстанавливаемой функции по какой- либо системе функций. 2. Основными задачами регрессионного анализа являются задачи оценивания неизвестных параметров β = = (pi β«) и σ2 модели (4.1) — (4.2), а в случае нормальной регрессии (4.3) — их доверительное оценивание и проверка гипотез о параметрах. Основным методом построения оценок для коэффициентов регрессии β является метод наименьших квадратов, в соответствии с которым оценки этих параметров находят из условия обращения в минимум квадратичной формы: ш
5(β) = S(X; β) = (Χ - Z'W(X - Ζ'β). (4.4) Точку β = (β,, ..., β*), удовлетворяющую равенству 5(ρ) = Γηίπ5(β), называют оценкой наименьших квадра- β тов (о.н.к.) параметра β. Определяющую роль в решении этих задач играет матрица А = ΖΖ'. В дальнейшем считается, что эта матрица невырождена (или, что эквивалентно, rangZ = = k). При этом предположении о.н.к. единственна, определяется нормальным уравнением Αβ = Υ = ΖΧ и имеет вид β = Α~ιΥ = Α~ιΖΧ. При этом оценка $ является несмещенной (Εβ = β) с минимальной дисперсией (т. е. дисперсии всех компонент вектора β минимальны) в классе всех линейных (т. е. линейно зависящих от наблюдений X) несмещенных оценок β. Более того, такими же свойствами обладает и любая линейная функция t = Γβ как оценка параметра t = Γβ (здесь Τ — заданная матрица некоторого размера rnXk); при этом D(?) = = σ°ΤΑ-ιΓ, в частности ϋ(β) = σ2Α_Ι Несмещенной оценкой остаточной дисперсии σ2 является [1, с. 181 —184) статистика σ2 = —[—г 5(β) = —1—■Х'ВХ, В = Εη-ΖΆ-'Ζ. (4.5) В задачах интерполяции, когда для неизвестной функции χ = f(t), связывающей переменные t и х, по измерениям (/,·, Χι = xt + εΐ), χι = /(/,■), ί = 1, ..., η, подбирают интерполяционный многочлен вида φ('; β) = Σ β,-α,<0. /=Ι где в качестве a^t), a^t), ... используют ортогональные многочлены Чебышева, о.н.к. неизвестных коэффициентов β, вычисляют по формулам h= 4г1,ада'''а'= Sia'{t,)·'= 1'2,"" (4.6) η η при этом величина 5(β) = 2 X? — Σ α/Ρ/ характеризует ί=Ι /=Ι точность приближения; первые три многочлена Чебышева имеют вид: а,(0 = 1, а2(0 =t-t, а3(0 = (t - T)(t - *"- -J) - s2. иг
где t = -i-2 U, sk = s*(<, in) = 4~Σ (t, - t)k ['!, с 189—191]'Г' Метод наименьших квадратов применяют также в случае, когда зависимость ЕХ; от β не является линейной. Пусть Xi = KU, β, β*) + ε„ t = 1, ..., η, где Εε,- = 0, Oek = σ2, cov(e„ ε,) = 0 (ί φ\)· Тогда о.η.κ. β параметров β минимизируют по β выражение Q№=i(Xi-f(ti, β,. ..., β*))2. 1=1 Таким образом о.н.к. р являются решением системы уравнений -«- = 0 ί - 1 к Приведем пример вычисления оценок параметров β. Пусть требуется определить неизвестные коэффициенты (βι, β2, ββ) функциональной зависимости *(0 = βΐ + Ф2 + t3fr. Будем предполагать, что в точках U = 2+—, ί = = 1, ..., η, измерены значения функции *(/). Ошибки измерений ε„ ί = 1 η, будем считать независимыми и нормально распределенными с Εε, = 0, ϋε, = σ2. В этом случае имеем линейную модель Xt = x(U) + ε,- = β, + /,β2 + <?β3 + ε„ ί = 1, ..., η, и оценки βι, β2, Рз удовлетворяют системе уравнений i(Xi-b\-Ubi-i%) = О, i(Xt-bi-tibi-tibi)ti = 0. (4.7) Σ {Χ, - β, - <& - <?&)<? = ο. ί=1 Пусть βι = 3, β2 = — 1, β3 = 1, σ2 = 0,04. Смоделируем ε, в случае η = 25, η = 100 и найдем соответствующие Χι. Из системы (4.7) находим: 113
1) η = 25 β = 2,983; % = —0,828; β3= 0,895; σ2 = 0,034. 2) η = 100 β,= 2,992; β2 = -1,007; β3 = 1,001; σ2 = 0,046. На рис. 5 и 6, относящихся к случаям η = 25 и п. = 100 соответственно, приведен график точной функциональной зависимости x(t) =■ βι + fat + β3^3, знаком О отмечены результаты измерений (ίι,^Χ,), а также приведены графики функций x(t) = β, -j- %t -\- β3/3. 3. Для схемы нормальной регрессии (4.3) о.н.к. β совпадают с оценками максимального правдоподобия (о.м.п.) параметров β. В этом случае γ-доверительный интервал для параметра β,- имеет вид где α" — /-й диагональный элемент матрицы А~\ а для остаточной дисперсии 5(Ρ)/χ2φ.η_Λ < °2 < 5(β)/'^.„_;· (4.9) γ-Доверительная область для вектора t = 7"β, где Τ — заданная матрица размера mXk и rangT = т, строится по формуле Gy(X) = [t:(T$ - t)'D~\Tb - 0 <-i3pS(&)A,v.«..-i}. (4.10) где D = ТА~1Г [1, с. 194—196]. Если требуется одновременно оценить некоторые линейные комбинации параметров β, τ. е. величины λίβ, г = 1, .... ш, где Хг — заданные векторы, то система совместных доверительных интервалов с доверительным уровнем, большим или равным у, имеет вид λίβ - и,(Х; λή < λίβ < КЬ + "ν№ Μ. г = 1, ..., т, (4.П) где «ν(Χ;λ) = [-^τ5(β)^ν.^_*(λ'Α-,λ)]'/2 [1, с. 198]. Наконец, для проверки линейной гипотезы вида Н0: βεΒ0 = {β: Γβ = to], где Т — заданная матрица коэффициентов ограничений размера mXk и rang Г = m, t0 — заданный вектор, используют F-критерий с критической областью вида 114
Рис. 5 где 5т = min 5(β) — условный (при гипотезе Н0) ми- β : Γβ = la нимум 5(β) [1, с. 199—200]. 4.1. Для линейной модели (4.1) в случае k = 2 записать явно выражения для о.н.к. (β\, β2) через (Хи ..., Хя) и ζ(') = (ζ(;), ζ«), ί = 1, ..., η. 4.2. Для модели простой регрессии Χι = β. + βϊ/.- + εί, ί = 1, ..., /ζ, найти явный вид о.н.к. (β\, β2), проверить их несмещенность и найти условие их состоятельности. 4.3. Вычислить оценку σ2 (см. (4.5)) остаточной дисперсии σ2 в задаче 4.2. Указать достаточное условие ее состоятельности. 4.4. Найти cov(|3i, $2) оценок β,, β2, определенных в задаче 4.2. 4.5. Значения функции x(t) = β, + β2/ + $zt2 измерены в точках ti (i = 1, ..., η): Χ, = β, + β2ί, + β3<? + ε,; Εε, = 0, De, = σ2. Найти: 1) о.н.к. βΙ; β2, ββ параметров βι, β2, β3; 2) Εβ„ D&, i = 1, 2, 3; cov(fr-, 3/)- 4.6. Является ли статистика ь T=\x{t)dt 115
О -0.5 0,0 OS W ϊ Рис. 6 ъ несмещенной оценкой интеграла / = \x(t)dt, где x(t) = а = βι + М + рУ2,*. a x(t), β,, β2, β3 определены в задаче 4.5? Найти D/. 4.7. Смоделировать наблюдения Л", = βι+β2</ + εί, ί = 1, ..., η, если η = 100, U = 2i/n, β, = 2, β2 = 1, ε,-— независимые равномерно распределенные случайные величины на отрезке [—1,386; 1.386J. Построить графики функций x(t) = 2+t, x(t) = β\ + & на отрезке [0, 2]; отметить также точки (/,-, А",), / = 1, ..., п. 4.8. Решить задачу 4.7 с ε,- распределенными нормально с Εε,- = 0, ϋε,- = 0,16. 4.9. В предыдущей задаче построить γ-доверитель- ные интервалы для параметров βι, ffo и σ2 (см. (4.8) — (4.9)), а также γ-доверительный эллипс Gy(X) (см. (4.10)) для вектора β = (β,, β2); считать у = 0,9 и у = 0,95. | Указание. Использовать решения задач 4.2—4.3. 4.10. По данным независимых равноточных измерений (X, /,-), i = 1, ..., η, значений некоторой линейной функции x(t) = β, + β2ί (погрешности измерений подчиняются нормальному распределению N(0, а2) с неизвестной дисперсией) построить доверительный интервал для интеграла от этой функции на отрезке —a^t^a (а задано). 116
Произвести соответствующие вычисления для следующих данных: (2,96; -2), (3,20; —1.), (3,41; 0), (3,63; 1), (3,79; 2) при а = 2 и доверительном уровне у = 0,95. 4.11. Координаты a{t) движущейся равномерно и прямолинейно точки в моменты t = 1, 2, 3, 4, 5 оказались соответственно равны 12,98; 13,05; 13,32; 14,22; 13,97. Предполагая погрешности измерений независимыми и нормальными N(0, о2), построить 0,95-доверительный эллипс для точки (а(0), ν), где и — скорость точки. 4.12. Смоделировать наблюдения X,■ = βι + faU + + β3/2 + ε„ i = 1, ..., η, с β, = -8, β2 = 10, β3 = —2, η = 100, U = 1 + 2i/n, ε,-независимые равномерно распределенные случайные величины на отрезке [—1,386; 1,386]. Построить^графики функций x(t) = βι + fat + fat2, *(0 = βι + Μ + fat2 на отрезке [1, 3J; отметить также точки (d, Xi), ί = 1, ..., η. 4.13. Решить задачу 4.12 в случае нормально распределенных ε, с Εε,- = 0, ϋε, = 0,16. 4.14. В предыдущей задаче построить v-доверитель- ные интервалы для параметров βι, fa, fa и σ (см. (4.8) — (4.9)), а также систему совместных доверительных интервалов уровня, большего или равного у, для βι, fa, fa (см. (4.11)). I Указание. Использовать решение задачи 4.5. 4.15. В четырехугольнике ABCD результаты независимых и равноточных измерений углов ABD, DBC, ABC, BCD, CDB, В DA, CD А и DAB (в градусах) соответственно таковы: 50,78; 30,25; 78,29; 99,57; 50,42; 40,59; 88,87; 89,86. Считая, что ошибки измерений распределены нормально N(0, а2), найти о.н.к. углов βι = ABD, β2 = DBC, fa = CDB и β4 = BDA. Построить 0,95-доверительный интервал для σ2. 4.16*. Доказать, что о.н.к. β является оптимальной оценкой β в классе всех линейных (т. е. линейно зависящих от X) несмещенных оценок β (т. е. дисперсии Οβ, минимальны V0; получить, что Όφ) = а А~х = σ2||α''||· 4.17. Доказать несмещенность оценки σ2 для остаточной дисперсии а2. Получить явную зависимость σ2 от X, указанную в формуле (4.5). Указание. Использовать разложение S(£) = = 5{β) + (β - β)Ά$ - β) и формулы covtff, β;) = = σα'1 (задача 4.16). 4.18. Пусть матрица плана Ζ обладает свойством, что ее строки ортогональны. Как выглядят в этом случае о.н.к. βι, ..., β* и их вторые моменты? 117
4.19.* Пусть имеются к предметов, веса которых βι, ..., β* неизвестны. Для определения этих весов взвешивают комбинации предметов: каждая операция (одно взвешивание) состоит в том, что несколько предметов кладут на одну чашу весов, несколько — на другую и добавляют равновес для приведения весов в равновесие. В результате получают соотношения z\:%+... + z^k = у, (для ί'-го взвешивания, i = 1, ..., η), где ζ^ — 1, —1, О в зависимости от того, лежит /-Й предмет на левой чаше весов, на правой или вообще не участвует в данном взвешивании, а у, — добавляемый равновес. Считая погрешности измерений независимыми и нормальными N(0, а2), оценить веса четырех предметов по данным следующей таблицы восьми взвешиваний: βι β2 β> β4 Вес 1 1 1 20,2 1 -1 -1 8,1 1 1 -1 9,7 1 -1 ι 1 1,9 1 1 ι 1 19,9 1 -1 ι -1 8,3 1 1 -1 10,2 1 -1 1 1,8 Найти матрицу ковариаций оценок, а также оценку для σ2. Сравнить точность этих оценок с точностью оценок, получаемых обычным способом, когда каждый предмет взвешивают несколько раз и в качестве оценки его веса принимают среднее арифметическое результатов взвешиваний. Указание. Использовать решение предыдущей задачи. 4.20. Для данных предыдущей задачи построить систему совместных доверительных интервалов для β,, ..., β4 уровня, большего или равного 0,95. 4.21. Найти оценки максимального правдоподобия параметров β и σ2 нормальной регрессии (4.3) и вычислить их смещения. 4.22. Убедиться в том, что γ-доверительный интервал для произвольной линейной комбинации λ'β = 2 λ/β,- κο- эффициентов нормальной регрессии (4.3) имеет вид (λ'β± ь +v)/2,,-*VτζΓΓ5(β) λ'Α~'λ)· 118
4.23. Построить γ-доверительный интервал для ординаты φ(/) = βι + β2^ линии регрессии в произвольной точке / (модель предполагается нормальной). Произвести соответствующие вычисления для данных задачи 4.8 для t = 1,5, у = 0,95. |Указание. Использовать решения задач 4.2—4.4 | и 4.22. 4.24. Убедиться в том, что интервалы ( & Г" & ±[^S(frf,*.„_*(a''- 2а'' + a'7)]"2). образуют систему совместных доверительных интервалов уровня >у для разностей β,— β„ i> j. 4.25. Построить систему совместных доверительных интервалов для средних значений всех наблюдений Х\, ..., Хп в модели нормальной регрессии. 4.26.* Пусть Г— заданная матрица размера mXk (m ^ k) и rangr=m, ta — заданный m-мерный вектор такой, что система Γβ = t0 совместна. Обозначим St = = mm S(B) и назовем обобщенной о.н.к. Вт то значе- β. гр = /о А ние β, при котором St = 5(βτ). Доказать, что βτ= β-A-TD-1 = (Γβ - ίο), где матрица D = ТА~]Т' положительно определена. Найти разложение ST = 5(β) + (Γβ - U)'D- '(Γβ - t0). 4.27. Убедиться в том, что критерий уровня значимости а для проверки гипотезы Я0: β2 = β2ο, фиксирующей значение наклона линии регрессии (в нормальной модели), задается критической областью *■- = {ΐβ2-β2οΙ ^ ^_а/2,;1_2л/5(Э)/[(п — 2)Σ (^ — О2]}. Указание. Использовать решение задач 4.2—4.3 и соотношение (4.12). 4.28. При каких значениях уровня значимости α отклоняется гипотеза Я0: β2 = 1,2 по данным задачи 4.8? 4.29. Значения независимых случайных величин Х^ (ι = 1, 2, 3, 4; /' = 1, 2) приведены в следующей таблице: 119
i \ 1 2 1 8,67 10,03 2 9,71 10,23 3 10,16 9,26 4 13,65 13,79 Предполагая, что L{Xf) = #(μ„ σ2) (все параметры неизвестны), построить оценки для μ,, μ2, μ3, μ4 и σ2 и проверить гипотезу однородности Я0: μι — μ2 = μ3 = μ4 (уровень значимости принять равным 0,1). 4.30. Построить интерполяционный многочлен вида k q>k(t, β) = Σ β/αΧ0 Для k = 1 и 3, где a,{t)— многочлены / = ι Чебышева (см. 4.6)) по следующим данным, отражающим неизвестную зависимость χ = f(t): t, Xi 0,40 0,39 0,52 0,50 0,61 0,57 0,70 0,65 0.79 0,71 0,86 0,76 0,89 0,78 0,95 0,81 0,99 0,84 Как изменяется точность интерполяции при переходе от k = 2 к k = 3? 4.31. Решить задачу, аналогичную предыдущей, для следующих данных: /, Χι 0 66,7 4 71,0 10 76,3 15 80,6 21 85,7 29 92,9 36 99,4 51 113,6 68 125,1 4.32. Выписать четвертый многочлен системы ортогональных многочленов Чебышева Указание. Использовать рекуррентное соотношение ατ + ι(0=(ί + α)ατ(0 + βατ-ι(0. π π где а = — Σ/ία?(/,-)/α?. β = — Σ ^-i(^W<i)/a?-i i= ι ;= ι (см. (4.6)). 4.33. Смоделировать наблюдения λ",- = /, + ε,, i — 1, ..., η, если η = 100, /, = 2-τ-0,1(ί— 1), ε,— независимые 120
равномерно распределенные случайные величины на отрезке [0; 0,7). 1) Построить по этим данным интерполя- ционный многочлен <р*(/; β) = £ β/α/(0 Для * = 2, 3, 4, где a,j{t), j = 1, ..., 4,— многочлены Чебышева. 2) Построить графики функций χ = /2, χ = φ*(/; β), k = 2, 3, 4. 3) Как изменяется точность интерполяции с ростом &? 4.34. Решить предыдущую задачу при ε,- нормально распределенных с Εε,· = 0, ϋε,- = 0,04. 4.35. Решить задачу 4.33 с X; = eh -+- ε,·. 4.36. Решить предыдущую задачу при ε,· нормально распределенных с Εε,· = 0, ϋε,- = 0,04. 4.37. Пусть У,, ..., У„ — независимые случайные величины с общей функцией распределения F0((x— βι)/β2), где FQ(x) — известная непрерывная функция распределения, а параметры сдвига β( и масштаба Рг > 0 неизвестны. Тогда Υ-, = βι-τ-β2^/, где случайные величины U\, ..., U„ независимы и имеют функцию распределения Fq(x). Записав для соответствующих порядковых статистик представление νω = β, + α;·β2 + β2ε;-, где ε;· = ί7(;·) — α;·, α;· = EU(j), / = 1, ..-, η, получить оценки параметров βι, β2 методом наименьших квадратов. Указание. Здесь случайные величины Υ = = (V(i), ..., Υ(η)) удовлетворяют модели линейной регрессии с коррелированными наблюдениями: cov(8;, ε;·) = cov(t7(,), U(j)) = gr, известны, поэтому надо перейти к некоррелированным величинам X = = G~U2Y, где матрица G = ||§,-,-||" является, по предположению, невырожденной. Главв 5 РЕШАЮЩИЕ ФУНКЦИИ 1. Пусть на выборочном пространстве X = [х] значений наблюдаемой случайной величины X задана функция δ(χ), значения которой принадлежат множеству D = [d] возможных решений, которые могут быть приняты при наблюдении конкретного значения X. В этом случае δ(χ) называют решающей функцией (правилом, процедурой). Пусть, далее, L(X)eF = [F(x; θ), ΘξΘ|, и для каждой пары (θ, ίί)εθχβ задано число L(Q, d) ^ 0, интерпретируемое как убыток (ущерб, потеря) от принятия решения d, когда X имеет распределения F{x\ θ). В этом случае говорят, что задана функция потерь 121
Ζ.(θ, d). Например, в задачах точечного оценивания решениями являются значения оценки параметра Θ, поэтому множество решений D совпадает обычно с параметрическим множеством Θ, решающая функция δ называется оценкой, а ущерб /.(θ, d) отражает расхождение между значением θ и оценкой d. Поэтому в таких задачах обычно полагают L{Q,d) = ω(|θ — d\), где ω— строго возрастающая функция ошибки \d — Q\. Величина R(Q, δ) = ΕοΖ.(θ, δ(Χ)) называется функцией риска процедуры δ, τ. е. это средние потери, которые имеют место при применении решающего правила δ, когда наблюдаемая случайная величина X имеет распределение F(x; θ). Если для двух правил δ' и δ выполняется условие /?(θ, δ') < R(d, δ) ν θ e θ, (5.ΐ) причем имеет место строгое неравенство хотя бы для одного θ, то правило δ' предпочтительнее δ; в этом случае правило δ называют недопустимым. Решающее правило, не являющееся недопустимым (т. е. для которого не существует предпочтительного правила), называется допустимым. В практических ситуациях ограничиваются рассмотрением класса допустимых решающих правил, никакие два из которых уже несравнимы в смысле (5.1). Для выбора наилучшего из допустимых правил в статистике применяют байесовский либо минимаксный подходы. 2. При байесовском подходе предполагается, что параметр θ — это случайная величина с некоторым (априорным) распределением Ζ-(θ), задаваемым плотностью распределения (или вероятностью в дискретном случае) π(θ). В этом случае можно вычислить полную среднюю потерю для процедуры δ: /-(δ) = \r{8, 6)π(θ)άθ (или 2^(f)i> δ)π(θ,) в дискретном случае), называемую I байесовским риском, и упорядочить все решающие правила по величине этого риска. В данном случае оптимальным, или байесовским, решением является процедура δ*, минимизирующая байесовский риск /-(δ). Алгоритм нахождения байесовского решения при заданном априорном распределении параметра π(θ) имеет следующий вид [1, с. 224—225]: а) для X = χ находят апостериорное распределение π(θ\χ) по формуле π(θ|х) = f(x; θ)π(θ)//(.χ:), где f(x) = 122
= E/(x; 0) = \f(x; 0)π(0>ίθ( пли £/(*; θ;)π(θ,-) в дискретном случае! ; б) вычисляют среднюю потерю для решения d относительно этого апостериорного распределения E(L(0, d)\x) = = S ДО, ίί)π(θ|χ)ίίθ (или 2 ЦО,-, «Oji(OiU)); i в) в качестве искомого выбирают решение d* = δ*(χ), для которого эта средняя потеря минимальна. 3. При отсутствии априорной информации о θ для сравнения допустимых решающих правил используют максимальный риск ι,ι(8) = supR(0,8), и наилучшим счи- тают правило 5, минимизирующее /η(δ); это правило называется минимаксным решающим правилом. В ряде случаев такое правило удается построить, если можно найти априорное распределение параметра π(θ) > О, для которого функция риска соответствующего байесовского правила δ* постоянна: /?(θ, 6*) = const (такое распределение π называют наименее благоприятным априорным распределением), в этом случае δ = δ* [1, с, 225]. 4. В важном частном случае Θ = {0,, ..,, θ*), τ. е. возможными распределениями наблюдаемой случайной величины X являются лишь конечное число k распределений F,(x) = F(x;Qi), i = 1, ,,,, k, и требуется выбрать одно из них в качестве истинного по наблюдению над X. Такие задачи называют задачами классификации. В данном случае множество возможных решений D = {dt, .... da), где решение di означает, что в качестве истинного следует выбирать распределение F„ i= l, ..., k, и каждое решающее правило δ(χ) порождает разбиение выборочного пространства X = W, (J . . (J Wk, Wi(] W; = = 0. Ίφ'ΐ, где Wi = [χ: δ{χ) = di), i=\, ..., k. При этом байесовское решение δ* определяется разбиением Λ: = U/*U-U Wf, в котором WT = [х- Ы{х) = min /г,(х)), i = 1, ..., k, где функции Λ/Μ = Σ Ό'1'Ή/.Μ- /(/Ίϊ) = Ζ-(θ.-, dj), π,- = π(θ,-) (если указанный минимум достигается при нескольких значениях /, то в качестве значения индекса i берут минимальное из них). Если потери t(j\i) = I, j φ-i, или же они неизвестны, или их трудно оцепить числом, то байесовское правило заменяется принципом максимума апостериорной вероятности: относить объект с наблюде- 123
нием х к тому классу, апостериорная вероятность к л,(х) = /|(^)л,/2 nsfs(x), i = 1, ..., k, которого максималь- 5=1 на, т. е. в таких случаях [1, с. 230]: W? = \х: 7iif,{x) = max Ji,f,(x)), i = \, ..., k. Для построения минимаксного решения δ ищут наименее благоприятное априорное распределение π = = (πι, .,., π*) из условия равенства компонент вектора риска /?(δ*) = (R\(8*) /?*(δ*)) соответствующего байесовского решения, где Я,{б*) = R(Q„ δ*) = Σ tU\i)Po,(X^ K)< ί= ' *■ 5.1. Пусть ВД = Bi( 1,0), Θ ={θ, = -j , 02 = -§}, множество решений D = {db еУ и функция потерь L(0,-, cf,·) задана таблицей rfl 0 2 rf2 1 0 1) Определить все допустимые решающие правила в данной ситуации и найти среди них минимаксное. 2) Найти байесовское решение δ* для произвольного априорного распределения π(θ,) = α, π(θ2) = 1 — α, αε [0,1], и построить график байесовского риска р(а) = г (δ*) как функции а. 5.2. Найти все байесовские решения в следующей ситуации: Цх) = в/(1,0), Θ ={θ, = -j-, θ2 = I], D = [du d2, d3) и функция потерь /.(θ,, dj) задана таблицей, Построить график байесовского риска ρ(α) = /-(δ*) как функции а = π(θ), α ΕΞ [0,1]. d, di d3 0 4 1 0 1 2 1 2" 124
Указание. Сравнить средние потери относительно апостериорного распределения, данного в решении 2 предыдущей задачи. 5.3. Пусть ЦХ) = Βί(Ι,θ), θ = |θ„ θ2}, D = [du d2) и функция потерь Ζ.(θ„ df) задана таблицей (а, Ь > 0), Рас- 2 1 3 1 смотреть два случая: Θ, = -у, θ2 = -ψ и θ, = -^-, 02 = -j- rfi d2 0 ft α 0 Убедиться в том, что в обоих случаях множества допустимых решающих правил совпадают, но во втором случае при любом априорном распределении параметра байесовское решение предпочтительнее. | Указание. Воспользоваться решением задачи 5.1. 5-4. Пусть ЦХ) = β/(3,θ), Θ = (θ, = 1(Г2, θ2 = КГ'}, множество решений D= \d\, di\ и функция потерь Ζ.(θ„ dj) задана таблицей d, d2 0 1 2 0 Рассмотрим следующие решающие правила δ,· = (δ,(0), δ,-(1), δ,(2), δ,(3)): δι = (d\, di, аг, di), δ2 = {d\, d\, аг, di), бз = {d\, d],d\, d<i). Убедиться в том, что эти правила между собой несравнимы и найти среди них минимаксное. 5.5. Пусть ЦХ) = βί(Ι,θ), θ = |θ,,θ2), D=[dud2) и функция потерь задана таблицей d, di 0 ft а 0 125
Определить минимаксную решающую функцию среди функций δι{χ) = ιά' "Ри * = ?■ .'•■■■•t'-1· w Ыг при χ = ι, ι + 1, ···, i= 1,2, 5.6. Убедиться в том, что если в условии предыдущей задачи заменить L(X) на пуассоновский закон Π(θ), то δ = δι при а{\ — e~°')^.be~ 2, а соответствующий вектор риска есть (а(\ — е-0'), Ье~°г). 5.7. Пусть -Υ — случайная величина, имеющая распределение F\(x) = F(x; θ,) либо Fi(x) = F(x; θ2); множество решений D= {db d2) и функция потерь задана таблицей 0 ь а 0 о, 02 Построить байесовское решение для заданного априорного распределения (πι,π2) и вычислить соответствующий риск (ср. с задачей 3.51). Рассмотреть случай, когда Ft — нормальное распределение Λ/(θ,·, σ2), i = 1,2. 5.8*. Предположим, что наблюдается случайная величина X, распределенная по нормальному закону с неизвестным средним θ и известной дисперсией а'2, множество решений D= [d\, di, аз) и функция потерь L(Q, d) задана следующей таблицей: \ d 0 \ <0 =0 >0 d, 0 1 2 d2 I 0 I d., 2 1 0 Рассмотрим решающие функции вида 'd\ при х<.а, &аЬ(х) = аг при а^.х^.Ь , аз при х>Ь, где а<0<6. Убедиться в том, что функция риска имеет вид 126
/?(θ, δ«*) = φ(-^-) +Φ(±=Α_) при θ<0, Φ(α/σ) + Φ( — 6/σ) при θ = 0, и построить ее график при Ь = — а. 5.9. Предположим, что θ = (0, 1), D= [d\ = [О, 1]и функция потерь имеет вид Ζ.(θ, d) = |θ — d\a, a^\. Рассмотрим класс решающих функций вида δ(χ) = const (т. е. решение принимается без предварительных наблюдений). Найти в данном классе байесовское решение, соответствующее априорному распределению параметра π(0) = = α, π(1) = 1 — α, ае[0, 1]. 5.10*. 1) Показать, что для риска баейсовского решения в задаче классификации (см. п. 4) справедливо представление (ниже для дискретной случайной величины все интегралы заменяются соответствующими суммами) /-(δ*) = \ min hi(x)dx. 2) Введем величины Доказать следующие оценки для л(б*): min(ji,f,(jt), n;fj(x))dx, I = max/(/|i). / = min/(y'|i). ΙΣ тахЛ;<г(б*)<С/ Σ Ir„ В каком случае эти оценки совпадают? I Указание. Использовать тождество (при доказа- I тельстве воспользоваться методом индукции по k) к к 2 ο,ι = max α,- + Σ ггпп(а,-, maxa,). 5.11* (продолжение задачи 5.10). Пусть F,{x) — функция распределения л-мерного невырожденного нормального Ν(μ('\ А) закона, i = 1, 2. Доказать формулу /12=я,ф(—f Uln-^) + + π2Φ(--^+-μΐπ^-). где ρ = (μ0) — μ(2))7ΐ_ '(μ(|) — μ(2)) — расстояние Махала- нобиса между распределениями^^'1, Α) πΝ(μ(2}, А). Вывес- 127
ти аналогичную формулу для [12 в случае двух пуассонов- ских распределений. 5.12* (продолжение задачи 5.11). Построить байесовское и минимаксное решения в задаче классификации с двумя нормальными распределениями, указанными в предыдущей задаче (ср. с задачей 3.52). 5.13*. Пусть X = (Хи ..., Х„) — выборка из распределения L(g)^.F = [F(x; θ), θεθ), а априорное распределение параметраА^еЕ^7*. Семейство распределений параметра F* называется сопряженным к ^(обозначается F*<lF), если при X = χ апостериорное распределение L(Q\x)^.F*. Убедиться в справедливости следующих утверждений л (ниже χ = 2 Χί)'· ;= ι 1) β(α,6)<]βί(ηι,θ), при этом L(Q\x) = β(α + *, b + + ntn — χ); 2) β(α,6)<]Si(r.6), при этом L(Q\x) = β(α + χ, b+nr); 3) Γ(α,λ)<Π(θ), при этом1(в|*) = r(-^-j-p , λ + jc); 4) Γ(α,λ) сопряжено к экспоненциальному распределению с плотностью f(x; θ) = Qe~°", х>0, при этомЦ0|дс) = 5) распределение Парето, задаваемое плотностью π(θ) = aac/(f + l, θ^α (α, α>0), сопряжено к равномерному распределению /?(0, Θ), при этом π(θ\χ) есть плотность Парето с параметрами (тах(а, хи ..., χη), α -\- η); 6) распределение Дирихле D(a), α=(α,, ..., ад/), α,·>0, i = 1, ..., Ν, задаваемое плотностью ^ = rt!i.:,;;" р!-'-сй"-' , е.+-+е,=ι. сопряжено к полиномиальному распределению М(п; θ = = (θ,, ..., ΘαΛ). при этом L(Q\h = (Λ,, ..., Λ«)) = D(o + Λ); 7) ΛΤ(μ, σ*)<| Ν(θ, б2), при этом Ц0\х) = ΛΤ(μ,, σ?), где Указание. Плотность любого распределения достаточно вычислить с точностью до нормирующего множителя, поэтому, используя для любой случайной величины ξ запись Д(/) = cp{t)^p{t) (здесь постоянная с определяется условием c\p{t)dt = 1), при нахождении апостериорной плотности π(θ\χ) = f{x; θ) n(Q)/f(x) достаточно ограничиться вычислением чис- 128
лителя π(θ) f(x; θ); аналогично следует поступать и при вычислении плотностей π(θ) и f(x; θ), 5.14. Рассматривается задача оценивания неизвестной вероятности «успеха» θ по наблюдению числа успехов X в η испытаниях Бернулли (таким образом, здесь Θ = D = = (О, 1)). Пусть функция потерь имеет вид ш d) = {d-0) ί-\Ό>α) 0(1-0) ' а априорное распределение параметра θ является равномерным на интервале (0,1). Доказать, что байесовское решение есть δ*(χ) = —. Является ли это решение минимаксным? 5.15 (продолжение задачи 5.14). Найти байсовское решение б* для случая, когда функция потерь Ζ.(θ, d) = = (d — θ}2, а априорное распределение Ζ,(θ) — β(α, b). При каких значениях параметров а и Ь решение б* является одновременно минимаксным? (ср. с задачей 2.6). Указание. Воспользоваться решением задачи 5.13 п. 1). 5.16. Рассмотрим задачу точечной оценки скалярного параметра θ с позиций теории решений, т. е. когда множество решений D совпадает с параметрическим множеством θ и решение d^D— это значение оценки параметра Эе0. Пусть функция потерь имеет вид Д6 d) = {d — θ)2, тогда функция риска /?(θ, б) = Ео(б(Я) — Θ) есть средне- квадратическая ошибка оценки 6(Х). Доказать, что при наблюдении X = χ байесовское решение (байесовская оценка) Ь*(х) имеет следующий вид: δ*(χ) = Ε(θ|χ) = \θπ{θ\χ)άθ, т. е. совпадает с апостериорным средним параметра, а соответствующий риск л(б*) = ΕΌ(Θ\Χ), где D(9U) = Ε[(θ - б*(х))2И = $(θ - &*{χ))2η{θ\χ)άθ есть дисперсия апостериорного распределения параметра, а математическое ожидание вычисляется относительно плотности (или вероятности в дискретном случае) f(x) (предполагается, что все соответствующие моменты существуют). Применить этот результат при решении задачи 5.15. 5.17. Пусть испытания Бернулли продолжаются до получения л-го «неуспеха» и X — число «успехов» в этих испытаниях. По наблюдению над X построить байесовскую 5-190 129
оценку неизвестной вероятности «успеха» θ в случае, когда функция потерь Ζ.(θ, d) = {d — θ)2, а априорное распределение L(6) = β(α,6). I Указание. Воспользоваться решениями задач | 5.16, 5.13 п. 2). 5.18. По выборке X = (Хи .... Хп) из пуассоновского распределения Π(θ) построить байесовскую оценку для параметра, если функция потерь ЦЭ, d) = (d — θ)2 и априорное распределение L(6) = Γ(α, λ); вычислить риск этой оценки и определить оптимальный объем выборки при цене Ο·0 одного наблюдения (т.е. минимизирующий общие потери л(б*) + сп). I Указание. Воспользоваться решениями задач I 5.16, 5.13 п. 3) и 1.39 п. 4). 5.19 (продолжение задачи 5.18). Убедиться в том, что если функция потерь Ζ.(θ, d) = (d — θ)2/θ, то байесовская η оценка при λ+ Σ ^<>1 имеет вид ι = ι β^ = ΐτ(?* + λ-1) и ее риск г(Ь*) = —^——. г ч па + 1 I Указание. При вычислении моментов восполь- I зоваться формулой Γ(Ζ + 1) = ΖΓ(Ζ). 5.20. Рассмотрим задачу оценивания параметра θ экспоненциального распределения с плотностью f(x; θ) = = Qe~ex, х>0, по выборке X = (Х\, ..., Хп). Пусть функция потерь Ζ.(θ, d)={— d\ и априорное распределение ЦР) = Г(а, λ), λ>2. Доказать, что байесовская оценка имеет вид и ее риск r{b*) = (α2(λ + η — 1)(λ - ΐχλ + 2))-'. Убедиться в том, что оптимальное число наблюдений при цене О 0 одного наблюдения равно α-^ο(λ- ΐχλ— 2) Указание. Использовать решения задач 5.13 п. 4) и 1.39 п. 2). 130
5.21. Пусть Χ = (Х[, .,., Хп) — выборка из распределения /?(0, Θ), где априорное распределение параметра θ есть распределение Парето с параметрами а и ос>2 (см. задачу 5.13 п. 5), Убедиться в том, что байесовская оценка θ имеет вид б*(дс) = —" + ot тах(а, ^п)), цп) = max xu вычислить ее риск; определить оптимальный объем выборки при цене с>0 одного наблюдения. Указание. Использовать решения задач 5.16, 5.13 п. 5) и 1.35. 5.22. Предположим, что по наблюдению X оценивается параметр θ равномерного распределения /?(0, Θ), когда параметр имеет априорную плотность π(θ) = ве~", θ>0. Доказать, что байесовская оценка в случае квадратичной функции потерь имеет вид δ*(,Υ) = X -\- 1, а ее риск г(Ь*) = = 1 . Указание. Записать среднюю апостериорную потерю для решения d в виде интеграла и продифференцировать по d; использовать формулу для гамма- функции Г(л + 1) = ( Ге-'dt = п\ 5.23*. Пусть вектор ν = (νι, ..., vw) имеет полиномиальное распределение М(п; θ = (θι, .... 6W)). Требуется по наблюдению над ν оценить Θ, если функция потерь Ф, d) = Σ (di - θ,)2, d = (d , dH), i = I в предположении, что априорное распределение параметра θ есть распределение Дирихле D(a) (см. задачу 5.13 п. 6). Показать, что при ν = h = {h\ hN) байесовская оценка δ*(Α) = (β;(Α),...,δ;(Α)) имеет вид β$Α) = -£tA i = Ν = 1 Ν, где α = 2 «ί. а ее риск '(δ*) = Σ«ϊ ■■ ι ο(α + 1)(ο + η) Указание. Воспользоваться решениями задач 5.13 п. 6), 1.52 и следующими формулами для моментов распределения D(a): 131
_ α,(α, + 1)-(α, + г - 1) tb' - α(β+1)...(α + Λ-1) ' Λ - ·■ J 5.24. По выборке X = (^ι X„) из распределения N{Q,b2) построить байесовскую оценку параметра, минимизирующую среднеквадратическую ошибку, если априорное распределениеL(Q) =Ν(μ,σ2); вычислить риск построенной оценки и определить оптимальное число наблюдений при цене с>0 за одно наблюдение. Указание. Воспользоваться решениями задач 5.16 и 5.13 п. 7). 5.25*. Рассмотрим задачу оценивания скалярного параметра Θ, если функция потерь L(Q, d) = |θ — d\, Q,d^.Rl. 1) Доказать, что при Χ = χ байесовское решение d* = = б*(л:) при любом априорном распределении L(Q) есть медиана апостериорного распределения £(θ|*), τ. е. такое число, что P(e<d*U)^-i-· P(G>rf*|x)>-i-· 2) Использовать этот результат при оценивании среднего модели Ν{θ, б2), когда £(θ) = Ν(μ, σ2). Указание. 1) Установить неравенство Ε(|θ — d||*)> mQ-d*\\x)Yde=Rl. 2) Использовать решение задачи 5.13 п. 7). Глава б СТАТИСТИКА СТАЦИОНАРНЫХ ПОСЛЕДОВАТЕЛЬНОСТЕЙ Бесконечная в обе стороны последовательность случайных величин [Χι], t = ..., — 1, 0, 1, .... называется стационарной, если выполняются следующие условия: Έ.Χι = m -+- const, co\(Xk+\,Xt) = Ε(Χ*+ι — m\Xi — in) = Rk. Последовательность чисел {Rk}, k = ..., —1,0, 1, ..., называют ковариационной функцией последовательности {Χι). При этом R-k = Rk при всех k и R0 = DXt = σ2 = = const. Будем предполагать, что оо Σ 1Я*1<°°. (6.1) * = ι В качестве оценок m и Rk по наблюдениям Х\, ..., Хп используют соответственно статистики 132
£ = Ο, 1 η — 1 . Для иллюстрации смоделируем η членов стационарной последовательности Χ, = ξ,_,-Η, + ξ, + 1, /= 1,2,..., η, (6.2) где ξ/, / = 0, ±1, ±2 — независимые случайные величины, равномерно распределенные на отрезке [О, 2Л]. Нетрудно проверить, что здесь ЕЯ, = ЗЛ, /?о = -тр /?ι = = -τ-, /?2 = -τ-, ^ί = 0, /^3. В табл. 6.1 и 6.2 приведены значения статистикой С*(гс)для различных п. Таблица 6.1 η 10 100 1000 X 1,70 1,41 1,50 Со(п) 0,372 0,270 0,262 С,(п) 0,264 0,185 0,178 £,(п) 0,088 0,092 0,089 £>(п) -0,128 0,026 0,002 С,(п) -0,242 0,047 -ι,οχ h = 0,5 (EX, = 1,5; Λο = 0,25; R, = 0,166...; R2 = 0,0833...) Таблица 6,2 η 10 100 1000 χ 2,040 1,690 1,800 е„(п) 0,535 0,389 0,378 Си 0,381 0,266 0,257 £ι(η) 0,127 0,132 0,126 <ο(Ό -0,184 0,039 0,003 С,(л) -0,349 0,068 -2,0 Χ хю-4 /ι = 0,6 (ΕΛΊ = 1,8; Λο = 0,3; R, = 0,2; Λ2 = 0,1). Важной характеристикой стационарной последовательности {Χι} является ее спектральная плотность f{x), представляющая собой преобразование Фурье ковариационной функции (/?*}: 133
/(*) = -j— Σ Rkcoskx, »е[-я, π]. (6.3) Λ = — οο Спектральная плотность (когда она существует) и ковариационная функция находятся во взаимно однозначном соответствии, В качестве оценок f(x) по наблюдениям Х\,...,Хп используют статистики вида Ux) = -s— Σ wn(k)Ck{n)coskx, (6.4) Δπ №<η-\ где {wn{k)\ — некоторая последовательность весовых коэффициентов (wn( — k) = wn{k)). В частности, при wn(k) = = 1 L-!— получаем периодограмму выборки. Если среднее т = EXt известно, то в формуле (6.4) X заменяют т. 6.1. Доказать, что среднее арифметическое X = = {Х[ + ... + Хп)/п является несмещенной и состоятельной оценкой для т = EXt. 6.2. Доказать, что статистика С*(л) = —Ц-Σ (Χι ~ т)(Хк + , - т), О < k < л, является несмещенной оценкой Rk. 6.3*. Доказать, что статистика n~R ι = \ является при л_->- оо асимптотически несмещенной оценкой Rk, т.е. ЕСк(п)—*■ Rk (k фиксировано). Π-»- оо 6.4. Пусть ξ и η — случайные величины с Εξ = Εη = = О, D£ = Ότ\ = σ2, cov(£, η) = 0. Доказать, что последовательность Χι = IcosXt + r]SinXt, t = 0, ±1, ±2, .... λ g (0, π), является стационарной и вычислить ее ковариационную функцию. 6.5. Пусть %t, t = 0, ±1, ±2, ..., — некоррелированные случайные величины, т. = Εξ,, σ2 = D|/. Является ли последовательность (ξ,} стационарной? Доказать, что последовательность г х, = Σ «/£'-/. ' = о, ± ι, ±2, ..., / = о является стационарной. Найти EXt и Rk. 134
6.6*. Для последовательности (6.2) доказать состоятельность оценки Ск(п), найденной в задаче 6.2. 6.7. Смоделировать последовательность (6.2) в случае, когда |/ распределены нормально с Εξ/ = 0,5, ΌΙι = 0,1; η = 100. Составить таблицу, аналогичную табл. 6.1. 6.8*. По значениям Χι, t = —η, — η + 1, ..., — 1, 0, предсказать значение Χ[ — найти оптимальный линейный о предиктор Xfn = Σ βΤηΧι, т. е. определить β/ так, чтобы /= -п 0 выражение E(Xi — 2 β'-^')2 было минимальным. Вы- /= -я числить аг(п) = Е(Я[ — Xf„)2 — минимальную средне- квадратическую ошибку прогноза. 6.9. Пусть v/, t = 0, ± 1, ± 2, ..., —стационарная цепь Маркова с состояниями 1, 2 ([2], с. 167). Доказать, что матрица ||рц(0И?1 где pij(t) = P(vs+/ = /Ivs = ί). '. /' = 1. 2. определяется формулой w<>»=Hll!!ll+<'-w||-i-!ll)· если Ί^·(ΐ)ΐΐ = || '""i-JI· ° <α<1· Найти стационарное распределение этой цепи. 6.10. Составить программу для моделирования последовательности v/, t = 0, 1, .... п, определенной в задаче 6.9. 6.11. Пусть (ν/} — стационарная цепь Маркова, определенная в задаче 6.9. Является ли стационарной последовательность (η;}, где _ ( 1, если ν( = 2, η' ~~ 1 — 1, если ν, = 1 ? Найти Εη; и /?«. 6.12. Смоделировать последовательность η,, ..., η,οο, l D где η, определено в предыдущей задаче, а = — Вычислить оценки величин Εη<, /?*. 6.13. Пусть ν; — цепь Маркова, определенная в задаче 6.9, ξι(ί), ξ2(0> * = 0, ±1, ±2, ..., — независимые 135
стационарные последовательности с Εξ,(/) = 0 и ковариационными функциями /?ϋ°, i = 1, 2. Положим η< = ξν,(ί)· Является ли последовательность (η<) стационарной? Найти Ех]1 и Rk. (Указание. Воспользоваться формулой полного математического ожидания. 6.14. Смоделировать последовательность ηι, ..., ηιοο, где η, определено в задаче 6.13, а = —, ξ,·(/) равномерно распределены на отрезке [—1, 1]. Вычислить оценки величин Εη<, Rk. 6.15. Решить предыдущую задачу для случая £(|,-(0) = = ЩО, 1). 6.16. Убедиться втом, что при условии (6.1) спектральная плотность f(x) (см. (6.3)) существует, непрерывна и определяет ковариационную функцию по формуле Rk = ) f(x)coskxdx. 6.17. Вычислить спектральную плотность для стационарной последовательности некоррелированных случайных величин. 6.18. Существует ли спектральная плотность у последовательности [Χ,], определенной в задаче 6.4? Убедиться в том, что в данном случае Rk = \ coskxdF(x), где — л F(x) — ступенчатая функция со скачками в точках ±λ и 2 величинами скачков ^—, F(— π) = 0, F(n) = σ2 {F(x) называется спектральной функцией последовательности {Χι}). 6.19. Получить следующее представление периодограммы (см. (6.4)) через выборочные значения последовательности {Χι} (среднее m известно): U*) = iM2W. Rn{x) = Α2η(χ) + Bl(x), где АпШ _2_у (χ, _ m)(cosxt 6,20*. Убедиться в том, что для математического ожидания периодограммы при известном среднем ηι = ΕΧι справедливо представление 136
Ejn{x) = \ kn(x — y)f{y)dy, — л где kn{x) = -~{s'm{nx/2)/s\x\{x/2)f— ядро Фейера. Доказать, что Efn(x)—=-*f(x), — π ^ χ < π. Замечание. Периодограмма является асимптотически несмещенной оценкой спектральной плотности и при неизвестном т, однако эти оценки не являются состоятельными. Состоятельные же оценки можно Получить при специальном выборе «весов» wn{k) в (6.4). 6.21*. Пусть среднее т = EXt известно и wn{k) = Доказать, что Jn(k) — асимптотически ('-^)- несмещенная и состоятельная оценка величины λ + e 4г ) f{y)dy, 0 < ε < π, λ <= [ — π + ε, π — ε]. λ — β Ι Замечание. Результат верен и при неизвест- | ном т. 6.22*. Пусть т известно и wn(k) = ( 1 Щ ( 1 — LJ-) при \k\ < /„ и wn(k) = О при \k\ > /я. Доказать, что если п, 1п -»■ оо, 1п/п -+0 и J \kRk\ < оо, то fn{x)— k асимптотически несмещенная оценка f(x). Замечание. Результат верен и при неизвестном т. Эти оценки при широких условиях являются состоятельными. ОТВЕТЫ И РЕШЕНИЯ Глава 1 1.1. Положить Х„ = 1, если U„ < ρ, и Х„ = 0, если U„ > ρ, где Uη — последовательность (1.5). 1.3. Отрезок [0, 11 разбить на N отрезков Δι, Δ2 Δν. где Δ, = [О, ρ,Ι Δ, = [ρ, + ... + Ρι-ι, Ρ\ + ... + Ρι\ / = 2, 3 Ν; положить Χι, = Ι, если ϋι e Aj, / = I Л/, где t/„ — последовательность (1.5). Числа Х\, ..., Х„ образуют реализацию η первых испытаний полиномиальной схемы с указанными в задаче вероятностями исходов. 1.4. Положить So = 0, S„ = S„_i + X», η > 1, где Х„ = 1, если Un =ζ -s-, -V„ = — 1, если U„ > -^- ,где U„ — последовательность (1.5). 137
1.6. Положить Х„ = — α1π(1 — U„), где U„ — последовательность (1.5). 1.8. Пусть ξ,, ..., ijm — независимые случайные величины, имеющие показательное распределение с параметром а. Тогда случайная величина η = ξι + — + ξ™ имеет распределение Эрланга с параметрами (ос, т). 1.9. Положить где U„ — последовательность (1.5). Удовлетворительное приближение к нормальному распределению получается уже при N = 12; это значение параметра N обычно и используют для вычислений. 1.12. Если ξι ξ* — независимые бернуллиевские случайные величины с параметром ρ (см. задачу 1.1), то L (|( + ■·- + ξ») = = Bi(k, ρ). 1.13. При больших л по теореме Муавра — Лапласа имеем / |ν„ пр\ ^ \ _ φ(ί) _ ф(_() = 2ф(;) _ { q \ VnP<7 ' или Ρ Следовательно, чтобы выполнялось соотношение Pi I —— — р\ ^ < 6,1 ?а γ, надо положить 6, = t ~\1-£-Ц а t = ф-1 (—ί-^-) = ξ ц ι+τ,. При у = 0,98 квантиль ц0в9 = 2,326 и для приведенных экспе- 2 риментальных данных граница 60.98 = 0,0183, а I —— -^-1 = 0,0069, т. е. согласие с теорией хорошее. 1.14. Согласно предположению, появление числа, не превосходящего 4, можно рассматривать как «успех» в испытаниях Бернулли с вероятностью «успеха» ρ = 1/2. Поэтому (см. решение задачи 1.13) граница в данном случае равна δ0,9β = 0,0116, а наблюдавшееся значение отклонения частоты «успеха» ■ -т-| = 0,0089 < δ0,98· Следовательно, согласие экспериментальных данных с теорией хорошее. 1.16. При больших п имеем Р(~\1—\Х — θ]| < ί1 « 2Φ(ί) — или Р(\Х - о,| < 6) « 2Ф| В данном случае а, = 6, μ2 = 3, л = 4096 и правая часть приближенного равенства равна 0,998 при δ = ~у —"о.999 = 3,090 V3/4096 = = 0,0836... Наблюдавшееся значение \х — 61 = 0,1389 оказалось гораздо больше этой границы, т. е. в данном случае в эксперименте наблюдалось маловероятное событие. 138
1.17. В данном случае (см. предыдущее решение) а, = 2, μ2 = ~ 5/6, δ = т/5/(6■ 4096)■ 3,090 = 0.044. Наблюдавшееся же значение \х — 21 = 0,003 укладывается в эти границы, т.е. с позиций этой характеристики данные рассматриваемого эксперимента лучше согласуются с теорией. 1.19. Используя результаты решения задач 1.13 и 1.16, имеем δ = Vfi7n"o.99 = УЧ.9167/500·2,326 = 0,359; наблюдавшееся же значение отклонения равно \х — ос, I = 15,942 — 61 = 0,058, т. е. согласие с теорией хорошее. 1.24. Поскольку F„(xo) = и /.(μ,,^ο)) = Bi(n, р0), где р0 = = F(x0), при η -*■ оо по теореме Муавра — Лапласа имеем /μ„(^ηρο <2\ φ(2)ι ίο = 1_ρο. Отсюда PdF.to) - Pol < ί/V^) -* Φ (~ρ==) - φ( г^=) = 4 VPo<7o' ч VP»9o' (-7=) ~ '■ 2Φ_ \/Ро<7о 1.25. Имеем cov(F„(*i). ^„(Jia)) = -^j-covfa„(*,), Δ„(λ:,, *2) + μη(*ι)) = = —г{соу(ц„(д:,), Δ„(*,, *2)) + Ομ^ι)]. Здесь Ομ,,(Α-ι) = я/^я,)^ — F(xi)) (см. решение задачи 1.24.), а л ΰον(μη(^ι), Д„(д:|, *2)) = Σ cov(r),, ς;). В силу независимости наблю- дений, при ί =?t /' индикаторы η,- и ς; независимы и, следовательно, cov(r);, ς,) = 0. Далее получаем cov(r),-, ς,) = Εη,ς,- — Εη,Ες,- = Ρ(η, = ς, = 1) — - Ρ(η, = 1)Ρ(ζ/ = 1) = - Ρ(η, = 1)Ρ(ς, = 1) = - F(x,)(F(x2) - F(xt)), поскольку {η; = ς, = 1) — невозможное событие. Отсюда οον(μ„(Λ:ι}, Δ„(χ,, хг)) = — nF(x,)(F(x2) — F(x,)). Объединяя эти формулы, приходим к требуемому результату. 1.26. Рассмотрим полную группу из N событий Е\ = (ξ ^ х{\, Ег = \хх < ξ < хг] Ец-, = (*«_2 < ξ < Χν-ι), Εν = (ξ > χν_,); их вероятности соответственно равны plt ..., ρν- Тогда ν„ очевидно, есть число реализаций событий £( в л независимых и однородных испытаниях, ί = 1, .... N. Следовательно, £(ν) = М(п; ρ , ρν). Далее Имеем / \ , , \ , \ ι \\ — лр,р2 = cov(v,, v2) = οο\ι(μ„(χ,), μ„(*2) — μη(·*ι)) = = ΰον(μ„(*,), μ„(χ2)) — Ομ„(^,). Отсюда οον(μ„(*ι), μ„(*2)) = Ώμ„(χι) — np,p2 = лр,(1 — ρ, — ρ2) = = nF(xt) (! - F(x2)), что эквивалентно результату, полученному в задаче (1.25). 139
1.27. Случайные величины Ύ*, ί = I, ..., л, при любом k независимы и имеют такое же распределение, как £*, поэтому cov(/U Ans) = ΙγΣ cov(A?, X',) = -^cov(|*, ξ1) = " './=1 = —(EE* + ,-EE*EE') = —(o»+,- o»o,). В частности, DZ = DA„t = — (o2 - α?) = -ui-. η η При исследовании выборочных центральных моментов можно считать, что αϊ = 0, и, следовательно, μ* = α*. Учитывая это, имеем ES2 = ЕА„2 - ЕЛ2, = μ2 - -^ = ^~μ». Далее, (S2)2 = А\г — 2Α2„,Α„ι + И Αι, и непосредственные вычисления дают ЕЛ22 = -^-Е( 2 Χ} + Σ XfX2) = -τ-(«μ4 + "(л - 1}μ22) = μ44-(η-1}μ| ел2,лл2 = \ε( Σ χ! + Σ Χιχ) Σ XI = " \,_ι ,*., '* = . ЕЛ1, = -U ( Σ *? + Σ W,V = Д-Е (( Σ χίΐ + 2 Σ xlxl) μ4 + («-1)μ22 , 2(11-1) , μ4 + 3(η-1)μ22 = _, + -^г—μ2 = ^ ■ 2 _ ε,<^2 ,rc2*s _ Η·4-Η·2 2(μ4 - 2μ2) , μ4-3μ|_ Отсюда DS2 = E(S2)2 - (ES2)2 = r* r' - ~чг< Г1"' + Л Л /ι что эквивалентно приведенной в условии задачи формуле. Наконец, cov(/V, S2) = Ei^VS2), так как по-прежнему можно считать, что ос* =0. Записав s2 = — Σ х%> — —τ ( Σ Χ\) = —-я— Σ х] — —τ Σ XiXh получим E(*S2) = 1Ζ±ε (Σ χ) (Σ *2) = ^-ε( Σ Λ?) = -U^V Для распределения Ν(μ, σ2) моменты α, = μ, μ2 = σ2, μ3 =0, μ4 = 140
3σ4, поэтому EX = μ, ΌΧ = —, ES2 = -σ2, DS2 л η = 2("71)σ4, cov(Jf, S2} = 0. 1.28. Рассмотрим r-мерные векторы ξ, = (^ι, ..., Χ£'}, s = 1, ..., η. Они независимы, одинаково распределены и Ε(ξι) = α, D(£i) = = ||cov(A'fr', Χίί)\\\ = J(q и J указаны в условии задачи). Следовательно, по центральной предельной теореме при η ->■ оо имеем (-L<S, + ... + ξ„ - па)) - Л/(0, Σ) Остается заметить, что ——(ξι + ... + Ел — «о) = л[п(А„1,] — ctj,,, ..., V" Л„4г, — О*,). 1.29. Можно считать, что<Х| = Εξ = 0 (см. решение задачи 1.27). Положим η» = -Jn(Sl — μ2) = ξ* -f- δ„, где l„ = -φΪΑ„2 — μ2), δ„ = = — -JnAl\. Поскольку при сделанных предположениях Щ„) -*■ N(0, Цч — μ§) (см., например, решение задачи 1.28), достаточно убедиться в том, что δ„ ->■ 0. Но Ρ(Ιβ.Ι > ε)<-Ε|β,Ι =^-ΕΛ2, = ^ОАя1 = JiL.-*o, ε ε ε еул что и требовалось показать. Асимптотика моментов следует из задачи 1.27. 1.30. События \Х(Г) < хи X(s) <; х2) и (μ„(^ι) ^ г, μ„(χ2) > s) эквивалентны, поэтому F,s(x\, χι) = Ρ(μ„(-Κ|) ^ г, μ„(^2) > s). Пусть сначала χι < Χι. Рассмотрим случайные величины νι = μ«(ίι). ν2 = \ίη(*ι) — — μ„(^ι), ν3 = η — μ„(χ2). Тогда (см. решение задачи 1.26) Ц\\, v2, v3) = Μ (η; pi. p2, Рз). где ρ, = F(x,), p2 = F(x2) — /^i). рз = 1 — f(*2). Отсюда Ρ(μ„(*0 > r, μ„(*2)> s) =ΣΡ(ν, = m, ν2 = /), где суммирование производится по всем m и /, удовлетворяющим условиям m^r,s^m + j^.n. Поскольку Ρ(νι = m, v2 = j)= ■"· pfp'jp»-»-), т!/!(я —m—/)! отсюда следует приведенная в формулировке задачи формула. Если же X] ^ *2, то событие {Х(,) ^ *i, -¥(S) ^ х2) = (-¥(j) ^ д:2); формулу же для одномерной функции распределения можно получить, например, из предыдущего результата: F,(x\) — lim F,s(x\, x2). х2-»-оо 1.31. Пусть г = 2 (общий случай рассматривается аналогично) и точки χι < х2 заданы. Событие {X(il)e(x\; х\ + d*,), Χ,ι,,ε^ι; *2+ + dx2)} осуществляется тогда и только тогда, когда k\ — 1 из всех наблюдений меньше х\, одно попадает в интервал (х\, х\ + djii), k2 — k\ — 1 наблюдений — в интервал между х\ + d-ti и д:2, одно наблюдение — в интервал (х2, х2 + dx2) и остальные η — k2 наблюдений больше Х2 + d*2. В силу независимости наблюдений, вероятность ука- 141
занного события при малых dx\ и dx? с точностью до членов, имеющих более высокий порядок малости, равна С*'-'F*'-'(*])(" - *ι + l)/(*,)d*,Cfcr<£.-i(F(*2) _ - F(x,))k'-k'-\n - k2 + l)f(x3)dx*l - Fix2))n-k>. Разделив на dx\dx3 и устремляя dx\ и dx3 к нулю, получим указанную формулу jumg^klxy, хг). 1.32. Обозначим ft, = \пр\ i = 1, 2, и пусть η„ι = (Ζηρ( — Qpi)V". ι'=1, 2. Совместная плотность распределения случайных величин η„ι и η„2 по формуле (1.2) равна (см. задачу 1.31) Ч>"(</|.</2) = —gb, + \.b,+ \Up, + -Ц^, ζρ, + -ηΗ = А){п)А£п)А3{п), где Л,(л η!ρ^'(ρ2-ρι)^-^-'(1-ρ2)η-^-' η*Κ*ι-*ι-1)Κβ-*s-l)l Из(м) -φι/ \ -φι IX тгЧ?- + -^)) \ ^—; (Ч л/Л/ \ η - ti - I —i=S—) · Из формулы Стирлипга следует, что А\(п)-+ ■ 2Wpi(P2-p1Xl-P2) Далее, так как плотность f(x) непрерывна, то Аг(п) ->■ /(ςρ,)/(ξρι). Наконец, поскольку F(** + -%г) = Р; + №«^ + №>>ft + °(т-)· ' = '■ 2· несложно получить, что + (|i'p^2-p/(4 = - ff_i «'W/.IO"! - l«.l-. Учитывая, что /(Sp,)/(?P,)/VPi(P2 — PiX' — Рг) = (det||a,7||) 2, окончательно может записать, что φΛί/ι. ί/s) -* -—7====Г€ХР { - -ψ- Σ σ"'ί/.<//}. т. е. в пределе имеем плотность двумерного нормального распределения с нулевыми средними значениями и матрицей вторых моментов ||6,;||. 1.33. Согласно решению задачи 1.31, совместная плотность распределения -¥(,) и X(„-s + \) равна (при х\ < хг) 142
χ[ΐ - /totf-'/toi/ta). Поскольку якобиан преобразования у\ = nF(x\), у2 = л[1 — F(*2)] равен /(*,, *2) = — пг1(х^(х2), по формуле (1.2) совместная плотность распределения случайных величин я, = nF(X(,)) и η„ = л[1 — — F(-V(„_J+|)}] имеет вид φ^,2) = ,,,-Ι+,(^(^)..-'(.-^))/ι/(.-(^-). 5-.Л уЛ\1 .. "' ^ У?"1 уГ' s, \ п))\ (n-r-Sy.n' + s (/--I)! (s-l)!A ν(ι .. у + уЛ-'-' . уГ' e-„ И"' r-u, \ я / (г-1)1 С*—1)1 ■ если л ->■ оо, а л, s — фиксированы. Таким образом, κ л и ηη, следовательно, Х(,\ и Ύ(η_,+ ΐ) асимптоти чески независимы; при этом £(κ„) -*- Г( 1, г), Цца) -»- Г(1, s). 1.34. Якобиан преобразования у\ = пх\, уг = (л — ΐχ*2 — х\) уп = х„ — л:„_| равен J(x\ хп) = л!. Отсюда по формуле (1.2), принимая во внимание указание, имеем, что совместная плотность распределения величин Y\ У„ есть ехр| — у\ — ... — у*}. Далее, Х(к) = = 2 У//(я — / + '). поэтому 1-1 Но среднее и дисперсия экспоненциального распределения Г (1,1) равны 1, следовательно, окончательно имеем "1 "1 В частности, если η-*■ оо, то Е*(л) = Σ-!-= 1пл + с + о(1), 1-1 ' с = 0,5772... — константа Эйлера, D*w= Σ|=τ+ο(Ι), ι-ι' 6 1.35. Согласно решению задачи 1.31, совместная плотность распределения случайных величин Х^) и Χ(η есть "<<*-■ *>) = (й-1)!(/-*'-1)!(,,-И"'(* - X^k"{l - "Γ~' О < jt| < л:2 < 1. Отсюда (см. (1.2)) совместная плотность распределения Y\ = X(k) и Υι = Xw — -V(t) имеет вид 143
**"■*) = (k-iW-k-i)i(n-/)iУ*-У»-*-'(1 - *" - *"Г'. i/l. </2 3* 0, J/, + 02 < 1. Теперь, чтобы получить плотность распределения У2, достаточно вычислить интеграл 0 < i/s < 1. Аналогично, плотность распределения Хщ равна 1-у, f Далее, так как среднее и дисперсия распределения β(α, Ь) равны a ab —ГТ и Ι—7~ПТ,—ι .. , ,\ соответственно, то а+Ь (а + Ь)\а+Ь+1) ру k nv k(n-k+l) l-k m WJ (n + iftn + 2) Наконец, поскольку D(A-(,) - *(11) = DXW + ΌΧ(η - 2cov (Xw, Xv)), из этих формул получаем, что „ IV V S k(n —1+ I) cov (Xw, Xw) = \ —-!— . (n + l)\n + 2) 1.36. Заметив, что L(— ) = /?(0, 1), можно воспользоваться решением предыдущей задачи. В данном случае Ху) = (Ь — а)Х{\) + а, Хм = (й — а)Х(„) + о, где Х{\), Х(п) — экстремальные значения выборки объема л из распределения /?(0, 1), совместная плотность распределения которых имеет вид Ы*ь *0 = п(п- 1)(х2-х^-\ 0<д:,<д:2< 1. 1.37. Р(А-(„ > х) = Р(^ > *, ,· = ι η) = [1 _ /Ч*)Г = е-"(^ )■ χ ^ д. Отсюда _,./'-* У а также Ρ(η,/,,(^(ΐ)-α)/6</) = 1-е-'", /JsO. Таким образом, случайная величина л"^.^) — α)/6 имеет распределение, не зависящее от объема выборки, именно распределение 117(0, а, I). Отсюда Ελο = a + br(l + -^n- 144
1.38. Слагаемые в F^x\, хг) независимы и имеют такое же распределение, что и пеличипа η = е(х\ — 1\)е(х2 — I2), поэтому EF„(xt, X2) = Εη = Ρ(η = 1) = Ρ(ξ, < χ,. Ι, < χ2) = F(xt, χ,), DF„(x,, χ,) =1θη = 1-[Εη -(Εη)2] = -±f(xt, χ2) (Ι - F(xu *,)). Отсюда согласно неравенству Чебышева P(\F„(xx, х2) - F(xt, х2)\ > ε) < -L DF„(x]t x2) -* 0 при η -* оо. е Обозначим, далее, через Xt = (Хи Х„,), j = 1, 2. Xh S] = S\Xt) соответствующие выборочные средние и дисперсии и через Si2 = 1 " - 1 " = — 2 №1 — %') №s ~~ X*) — — Σ ^ii^is — Х\Хг выборочную ковз- ί=Ι ί = I ривацию. Тогда статистическим аналогом для коэффициента корреляции ρ = cov(£i, b)/^Dl\Db является ρ„ = S12/S1S2. Если Ε(ξ¥ξ1)-=с оо. то существует D(_Ly χ,,χΛ = _!_0(ξ,ξ2) н из неравенства Чебы- шева следует, что 1 " — У ХцХ,г -5- Щ\Ь) при η ->- 00. " /= 1 Но также Χι-*- E|j, S2(X/) -*■ D|;, /' = 1, 2, поэтому p„ ->■ ρ, если только D|, > 0. / = 1,2. «,.--£-,■ 1.39. 1) Если L&) = Λ(μ. ι2), то Ее"1 = е 2 . 2) Если £(£) = Γ(α, λ), то Ее" 3) Если L(v \N) = М(п; р\, ... , ρΝ), то Е(* ι'...*,/') = (хф\ + 4) Если U\) = Π(λ), то Ех1 = г*1 ~ ". 5) Если Щ) = βΊ(Λ, ρ), то E*s = ^ 1.40. Пусть U — ортогональная матрица, приводящая 2 к диагональному пиду: U'^U — D. Обозначим В = l/Dl/2; тогда 2 = ВВ' и У = BJf + μ. где компоненты вектора X незааисимы и нормальны М(0, 1). Далее. Q = Я'В'ДВЛг =.ГД,Лг и по условию Д? = ΒΆΒΒΆΒ = = B'i4B = i4i, т.е. матрица А\ идемпотентна. Следовательно (см. утверждение 2°, п. 6 гл. 1) L(Q) = /2(trΛ,)- Но ίτΛι = tr(ABB') = = (r(i42) = m· что и требовалось показать. 1.44. Совместная плотность распределения Х\ и ^ равна ^, - \xh - I _ л-1 -f-Д-, 1(х\, хг) = ■ . , . е " , л·,, λ·2 > 0. αλ' + λΤ(λ,)Γ(λ2) 145
Рассмотрим преобразование у\ = х\ + хг, Уг = ■ Оно взаимпо- Х\ + Χι однозначно отображает область {х\. хг > 0) на область [у\ > 0, 0 < «2 < 1) и его якобиан J(x,. хг) = . Отсюда по формуле х\ + *2 (1.2) плотность совместного распределения Y\ и Уг равна .Λι + λ, - I -yi/a λι-Ι,, νλ, - ι Γ(λ, +λ2)αλ'+ λι β(λι. λ2) 1.45. Формула для моментов следует из общей формулы для моментов распределения Γ(α, λ) при а = 2, λ = -=- и свойства гамма-функции: Γ(λ+ О = λΓ(λ). В частности, Εχ? = 1, ΌχΊ = 2. Согласно свойству воспроизаодимости гамма-распределеиия χ?ι = ξι + ··· + 1<ι. где слагаемые независимы и имеют одинаковое распределение Γί 2, —1 = χ2(1). Следовательно, по центральной предельной теореме случайная величина (χ2, — п)/^[2п при η -*■ оо асимптотически нормальна JV(0, 1). 1.46. Первое утверждение есть прямое следствие формулы (1.4). Во втором случае имеем Р(Д + tg I < х) = P(s < arclg (χ - α)) = ^-("f + arc,g (' ~ α))· Отсюда искомая плотность распределения равна 1 . ,, > 1 1 — arctg (* — α) = —· , t ^ -j-. π ° π 1 + (* — ay 1.47. Поскольку Ε/Γ = η'Εη2'Ε(χ2)-', при 2г < η из общих формул для моментов распределений N(0, 1) и Гf 2, —J имеем r(f-) Εη2' = 1.3-(2/-D, Ε(χ2)-'=- W -^- - 2'Γ (f) (я_2)(я-4)...(и-20 ' Остальные утверждения о моментах следуют из вида плотности s„(x). Утверждение о сходимости плотности s„(x) является следствием соотно- шепи «"-"Ч^)/г(1)-^и(,+4) ^ -^п-^П Наконец, по закону больших чисел χΐ/η ->- 1. Но тогда ил/η/χί-*! и. следовательно, £(/„) -* Цг]) = #(0, 1) (см. утверждения 1° и 2° в), п. 4 гл. 1). „2 „2 /,„2. ,_ „г.,. „„„„., ρ "S_ X»I 1.48. Положим К = χ2,,/(χ,2,, + Y.h): тогда F„, „2 = -ί- -i£- "' Χί2 л2 У щ 1 - К 146 . Но (см. задачу 1.44) L(Y) = β( —-, -£) , поэтому
(П\Х \ / П\Х П\ Ло\ ^1|."J „ , / ^"1 "2 \ \ ' Л1.Л2 Η Ι- ' Поскольку Υ = : , имеем также равенство £ ρ 4.-22. V 2 ' 2/ при «2 > 2 Ε/"ηι.„2 = —- —, а при ns > 4 имеем Dfni, „ Моменты можно вычислить по формуле EF'niJli = = ( —1 Efxn^'EfxJ,}-', используя формулу для моментов гамма-распределения. При этом моменты существуют лишь при ^- <.г<-~ и они равны EF'„. „ = ( —) - *—|——L . В частности, П2 П2 — 2 2n|(n,+пз-2) П,(П2-2)2(П2-4) ' 1.49. По теореме о среднем значении имеем При 6-+ю и фиксированном а по формуле Стирлинга—=т——- -~ 6е, 1(6) следовательно, ±!п[1-В(*; α. 6)] = In (1-*)--!■ In 6+-J-In Γ("+&) + Второе соотношение является следствием первого соотношения и задачи 1.48. 1.50. Распределение случайной величины /„ = η/Vxv" симметрично (так как распределения —η и η совпадают), поэтому Р(/2„ < *2} = Р(_ |*| < /„ sg |*|) = 2Р(/„ < UI) - 1 или Р(/„ < |*|) = -~- + -~-P(tl < х1}· Отсюда при * > 0 получаем Р'С" < х) = ίγ Р'С" < *2)· τ· е- s«(x) = */ι."(-κ2}· Из этих соотношений также имеем, что Р(/„ > d-φι) = [1 — F(d2n; I, л)]/2. Отсюда и из задачи 1.49 следует указанное предельное соотношение. 1.51. Поскольку l{^(X> + ... + Χι]) = х\21], £ (J-(*/+ ι + ... + + Χι + m)J = X2(2m) и указанные случайные величины независимы, утверждение следует из определения закона Снедекора. 1.52. Обозначим <ρ(*ι χ») = E(*V ..*«*) -=-(х\Р\ + ···+ Χ/ψκΎ 147
Тогда Е(^'...^') = <р(*. **. 1 1} = = (Χ\Ρι + ... + хьрь + 1 — pi — .· — ρ*}2. Далее, ^iri + ... + tv Ε(ν,)»,...(ν„)»„- д^, ^ a<*V Φ(*' **)|x,_..._xw_| и непосредственное вычисление этой производной приводит к искомой формуле. Наконец, N Εη' = Σ с'пР> — nc'< i = 1 Ν Ν coy (η1, η2} = Σ CiC2cov(v,-, ν,} = л J] c}c?p,(l — Pi) — ι. / = 1 i=l /V A/ — «2 C'C/?P'P/ = "( Σ clef ρ,— Σ c'c/PiP;) = i Φ i V = I I./ = I ' = n(c"? - с1?}. 1.53. Обозначим ν* = (vf ν?}, где vf = (vj — np/}/V". / = Ί ··· . k. Достаточно показать, что характеристическая функция Ee''4'-*exp{--i-/'2^}. f = (/. <*). для любого фиксированного t. Из предыдущей задачи следует, что Ев«ч· = e-;^'"P[i + Σ ?fj,lN~n- ijj" ρ = (ρ, p4). Логарифмируя это соотношение и учитывая формулу 1п (1 + ε) = е — ^-+0(ε3}, е ->■ 0, в условиях задачи получаем к 1п Ее"'"' = - /τ/n ί'ρ + я Σ Pi(*"'/V!i - 1) - что и требовалось показать. Наконец, в результате непосредственных вычислений получаем I 2irl = pi ... р*(1 — Ρι — ... — ρ*} φ 0 при k < Ν. 1.54. Для любых целых неотрицательных й|, ..., kN таких, что k\ + ·■· + кц = /ι, имеем 148
Здесь числитель в силу независимости ξι t,N равен Π е-Ч''/Α,-Ι = ^λ Π -ζτ ■ λ = λ, + ... + λ» /-ι ί=ι Λ'! Далее, так как (см.задачу 1.39 п. 4} t (ξι + ... + in) = Π(λ), то Знаменатель равен е~\п/п\. В результате искомая вероятность равна ■т-: т—г Pi — ρ ν, что и доказывает утверждение. Ail... Ам< 1.55. Вычислим безусловные вероятности Ρ (ξ = й), й = 0, 1,... Имеем _ Г(й + а}/ α V + ' 1 _ г* / α V + 1}' ' 1.56. Вектор [Χ, Х\ —Χ, ..., ΧΛ — Χ) распределен по нормальному закону, поскольку представляет собой линейное преобразование .нормального вектора X. Здесь cov (Χ, Χι — Χ) = cov (X, Xi) — D X = = — D Xi — DJf = = 0, i=l n, следовательно, nep- n η η вая компонента не зависит от остальных. 1.57. Пусть U — ортогональная матрица, приводящая Αι к диагональному виду: U'A, U = Di, где по условию η — nt = п2 диагональных элементов Di равны нулю. Введем вектор η = U'X; тогда Л = t/η и можно записать, что Q = ηη' = η'£/'Λ,ί/η + η'£/'Λ2£/η = = η'£>,η + η'ί>2η· где ί>2 = U'AiU = £„ — D\ является диагональной матрицей и rang Di = rang Αι = n2. Отсюда следует, что диагональные элементы матрицы ί>2, отвечающие нулевым элементам Di, равны 1, а остальные — нулю. В свою очередь, это означает, что все ненулевые элементы Di равны 1. Следовательно, матрицы Αι и Αϊ идемпотентны. Из предыдущих рассуждений также следует, что DiDi = 0, а значит, и ΑιΑί = 0. 1.58. Если перейти к нормированным величинам Х\ = —'■ '—.то ς вид η не изменится, поэтому можно считать, что параметры (μ, σ) = = (0, 1). Пусть В — матрица размера η χ п, все элементы которой 1 " равны — . Тогда nS1 = 2 № — %f = ΧΆΧ, где матрица А = Е„ — п ;=1 — В идемпотентна, следовательно, rang Л = tr А = η — 1. Отсюда следует, что φ — 1 собственных чисел А равны 1 и одно рапно 0. Пусть U|,...,u,_i — собственные пекторы А, отпечающие собственному чис- л-1 лу 1; тогда спектральное представление А имеет вид А = 2 "*"*· Ь= I 149
При этом непосредственно можно проверить, что и\ = ~\1 -I η ) у η — t = I Таким образом, обозначая Уц = uJJf, A = 1,..., η — 1, получим представление X, -X Jn ■ Υ>ΝΠ + ... + Yi- причем У| У„-1 независимы и нормальны W (0, 1), которое можно записать также в виде η = Y\/^Yi + 5Й-2. гДе χϋ-s не зависит от У( и t(Хл —2) = χ2(π — 2). Отсюда уже с помощью непосредственных вычислений можно иайти распределение η. Прежде всего заметим, что это распределение сосредоточено на интервале (— 1, 1} (так как η2 < 1} и симметрично (так как распределения —Υ\ и У ι совпадают), поэтому для 0 < и < 1 Ρ (η >U) = J Ρ(η2 > И2) = 1 Р((^У7Г<1^?) = где F(x\ Πι, пг) — функция распределения закона Снедекора S (n(, н2). Воспользовавшись результатом задачи 1.48, можем записать 'Шъг·- ..|)-»(.-*-Ч*4> Таким образом, окончательно имеем, что для 0 < и < I ,,(„,-.-|д(.-в.:^4). Для отрицательных значений и F^(u) = I — /%( — uj. 1.59. а) Совокупность случайных величин (ΛΊ, Х2, Хц — Х{, Хц — Л&, ί = 1, .... п) имеет нормальное распределение, поскольку пред- ставляет собой линейное преобразование нормального вектора (Хп, Хп, /_=!,„., п). Непосредственно проверяется, что первые две компоненты Χι и ^2_некоррел21рованы с остальными. Отсюда следует, что (Х\,Хг) и Ο^α —2(ι. Χα —Χι, ί = 1. ···,«} независимы, следовательно, независимы (ΑΊ, *2) и (S?, S,2, SO. 6} Из предыдущего следует, что £(ΛΊ, Ла) = Λ?((μι. μ2). —Σ) < поэтому, положив в задаче 1.40 Λ = π Σ- , получим требуемое утверждение. -2 2 в) Имеем S? = — (У? + Ъ). S,2 = ^- У,л/^. S| = — У3 и модуль π η η якобиана указанного преобразования -5/2 3 пг 1^1 == Т—5—г" ' ~ t—F^ 0|0|у*2 (σισ2) Vi/з По формуле (1.2) плоность совместного распределения (У|, Уг, Уз) равна 150
φ (ift. №. Уг) = -^ е ''у, 2 е "* у3 2 е "/2 У2л Г (п -2). т. е. представляет собой произведение плотностей распределений Λί(0, 1), χ2(η — 2} и χ2(η — 1} (здесь коэффициент преобразуется к нужному виду с помощью формулы Г(р) Г (р + — J22'1-1 = -у/л Г(2р}}. Таким образом, случайные величины У(1 У2 и У3 независимы н при этом L(Y,) = Λ/(0, 1), £(У2) = χ2(η -2), £(У3} = χ2(η - 1). Отсюда L (Τ = У,/УГ2/(п-2}} = S (η - 2). Г Наконец, рл = — ——, и по формуле (1.3) находим, что плот- Уя - 2 + Г ность распределения случайней величины рл равна , , / Ул — 2j/ \ Ул — 2 r(^i) ·(! - </5} 2 . - 1 < У < 1. VH г(^) 1.60. Как следует из решения задачи 1.29, асимптотические распределения статистик S2 и А„г одинаковы, следовательно (см. задачу 1.28), совместное распределение X и S2 асимптотически нормально. Отсюда и распределение любой их линейной комбинации асимптотически нормально, поэтому достаточно вычислить среднее и дисперсию П С2 разности X Г Используя решение задачи 1.27, находим D <* - ^Т ^) - ™ + (^t)W - -ёп »ν (К S2) = _ μ2 1 / η - 3 Λ μ3 .. 2λ2 Отсюда заключаем, что L (ςη) -* Λ? (0, 1). Но 7"„ = ς„Χ/λ, откуда с учетом указания получаем утверждение. 1.62. Здесь ί,(ξ,) = Λί(μι. σ?) и jb(x) = гщ^ exp ( - ^К} · J»**· У) = 2ла,а2У1-р^ 6Хр { " "2(1^7} Г~3\ _ 2 (* - μι) (у - \ч) + (у -Ы1 1). σιθ2 σΐ J/ Далее, в результате простых вычислений приходим к указанному выражению для условной плотности /t,it,((/U}. II Ε"1 Ο Μ 1.63. Пусть L = . с|2| ~~ матрица указанного преоб- 151
разования. Тогда £()*». У<2>) = *(*.({£>), ί-Σί.') и в результате непосредственных вычислений имеем Μ ΕΙΙΙ П II II V V || /Y/'_- U II *ΊΙ *Ίί ί-Ζί. _ 'ν ν · Π — Л С Ν | | 2,2| 2.22 Ι Ι II Ε'" -Α'|| _ Μ Σ" Οιι II ο ε'2» II ~ II о в II (в частности, это приводит к следующему равенству для определителей: |Σ| = |ΣιιΙ·|Β|). Из структуры матрицы ί,Σί/ следует, что У" и У21 некоррелироваины, следовательно, и независимы. Кроме того, отсюда получаем, что L(Y-2') = Ν(μ2 — Αμν\ В). Но тогда L(Xm\X0) = *(|)) = L(Y(2) + лх<") = М(М(х0)). В). 1.64. По формуле полной вероятности вероятность указанного события Ρ(ξ = Λ}= \Р'(- Σ1π υ· < Λ·Ρ(— In f/* + l > X-t)dt = О ч ;=i ' = ( L—.e-i + 'dt = е~*_ i (A-1)1 ft I 1.65. Так как Ρ (c£/, < / (*)) = /М/с и ι P(ct/, > /(α + (6-α) i/2}} = Ul -_/(e + (ft-e)*))d* = 0 \ с j = I - l/c(b - a). то плотность распределения случайной величины ξ в точке x£[a,b] вычисляется по формуле Глава 2 2.1. Решение сформулированных задач сводится к вычислению средних, а также дисперсий указанных статистик и асимптотическому при η -> оо анализу этих характеристик. Например, в задаче а) имеем /^„(д;) = ', где μ„(χ) — число элементов выборки X = (Х\ Х„), η принявших значение < д;, т. е. ί.(μ„(χ}} = Βί(η, F(x)); отсюда EF„(x) = Εμ„(χ)/η = F(x), DF„(x) = Ώμ„(χ)/η2 = F(x)(l -F{x))/n-*Q при n-> оо, что означает несмещенность и состоятельность F„(x) как оценки F(x). Рассмотрим задачу г). Так как (см. задачу 1.27) Е52 =__μ2 = μ2 + о(_). DS2 -l—iL^- — ^ = = О ( —J при μ< < оо. 152
то S — состоятельная, но смещенная оценка μ2; чтобы устранить смещение, надо использовать статистику — S2 = S' , дисперсия кото- , 2 рой равна f—^—\ DS2 = θ(—V т.е. S оценка μ2. 2.2. Согласно решению предыдущей задачи п. б, ,2 также состоятельная Т„(Х) -5- V<*2/2 = αι^Λ/μ! = α т. е. когда среднеквадратическое отклонение и среднее равны. В частности, это имеет место для распределений Г(а, 1), Ν(α, α2), α > 0. 2.4. Для случайной величины η = ξι + h выборочными данными являются (У, = Хп + ^,2, i = 1 п), а ее дисперсия Οη = D£i + + D?2 + 2cov (ξι, £г). На основании задачи 2.1 п. г) несмещенной оценкой Οη является статистика Σ м - ?f = -г^-г Σ № - *■)' + —Ц- Σ (χ» - ί2}2 + «-'/Г, "-'/Γι "-'( = ! л Σ (*,■■ ->г.)(^ч-Л). 1 Первая и вторая суммы этого разложения являются несмещенными оценками дисперсий Щ| и D^2 соответственно. Поэтому 1 " Е——т Σ №ι - *i) №2 - &) = "ν (ξ,, ξ2). ί = ι что и требовалось установить. 2.5. Для произвольной статистики Т(Х) имеем E07-(AQ= Σ Τ(χ)0Σχ{1 - 0}"-Σ" = χ, = 0. Ι. ί = 1, .... я л = Σ ο'(ΐ-ο)"-' Σ Άχ). /■ = 0 χ : Σχ, = г Здесь правая часть представляет собой полином от 0 степени не выше п. Следовательно, в данной модели несмещенные оценки можно строить S лишь для полиномов τ(0) = Σ α*0* ИРИ s ^ η· к = 0 2.6. Вычислим первые два момента статистики Т. Так как LJ,r„) = = Bi(n, θ), то EJ = |— (Е0л„ + α) η0 + α n+β ι+β J ("Τβ? Ε»7-2 = ,- , «Λ (°»Α" + (Ε»Α")2 + 2αΕ°Α" + α2) η(η- 1)02 + (2α+ l)fi0 + ct2 (я + β}2 153
Отсюда Δ(ο, β; θ) = Εο(7· - θ}2 = Ε0Ρ - 2ΘΕ07· + θ2 = θ2(β2 - η) + θ(η - 2αβ) + α2 (« + β}2 В частности, Δ(0, 0; θ) = D„(-^-) = 6('~θ) , δ(^. τ/λ; θ) ι не зависит от θ. 4(τ/ή + Ι}2 η τ, r-„ + Vn/2 Рассмотрим оценку / = —ь—. Ее среднеквадратическая п +V" ошибка меньше ошибки несмещенной оценки Т* = —— (т. е. == < η Ц-φι + ΐγ < -) при θ £ I —-± ν 1—■—1 · длина этого интервала стре· n \2. 2(V^+1)/ мится к 0 при п-+оо. При остальных же значениях параметра θζ(0,1) более точной является оценка Т*. Таким образом, по критерию средне- квадратической ошибки оценки Г и Т* не сравнимы, и чтобы выбрать какую-то из них, необходимы дополнительные рассуждения. Например, можно принять правило считать ту оценку лучшей, для которой максимальное значение среднеквадратической ошибки меньше (принцип минимакса). Поскольку тахв(1 — θ) = — и < ——, согла- о 4 4θ+1}2 4п сно принципу минимакса оценка V лучше оценки Т*. 2.7. Условие несмещенности ЕаТ(Х) = τ„(θ), V6 6(0, 1), принимает в данном случае вид ь Σ 7-(/}αθ'(1-θ}*-< = 0'(1 -θ}', V6€(0, 1). I = о При любом Τ выражение в левой части этого тождества представляет собой многочлен от 0 степени не выше k, следовательно, указанное тождество может иметь место лишь при г + s < к. Далее непосредственно проверяем, что указанная в формулировке задачи статистика удовлетворяет условию несмещенности. Покажем, наконец, что это единственная несмещенная оценка в данной задаче. Предположим, что Т'{Х) — другая несмещенная оценка. Тогда статистика Т\(Х) = = Т(Х) — Т'{Х) удволетворяет тождеству * У/,(/)С1<У(1-0)*-' = 0, 0<0<1, i = о * θ или У 7"i(/)Cu' = 0, 0<д:< оо, где д; = - ^. Но из тождест- 1-0 ' - θ венного равенства нулю многочлена следует равенство нулю всех его коэффициентов, т. е. Т'(}) = T(j) для / = 0, 1, ... , к. 2.8, Согласно свойству воспроизводимости биномиального распределения L^T) = Bi(kn, θ), поэтому kn Е0Я(П= 2 Я(/)С1„<У(1-0)*"-'. i = 0 154
При любой функции Η это среднее представляет собой полином от θ степени не выше kn, следовательно, несмещенные оценки вида Н(Т) в данной модели можно строить лишь для функций вида τ(θ) = s = 2 а>®' ПР" s < ft'i· Пусть τ((0) = 0', / < kn, тогда, поскольку ' - ° Еп(7"}; = C"i};0' (см. задачу 1.52), несмещенной оценкой для τ,(0) является статистика if = (T)j/(kn)j. To, что это единственная несмещенная оценка, являющаяся функцией от Т, устанавливается так же, как аналогичное утверждение в предыдущей задаче. 2.9. Первое утверждение следует из цепочки равенств Чтобы убедиться в справедливости второго утверждения, запишем условие несмещенности ЕаТ(Х) = τ(0) V0>0, в виде Σ ?W-V= Σ-τγ vo>o. Ясно, что не существует не зависящей от 0 функции Т(к), удовлетворяющей этому тождеству. 2.10. Условие несмещенности Έ.αΤ(Χ) = τ(0) V">0 принимает в данном случае вид 2 T(k)^- = e\i -О2 = *' + '-" -2 = 2 2 "^ V0>0. к = 0 ' г = 1 ^ ·'· Единственная функция Т(к), удовлетворяющая этому тождеству, имеет вид \2 при k > 0 четном, остальных случаях. Такая несмещенная оценка Т(Х) практически бесполезна. 2.11. В дампом случае условие несмещенности ™-ег Σ 7W С", + „ _ ,0*(1 - 0}' = 0s V 0 6 (0, 1} к = 0 можно переписать в виде 2 7-(ft)C? +*-,()*= °' = 2 C{ + i-i0' + 'V06(0. Ι}· ь - о (' —ϋ; / = о Из тождественного равенства двух степенных рядов следует равенство соответствующих коэффициентов, поэтому единственной функцией Т(к), удовлетворяющей этому тождеству, является функция T(k) = C' + i-.-i/C' + i-,, k = 0, 1 Отсюда следует, что единственной несмещенной оценкой в данной задаче является статистика (0 при X^s—\, \(Xl/(X + r-l)s npnX^s. 155
Если г = 1, то эта статистика принимает лишь два значения 0 и 1, не принадлежащие параметрическому множеству модели θ = (0, 1), поэтому она практически бесполезна. 2.13. Так как ЦХ) = n(q. —\ , то Ε„(Ύ2} = DeX + (E<,Xf = η + θ , откуда и следует утверждение. 2.14. Для распределения νν(μ. О2} второй и четвертый центральные 4! 2Ϊ27 моменты соответственно равны μ2 = θ2 и μ4 = '; б4 = 3Θ4, поэто му (см. решение задачи 2.1) E^ = .lnie', Do52= ^т'У Отсюда имеем , θ4 2n- + T" = ^~ Εβ(τ* - θ2)2 = Dot* = 1 D^, - μ)2 = -^=^- = £ Θ4. Таким образом, Eo(S2 - О2}2 < D0T* < Do(S'2}. т. е. согласно критерию минимума средиеквадратической ошибки статистика S2 точнее оценивает теоретическую дисперсию θ2, чем статистика т*, ио в классе несмещенных оценок т* точнее, чем S'2. 2.15. Так как La( *'Γμ) = W(0, 1), то EoU, - μ| = 0-1- \ \x\e-*'4x = θ~λ/ — f xe~*~dx = θ~λ/ —, Dol^-μΙ = Ε0(Ύ1-μ}2-(ΕοΙ^-μΙ}2 = -^^-θ2. несмещенность; ί = ι состоятельность. Отсюда £0ЗД = \γ~ Σ EalX' - μΙ = θ - ί = 1 ΟοΓ„(ΑΟ = |..1.00№-μ|=^θ2 = θ(1.) - 2.16. Из того, что ί.ο(Γ/θ2} = χ2(η) = г( 2, γ), и из формулы для моментов гамма-распределения следует, что E^-B**r(i±±)/r(i). т. е. несмещенность указанной оценки. 156
Чтобы сравнить оценку nf для θ с оценкой Т„, полученной в предыдущей задаче, надо вычислить D0Tf = E^if)2 — θ2. Имеем Ε<ι№ '(τ) (здесь использована формула Г(х + 0 = *Г(х)), следовательно, D0tf л-2 2п m 1) θ2. Это выражение надо сравнить с D07"„ = θ2 (см. решение задачи 2.15). При η = 1 обе статистики (и их дисперсии) совпадают; D0tf = ( — — 1)θ2 я; 0,273·θ2 при η = = 2, a DaTi = ί — —) В2 «< 0,285· θ2, т. е. оценка if точнее, чем Τ г (здесь учтено, что Γί — 1 = л[к. Г(1) = 1); аналогично, D0Tf =(-5 — 1)θ2 ss 0.178Θ2 < D07\j « 0.190-02 при η = 3. Доказательство того, что оценка if при любом η точнее оценки 7"я, следует из результата, полученного в задаче 2.64. 2.18. Средиеквадратическая ошибка произвольной оценки 7Λ равна Ео(7\ - θ22)2 = EoMS'2 - θ!) + (λ - l)6l)2 = *2De(S'2) + (λ - 1)4 = <^λ)θ5 2λ2 (см. решение задачи 2.14, где вычислено Do(S'2)), φ(λ) = п __ [ + + (λ — I)2. График функции <ρ(λ) изображен на рис. 7. Поскольку φ(λ)<φ(1) для ——-ρ < λ < 1, при этих значениях λ Ee(7\ - θ|)2 < Eo(S'2 - θ?)2 п-2, η Для определения k имеем условие —r-l—— < ■< которому л+ 1 η + * удовлетворяют лишь значения k = 0, I, 2, 3. Наконец, min Е^Т"; _2_ /7-7 2 л+7 Л-.7 л*7 ZLZ. л+7 / /? Рис. 7 157
о — О2)2 = φ(λ*)02 = -О4, и, следовательно, оптимальной по крнте- п -\- 1 рию минимума средиеквадратической ошибки является оценка 7V = 2.19. Из решения задачи 1.45 имеем Ео7\ = А.0г, Ε07"ι = λ —_ . 9г, UTl = ,з("+1)(п + 3) = λ<(η+1)(η + 3)(η + 5)6. (η - Ι)2 (η - Ι)1 Отсюда мера 6,(θ) = Εο(7·χ - 0|)4 = Έ.ΙΤΙ - 47-gei + 67|Θ5 - 4ΛΒ! + θ!) = Ψ(λ)β|, где ψ(λ) = λ<(η+1)(η + 3)(η + 5)_4λ3(η + Ι)(η + 3) λ2^Μ,_ (я - Ι)3 (η - Ι)2 η - 1 -4λ+ 1. Уравнение Ψ'(λ) = 0 с помощью подстановки λ= —|Η 1 η -\- 5 \ η/ приводится к виду *» + Зр.* + 2* = 0. />» ="7^· «7» = -(η + 18"(η + 3)· Поскольку дискриминант D„ = ρϋ + о\ > 0, это уравнение имеет единственный действительный корень *« = V-9» + -fD" + V1?» - Va, . (формула Кардана), для которого при больших η справедливо асимптотическое представление х„ = ——|- 0(—^). 3 \ V" / Таким образом, оценка, минимизирующая в классе статистик (Τλ = \Sn] меру δ,(θ), имеет вид Т* = ^-^-(1 + — V2. п + 5 \ η / Рассмотрим теперь вторую меру δ2(θ) = Е0|7\ - 0|| = χ(λ)θ|, где χ(λ) = Ε I -^грс?,-. - 1 | , i(xJ_i) = У.> - 0· Если Лг„ _ ι(/) — плотность распределения χ2(η — 1), то л -1 xw = 5 —^-r-t - 11 k„-,(i)dt = j(i —Ц-/)йл_,(/)л + о " l ' о "—ι + 1 {-7Г~Г' - П*--|('У". /1-1 λ 158
Отсюда следует, что уравнение χ'(λ) = 0 эквивалентно уравнению \ tkn-i{t)dt = \ /*„-i(/)d/, О /ι- Ι которое определяет единственное значение λ* = λί, а тем самым и оптимальную оценку Т%*. 2.20. Поскольку L^nS1/di)=x1(n — 1) = Г (2, "~ ), из формулы для моментов гамма-распределеиия имеем «* - >· (**)*- ^{Щ^Н^г-)- откуда следует несмещенность указанной оценки. При η = 2 S = I 0\Х\ — Χϊ\ и поэтому ^ '*, -ΛΙ*. 2*-/*± (Τ') Отсюда EolA-, — А-2| — Θ2 = 2 /π62. Vn 2.21. Утверждение непосредственно следует из того факта, что LJJ) = Г(0, in), и формулы для моментов гамма-распределения. 2.22. Если L^l) = Г(0, 1), то Щ/в) = Г(1, 1) и по задаче 1.34 случайные величины У, = (-¥(,) — ,¥(,_])), г = 1, ..., п, независимы и L&y,) = Г(1, 1) для любого г. Отсюда (см. решение задачи 1.34) Xw = θ £ У/Л" - / + 1) и поэтому Τ = 7ХХ) = Σ λ**№ = 0Σ -JV, , У». Λ< - Σ ^. * = 1 г 4-1 I-I "г *-,' Из этого представления сразу получаем, что EJ = θΣΛ,/(η - ί + 1), D07· = 02ΣΛ?/(η - l + Ι)2 ί-Ι i-l г Условие несмещенности эквивалентно условию "£А</(п — i + 1) = 1. ι = ι при котором надо минимизировать выражение ΣΛ?/(η — i + I)2 ι- ι Метод неопределенных множителей Лагранжа дает следующий результат: оптимальный выбор Λί таков: Λ,* = , ί = 1,..., г Окончательно получаем, что оптимальная несмещенная оценка 0 имеет вид 159
τ* = i-2 υ, = i-2(n - « + !)№,■, - *„_„) = -J-2^0 + -V1^)· η τ* °2 и ее дисперсия D0/* = — . 2.23. Из решения задачи 1.36 следует, что Ό J = aaD„*<,) + P'DoX,,, + 2аЭсоу(А-(„), *<„) = " (η+ !)»(„+ 2) [α +β +—)■ Отсюда имеем, что оптимальными являются значения α и β, минимизирующие форму α2 + β2 + 2αβ/η при условии a'fin + \)/{п + 1) + + β(η + 2)/(ч + 1) = 1. Решение этой экстремальной задачи (например, методом неопределенных множителей Лаграижа) имеет вид 2(я+1) п+1 _ α = ——г~г-· β = τ—г~г· Таким образом, оптимальной несмещен- 5п + 4 5п + 4 ной оценкой 0 в рассматриваемом классе оценок является Т* = = 1Й7<*" + 2*">} " еС ДИС"ерСИЯ ^ = (п+2?5п + 4)· 2.24. Несмещенность указанных оценок непосредственно следует из О2 л задачи 1.36. Далее, D07"i = —-.——τ- < D07"2 = —г-^-°2. τ· е· оценка П[П~\-2) rt + 2 7Ί точнее. Более того, Dq7"i -> Q при η -»■ оо, т. е. оценка 7Ί состоятельна. Оценка же Тг не обладает этим свойством. Действительно, так как РоМо < /) = 1 - ( 1 - 1-Л\ 0 < / < 0. Prfir, - οι < «) - Ρ-(4ϊτ < '"> < 4ΐτ) - _ Λ _ e~E V _ Λ _ ° + Ε \ " —(ο —,ι/ο _ —(ο + .ι/ο ,, ι - \ о(п+1)У ^ ϋ(η + ιν 7^ 2.25. Из формул задачи 1.36 непосредственно следует несмещенность указанных оценок и следующие выражения для их дисперсий: D07", = i(D„*0) + ОД,„, + 2cov(*0). ХМ))= 2(η(^7χη + 2) ' DJ2 = ^^±1V(D„X(1) + DoAfw - 2cov(A-(1). XM)) = 2(62-Q,)2 (n_i)(„ + 2) · При η -*■ oa эти дисперсии стремятся к нулю, т.е. обе оценки состоятельны. 2.26. Из формул, приведенных в решении задачи 1.37, непосредственно следует несмещенность указанной оценки и тот факт, что Dq7"-»-0 при η-*■ оо, т.е. ее состоятельность. 2.27. В данном случае теоретическое среднее совпадает с 0. поэтому результат следует из решения задачи 2.1 п. б). 160
2.28. Согласно свойству среднего арифметического для распределения Коши L((X) = К(0), т. е. распределение статистики X не зависит от η и поэтому величина Ρο(ΙΎ — θ| > ε)одна и та же для всех п. 2.29. Из задачи 1.52 следует. Е.7-, = р„ VJ, = ΛίΕ»ν,(ν, - I) + E„v, - (Ε0ν,)2] = Ρ'('~Ρ,) - 0 η η при η -*■ οο , что доказывает утверждение п. а). Далее имеем Ε·»<Γ Ч-.?..-."(^ ^)га'''··* Здесь при любой функции Η правая часть представляет собой многочлен от ρι, ..., ρ и степени ^ л, следовательно, несмещенные оценки в данном случае можно строить лишь для многочленов степени < л от параметров р\, ..., рц. [ N N Наконец, если Η = —Σ£,ν'· то ^α// = Σε'Ρ< — τ(^). DoW = п i-i ί-ι ±(i*- τ (0))-»-0 при л -»- оо, т.е. // — несмещенная н состоятельная оценка τ(0). 2.30. Так как а, = а2(0) = О.Г(02 + 1)/Г(0я) = 0]0а, <*2 = а2(0) = = 0?Г(02 + 2)/Г(02) = Ο?θ2(02 + 1), откуда 0, = (о2 - а? /о,, 02 = = а2/(а2 — ctij, то искомые оценки имеют вид 0, = (Аяг - Α*,)/Α„ι = S2/X, 03 = Ah/(Am - Λ5ι)= *7$2· Этн статистики представляют собой непрерывные функции от выборочных моментов, поэтому они являются состоятельными оценками соответствующих параметроп. 2.31. Здесь о,(0) = Е„| = -ί(0, + 02), о2(0) = Е0|2 = у(02 + + Ог + Οι + 02) и решением уравнений ajfi) = /4„», к = 1, 2, являются оценки Οι. ι = Л„, Τ -jA„t - Λ2, - Λ„, = * + VS2 - * · Для приведенных данных 0( = 2,17 ..., 02 = 3,57... . 2.33. Если D ^ /г0, то условия несмещенности эквивалентны системе уравнений о 2ДА)/(*; D, л) = τ(0), D = О, 1 А0. 4-0 Это треугольная система, в которой диагональные коэффициенты f(D; D, л) = C%zPD/C% φ 0. Следовательно, значения Т(0), T(l) Т(к0) отсюда определяются однозначно. Для значений же m > ka полагаем T(m) = Гт(т) - Σ T(k)l(k\ т. яЛ/Γΐ - Σ Ρ: т. ιι)1 . L 4-0 -* ■- *-о J 6-190 161
Α. что возможно, так как I — Σ /(*! т, η) Φ 0 при т "> ка. »=.о Далее, поскольку Σ АДА: о, п) = J£. 4-0 '* (см. формулу для среднего гипергеометрического распределения H{D, Ν, η) во введении к гл. 1), для случая x(D) = D функция Τ имеет вид T(k) = kN/n при k = О, I, .... fto и 7-(«) = \m - ^ 2 */(*: m. /ijl / fl - Σ /(*: «· ")] L 4-0 ·· L 4 = 0 J при m > ka- В частности, если положить ft0 = η (контроль всей партии не предусматривается), то несмещенной оценкой для числа D дефектных из· /V делий является статистика 7"(ξ) = —ξ. η 2.34. а) Поскольку (см. указание) Εγ(ΐί) = P(u e s) = л(н), из представления e(s, χ) — ^y(u)x(u)/ii(u) следует несмещенность оценки Горвица — Томпсона. Далее, так как Σ о(и)х{и) = Σγ(«Μ")*(")> то uej и условие несмещенности такой оценки означает, что Σπ(ίφ(ίφ(κ) = Σ*(") V* е Λ". Μ li Выбирая, в частности, в качестве χ координатные векторы евклидова пространства /?", получим, что л[и,) а{и,) = 1, ί = 1, .... N. Таким образом, оценка Горвица — Томпсона является единственной линейной несмещенной оценкой для Т(х). б) Так как ?(s, χ) = Σ ν(")4(ν)χ(αΗυ)/φΗν) + Σν(")*2(«)/π2(") и Εγ(")γ(υ) = P(u esjes) = n(u, ч), « ^ u, то De(s, x) = Ee2(S, x) - (Εφ. χ))1 = Σ ^"^ «ΜΌ + + Σ*2(")/*(") - (ς^"))2· что эквивалентно указанной в формулировке задачи формуле. в) Несмещенность следует из представления Μ^) = Σν(^(^-ι) + , „ ... v x(u)x(v) / n(u. υ) Λ 162
г) Указанные формулы являются пряным следствием представлений «Μ = Σν(Ά n\s) = Σ viuM") + Σν(<4 2.35. а) Для любого фиксированного элемента и существуют η(Ν — 1),_ι различных выборок, содержащих этот элемент, поэтому φ) = η(Ν- 1)„_,/(Л0„ = ^-. Ь) Формула для Dx следует из общей формулы для De(s, χ) (см. п. б) предыдущей задачи), если учесть, что в данном случае . . л(л-1)(Л/-2)„_2 n(n-l) π("·ϋ) = (до. =-Щ=ТТ- и Ф ΰ· в) Несмещенность o3(s, r) следует из представления S\s, χ) = ~-Σ v(")W") - μ)2 - „,„!_ η Σ ν(«Μ")W«) - μ)Μ») - μ)· 2.36. Вычислим сначала Εμ,. Используя указание, имеем Εμ,= = ΝΈ.ΐ\'] = NP(l\r) = 1), где, согласно классическому определению вероятности, Р(й"= i)= ^(Сг!г(СХ_,Г-'/(СТ = c;(-^-)'-(i - -J)""· Окончательно имеем ■-««(тУО-тГ Отсюда следует, что среднее любой линейной статистики имеет вид Ε'=».5/·«(ΐ)'('-!Γ· т. е. представляет собой многочлен от -тт степени ие выше η — 1. Это означает, что несмещенные оценки в классе L могут существовать лишь * для параметрических функций вида τ(Λ/) = Σ c-,/Ν1 при k^n—l. /-ι Пусть τ(Λ/) — такая произвольная функция. Тогда условие несмещенности означает, что Отсюда следует, что коэффициенты /, искомой оценки однозначно определяются через коэффициенты С\. То, что I, имеют указанный в формулировке задачи вид, можно проверить непосредственно, учитывая (см. задачу 1.52 п. б) формулу 1М1)'('-тГ=Ч1)'<='·*·-· 2.37. Найдем сначала распределение случайной величины η. Обозначим через Л, событие, состоящее в том, что ί-й элемент не б* 163
наблюдался (ί = 1 /V), и пусть μ0(η, m, /V) = Ν— η — общее число ненаблюдавшихся элементов. Тогда по формуле для вероятности суммы событий [2, с. 109] Ρ(μ„(η, т, N) > 0) = р( Μ Α,) = £ (- 1У+' Σ ΡΗ*/-»^· Во внутренней сумме все слагаемые равны (С™_/)"/(С™)", а их число равно C'N, поэтому РЫп,т. N)>0) = Ei-O'+'C'^C^^ACX Далее, Ρ(η=Α) = Ρ(μ0(η, т, /V) = Ν - ft) = Σ Р(<4,1..Л1-„_И/1-ЛУ. Ι<ι,<...<ιν-*<" где {/ι /*) = {! Λ^)\{'Ί iv-tl· В последней сумме все слагаемые равны друг другу, а их число равно С% По теореме умножения вероятностей Wil...AIN_Jil...Ajl) = P(Ail...Alll_liP(Aii...A,k\All...Airl_J. Здесь p(/i,l..yilv_j = (CZr/(Q" Η ^„...^М/,..^/*-J = РЫ«. m. A) = о) = χ (- lycKcp-jJViCD". 1-0 Из этих соотношений окончательно получаем, что Ρ(η=έ) = C*v 2 (- 1)*"'С1(СП7(СХ)", * = m, m + 1, .... min(m n, /V). Пусть теперь N < mn, тогда Ετ'= Σ τ'(*)Ρ(η=Α) = (СХ)- Σ С« Σ (-"Г'СЦСПМЛ = Α — m A = πι у — m = (св- Σ t(/)(C7)"C'v s'i-irc'v.^xiw), /=m л - 0 так как v'c iw - /°· ΠΡ" '<Λ/· ,±V IJL"-/-1 I при j = N. Если же N~>mn, то, учитывая свойства функции /(/V), можем записать, что Ет' = (Сэ-£ с*2(-|)*--'С1/0-). 4=0 ,=0 Для этого достаточно убедиться, что Σ (-1)*~'С1/(/) = 0 при /г > тп. 1-0 В свою очередь это следует из цепочки равенств Σ (-\)к-'сы = (ft),*ς'(-ι)'-γ_Ώ_, = о, г<а. /-0 s=0 164
Теперь имеем Ex' = (Q- 2 ЯЛС'/ς '(- 'ГС'*-,- = (C$-f(N) = τ(Ν). I - α г - α 2.41. Так как для схемы повторных независимых наблюдений распределения векторов X и лХ совпадают, то ЕаТ' = Ео7" = т,т. е. Τ — несмещенная оценка τ(θ). Пусть теперь ΌαΤ{Χ) = б2, тогда и Do7"(ilK) = = δ2, а согласно неравенству Коши — Буняковского cov0 (ЦтцХ), Цп2Х)) <С ^OJ^XlDJimX] = δ2. Отсюда Do7" = T^vr \Σ DJ(nX) + Σ cov° №'*)· ЦпгХ))] < ("■У 1- л л, * л, J ^ ., н!+н!(н1-1) _., <δ—щ ό· Таким образом, для любой несмещенной оценки можно указать симметрическую несмещенную оценку, дисперсия которой не превышает дисперсии исходной оценки. Следовательно, оптимальную оценку (когда она существует) надо искать среди симметрических функций наблюдений. 2.42. I) Рассмотрим статистику Τι = Τ" + λψ, которая при любом λ является несмещенной оценкой τ. Тогда, в силу оптимальности Т, ЭоГц = Dor + b2D0t|> + 2λΰον0(Γ, ψ) > D07". Но это возможно при всех λ только лишь, если cov0(T*, ψ) = 0 V0- 2) Пусть Τ — произвольная несмещенная оценка τ. Тогда статистика ψ = Τ' — Τ имеет нулевое среднее и на основании предыдущего О = covo(7"*, Г - Τ) = D0 Г - cova(f. η. («- д)' 2.43. Для модели Λ/(θ, δ2) функция f(x; θ) = ———е 26' . От- γ2πδ сюда Txrlnftx; θ) =—j- и поэтому ί(θ) = -ττ· Для модели Ν(μ, θ2) ου о о имеем \2 Д Infix Ш 3(*~μ) ' откуда F^'-^-^-i Для модели Γ(θ,λ) функция /(χ;θ) = щ^с— и . _j-|n/(x: 0) = · Ίχ λ 2 λ λ = -тя οΓ· Отсюда ί(θ) = -^з-ЕД| — —5- = —,-. Для модели Коши д , tl „, 2(χ-θ) ao ■"/(*:") = , ^(x_e)». поэтому .... 4 г (χ-θ)2 . I , .«, I ,W = lTJ,[i+(x-eff<<x=ararctgxU-°T· 165
^2 Для биномиальной модели j(x; 0) = С'к0'[\ — О)*-', :^Нп/(*; °) = <30 X k — X = -тлг + yj ηττ-. следовательно, Для пуассопопской модели f(x;Q) = e~G—-, -гр-1п/(л:; 0} = -^-, следовательно, i(0)=JrE0X,=i-. Для модели В((л, 0) функция /(лг; 0) = Ci+I_i0*(l — 0}', ή2 X г — _^j-!n/(x; Q)=-^r+ (1 _Q.a , следовательно, 0(1-0) ' (I-О)2 0(1 — 0) 2.44. В данном случае И(«: о)= ~(*2^j0l)' - ш(о«л/2й) и -E»(^c5c7ln№;°3) = iE^'-0') = 0· Этот факт с учетом предыдущих результатов приводит к указанной в формулировке задачи формуле. 2.45. Вид матрицы /(0) следует из формул ΕΛξ,α,) = РЛ = «О = р, _^J^L^J(^i_+M^L·, σ/7, Pi ρ н a'lnffi.g,) 6(1, α ύ — = J , I =f=- J ', δρ,όρι Ρ Ν непосредственно можно проверить, что /_|(0) = Σκ_ι, где матрица Σκ_ι определена в задаче 1.53. 2.46. Пусть F — экспоненциальная модель. Тогда при 0 = (0ι, ..., 0,) Uix-, θ) _ *4„фг; θ) = Σ 4г**КХ.: θ) = -^-TW + n-dC(6) (30, κ ' ' ~\ «30, n " ' d0, v ' ' d0, ' где T[X) = 2 fi(-Vi). Отсюда, полагая a,(0) = ("-^ί ) . /= I '. можем записать, что Обратно, если имеет место представление а'(0){/(Я; 0} = Т„(Х) — τ(0) при некоторых а(0) = (αι(0), ..., α,(θ}), 7"„(Х) и τ(0), то, в частности, s«wa'nffi0) -ад-ко). 166
Отсюда следует, что функция {(χ; Θ) имеет указанный вид. Чтобы получить формулу для дисперсии 00τ*, заметим, что jrj&_ = К(ж)а1пц«;е)цх. β)ώ = E<iAX)Uj(X. θ)) = = cov„(t*(*), £/,(*; θ)), так как Е0£/,-(ЛГ; θ) = О V θ. Отсюда Ι ν дт(в) / (3/1(0) . . ,....,.ν ... Τ 2,-авГ/ ~δδΓ = COVo(T' (' ( ; θ)) = = cov0(t\ τ* — τ(θ)) = D0t*. 2.47. Так как дисперсия эффективной оценки совпадает с границей Рао—Крамера, то из предыдущей задачи имеем соотношение , = Α (Θ) — - откуда следует указанное выражение для ί(θ). Далее, т ■Щ = _ Е0 ΰ?Ιη^: θ) = -А'ЩЕ^вт - С"(0) = -£Ц-/Г(6) - С'Щ т.е. Е.В(|)=-С/(в)/Л'(в). 2.48. Проверка состоит в^ прямом применении результатов задачи 2.46. Например, для модели В!(г, Θ) 1{х; 9) = exp[xln9 + /-ln(l - Θ) + In CJ+I_,1, т. е. /1(θ) = Ιηθ, В(х) = х, С(в) = л1п(1 — Θ). Следовательно, τ(θ) = -СЩ/АЩ = ,θ/(1 - θ), χ" = X. О*' --Σ^--^-^-. 2.49. В рассматриваемом случае г^—-=2/(χ;θ), откуда σο /(e)=2J>^=2I(iw4· Далее, 00 00 л D„? = Ео(| - Θ)2 = \(х- θ)2/(χ; 9)dx = J х*е~'(\ + е~х)-^х = у. — 00 — 00 Отсюда Зп пЩ η 2.50. Утверждение следует из равенства 1 г 2θο2 at . σ4 ΰ21 Τ -, σ2 1 г 2Ио' dL LI η <?θ +' ΰθ' θ2 и критерия Бхаттачария. 2.51. Иэ задачи 2.21 следует, что ЕВГ = θ2, θοΓ = } „V* "*" J θ4. Нижняя же граница Рао—Крамера для функции τ(θ) = θ1 равна (см. задачу 2.43) Θ1, что меньше Do7", следовательно, 7" не явля- λη 167
τ[ ется эффективной оценкой т(0). Оптимальность Т" следует из равенства Ι Γ2Θ3 6L О* d1L-i_ η _г 2 кп дв + кп(кп + I) 50' J — λ(λη+ Ι) Χ и критерия Бхаттачария. 2.52. Оптимальность указанных оценок следует из равенств Θ2· d\nL - „ ■ = Χ — θ|, П <5Θ| ι / ej at ej d2L\_ ι " L\n-\ ΰθ2 η(η-ι) "аё77 η-ι ,^,1* ι ΰθ2 η(η—ι) aef и критерия Бхаттачария. Информационная матрица модели Ν(θι, θ?) вычислена в задаче 2.44., откуда получаем, что граница Рао—Крамера для функции ΐ|(θ) = θ| равна 02/л, что совпадает с D^X, т. е. X — эффективная оценка τι(θ). Для функции же тг(0) = вг эта граница равна 202/". что меньше Do(S'2) = 2в*/(п — I) (см. решение задачи 2.14), т.е. S'2 не является эффективной оценкой Τ2(θ). 2.53. На основании предыдущей задачи наилучшей оценкой для среднего является выборочное среднее объединенной выборки, т. е. статистика Х =—(п\Х\-\-пгХг), где η = πι + пг, при этом д2 DoX = —-<mm(D0Xi. D^Xi) = 02/max(n,, nt). Аналогично, наилучшей оценкой для дисперсии, учитывающей всю информацию, является статистика Но nA„i = гцА^д + П2-4™2, где Л',?* — выборочным момент пторого порядка i-й выборки, /= 1,2. Из формулы *' = -^Ьг(АУ12-Х?) имеем п,-/4<^2 = (пу—l)S/J+ η,·λ?. Отсюда окончательно находим, что S'2 = -^ГГ*" - ')Si2 + («2- I)552 + Щ*? + n2i*2 - пХ2). При этом (см. задачу 2.14) 2 „4 _...._,«, г.,1, «,о,2^_ 29J Do(S'2) =—Ц-θί < min (Do(S[2). D„(S2: I max(fi|, Пг) — I 2.54. I) Пусть λ известно. Рассматриваемая модель является экспоненциальной, для которой (см. задачу 2.46) В(х) = х, /4(0) = λ λ = —ζζτ· C(ty =-гГ* где θ = μ. Следовательно, в данном случае эф- 2Θ U _ фективная оценка τ' = X, а соответствующая параметрическая функ- I Ι μ3 ция τ(θ) = μ. При этом Dt' = — DX\ =—. ,,.х =-Ц^. Если λ неиз- ^ г η пА (0) ηλ вестпо, то, вычисляя US
dlnL kn - и применяя критерий Бхаттачария, получаем утверждение. 2) В данном случае мы имеем экспоненциальную модель с Β(χ) = φ- + ~Γ.Λ(0)=-γ. С(0) = £ + у!пО, где 0 = λ, и согласно решению задачи 2.46 статистика τ* = I п / X I \ 12 = — У (—<г + -у-) есть эффективная оценка для τ(0} = —Η . 2 Следовательно, искомая оценка имеет вид τ* . μ 2.55. При Vе 6 Rm согласно неравенству Рао—Крамера для скалярных оценок где *(0) = ( D„(cT) > 6'(θ)/„-'(θ)6(θ), ■ 5τ(θ) 5τ(θ} ч _ / - ее. ее, ) К** .... Σ **gSW (5τ,(0) Й0| Таким образом, c'D0(7> > с'В'(в)/„-'(в)В(в)с или c'[Do(r) - Β'(θ)/«~ '(θ)Β(θ)]ρ ^ 0. Это и означает, что матрица Do(7*) — Β'(Θ)/,Γ'(Θ)Β(Θ) является неотрицательно определенной. 2.56. Если для τ(0) существует эффективная оценка, то модель является экспоненциальной (задача 2.46). Следовательно, Цх; 0) = ехрИ(О)Дж) + пС(9) + Σ D(x.-)). Π*) = Σ Β(χ,), 1-1 /-1 и согласно критерию факторизации Т(Х) — достаточная статистика. 2.57. Достаточность статистики 7\, следует из задач 2.56 и 2.48. Проверим ее полноту, т.е. покажем, что условию Ε0φ(7'„) = 0 V6 е е (0, I) удовлетворяет лишь функция φ, для которой <р(/) = 0, / = 0, \, .... гп. На основании свойства воспроизводимости ЦТп) = Bi(kn, G), поэтому е«ф(Гя)= StfOCkoO-e)"-' 1-0 эквивалентно условию ι Σ ф(0С*„*/ = 0 V* > 0, χ = и условие полноты эквивалентно условию *" О I —θ Но из тождественного равенства нулю многочлена следует равенство нулю всех его коэффициентов, что и требовалось установить. Из пол- 169
йоты статистики Тл следует, что в данном случае несмещенные оценки существуют лишь для таких параметрических функций τ(θ), которые имеют вид ЕвН(Тл), т. е. представляют собой полиномы от θ степени не выше kn. В частности, несмещенной (а следовательно, и оптимальной) оценкой степени Θ' при j ^ kn является статистика (Tn)i/(kn)j (см. задачу 1.52), а из линейности свойства оптимальности следует, г что несмещенная оптимальная оценка полинома τ(θ) = Уа,в'прил< 1 = 0 ^ kn имеет указанный в формулировке задачи вид. Этот результат обобщает и усиливает результаты задач 2.5, 2.7 и 2.8. 2.58. Достаточность статистики Тл следует из результатов задач 2.56 и 2.48. Имеем, Ldja) = Π(ηθ) и поэтому условие полноты экви- валентно условию Σ фМ — = 0 Vе > 0- Но из тождественного 4 = 0 *" равенстпа нулю степенного ряда следует равенство нулю всех его коэффициентов. Таким образом, этому условию удовлетворяет лишь функция φ, для которой <р(й) = 0, k = 0, 1,2 Это означает полноту статистики Тя. Из задачи 2.9 следует, что оптимальной несмещенной оценкой для Θ' является статистика (TJi/n1; отсюда, учитывая линейность свойства оптимальности, получаем последнее утверждение задачи. 2.59. Из предыдущей задачи имеем, что для функции τ(θ) = 00 = Σ (θ(2 — ')///' оптимальная оценка имеет вид 1-0 (Γ,)/ζ—1У й„, (ζ-'У Л , 2-'\Г\ оо Q4 + / для функции π*(θ) = ]£(~*У ь| ■■ следующий вид: оо для функции τ,(θ) = Σ "*(θ) — эта оценка такова: «= г О при Гл < г. : = Σ ηί = 2.60. I) В данном случае /(χ; θ) = βχρ{/|(θ)β(χ) + С(0) + D(x)) при /1(0) = InΘ, В(х) = ж, С(0)=—1п/(0), £>(х) = 1па(х), и утверждение непосредственно следует из результата задачи 2.46. 2) Из вида функции правдоподобия Цдс.в) = вг"(х)/-"(в)Пв(х,) иа основании критерия факторизации следует достаточность Тп. Чтобы найти распределение Т„, заметим, что ее производящая функция равна (см. указание) 170
Е„гГ" = ф"(2;0) = /"(гО)/Г(0). Выделяя в правой части коэффициент при г', получаем указанный результат. Пусть теперь <p(f) — произвольная функция, заданная на множестве (n/, nl+\, ...], и такая, что E<jtf(Ta) = О V0 е θ, τ· е. 2 ЧСЫФ' = О V0 е Θ. t-.nl Отсюда следует, что <p(f) = О для всех /, для которых b„(t) Φ 0, т.е. φ(ί) = 0 на множестве всех возможных значений статистики Т„. Таким образом, Т„ — полная достаточная статистика. 3) На основании предыдущего пункта достаточно проверить, что Ε0τ] = θ*. Имеем Ε»τ*= Σ bJiu\*] *&· = ')= Σ b„(t-s)o'/ne}= 1_л|+1 "Μ1? t = nl + , = 0'2 W'//"(0) = 0«, поскольку 2 δ»(')0' = /"(0). |=л| 4) Используя линейность свойства оптимальности и результат п. 3, находим [ Ь7\Т„) V а,Ь„(Т„ - /) при Г. >/>/ + /■, τ' = Σ α'τ<* = '-' ι-г О при Гл <«/ + '· Наконец, если 7л>(я + 1)/, то отсюда в силу указания следует внд оценки /'. 2.61. Рассматриваемая модель—частный случай модели предыдущей задачи (при /(θ) = β°—I), поэтому τ* = Ь„(Т — \)/Ьл(Т), где 6„(ft) = сое/0^е0 - I)" = Σ {-\)""С'У/к\ = A"0*/ft! при ft^n и 6„(ft) = 0 при ft < п. Отсюда следует сформулированный результат. 2.62. Полагая в задаче 2.60 /(0) = (I — 0)"' и учитывая разложение 00 /"(Θ) = (Ι —0)-'"= 2 С;„+|_|в' (см. задачу 2.11), находим, /-о . Ι ϊ_' (Ta)s T'rT"~s /Гт" —) Π /T , "'—fT- при Г, > ι, I 0 при Τη < s (это значительное усиление результата задачи 2.11). Далее, тг(0) = /_'(0) и аналогично задаче 2.60 устанавливается, что τ*, = б„-.(Г„)/б„(Г„) = с,7·, _ 1)+ Тш _ ,/с:;+г._, = π (γ,(+^-ι> ■ 2.63. Из указания к задаче 2.45 следует, что функцию f(x; 0) можно представить в виде 171
/(*; 0) = exp { J θίδ(χ,α/) + Ιηρν), где B/=ln-^-, / = I Ν — Ι. Рн Отсюда по критерию для л-параметричеекого экспоненциального семейства (при г = N — \) следует, что Τ = (7Ί Τ„ _ t), где Τι = = 2 δ(Χι, α/) = Vj, /=l Ν — Ι, — минимальная полная достаточ- иая статистика. Следовательно, в данной модели несмещенные оценки существуют лишь для таких параметрических функций, которые имеют вид Ец//(Г). Но класс таких функций совпадает с классом полиномов от р^ pv степени <п (см. решение задачи 2.29). Из задачи 1.52 п. 6) следует, что оптимальной оценкой τ(θ) = pt '...р^ при fti +■■■ + *v^ n является статистика τ* =Cvi)<.,...(vv)*A,/(n)*1+ ... + *№ Оценки для произвольных полиномов строят с помощью линейных комбинаций этих статистик (па основании линейности свойства оптимальности). 2.64. Модель Ν(θ, σ2) является моделью зкмпоиеициалыюго типа и выборочное среднее X является для иее полной достаточной статистикой, поэтому 7"— оптимальная несмещенная оценка функции τ(θ) = θ (ср. с задачей 2.50). Аналогично доказывается оптимальность оценок, указанных в задаче 2.16, поскольку Т2 — полная достаточная статистика для модели Ν(μ, θ2). 2.65. Имеем Ε07Ί = Ρ0(ξ < хо) = τ (θ), т. е. 7", — несмещенная оценка τ (θ). Поскольку в данном случае (см. решение задачи 2.64) полная достаточная статистика есть X, оптимальная оценка может быть вычислена по формуле τ* = Ε0(7ΊΙλ) = Ρ0(ΛΊ -Х< χα - Х\Х). Но X, — X и X независимы (см. задачу 1.56) и La(X\ — X) = = N(0, σ2), поэтому τ* = Р„(Х, - Х< хо - Χ) = Ф(У„1, Ха~Х)- 2.66. Функцию f (x; 0) можно в данном случае записать в виде . л ft I / (χ- о) = exp J θ;* + Θ5*2 + с (θ;, вц , ο: = -gi-, Θ5 = - -^-. Согласно критерию для л-параметрического экспоненциального семей- п η стпа отсюда следует, что ( 2 ^'· Σ ^П ~~ минимальная полная до- М= I "=1 ' статочная статистика. Таковой же является и эквивалентная ей пара (X, S2), поскольку эти две статистики взаимно однозначно определяют друг друга. Отсюда следует оптимальность указанных в задаче 2.20 оценок. 2.67. Так как Е0(52) = " ~ ' и2 = -^-γ2θ2 (см. задачу 1.27), η л Ей(Р) = θ2 + — ν202 = "+V О2 (см. задачу 2.13), то V θ Εοφ(7·) = η η = 0, т. е. критерий полноты ие выполняется. 2.68. Пусть X = (X Х„) — соответствующие измерения; тогда 172
A — выборка из распределения ЛЛ (Θ|, Θί) и речь идет об оценивании параметрической функции τ (θ) = — θ?. Используя решение задачи 2.67, находим, что Ее( Лг S I = θ| Η "г — · θ2 = θ|. \ η — Ι/ η η — Ι η Отсюда в силу полноты достаточной статистики 7 = (X, S2) (см. задачу 2.66) следует, что оптимальной несмещенной оценкой τ(θ) является статистика τ* = —- [X2 S2). 4 V η — I J 2.69. Для любого события А условная <рл[Т) = Ρβ(7Ί ζ Α\Τ) и безусловная ул = Ρο(7Ί ζ /1) вероятности по условию не зависят от параметра Θ, при этом ΕαφΛ(Π = ул, т. е. Eag(7) = 0V Θ, где g (Г) = = ч>л(7") — ул. Отсюда и из условия полноты статистики Τ следует, что <рл(Т) ε~ yAl т. е. указанные условная и безусловная вероятности совпадают. Но это и означает, что статистики Т\ и 7 независимы. 2.70. В данном случае Т2 = X2 + XI + XI — полная достаточная статистика (см. решение задачи 2.64) и Т, = ЦХ, ^ х0) — несмещенная оценка τ (θ), поэтому оптимальная оценка Е0(Г,|7)=Р0(А <-^|?"). Здесь статистика η = Χ,/Τ = Υ,/^Υ2, + χ£, где Υι = Χι/β, i = I, 2, 3, χΙ = Y\ + Y\, имеет распределение, не зависящее от параметра θ (поскольку Lt(Yi) = Л/(0, Ι), ι = 1,2,3), следовательно, по теореме Басу (см. задачу 2.69), η и 7 независимы. Итак, τ* = /^f-^-J, где F^(uj = = Ρ(η < и). Вид функции распределения Рц(и) получен в задаче 1.58 для произвольного объема выборки. Полагая п = 4, в данном случае имеем: при 0 ^ и ^ I ' о{, .А. . "\ _ ' +« при — I < и ίζ. 0 I + и /\(u) = I - f,(-«!)=■ 2 Таким образом, L (η) = R (— 1,1) и окончательно получаем, что I при Хо > 7", т*= -L(l +ii) при U„| < 7, 0 при *о < — 7. у ft 2.71. Введем случайные величины У; = —д , ' = I η· рас- "2 пределеиие которых ие зависит от 0 = (θι, вг). Тогда Χι -X Y,— V S (X) S (У) I п, т. е^_ распределение статистики U ие зависит от Θ. Поскольку Τ = = (X,S2(X)) — полная достаточная статистика для модели Λί(θι,θΐ) 173
(задача 2.66), по теореме Басу (см. задачу 2.69.) Τ и U независимые 2.72. Так как Ε07Ί = Ρ0(ΛΊ ^ ха) = τ (θ), то Τ, — несмещенная оценка τ (0). Следовательно, оптимальная оценка может быть найдена по формуле (далее Τ = (X, S2)) τ* = Ε0(7·,ΐη = Ро(Х, < xa\T) = Ρ0(η < u0\T), Χ\ — Χ Χα — Χ где η = —== , Μο = ——= . Но иа основании решения пре- V" - I S Vn - I S дыдущей задачи статистики η и Г независимы, поэтому τ* = Fn(uo). Функция распределения статистики η вычислена в задаче 1.58. Используя этот результат, получаем I J β (' _ "°; —2—' ~2~)' 6СЛИ Х < Х°' ■уВМ — ul; —J—· у)· если х^ *»· Для расчетов можно использовать таблицы функции бета-распределения В (ί; а, Ь). 2.73. Модель Г (θ, λ) является моделью экспоненциального типа л и для иее Г = £Xi— полная достаточная статистика. Следовательно, 1-1 указанные в задаче 2.21 оценки — оптимальные. Чтобы доказать dto- рое утверждение, достаточно убедиться в тон, что не существует функции Η (Г), удовлетворяющей условию Е0Я (Г) = θ~°ν θ > 0. Это условие можно запвсать в виде \H\(x)e~"dx = z—^Vz > 0, где 2 = -i-, Н,{х) = Η(χ)χλα-]/Γ(λη). Если m = а — λπ + I — целое, то, продифференцировав это тождество по 2 m раз, получим 00 \Hi(x)xne-"dx = 0. α Этот интеграл есть преобразование Лапласа от хтН\(х), поэтому отсюда следует, что Н(х) = О при χ > 0. 2.74. Так как Τ — полная достаточная статистика, то достаточно убедиться в несмещенности оценки; τ* Le(T) — Г (θ, λπ), поэтому 00 Ε0φ(ί7·) = \φ(ίχ)χ'-,'-,ε-"α(Ιχ/(Γ(λη)β>-η = θ Далее, 174 je-?(l _ ,)*-'>-^-= ·~1(τΓ"°Γ№("-,)>
Отсюда, так как можно менять порядок интегрирования, получаем Erft* =Тщр\ч(У)УХ-'е-у">с1у = Еоф(У. В частности, если а > — λ, то для φ (χ) = х° среднее Ε0φ (ξ) существует и τ (0) = Εοφ (ξ) = (* * а) Θ" Поэтому ___П?£)11__ г λ + 0_, _ «..„_,. Г (λ") Г (а + λ) - Γ(λ)Γ(λ(«-Ι)) V U t; at - Γ (λ) Γ (β + λπ) ' ■ При а = 1,2 приходим к результатам, полученным в задачах 2.48 и 2.51 соответственно. 2.75. Имеем τ (θ; t) = Е0е(| - ί) = Ρ0(ξ ^ ί). поэтому Γ(λ)Γ(λ(π-Ι)) \e[ ' П1Г U *; a- Здесь e (xT — t) = \ ο χ ~^& t/T, поэтому, если Τ < ί, то интеграл равен нулю, а при 7" > ί он выражается через функцию бета-распре· деления: \ Xх-Ч\ - x^-^-'dx = В(КЦп - 1))(1 - Β{~;λ, Цп - I)). В частности, при λ = I Β(Χ· '■"-')= β(|,'η-|) S(l -z)"_2dz = I -С -*)-'. что позволяет получить указанный результат для экспоненциального распределения. 2.76. Записав функцию правдоподобия в пиде согласно критерию факторизации получаем, что 7 — достаточная статистика. Найдем ее распределение. Заметив, что PdC2 (ξ/0)λ < χ) = Ρο(ξ < 0 (γ)'/λ) = I - <^Τ. т. е. £0(2(ξ/0)λ) = χ2(2), находим £0(27/0λ) = χ2(2η). Отсюда плотность распределения 7 имеет вид Условие Ε0φ (7) = 0 V 0 > 0, в данном случае имеет вид \<f(x)xn-,e-"dx = 0V2 > 0. α Отсюда следует, что φ (χ) = 0, χ > 0, т. е. статистика 7 - полная. Теперь достаточно проверить равенство Е0т* = τ (0). Имеем (так как можно менять порядок интегрирования) 175
Ε0τ* = („ _ i) J(| _ ί)-Ί \φ((ίχ),/χ)Ιτ(χ)άχ]άί = α L о J = -ρ- Ϊφ to) У'" 'β" "^ = Ε0φ (ξ) = τ (0). При φ (χ) = Xх функция τ(0) = θλ и τ* = (π - 1)7" J ι (Ι - ()"_2ίίί = α = Τ/η. 2.77. Используя функцию Хевисайда е (х), функцию правдоподобия можно записать в виде M^0)=lin^^-01)exp(--^ur^j = "ез'е(*(|)- 0,;|ехр{ _~о7(* ~ 0l)}- _ι_ Согласно критерию факторизации отсюда следует требуемое утверждение. 2.78. Как и в задаче 2.77, запишем функцию правдоподобия в виде л /-Ι Отсюда видно, что одномерной достаточной статистикой может быть только Хц) и только в том случае, когда / (хг, Θ) разлагается в произведение двух функций, одна из которых зависит от χι, а другая — от Θ, т. е. если / (*; Θ) = g (x)/h (θ). 2.79. Достаточность Х{п) следует из решения предыдущей задачи. Чтобы проверить полноту, надо сначала найти распределение Х(„> Имеем Р0(Х,Я) < х) = Р8(Х, < х) = (-■)", 0 < χ < 0. Отсюда, если о Εοφ (Х,„|) = -^r U M *"-'rf* = 0 V б > 0, θ о то, продиферепцировав по θ тождество }φ (x) x"~,dx = 0, получим, что α φ (θ) = 0, θ > 0. Это означает полноту Х(„> Поскольку Т* — несмещенная оценка θ (см. задачу 2.24), она как функция полной достаточной статистики является оптимальной среди всех несмещенных оценок. Далее находим Εβ(7·λ - θ)2 = Ε,[λ(7··- 0) + (λ - I) Of = = \2DJ* + (λ- Ι)2Θ2 = ψ(λ)02, 176
?2 где (см. решение задачи 2.24) ψ (λ) = —г-Ц-— + (λ — I f. Здесь η \η ~\- 2) min ψ (λ) = ψ (λ*) = 3 и λ* = ; . Таким образом, для τ η + 2 ν оценки 7\· = ;—г ■"(«ι средиеквадратическая ошибка л + I < , ■ m = D07·*. (η + Ι? η (π + 2) 2.80. Для модели Λ(θ|,02) функция правдоподобия может быть записана в виде L(x; θ) = β(θ2 — Х(П))е(хц) — θι)/(θ2 — θ|)", где е{х) — функция Хевисайда, следовательно, Τ = (X(d,X(„i) — достаточная статистика. Плотность распределения Τ задана в задаче 1.36. Используя этот результат, получаем, что условие Ецф (Т) = 0 V θ эквивалентно условию ί(φ(- :*i.*s)(*s - x,)°-2dx2dx, = 0 V θ. Дифференцируя это тождество сначала по θ|, а затем по 02, сводим его к тождеству φ (Οι, 02) (02 — 0|)°-2 = О V Θ. Отсюда следует, что φ(θι,θ2)= 0 V Οι < 0г, т. е. статистика Τ — полная. Оценки, построенные в задаче 2.25, представляют собой функции от Т, следова- U π С V "θ, +θ2 тельно, они оптимальны. Наконец, поскольку с.цХщ = ■—— , η + I Е<^(«) = ——г~Г~ (см· задачу 1.36), статистики {η(Χω — Х^Цп — I) η ~\- I и (п (Xw — Χ(ιι)/(π — I) являются несмещенными, а следовательно, и оптимальными оценками параметров 0| и 02 соответственно. 2.81. Записав, как и в предыдущей задаче, функцию правдоподобия в виде L (х; 0) = е (х(|, - а (0)) е (Ь (Θ) - х{„ЩЬ (Θ) - α (Θ))", убеждаемся в достаточности статистики Т. Если α (Θ) \, Ь (0) \ при возрастании 0, то (*,„> α(θ). *с,< *№))-»■( θ < α-'(*(ΐι). 0 < ft-1 («и!» "*>(0 < Τ, {χ) = тт(а-'(*<11),&-'(*<л1))], следовательно, функцию правдоподобия можно переписать в пнде L (χ; Θ) = е (7Ί (х) — θ)/(6 (θ) — α (θ))". Это означает, что в рассматриваемом случае существует одномерная достаточная статистика, именно статистика Tt(X) = min (α-1 (Х(н). Ь-|(Я(лй. Аналогично, если α(θ)|, й (Θ) | при возрастании Θ, то одномерная достаточная статистика существует и имеет вид 7"2(Jf) = max (a~'(X(l)), b~'(Xinj)). Этими двумя случаями исчерпыпаются ситуации, когда в модели R (а (0), Ь (0)) существует одномерная достаточная статистика. В частности, для модели /?( —Θ, 0) статистика Тг(Х) = тах(-Х(|),Х(я1) = тах(|Х„,|, U(,„|). 2.82. Условие Ε0φ (X) = О V 0 можно записать а виде 00 φ(-ΐ) м _bj + Σφ(*)β' » о нли (см. указание к задаче 2.11) π пиде 00 ф(0)+ SfoM + *<р(-1)]0* = 0. .■-I 177
Этому условию удовлетворяют функции φ, для которых φ (0) = 0. φ (χ) = — χφ(— Ι), χ = 1,2.... Единственной ограниченной функцией такого типа является функция <р(дг) = 0, χ = — 1,0. I,... . Таким образом, X — ограниченно полная достаточная статистика, не являющаяся полной. , 2.83. Распределение выборочных данных μ = (μ, μ„) сосредоточено на множестпе Ln.n векторов / = (/ /„) с целыми неотрицательными компонентами, удовлетворяющими условиям /ι + - + 'п < Ν, /, + 2/2 + - + л/л = п. Найдем функцию правдоподобия ΡΜ(μ = /), / GE Ln,- Общее число возможных исходов эксперимента равно jV", число же исходов, совместимых с событием (μ = f), можно подсчитать следующим образом. Зафиксируем сначала те ft = /, -+- ... + Ь элементов совокупности U, которые будут предстаплепы в выборке. Это можно сделать Сн различными способами. Для каждого такого подмножества элементов существует одно и то же число способов формирования выборки из элементов данного подмножества с заданным значением / статистики μ н при условии, что псе ft элементов должны войти в выборку. Обозначим это число А (/; ft, π). Тогда общее число благоприятных исходов равно CjtjA (/; к, п) и согласно классическому определению вероятности ΡΑ,(μ = 0 = g{k; N)A(l;k,n), где первый множитель g (к; N) = C^/V-" зависит от параметра N, а от выборочных данных зависит лишь через k = /ι + ··· + /„-совместимое с событием (μ = ί\ значение статистики η, а второй множитель А (/; ft, л) от параметра N не зависит. Согласно критерию факторизации η — достаточная статистика для N. Для доказательства полноты η требуется проверить, что для всякой функции φ (ft) из Ем<р(ц) = = OtyN следует, что φ (ft) = 0 для всех возможных значений статистики η (при всех N > I). Распределение η имеет (см. решение задачи 2.37) вид * Ρ4η = ft) = g (ft; N) 2 (- l)'-'Ci/\ k e= Κ (Ν) = (I. 2 min (n, N)\, i-° следовательно, надо проверить, что φ (Ι) = φ (2) = ... = φ (π) = 0. Если Λ/ = Ι, το Ε,φ(η) = <p(l)g(l; 1) = φ (I) = 0. Если N = 2, то Ε2φ (η) = φ (Ι) Ρο(η = 1) + Ч>(2) Ρ2(η = 2) = φ(2) g (2; 2) (2" - 2) = 0. т. е. φ(2) = 0. Аналогично, полагая Λ/ = 3, находим, что условие Езф (η) = 0 вместе с уже установленными равенствами φ (Ι) = φ (2) = 0 означает, что φ(3) = 0 и т. д. Таким образом, последовательно проверяется, что φ (ft) = 0 при всех ft < л, т. е. статистика η — полная. 2.84. Согласно критерию эффективности справедливо соотношение откуда следует, что 0„ удовлетворяет уравнению τ (0) = τ*. Чтобы доказать ее однозначность, вычислим вторую производную <?21п L(x;Q) _ α (Ο)τ'(Ο) + (τ* - т(0))д'(0) Ш ~ ?(0) Так как в данном случае речь идет об экспоненциальной модели (см. решение задачи 2.46), то D0t* = α((1)τ'(0) > 0 и поэтому 178
dHnL{x, θ) I ,m „ -τ-;—^тл -1 , < 0, т. е. каждое решение урапмеиия τ (0) = т* яв- ляется локальным максимумом функции правдоподобия. Если бы было больше одного максимума, то между последовательными максимумами должен находиться минимум (т. е. в точках минимума, которые * * д*1п L также удовлетворяют уравнению τ(Θ) = τ*, должно быть —-^—> О), а так как минимумов нет, то не может быть более одного максимума. Отсюда и из задачи 2.48 получаем следующую таблицу значения ff„ для некоторых моделей: Модель О. Ν(θ,α2) X Ν(μ.θ2) τ, Γ(ο,λ) Χ/λ β(Ο,Ι) Τ2 Bi(k. 0) X/k Π(ϋ) χ Bl(r.O) X/(r+X) Здесь T*-[i;i(*-*fY.T,=[-±hnx]- 2.85. Для распределения Ν (Ά, α2} точка θ является теоретической медианой, поэтому выборочная медиана Т„ является в данном случае состоятельной оценкой 0, распределение которой, согласно решению задачи 1.32, удовлетворяет асимптотическому соотношению Lo(T,) ~ ~ JVr в, ——J, т. е. для нее σ2(θ) = πσ2/2. В данном случае о.м.п. @а = X (см. решение предыдущей задачи) и £ц(0,,} = TV ί 0,—J. Отсюда eff (7·„; θ) = σ2/σ2Γ(θ) = — я 0,637... Это означает, что при больших η выборочное среднее Хдля выборки объема п' = 2п/п оценивает θ с такой же точностью, как и выборочная медиана -^о/гц-1) выборки объема я, независимо от значений θ и а2. 2.86. В данном случае L (х; 0) = —— ехр (л/2л 02>" {- ж ■Σ(* (λ^θ2)" ехР { - -^нт к2 + (χ η ~W2 - θ,}2} = θ,}2}}, где s1 = — Σ(*< — χΐ· χ = — Σ-*'· П0ЭТ0МУ уравнения правдоподо- dlnL dlnL бия ——— = ——— = 0 имеют вид σϋι σϋ2 х = ο,, θ! = s2 + (* - в,)г. Они однозначно определяют решение θ = (χ, s). Покажем, что в этой точке функция правдоподобия достигает максимума. Для этого перепишем формулу для Цх; Θ) в виде Цх; θ) = (2πΜ2}-"/2εχρ(-ηψ(«; Θ), 179
где ψ(*;θ} = -— ι/ +-ό-(τπ—' )~lnfT· и будем минимизировать ZU2 ^ \ U2 / U2 по θ = (θ|, Θ2} функцию ·ψ(«; θ). Используя легко устанавливаемую оценку 1πα<α— 1 Va>0, из которой при а = Ь2 следует также оценка In b < (б2 — 1}/2 V 6 > 0 (знак равенства имеет место лишь при й = 1), получаем, что ψ(«; θ) ^ 0 (знак равенства имеет место только в точке θ = (ж, s))._ Следовательно, Цх; θ}< (2nes2}_"/2 (знак равенства лишь при θ=(χ, s}}. Тем самым доказано^ что о. м. п. ®„ = (6ι„, θ2„) существует, единственна и пр^и этом 6„=(Х, S). 2.87. Вид оценки т„ следует из задачи 2.86 и свойства инвариантности оценок максимального правдоподобия. При n-»-oo L^-fyia — — τ(0)))-+Ν(0, σ2(θ}}, где (см. задачу 2.44) σ?(θ) = ьщг '(θ)6(θ) = (■*$-)** + (-ζ^)*/* = (*о-е,); . ~2ле L+ 201 J' Λ 2.88. Вид оценки θ„ указан в задаче 2.84. Используя этот результат, можем записать, что θ„ = τι, где τ* — несмещенная оцен- Mi) ка Θ, полученная в задаче 2.16. Отсюда имеем, что ЕД, = с„0, с„ = Используя формулу Стирлинга для гамма-функции Mi) Γ(ζ)~-\J2nzz'~'е-2, z-»-oo, получим с-»-1 при п-*-оо, таким образом, имеет место асимптотическая несмещенность θ„. Далее имеем DS„ = Εοθΐ - (ЕД,)2 = θ2(1 - cl) — 0, η -* оо, откуда следует состоятельность б!,. Наконец, £е(л№ - б}} >- N(0, Г'(θ}} = Ν(0,θ72}(εΜ. задачу 2.43}. Рассмотрим теперь оценку 7"„ = "у-д-·—Σ 1^' — и1 (см. задачу 2.15). По центральной предельной теореме, используя решение задачи 2.15, находим, что ЦТп)~ ν(θ, —^—θ2) при η-»- оо. Отсюда ее асимптотическая эффективность eff(r„-,e} = -el/^G2=^ = 0.88... 2.89. Здесь функция правдоподобия ^е)=(4^еХМ-ж|^-е)2} и уравнение правдоподобия имеет вид θ2 + 2Θ - Т„ = О, Г, = — У *2. п 1-х 180
Это уравнение имеет единственное положительное решение 0„ = = VI + Та — 1. которое максимизирует Цлс; Θ). По закону больших чисел Тп сходится по вероятности при п-»-оо к ЕцХ\ = DoXi + + (Έ,αΧι}2 = 20 + б2, следовательно, 6nJj^Vl +20 +О2 -1=0. 2.90. В данном случае плотность распределения наблюдений Д,;Г,0) = ^ехр(-^^ + -^7Г-11пИ.-р2)]}, учитывая указание, можно записать в виде /(*. У. Я) = "27ехР М*2 + У2) + <7*«/ + τ(<7))· где τ(<7) = -^-1п(4<7? — q\). Отсюда уравнения правдоподобия для па- хождения оценок (j\ и q2 имеет следующий вид: - Σ (* + </,}- —^— -4,ϊ-,!· -я- Σ *<л = - ΰτ(<7) <72 ., <?<72 4<7? — <72 ' υ 2 2<7ι <72 , Но σ = ——3 J-, ρ= —-—, поэтому из предыдущих уравнений сразу получаем искомые о. м. п.: °2 = 4т Σ Ш + Я), ρ = 2 Σ хм/ Σ (*.? + γ')- ίη ι-ι 1=1 ί=ι 2.91. Для данной модели плотность распределения имеет внд К*' У: θ) = 2л(1-^)'^еХР { _ ~27Τ^θΤ{χ2 ~ 2°ХУ + у2))· поэтому lnL= -η1η(2π)--|·1π(1-02)- ^ (Г„ - 26Г|Д + ^, где Г"=7^''Ь = 7Е *№. Г» = ~- Σ </?■ n if ι n ,-f ι П ,f ι Отсюда находим и уравнение правдоподобия приводится к виду 0(1 - б2} + (1 + 62}7\2 - 6(7·,, + 7-22} = 0. Это кубическое уравнение, имеющее три кория, два из которых могут быть комплексными. Если все три корня действительны и принадлежат интервалу (—1, 1), то в качестве ff„ выбирают тот из них, который максимизирует функцию правдоподобия L, С помощью подстановки 181
О = χ +-Τ-7Ί2 урапиение правдоподобия приводится к каноническому виду х3 + Зр„х + 29„ = 0, Зр„ =7·,, + T^--jTh- 1. 2«7„ = 7-|2(-i-(7-|l + 7-„)--^-Ti2-l)- Условием того, что оно имеет единственный действительный корень, является неравенство р\ + ?» > 0, (см. решение задачи 2.19), которое очевидно при р„ > 0. Так как выборочные моменты сходятся по вероятности при п-*оо к соответствующим теоретическим моментам (см. решение задачи 1.38), то р„ сходится по вероятности к величине — (l + 1 — —О2 — 1 J =-o"(l — -о-О2) > 0- Таким образом, при больших значениях η уравнение правдоподобия имеет один действительный корень, который и является оценкой максимального правдоподобия 0„. Асимптотическая дисперсия оценки 0„ равна 1/ί«(0), где с521п£_ с [- 1+02 , 40 т 1+3Q2 /7, ''(°)=-Ео-5дГ - -nE»L(i_oV + (l—oY ,!~ (1-θΥ(Γ"_ -20Г,2+^)]=-п[т}^+1Т^т-т^(2-202,] = 1 +02 -"(ΐ-θΤ- Таким образом, при η-><χ> (1-02)2· i-niV^Oi. — о»-*w (°· ι + о" )· 2.92. По центральной предельной теореме статистика Т„ асимптотически нормальна с центром в 0 и асимптотической дисперсией — Da(X\Y\) = —ГЕ0(Х2У2) — 031, поэтому задача сводится к вычислению η η смешанного момента Е0(Х2У2). Так как в данном случае характеристическая функция ср(/,/2)= Е„е'С'х' + '^')= exp|--i-(/f + 20/,/j + /|J. то UxW) = ^h£L\ = 1+20,. dtidti ' ), = /, = о Таким образом, Ό£Χ\Υ\) = 1 + О2 и поэтому асимптотическая эффективность оценки Тп равна 2.93. I) Рассмотрим функцию правдоподобия η Ц*; о)= Ш(*ь °) = [(2"ΠΣ |]~Τ«Ρ {_Τ Σ (*ι-μ)'Σ ~ '(*/-μ)}- 182
Здесь Σ (* - μ)' Σ " '(*■ - μ) = Σ (*< - *)'Σ " '(* - *) + 1=1 1=1 + η(*-μ}'2_,(*-μ}· Использовав легко проверяемое равенство у'By = \.т (BY), где Y = yy', и линейность оператора tr, получим Σ (*<-*)' Σ ~ '(*<-*) = ηtr (ς _' £(*))■ 1=1 ч ' Из этих соотношений следует приведенная в указании к задаче формула, из которой следует, что максимизация по θ функции L(x; θ) эквивалентна минимизации по μ и Σ функции *(*■■ μ. Σ) = (*-»»)'Σ (*-μ)+[ίρ(Σ Σ(*))-*- -ΐπΐΣ~'&*)Ι]- Обозначим через λι λ* корни характеристического уравнения ΙΣ Σ^Μ ~ λ^* | = О или, что τα же, уравнения | Σ^(*)~ λ Σ Ι = 0· Тогда выражение в квадратных скобках равно λι + — + λ* — k~ — Ιπ(λ|...λ») и можно записать Ч>(*; μ. Σ)= (ί - μ)'Σ"'tf - μ) + Σ &- ι - ΐηλ,)- ί = ι Поскольку Σ положительно определена и λ— I —In λ > 0, νλ > О, из последнего представления получаем ψ (ж; μ, Σ)>^· пРичем равенство нулю имеет место только при μ = ж и λι =...= λ* = f, т. е. когда Σ = Σ»· 2) Согласно задаче 2.4, несмещенной оценкой σι; является статистика j-Sji, следовательно, Е| τΧ) = Σ· η—ι \η—1*ν 3} Чтобы получить указанное выражение для maxL(jc, θ), доста- θ точно учесть, что tr(2 (*)Σ(*))~ t["E* = k- 2.94. Моменты Ео(Х*} можно вычислить следующим образом. Пусть <р(г) = Ε#"γ' = ехр ί//θ, - -^-θ2-}; тогда Erftf) = ε**"' = φ(4·) = εχΡ {*θ' + τθ')· Отсюда т,(в}=ЕвХ1=ехр(В, + в^/2), τ2(θ) = D Д, = Ε„*? - (Ε Д,}2 = A0)(eal - 1). Поскольку о. м. п. (6|„, 6L} = (F, 52(У}}, то согласно свойству инвариантности оценок максимального правдоподобия ϊ,„ = exp(F + 52(У)/2), ΐ2. = xl(es'm - Ι)· Ш
Далее, поскольку V и S2(Y) независимы (см. задачу 1.56), Eoim = = Еце EoeS tK)/2. Здесь L,iX) = n(§\, —J и аналогично предыдущему Εοβ? = ехр(0, + 022/(2п}). Далее имеем L,HnS\Y)/O^) = χ2(η-1) (см., например, решение задачи 1.58), поэтому Eofis tK'''2 = t|)(0j/(2in)), где ψ(ί) = _ "- ι = Ее"*--' = (1 — 2ii) "5_ (см. решение задачи 1.39). Отсюда (q2\ я— 1 1 -I 2 . Из этих соотношений окончательно находим Ε0ί,„ = τ,(0)εχρ(-^-θ|) (.--Ι)" 4^"^ 2.96. Здесь -|j-in ц*; θ) = -A-in (θ"7-"(θ) Π «(*')) = -тг - "f'(θ>//(β) = τ(Γ-μ(θ»· где μ(0) = θ/'(θ)//(θ) = 2χα(χ)07/(0) = Εοξ· Отсюда следует, что урапне- χ пне правдоподобия имеет указанный вид. Асимптотическая дисперсия оценки 0„ равна (ш'(0))-\ где функция информации равна ВД= -Ε^ϋ,№. 0)= Ц-Ц^ + Л) = μ4θ)/θ. В частности, для распределения Bi(r.Q) среднее μ(θ) = -—— и решением уравнения μ(θ) = X япляется 6„ = Х/(г + X) (что совпадает с результатом, полученным при решении задачи 2.84), а /(0} = — „ = tj- (результат, приведенный в задаче 2.43). ~ θ(1-β)' 2.97. В данном случае Ш = е°— 1, μ(θ)=0/(1 — «τ") и уравнение правдоподобия 0 = х(1— е- ) точно решить нельзя, поэтому для приближенного вычисления о. м. п. 0„ можно воспользоваться методом накопления. Здесь (см. решение задачи 2.96) (/(,; о) = ^.щ цг. о) = А(г_ μ(ο)), ,-(о) = ' "/l^y и искомые урапнеиия имеют вид 0*+, = 0* + ΙΑ*; 0»)/(ηί(θ*)), k = 0. 1,... 2.98. Для записи этих уравнений (см. решение предыдущей задачи) ,,.„, ulnL(O) надо знать функцию вклада 1/(0} = . и функцию информации ί„(θ)=—Ео— д . В данном случае ,(0) = _ £ wm-mv ΕΛ = п £ ^.^ 1-1 ρ7(Θ) поскольку ΕοΛ, = ηρ,(0) и £р,(0} = ' V0. 1 = 1 184
2.99. Так как здесь i(0} = '/2 (задача 2.43), то итерационная процедура имеет вид o4+l = o4 + 4f/(ot). *-ο.ι ^) = 2|1+V_V В качестве начального приближения θ0 можно пзять значение выборочной медианы Т„ = Х„,у , + (. которая япляется состоятельной оценкой Θ, поскольку теоретическая медиана в данном случае совпадает с 0. Из задачи 1.32 следует, что при п-»-оо Следовательно, eff(7V, θ) = -^ = 0,1 2.100. Записав функцию правдоподобия в виде Цх; Θ) = еф — — Х(л)}/0" (см. решение задачи 2.78), видим, что оиа монотонно убывает по В для θ ^ Х(„), т. е. принимает наибольшее значение при θ = χ(η). Отсюда следует, что 0„ =-¥(„>. Используя решение задачи 2.24, находим ЕоОп = —г-г0 = 0 г-г, т.е. оценка 0„ асимптотически несмещена, п + 1 п+ 1 DoO„=-— .ι—-^τ-θ2-»-0 при n-*-oo, т.е. оценка б„ состоятельна. (η+ Ι) (η + ί) Функция распределения Х(„) приведена в решении задачи 2.79. Отсюда имеем, что при / Js 0 *(^«')-"(*«*·('-τ))-'-('-τ)'-*'-·-· т.е. в данном случае распределение 0„ не япляется асимптотически нормальным (модель не регулярная). 2.101. Из вида функции прапдоподобия Цх; 0} = е( 0 +-,— *«) X У.е(jtj,)— 0 + -ψ) следует, что Цх; 0) = 1 при всех θ <ξ Iхм — -у, х0) + "о" I, следовательно, любое значение 0 из этого интервала максимизирует Цх; 0). Произвольная точка Г е Ji(,)—^-, -^(о Η—^" I может быть записана π виде Τ = а( Х(„) — 1 + С — <*}( Хщ + у), α е е [0, 1]. Отсюда для нахождения несмещенной оценки θ получаем условие Ε J = у - α + аЕД(„) + (1 - α)ΕοΧ(,) = θ \>0. Воспользовались решением задачи 1.36, получим, что а = —, . Τ = -~-(-V(i) + -V(„)) — средняя точка интерпала. 2.102. В данном случае плотность f(x\ θ) = b~"a(x — θ)""' Χ Хехр( — Ь~"(х — 0)"), χ ^ 0, поэтому функция правдоподобия имеет пид 185
Цх; о) = Ш*:е) = ь-°"*"е(х, - о)П (*<„- аг-*ехР\-(±<Ц^ i -1 ί = 1 Она монотонно возрастает на интерпале — оо < θ ^ *(ΐ> и равна нулю при θ > χμ), следовательно, 0 = д:(|) — точка ее максимума. Рассматриваемая модель не является регулярной, но о. м. п. б„ = Хщ в силу решения задачи 2.26 асимптотически несмещена и состоятельна. Асимптотическое распределение Х^) припедено в решении задачи 1.37 н оио ие является нормальным. 2.103. Здесь функция правдоподобия Цж, 0} = ( — 1 β_7/°Π*/, Т= Д ι dlnL = 2j χι и уравнение правдоподобия ——-— = 0 имеет единственное ре- шеиие θ„ = Τ/η, максимизирующее Цх\ θ). Данная модель является частным случаем модели Вейбулла слнеизвестным параметром масштаба (см. задачу 2.76), поэтому о. м. п. Θ, совпадает с полной достаточной статистикой и является, в частности, несмещенной оптимальной оценкой Θ. 2.104. Вид о. м. п. ΐΛ следует из решения задачи 2.84 и свойства инвариантности оценок максимального правдоподобия. Из решения „ ., Λ λη . λη — I задачи 2.21 имеем, что τ„ = -τ\, где τι =—= несмещенная λη— 1 / оценка В-1. Отсюда имеем Εοΐ„=Τ^4-θ-' = θ-'+- ' λη-1 Γ (λη-1)θ ' т. е. ϊ, — асимптотически несмещенная оценка. Далее получаем D-=G^t)'d°- = (х£т)' к*- - '>2^-2 - θ"2ι= - *«fQ->^=W>f (CM. задачу 2.43). (λη- lf{kn — 2) λη πί(θ) Отсюда следует состоятельность г„ к тот факт, что L<(t„}~All—, fl!i ). 2.105. В данном случае максимизация функции правдоподобия сводится к минимизации суммы J \х'~ θ I = Σ ИС)- 0[. откуда сле- дует, что θ„ совпадает с выборочной медианой. Здесь нельзя воспользоваться теоремами об асимптотической нормальности о. м. п. или выборочных квантилей, так как плотность f(x\ θ) в точке θ не дифференцируема. Тем не менее, справедлив следующий результат: Ldfi„)~ ~ All θ, —1, имеющий такой же вид, как и в задаче 1.32. 2.106. Найдем предельный закон распределения оценки Т„ npf η ->■ оо. Имеем Ро ШТ„ - 0) < х) = рЛЩ,Х - Θ) < *) + (I -p,)P^bX - θ) < 4 где р„ = Ροα*ι»αη)=otv^e - β.»+Φ(-νί<θ + ^))—^ j J ππρρ1 'θθ| >0°· 186
Таким Отсюд; образом, а при п-> оо Li-RTn - е[цТ„; 0})- :0} = ШО, U(0 {.'- 1} при при при 101 >0, 0 =0. |0|>0, 0 =0, т. е. eii(7„; 0} ^ 1 при \Ь\ <1; строгое неравенство имеет место π точке 0 = 0, которая является, следовательно, точкой сверхэффективпости. 2.107. Для модели Я(0, 0) (см. решение задачи 2.100) б„ = Х^ и DuOn = 0(и~2). Для модели Вейбулла (см. решения задач 2.102 и 1.37) при 0 < α < 1 о. м. п. 0„ = Ύ<!) и DuS, = 0(η -2/а), т. е. в данном случае в зависимости от значения α дисперсия о. м. п. может иметь сколь угодно вьгсокий порядок малости. 2.108. Здесь (см. решение задачи 2.84)_0„ = X и согласно инвариант- иости τ„ = χ-'. Но (см. задачу 1.39)ЦпХ) = П(п0), откуда 9^X = Q) = = е~"" > 0. Следовательно, с положительной вероятностью при любом η случайная величина X принимает нулевой значение, и поэтому статистика т„ не имеет конечных моментов. В то же время ее асимптотическая дисперсия равна [x'(0)f/[ni(0)]= (03п)"' |см. задачу 2.43). 2.109. Асимптотическая дисперсия о. м. п. τ„ не зависит от параметра 0 тогда и только тогда, когда функция τ(0) удовлетворяет соотношению σ?(θ) = (т'(0))2/('(0) = const. Отсюда и из задачи 2.43 следует, что для модели Bi(ft, 0) соответствующее уравнение имеет вид τ'(0) = = c/V0(l —0}. Решением его является функция τ(0) = arcsin -*/сГ(с точностью до постоянного множителя), для которой σ?(0) = ——. Аналогично, для модели П(0) искомая функция есть решение уравнения с 1 т'(0) = , т.е. τ(0) = νό~~Η σ?(0) = —; для моделей Ν(μ, О2) и Г(0, λ) Vo 4 имеем уравнение -['(0}=—, т.е. τ(0)=!π0. Таким образом, используя решение задачи 2.84, имеем следующие, полезные для многих целей аппроксимации: для модели Bi(li, 0): Lo(arcsin sj8„) —< N[ arcsin д/0 , ——1 , 0„ = = X/к; для модели П(0): L^O») ~ Ν (ι/δ, -j-V 0„ = X; для модели Λ^μ,Ο2}: t„(ln6»)~jvfΙπΟ,-^-V 0„ = Γ-i- Σ (Χ,- μ)2] для модели Γ(0, λ): L<H\n0„)~N (ln0, -Μ, 0„ = Х/к. 2.111. Из решения задачи 2.83 следует, что при заданном r\~k (числе различных наблюдавшихся в выборке элементов) максимизация по Ν ^ к функции правдоподобия эквивалентна максимизации по N функции g (к; N) = CkNN~" нлн, что эквивалентно, функции 4-1 /(/V) = 2 \π(Ν — /) - η In Ν, Ν 3? к. Ά 187
Рассмотрим разность Δ/ (Ν) = f (Ν + 1} - f (Ν) = In -jy^py - η In -^±- = [S(N,k) - η]1π- + Здесь функция S(N,k) монотонно убывает по /V при ft > 1. Действительно, если ввести функцию <р(д:} = 1п Π о)/*п ( ' .,,.), О < χ < N + '. то неравенство S (N, ft) > S (Ν + '. Ό будет эквивалентно неравенству φ (ft) < φ(1). Функция же φ (χ) монотонно убывает, так как неравенство у'(х) <. О сводится к неравенству которое вытекает из того, что функция ψ (у) = (у — х) In ( 1 ), у > χ, монотонно убывает (ψ'(ι/) = In [ 1 J Η < 0). Таким образом, при заданных значениях η и ft > 1 неравенства S (N,k) ^ η < <S(N—\,k) однозначно определяют целое число Na = Na(k,n). При этом Ц (N) > 0 при Ν < ΛΌ — 1, Δ[(ΝΒ) < 0, bf (Ν) < 0 при Ν > /Vo + 1. Это означает, что функция \ (N) монотонно возрастает при N ^ Na и монотонно убывает при Ν > Να, если S (Na, ft) Φ η. Если же S (jVq, ft) = η, то f (N0) = [ (N0 + l), а левее /V0 и правее /Vo + 1 значения f (Ν) < f (No). Таким образом в любом случае max f (Ν) = f (Να), что и требовалось установить. Пусть, наконец, Λί ft = 1, Тогда g(l; N) = 1/ЛГ-' и значение этого выражения достигает максимума при N = 1. Значение Я = η тогда и только тогда, когда выполняется уело· пне S (η, η) ^ η (так как S (η — 1, η) = оо по определению функции S), которое можно переписать а пиде 1п (η + '} ^= п 'п — · В указанных асимптотических условиях приближенное решение для а получим из соотношения — In I 1 α) «* n In J· « а. V п / N Отсюда — « α (а), где а (а) = , или, обозначая α~'(ί) фупк· л α цию, обратную к α (а), имеем а л а_1( — )· Для произвольного значения m вместо функции g (ft; /V) получаем функцию gm(ft; /V) = Скн(Сн)~", максимизация которой по N приводит к следующему результату: при ц > m о.м.п. Ят определяется неравенствами Sm(Nm, π) < η < S„(AL - 1, η), где S.Mb) = ln N+Y_k/b /+\!_я при *=>*>«, Sm(k- l,ft) = oo; если же η = m, то Ят = т. 188
2.112. Если μ2 = /г, то Я есть значение Ν, максимизирующее функцию правдоподобия Ρν(μ2 = А) = Cl,C7-llC7 β g (Ν). ο S(«) (Л/-т,)(Л/-т2) Здесь —-rj rr- = ν —rr-, откуда получаем, что нера- веиства g (JV) 3; g (/V — 1} эквивалентны соответствующим неравенствам JVft ^ mim2. Обозначим Л/0 = —!-—, где [· ] означает целую часть. Тогда из предыдущего следует, что при Ν ^ Ν0 функция g (N) воз· растает, а при Ν ^ Ν0 убывает, если число —τ— ие целое, т. е. в этом 17 г- 17 ГП\ГТЦ случае Ν0 — точка максимума. Если же Ν0 = —, то максимум достигается в двух точках: Να — 1 и Να. Таким образом, в любом случае значение оценки N совпадает с Ν0. В задаче 2.36 было найдено, что (при η = 2 и mi = тг = т) едииствеииая несмещенная оценка для τ (Ν) = l/N, являющаяся линейной функцией от μ2, имеет вид рг/т2. Оценка же максимального правдоподобия τ =1/Я = , Г т2 -\ μ2 /. βμ2\-ι т2 Г т2 1 = 1 / = -*-т I 1 Ц-) , где е = , и является, L μ2 J m' V т ) μ2 L μ2 J таким образом, смещенной. 2.113. 1) Докажем сначала, что d„ — достаточная статистика и воспользуемся для этого критерием факторизации, т. е. убедимся в том, что для функции правдоподобия Ро(Х = *), * = (* *л), *( = = 0, 1, i = 1 η, справедливо представление Ро(Х = *} = я = g (d„; D) h (x), d„ = J*'· По формуле умножения вероятностей моги ι жем записать Р„(Х = χ) = Ρ0(ΑΊ = л) Р0(Л-2 = л:2| А", = *,)... Р0(Х„ = Хп\Х, = ж,, -^2 = Хг Xn-I = *г.-|}. Далее имеем D — х\ — ... — Xj-\ PD(Xl = Xl\X,=X Х;-\=Х;. N4+l ПРИ*< = . D — χ, — ... — χ,-, . = (D-x,- ... - x^YiN - D - } + 1+*,+...+*,_,)'-*/(#-/+1). Из этих формул находим Ра(Х = χ) = D"(D - χ,)"... (D - χ, - ... - *„_,)*'(Л/ - D}'-"X χ (Λ/ - D - (!-*,))'-'■...(# - D - („-l-^,-...-^.,}}'-'^/^,. Наконец, воспользовавшись формулой л Λί*'(Λί - е,}1'... (Μ - ε, - ... - ε„_,}" = Μ (Μ - 1}... (Μ - £ ε, +1} = /-ι л = УИ!/(М- S *i) '■ 189
где tj = О, 1, которая легко устанавливается, например, нпдукцней по л, окончательно получим, что ρ /у х) DHN-D)l(N-n)i _ .„_„.„ Убедимся теперь, что da — полная статистика. Для этого требуется доказать, что если Ε0φ(ίί„} = 0 при D = 0, 1 Ν, то φ (ft) = 0, /ге(0, 1,..., min (D, л)} при всех возможных D, т. е. при ft = 0, 1 л. Поскольку Lo(d„) = Η (D, Ν, л), предыдущее условие имеет вид min (D, я) 2 φ (ft) C»0CV.-VCv = 0, D = 0, 1 Ν. к - о Полагая здесь последовательно D = О, D = 1 и т. д., последовательно получаем φ(0) = 0, φ(1) = 0 и τ. д. Пусть теперь τ (D) — подлежащая оцениванию заданная функция параметра £>. Тогда оптимальная оценка для нее — это решение уравнения несмещенности Е07 Щ = τ (D), D = 0, 1 Ν. (*} Но при любой функции Τ левая часть здесь является многочленом от D степени ^ л, поэтому, если τ (D) не многочлен степени ^л, то уравнение (^с) решения не имеет, и, следовательно, для таких функций несмещенных оценок не существует. Пусть, наконец, т (О) — указанный в формулировке многочлен. Проверим, что τ* удовлетворяет уравнению (*}. Имеем л edt*. = 2Г(*Ж*:0.л) = -Σ«;#Σ(*)ί/(*·.ο.«)· Здесь Σ (ft)//(ft; 0,л) = ED(d„)i = (D)tn)i/(N)„ *-/ поэтому л Ε0τ*= ΣΜΟ), =t(D)V D. l-o Следовательно, τ* как функция полной достаточной статистики является оптимальной оценкой τ (D). 2} Функция Ti(D) является многочленом первой степени, поэтому несмещенная оценка для нее всегда существует и имеет, очевидно, вид tf = Nd„/n. Функция xi{D) = (Ν— 1} D — (D}2 является многочленом второй степени, поэтому для нее несмещенная оценка существует, если объем выборки η ^ 2. В этом случае из предыдущего находим, что τί = = d„(n-d„}(JV)2/(n}2. 3} Чтобы получить о.м.п. параметра D, надо максимизировать по D величину g (dn\ D) = С%Z„/Ci Но 190
g(d„; 0 + 1) ^ (D + \)(N-n+dn-D) g (d„; D) (D + I - <*„) (Λ/ - D) ' откуда находим, что g (d„; D + 1) ^ g (d„\ D) при D ·§ — (/V + I) — 1 ■ Следовательно, если число ——{Ν -f- 1) не целое, то максимум дости- п гается в точке Da = j—— (Л/ + Of В противном случае максимум достигается в двух точках: Do ч Do— 1. Таким образом, в любом случае о.м.п. В„ = Do. 2.П4. Функция правдоподобия для у'-й выборки имеет вид L, = Ш θ*. OJ -^^ exp{ —fa (sf + ft - ifJ·)} (см. решение задачи 2.86), а поскольку выборки независимы, функция правдоподобия для всех выборок равна L = Π L, = (2ne/»)" ^ехр {-4ς "/А - °/02 - -(т(4-0-«-Ё)}· 1 * где t1 = — 2n/s?· я => я, + ■·· + "*· Отсюда следует, что показатель " /-1 экспоненты не положителен и обращается в нуль лишь при Oyi = *;, / = 1 k, Вг = t. Тем самым о.м.п. параметров имеют указанный в задаче вид. Далее имеем (см. решение задачи 2.1) е0(Р2 = 1 ς n^s? = 1 ς п,^^- ol = -^ ol. следовательно, несмещенной оценкой 0\ является в данном случае статистика — Щ. π — k 2.115. Учитывая выбор константы су, имеем γ= P„bJn\X- В|/_0< су) = Рв (0(1 - сУЫ~п ) < Х< B(l +cv/V")) = = Р0 {*/(! + е,/У«) < 0 < Л/(1 - cr/V« ))· В случае модели с отрицательным параметром исходное равенство принимает вид у == Ро (Vn I^C- 2I/ISI < cvL откуда следует, что искомый интервал (Х/(\ — <:,/->/«), Х/(\ + ст/т/п )). 2.П6. 1) Здесь L„(G(X; В)) = N (0, 1) и, следовательно, G (X; 0) — центральная статистика. Далее получаем P0(g, < G (X; 0) < gi) = Φ (gs) - Φ (g,) = 7 и решениями урапненнй G (X; 0) = g,, g2 являются 7", = Χ ;=g2, V" Т2 = X F^ffi· Тем самым допернтельный интервал Δν(.ϊ) имеет 191
указанный вид. Его длина / = —p^(g2 — gi), поэтому, чтобы по- V" строить наикратчайший интервал, надо минимизировать разность Ё2 — gi при условии Φ (g2) — <D(gi) = у. Для этого воспользуемся методом Лагранжа нахождения условного экстремума. Составив функцию Лаграижа Η (gl, gs. λ) = g2 - gi + λ (Φ (g2) - Φ (gi) - γ) и приравняв все ее частные производные нулю, получим систему уравнений <D'(gi) = Ф'^г), Φ (g2) — Φ (gi) = γ. В силу четности функции Ф'(д:) = ι. е~" , из первого уравнения получим gi = — g2. Учитывая второе уравнение и соотношение Φ (— *) = 1 — Φ (*), получим 2Ф (g2) — 1 = Υ, откуда g2 = су. 2) Длина / доверительного интервала Δ?(.К), доверительный уровень γ и объем выборки η связаны в данном случае соотношением / = —;=?-. Отсюда имеем, что при заданных / и γ необходимое число 2 2 наблюдений л = η (Ι, γ) = 4 —-j-^- , а при заданных л и / доверительный уровень γ = γ (л, ί) = 2Φί—^—) — 1. В частности, Со.99 = 2,5758 и для / = 0,5 (при σ = 1) величина л = 106, а для / = 0.1 л = 2653. 2.117, Плотность распределения случайной величины Т/в, являющейся в данном случае центральной статистикой, равна д ах р» (4" < *) = -k Р° (£ < *0 = 2jt*^· поэтому Р0 (Θ е= δν(*)) = Ρ„(αι < -^- < as) = 2$*ft„(*V* = γ. Наикратчайший из рассматриваемых интервалов находится мипимиза- цией отношения аг/αι при условии }xk„(x2)dx = -=-. Метод иеопреде- ленных множителей Лагранжа приводит в данном случае к уравнениям a?ft„(a?) = alMai). J Μ*) d* = γ, =1 которые в обозначениях α2 = χ2,.„, al = χ1-α,.η сводятся к уравнениям Χί-α,.»/χ!α,.» = ехр! — (χί-α,.η — X«,.Jj , «Ι + «2 = 1 — V- Эти соотношения однозначно определяют ос! и аг, а тем самым и χαΐ.я. xl-ot.я С, с. 86, табл. 2.3]. Таким образом, оптимальный интервал имеет вид 6*(Х) = (77, 71) = (Γ/χ, _«,,.„, 7/χα,.„). 2.118. Здесь центральная статистика G (Χ; τ) = Τ"2/τ, τ = О2, и утверждения следуют из предыдущей задачи. 192
2.119. На основании указания имеем Р0 (V" ~ Ц·^ ~ Oi)/S ^ *ζ /ν л -1 )_== -у, откуда следует что нижний γ-доверительный интервал таков: _(Л' — /,. „_,5/д/п — I < 0,). Аналогично, из соотношения Ро (Vя — U^ ~ 0, )/5 ^ — ίγ,η-ι) =_1Получаем верхний γ-доверитель- пый интервал: (0, < X -f ty. „~iS/^Jη — Ι). Наконец, центральным двусторонним у-доверительиым интерналом является интервал (X ± + iij_v _ S/V" — I). Этот интервал имеет наименьшую длину среди 2 ' "~ всех γ-доверительных интервалов вида (X— a,S. X -f- a2S), что доказывается так же, как аналогичное утверждение в задаче 2.116. 2.120. Здесь nS2/x — центральная статистика и центральным γ-доверительным интервалом является интервал («S'/xi + T ,, iiS'/xi-, ,), а нижним и верхним γ-доверительиыми интервалами — соответственно интервалы (nS2/%i „_, < 0?) и (Ог < nS3/x?-γ, »-ι). 2.122. Выборочные средние X и К независимы и нормальны УУ(0">, о?/я) и N(Q^\aljn) соответственно, поэтому ЦХ — Υ) = Ν(τ, а2). Следовательно, (X — Υ — τ)/σ — центральная статистика для τ; ■^доверительный интервал находится, как и в задаче 2.116 и имеет вид (X - Υ ± c,Voi/n + affml ,т.п™ьч1(£^)^(„.±).,(Ц^) . - Ч°- i)· Ч ϊΡ™) - "> - "· г( ?'т) -Л" - "■ ■ силу независимости выборок \ 02 / \ я m/ \ V т + л Ог / = N(0, 1), L (-1 [nS2(X) + mS2(y)]) = Х2(т + η - 2), при этом случайные величины X — Ϋ и «S2(X) -J- mS2(F) независимы (на основании теоремы Фишера). Отсюда L(tm + „ _ 2) = S(m + η — 2) и, следовательно, („, + „_ 2 — центральная статистика для τ. Соответствующий γ-доверительный интервал строится так же, как и в задаче 2.119, и имеет вид (X-?±t, + J Г ,т + " -. (nS2(X) + mS\Y))\/2). \ !~^-. - + » - 2|_ тп{т + η — 2) J ) 2.125. По теореме Фишера Z.(nS2(X)/021'5) = χ2(η - 1), L(mX X S2(Y)/W ) = гг{т — 1) и S2(X) и S2(y) независимы, следовательно, L{F„ _ ι. я _ 0 = S(n — 1, m— 1). Отсюда получаем, что центральный γ-доверительпый интервал для τ имеет вид fn{m-\) S\X) ( n(m-l) S\X) I \ \ m(n - 1) S\Y) Ι 4γ· " - ■■"' - '' m<" - 4 WT/ "HP »-·.»-·/" 2.127. Так как Ζ.(2ηΧ/θ,) = χ2(2η), L(2m?/B2) = x2(2m), то ЦтХ/?) = S(2n, 2т). Отсюда, как и в предыдущей задаче, получаем, что искомый интервал имеет вид η · 2п, 2гл λ in. im 7-190 193
2.128. Так как Pd.Xw > χ) = Р"0(Х, > χ) = е-"<<-°> при л; 3s θ, то Po(0</(i)<x + 0)= 1 -β-"' = γ при л: = -—Ιπ(1 -γ), t.e.(Χω + Η 1π (1 —γ), /V(o) — искомый интервал. 2.129. Так как L0(X,/Q) = R(Q, I), то L0(XM/Q) = β(η, 1) (см. зада- ' чу 1.35). Отсюда при 0 ^ t ^ 1 ι·'" Р0((Х^№" < 0 = Р0(Хм/о < (,/я) = я \ χ" - [dx = t. Отсюда Яо(^(Я) < 0 < Х-„)/М'\^у) = /f-^м/О)" ^ 1 - γ) = γ. 2-130. Так как La{2T/tf-) = χ»(2«), то я/х^ <4^< 2 \ ~Г 2« τ(θ) < Χι + ν ) = γ, что эквивалентно утверждению. 2 ' " 2.131. Имеем Л,«о„ T)€Cvw) = p0(V^l^-o,lo2-|<Ci,„ χ2 <ns2/e|< <Х±±х> „ ,) = PJcfn IX - Q, | Q2-' < C>,)P0(X2 _ ., <nS2/0?< <χ1^.„_1)-(Φ(^-Φ(-^)(^±ίϊ--1^.)=ΥιΥ2 = γ. 2.132. Согласно задаче 1.59 п. б) квадратичная форма Q = т-^Г-^№ - °''2 - — № - <М № - θ2) +-!-(& - e2fl 1 — ρ L σι σ,σ2 сг2 J при любых θ и 2 имеет распределение χ2(2), поэтому ? = *о(<з<хЬ)-я0(еес,(*)), где G,(tf) = (Θ : Q = л(*, - θ„ *2 - 02)'Σ-,(^ι - 8„ Я2 - 02) ^ х?3). Таким образом, С,(Х) — искомая 7"Д°веРительная область для В = = (θ,, θ2). Это внутренность эллипса с центром в случайной типке (X,, Х2), граница которого задается уравнением Q = х,2. 2.133. Поскольку L^nT) = Βι(ιι, θ) (задача 1.39 п. 3)), случайная величина Τ принимает значения 0, l/л, 2/л, ..., п/п и ее функция распределения F(—-Q)= Σ co'(i-o)"-r является непрерывной и монотонно убывающей по θ (при k < η): р(А; е) = —nci_,e'(i -еу-'-1 <о, к<п. Следовательно, искомый интервал определяется решениями уравнений F(T; θ) = 1 — F(T — 0; θ) = (1 — у)/2, которые имеют указанный в формулировке задачи вид. 194
Если число наблюдений η велико, то для быстрого нахождения приближенного доверительного интервала для θ можно воспользоваться асимптотической теорией оценок максимального правдоподобия. В данном случае о.м.п. θ„ = X (см. задачу 2.84) и функция информации ι(0) = [θ(1 — θ)]-' (см. задачу 2.43), поэтому искомый интервал имеет вид (X ± сч^Х(\ -Х)/п). 2.134. При больших η имеем Y»P„(Vd*-OI/V~e(l-S)<cv) = Ραί(Χ-θ)2<-^-0(1-0)\ = =4θ2(ι+4)-2ο(*+<έ)+*2<°) = = Ρ0((θ - Γ,) (θ - Τ2) < 0) = ΡΒ(7·, < θ < Τ2), где Таким образом, (7Ί, Тг) — искомый приближенный γ-доверительнын интервал. Если пренебречь членами порядка l/л, то этот интервал сводится к интервалу (X + су-^Х(\ — Х)/п), полученному в предыдущей задаче. 2.135. Поскольку 10{2л/п(т{Х) — τ(0))) ~ N(0, 1), прн больших л у » P0(2fn\x(X) - т(0)| < с,) = Ρ J x{X) %- < τ(θ) < т(Х) + -£-) , \ 2V^ 2φι J что эквивалентно утверждению. 2.137. Решение данной задачи аналогично решению задачи 2.133. Здесь L(nT) = Π(ηθ), величина Τ принимает значения k/a, k = 0, 1, 2, ... , и ее функция распределения ir- ·) -.?. непрерывна и монотонно убывает по Θ: с-„ЛпвУ F'l±; θ) = -„е-»^^<0. Поэтому искомый интервал определяется решениями уравнений F(T; θ) = 1 — F(T — 0; θ) = (1—у)/2, которые имеют указанный вид. Прн больших п (согласно задачам 2.84 и 2.43) справедлива аппроксимация для о.м.п. 0„ = X: £о(У4(*-в)) ~М0. 1). откуда следует указанный вид приближенного интервала. 2.138. В первом случае имеем V« P0(2V«lV^ — Vol <cv) = Pjmaxio, УХ ηΑ < VI <. д/Х + 2л/я/ 195 + -'*
Отсюда следует, что ( max I 0, л[Х 'η=Λ , ( лр( -\ ^-1 ) — искомый приближенный γ-доверительный интервал для 0. Аналогично, используя другую аппроксимацию, имеем V» Po(V^I-?-0|/Vu<cv) = РоШХ ~ О)2 < су0/п) = = Ρ<1(θ2-2θ(χ + ^λ + Χ2 <о) =Р0(Г, <θ<7·2), где Г,., = TUX) = Х+4-+\Х —+ А■ Таким образом, (7Ί, Т2) — другой приближенный γ-доверительный интервал для Θ. С точностью до членов порядка \/п оба эти интервала эквивалентны интервалу [X + су-^Х/п). полученному в предыдущей задаче. 2.140. Из решения задачи 2.96 следует, что искомый интервал имеет вид (б„ ±£νν6η/ημ'(6»)), где 0„ — решение уравнения μ(θ) = X. а μ(0) — теоретическое среднее распределения. В случае распределения В\(г, Θ) этот интервал принимает вид ( 1- с7\1 ). V r + Χ ~ V nir + Xf/ 2.141. Искомый интервал имеет вид (&„ ± c4/^fni(Q„)), где в данном случае ί(θ) = λ/θ2, θ„ = Χ/λ. В результате получаем интервал ХХ-\1±сч/л/Ы). Если воспользоваться аппроксимацией L^-\/Xn(\n 0„ — Ιπ θ)) ~ ~ N(0. 1) (см. решение задачи 2.109), то будем иметь V да P0(VXnlln 6„ —In θ| < су) = ρ(Ίπ 0„ ^<1п θ <1π б„ + т. е. другим приближенным γ-доверительным интервалом является интервал (Χλ-^''·/^, Хк~^ес11^"), который с точностью до членов порядка Ι/rt совпадает с предыдущим. 2.142. Согласно решению задачи 2.109, имеем V»Pu(V2rtHn б„ —In 0| <cv) = P0(u„expi--^}<0<0„exp(-^}j , г ι ^ι τ' где 0„ = — 2j^'~M·)2 · Полученный интервал сводится к иптер- L п , =, ι J валу 0„(1 ± Cy/^[2ri), если пренебречь членами порядка, \/а и восполь· дартнон аппроксимацией 1^(0„)~Ν[ ΰ, -тг-) (см. зада· зоваться стан чу 2 2.143. Из решения задачи 2.87 следует, что искомый интервал, основанный на стандартной аппроксимации для о.м.п,, имеет вид (τ„ + cvcr,(0„)/V"). где τ„ = φί-^^-λ , §„ = (6„„ 02„) = (Χ, S), σ2(0) (см. задачу 2.87), 196
2.144. В данном случае модель определяется (N — 1)-мерным параметром Θ = (р Р«_|) (см. задачу 2.63) и функция правдоподобия выборки X = (Χι, ... , Хг) равна L(X\ 0)= Π р;'=е*р( Σ ν,ΐη Ί El + ; = ι l;= ι l -p. -·· -Ρα,-ι + «In (1 — p, — ... — pw_ ,)|, где v; — число членов выборки X, равных α(, / = 1, ... , Ν. Отсюда непосредственно находим, что решение урапнеинй правдоподо· din L . бия — = 0, / = 1, ... , N — 1 (т.е. оценки максимального правдо- др, подобия параметров ρ ρ«-ι) имеет вид р( = —, /= 1 Л/—1. Информационная матрица этой модели /(θ) указана в задаче 2.45. Согласно асимптотической теории оценок максимального правдоподобия, Lo(^fnlfi„ — θ)) ~ N(0, /_|(0„)) при η -*■ оо и поэтому La(Qn(B)) »■ χ2(Λ/ — 1), где квадратичная форма Л/- I <3„(θ) = n(8„ - Β)'/(δ„) (В„ - θ) = η Σ Φ'- Ρ') (Ρ> - Ρ*)/Ρ * + r.s = Ι Ν — Ι Ν + η Σ iP'-P-f/Pr = Σ Κ-ηρ,)7ν,. Отсюда Яв( Σ (ν,-«ρ,)7ν,<ΧΪ.*_.) —Υ· Это означает, что искомая асимптотическая γ-доверительная область для параметров ρ ρν имеет вид Г W Су(Х) = (ρ pa,)·. Σ (v,-np02/Vr<X2».«->. 0<р,<1, W 1 г= ι, -., λ/, Σ ρι= " · В данном случае эта область представляет собой пересечение внутрен· /ν ^ (ν, — ПО,) , ности Λ/.мерного эллипсоида 2j — <Χν.ν-ι с гиперплоско- г = I ' стью р, + ... + рл = 1, принадлежащее зоне 0 < р; < I, / = 1 N. При Л/ = 2 полученное решение аналогично результату задачи 2.134. 2.145. По теореме Фишера (см. также задачу 1.56) I и S2 независимы, следовательно, независимы также Х — Хп + \ и S. Но L&X ■ . χ„ + ,) = ινίο, 022 " ^ ' \ , a f о(-^-) = х\п - 1). Следовательно, -Г + 1 отношение Стьюдента („ _ ι = "Д/ ;—; п" + ' . Отсюда имеем 197
_ /-.In-1 \х-хя + ,\ , \ = p.(^-^i„_1sV^<^ + .<^+^.._1sViii). что и требовалось показать. 2.146. Для приведенных данных jc = 4,196; s = 0,226; (0.975.4 = = 2,776; следовательно, искомый интервал (3,43; 4,96). 2.147. При больших η у * p(-KgL < с») = я^ - ">2 < 2,гс?) = = Я(л2 - 2п(| + с?) + I2 ^ 0) = Р(п, < л < л2). где /ti.a = Л|.а(5) = ξ + Су Τ cvV2? -f с?· В данном случае С09 = 1,645, поэтому искомый интервал (131; 189). Глава 3 3.1. Имеем две группы данных с частотами k\ = 2048 и /г2 = л — — h\ = 1992. Для проверяемой гипотезы На : ρ = q = 1/2 и ожидаемые частоты равны пр = л<7 = л/2 = 2020. Статистика критерия Х\ = Σ — ^— = 0,776. При больших л эта величина распределе- , = , "Р· на приближенно по закону хи-квадрат с одной степенью свободы. По таблице квантилей распределения χ2 находим χο.95; ι = 3,84; χοι9. ι = = 2,71. Поскольку 0,776 меньше значений этих границ, можно считать, что данные совместимы с гипотезой На. 3.3. Статистика критерия Xl = Σ—! ——= 4,13 сраннннает- ,·_. пР< ся с критической границей хо,оз: г = 6,99. Поскольку 11,13 > 5,99, гипотеза Но отвергается. 3.7. Ожидаемое число показаний часов в каждом интервале равно /гр, = 500/12 = 41,67. Статистика критерия равна XI = 10,00, что меньше критической границы Хо,9:п = 17,3, т. е. согласие хорошее. Гипотезу Н0 не отклоняют при уровне значимости α < 0,55. 3.8. Здесь статистика критерия Х\ = 0,47, χο9. 3 = 6,25, т. е. согласие при α ^ 0,9 хорошее. 3.10. Границы интервалов находим из уравнений 1 — е~*' = = 1/4, e~xi — <?-■*/ + ' = 1/4, / = 1,2. Имеем х, = 0,288, хг = 0,693, хъ = 1,386. При группировке по этим интерналам получаем вектор частот h = (9, 9, 17, 15). Статистика критерия Х\а = Σ (/ίί~"Ρί) = 4,08 меньше критической границы /о.о.з = 6,25, т.е. гипотеза Н0 не отвергается. 3.11. Поскольку Р(| <: х\Н0) = 1 — е~*/0, вероятности р,(0) = = Р(| е £/|//о) в данном случае равны Р/(В) = е-('-1У/о(1 _ е-°/\ Ί = 1, .,., N - \, ρΛ(θ) = е-(л/- ι^/ο, 198
и уравнение для нахождения мультиномиальной о. м. п. О, имеет [1, с. 116) вид Σ-ν;(0)/ρ,(θ) = Σ Hi - 1 - \е-"й)!{\ - е-'") + (Л/ - 1)/ι,ν = 0. i = I /=1 Обозначая ζ = е'а^1 отсюда находим ζί Σ /Ay — Ад,) = Σ /Λ/ — η. 4/-ι ' /-ι Следонателыю, о. м. п. ζ„ = Ι Σ/А, — ")/( Σ/А; — Ад,], а соответ- ствующие оценки вероятностей р,(0) имеют вид р, = Н-'(1 - 2а,), /= 1 Λ/ -Т,'рА,= δ»-'. В соответствии с общей теорией [1, с. 115—116], если при больших значениях η выполняются условия Λ, ^ 5, / = 1, ..., Ν, то соответствующий критерий согласия χ отклоняет гипотезу Н0 тогда и только тогда, когда χ» = Σ (А; - лр,)7("Р,) ^ χ?-* а,-2, где α — выбранный уровень значимости. Для данных задачи 1.21 при указанном выборе параметров N и а имеем /г, = 28, /г2 = 16, Лз = 6, 2„ = -^ г— = -гтг и значение ста- 2л — А| 18 тистики Уп = 1,96... Поскольку хо.з, ι = 2,71, при уровне значимости α sg 0,1 гипотеза На подтверждается данными. 3.12. Здесь имеет место полиномиальная модель с JV = 4 исходами и вероятностями р\, ..., р<, имеющими при нулевой гипотезе На указанный вид, т. е. являющимися функциями одного неизвестного параметра. t Для оценивания параметра θ надо решить уравнение Σ Α,ρ,'(0)/ρ,(θ) = i=1 = 0, которое в данном случае принимает вид Αι А2+Лз Ju_ _ „ I Q 2+Θ 1-( Это уравнение сводится к следующему. φ(Θ) = ηθ2 + (А, + 2/г2 + 2/г3 - ή,)ϋ - 2/г4 = 0. Поскольку φ(0) = — 2/г4 < 0, <р(1) = 3(/г2 + А3) > 0, последнее уравнение имеет в интервале (0, 1) единственный корень б„. Следовательно, в данном случае критерий согласия χ1 при уровне значимости α отклоняет гипотезу //о лишь в случае Σ А,?/('«Р,<0«)) - " 3*ΧΪ-«:ΐ / = ' или Л?/(л(2 + О,)) + (Л22 + hl)/{n{\ - 0.)) + АЗ/(ч&0 ^ (χί_„.. 2 + «0/4. 199
3.14. Используем критерий согласия χ2. Оценка параметра В равна β1 В = 2 М' = 3,870. Вычисляем оценки оероятпостей р, = е~®-гт-, ί = 10 <h — " \г = 0, 1, ... , 10, и значение статисики XI = 2 —'—А = 13,05. ; = о яр,- Число степеней свободы k = 9. Поскольку χο,95,9 = 16,9 > 13,05, гипотеза Но не отвергается. 3.15. Здесь θ = 1,54, Х\ = 7,95, k = 6, χ20,,. 6 = 10,6. Согласие имеет место. Л л/ 3.16. Здесь θ = 0,928, ρ, = е"0-^-, i = 0, 1 5, Х\= 2,172, ί! k = 6,7. о,95; 4 = 9,49. Согласие хорошее. 3.17. Для Щ) = S/(2, θ) вероятности исходов имеют вид р,(В) = Ρ(ξ = 0) = (1 - θ)2, ρ,(θ) = Ρ(ξ = 1) = 2ϋ(1 - θ), ρ,(θ) = Ρ(| = 2) = Β2. Отсюда уравнение для нахождения оценки параметра θ имеет следующий вид: Σ />,ΡΙ(Θ)/Ρ,(Θ) = - -gL·. + -^^h2 + ^L = 0. а сама оценка B„ = (fi2 + 2h3)/2n. л В данном случае h, = 476, hi = 1017, ή3 = 527, η = 2020, поэтому Β„ = 0,513. Далее имеем з Я = Σ № - np$.)f/(np$H)) ~ 0,116; i-l результат надо сравнить с χ?-«ο ι· Поскольку хо,з; ι = 0,148, при любом уровне значимости α < 0,7 гипотеза принимается. 3.19. Подставляя в формулу Е(Х1\Р) = η Σ (Pi - ρΊΫ/ρϊ + Σ Ρ,Ο - Ρί)/ΡΪ Ν значение ρ = ρ\η) = (р\"\ ..., ρ'ί') и учитывая равенства Σ ρ? = 1. /=ι и Σ β/ = 0, найдем Ν Ε(*ΪΙρΜ) = Λ/ - 1 + Σ β?/ρ? + 0(1 /λ/4 /=ι Далее, так как R>s = Σ р^/рГ = Σ ρΡ'-'( ι + -^-V, «32= 1 + ^-Σβ?/Ρ?+ 0(«-3^, 200
Ru = ι + -J-Σ β?/Ρ?. β22 = Λ/ + 0(1/V^), Ли = Ν + 0(1/V^), /?,2 = 0(l). Отсюда, используя формулу ЩХ1\р) = 1("~'Х"~2)(/?з2 - Rh) + 2^—UiR22 - 2R2iRu - /&J+ + -^Л|2 - Л?.), л найдем D(*2lp<"') = 2(W - 1) + 4 Σ ρί/ρί + 0(1/лЯ / = ι 3.20. Так как (1 - р<">)" = ехр{л1п(1 - р<">)| = ехр{-лр<"> + 0(η"')) = = exp{-p-pfti/n,'", + 0(n-,))= е-"|1 - pft,/n,/4 + р2Ь,2/2л/п + + 0(п-3<")], А/ то, подставляя это разложение в формулу Ε(μ0|Μ"') = Σ(1 — pi™3)", i = i получаем искомое представление для среднего. При анализе дисперсии будем исходить из формулы и D(l0 = 2 Σ [(1 - ρ, - Pi)" - (1 - Ρ,-ΠΙ - Ρ,)"] - Σ (1 - Ρ,)2" + Εμ„. κ; / = ι Асимптотика второй суммы при гипотезе Н\п) находится так же, как и выше, и с точностью до главного члена она равна Л/е_2р. Оценим общий член первой суммы. Имеем (1 - Pt - Ρ,)" - 0 - Р-П1 - Pi)" = (1 - Pi - Pi)" - (1 - Pi - Pi + p.PiY = = exp(-n(p, + ρ,-) - -|(Pi + ρ,)2 + 0(ЛГ2))- ехр(-л(р; + ρ,) + ηρφ, - - \{Pi + Pi)2 + 0(N-*)\ = exp(-n(p, + p,) - -£(p. + Pi)2fexp(0(A/-2 -ехр(пр,Р)+0(Л/-2)1]= -npipie~nP'~ni4+0(N-2). Отсюда первую сумму можно представить в виде -2л Σ P/Pi«"""""'"" + 0(1)= - п( Σρ/ί-"ρ')2 + ·</ \ /-1 / + ηΣρ^-2""' + 0(1). 1 = 1 и В этом разложении второй член есть 0(1), a ^p,e~npi = е'р + о(1). /=1 Таким образом, все выражение равно — Л/ре '-'■' + 0(1). В результате имеем 201
ΕΚμοΐΜ"1) = - Npe~2ρ - Л/е"2" + Ne~> + 0(Л/|/2). 3.21. Применим критерий однородности χ2. Статистика критерия XI = п,пг Σ ! (ν,ι/«ι - ν,2/η2)2 = 2,18 (s = 4, ft = 2), (=1 V,i+V,2 критическая граница критерия x2_«;(s-ixi-i) = х2,9:з = 6,25. Таким образом, Х\ < χ29;3, согласие хорошее. 3.23. 1} Рассмотрим разность Δί; = ν,; — ν,.ν./η, ί, j = 1,2. Непосредственно проверяется, что все суммы Δ^ + Δί2 и Δι; + Δ2/- равны 0. Например, . . VIVI V|.V.2 Vj. . Δ|| + Δ|2 = V|| + V12 = V,. —{V. ι + V.2) = η η η = ν,. — ν,. = 0. Таким образом, модули всех четырех величин Д^ равны и поэтому 2 Л2- 2 ι XI = „ Σ -^- = ηΔ2, Σ —— = = n3A|l/(v,.V2.V.,V.2), Далее имеем «Δι ι = ν.ι — (v., + v.2) — (ν,, + v,2) = v.,v.2( — — Γ, L v.i J \ v.i v,2/ окончательно получаем Х1 = Л^У±(^__^Л1 =z; v,.v2. \ v., v.2/ 2} Заметим, что случайные величины vn и vu независимы по условию и i(vi/) = Bi{n-h ρ) при некотором ρ е (0, 1), j = I, 2, если справедлива гипотеза Н0. При nt, пг -*■ оо по теореме Муавра — Лапласа i(vi/) ~ Щп,р, n,pq), q = 1 — ρ, или Цуч/rij) ~ Λί(ρ, ~-J. У = I ° Отсюда \ «I «2 / \ П,П2/ Таким образом, при гипотезе Но случайная величина / Vll _ У|2 \-Л / "'"2 л f ■ - ι τ ι Ι ς""" ■ - - ■ - npq асимптотически нормальна N(0, Ι). Но 7 - с -\/ "2pq „ νι. ρ -л / " Ρ<7 ρ , ,-, и при гипотезе Но имеем >-р, следовательно, \/—ι >- I. От- η ν ν,.ν2. сюда следует, что при гипотезе //0 предельные распределения случайных величин Z„ и ςη,„, совпадают, что и требовалось показать. Наконец, при рассматриваемой альтернативе среднее значение раз- 202
Vι f V| 2 ности равно р\ — рг > О, следовательно, проверяя Н0 про- тив //,, критическую область разумно выбрать в виде [Ζ„ > ία). Поскольку Ρ(Ζ„ > ta\H„) ss φ( — ta), при уровне значимости α критическая граница имеет вид /« = — ф-'(о) = ф-'(1 - а) = и,-.. ■ За мечание. Так как при любой гипотезе, задаваемой вероятностями pi, рг, при л,, п2-*■ °° \ «I «2 / \ «I «2 / то, рассуждая аналогично, можно показать, что для «близкой» альтернативы Ηψ*: (ρ, — p2)/Vpi<7i = a/V"i α =^= 0, L(Z„|///"') -у MWv(l - Υ). Ο. Υ = lim —. «-.со П Этот результат позволяет вычислять мощность критерия при таких альтернативах. 3.24. Так как (см. задачу 1,54} £(vi, ..., vw|vi + ... + vv = η) = Μ(η; ρ, ρΝ) Ν при pi = 0//Σ 0.. / = 1. ■■■! Λ/, то гипотеза //о эквивалентна в данном 1=1 случае гипотезе о равновероятности исходов в полиномиальной схеме. Следовательно, критерий согласия χ2 имеет вид *-£,!,("-£)■>"-- ι Замечание. Обозначим через v, S2 = — 2(ν/ — ν)2 выбо- /=| рочные среднее и дисперсию; тогда статистику XI можно записать в виде XI = NS2/v. При гипотезе Я0 теоретические среднее и дисперсия наблюдений равны, поэтому, если гипотеза Н0 верна, то S2/v ->■ 1 при η ->■ оо или XI -*■ N. 3.25. 1} Так как в данном случае совместная плотность распределения порядковых статистик -¥(и Х(„) есть g(x]t .... х„) = л!, 0 < Χι < Х2 < ... г?С *,, < 1, (см. задачу 1.31), то по формуле полной вероятности безусловное распределение вектора κ = (κι, ..., κ,,+ ι) имеет вид ml г"+' Р(*. = к,, ί = 1 л + 1) = -r-r—г j- Π (*/ - *i- .)*'£(*. Й|! ... Кл+ ll J;_ , mf ^f Jl I (. л + 1 ...,^)^ι...^„ = .,',' ЛахЛахъ- \ П(*/ — Щ-\) 'dxa (здесь fe, + ... + fe„+, = m, *0 = 0, x„+, = 1). Интегрируя последовательно по х„, х„-\ и т. д. и применяя формулу ! (' - ")'(! - *)'** = (r + s+\y.{[ - a)' + S+'· 203
получим ρ (κ, = ft,, i = 1 η + 1) = *,!...*„+,! (ft„ + ft„+,+ l)! fe„_i!(fea + fe„ + ,+ l)! *.,](fe2 + ... + <:„ + 1 + n-l)l (ft„_,+ft„ + ft„+, + 2)! '" (fe,+A2 + ...A„+1 + n)l - (m + n)! - (C"' + ") С другой стороны, при Αι + ... + ft„ + i =m и произвольном ре(0, 1), q = 1 — ρ, P(i, = *,< = l ,.+ МЬ+-+Ьн—т)- р(Г^'Г'П+'! = Ρ(|ι + ... + i„+i = m) »+ ι Π (P*'<7) = T^i—„„■„+! = (Ci + m)-', поскольку t(|, + ... + £„ + ,) =B, (η + 1, ρ) Ья + тр ί/ (см. задачу 1.39 п. 5)). Таким образом, если справедлива гипотеза однородности Но, то £.(*) = t(|, i,+ ilii + ... + |„ + ι = m). 2) Вычислим л+l P(so(n,m) = /г) = P^2 / (i;= 0) = *|ξ, + ... + !„+, = m) = Pv ( Σ ' (i< = 0) = ft, Ι, +... + !„+, = m) /P(|, + ... + |„+, = m). Чтобы найти числитель этого выражения, зафиксируем номера тех величин, которые принимают значение 0. Это можно сделать Cj+i различными способами. Тогда числитель можно записать в виде С;+,Р(|, > 0,/= 1 п + l-k.li = 0, j = n + 2-k п+ 1)Х ХР(|, + ... + |„ + 1 = т||, > 0,i= I п+ 1 - ft, |, = 0, / = /1 + 2-* л + 1) = с;+,ря+,-уР(1, + ... + !„+,-* = m) (см. указание 2). Здесь при г = 1, 2, ... Р(1, =#■) = Р(|, =0/p(ii > о)=-у- = p'"V т. е. £.(!,) = £.(!,+ 1). Отсюда £(|, + ... + L) = £(|, + ... + Ь + s) и поэтому Р(|, + ... + !, = m) = Ρ (!, + ... +|s = m-s) = C^q'p"-. Окончательно можем записать, что р(50(п, m) = ft) = cί+,9y+'"'α-v+'~У"+'_n-7α+mp'v+, = С Λ /~>n — k //~>π η+ lL«i- l/Un + m. Таким образом, L (s0{n, m)) = Η (η + 1, η + m, η). Используя формулы для моментов гипергеометрического распределения, получаем, что при гипотезе //0 204
с / \ п (" + 0 r> / \ m(m—1)/](п + 1) Es0(n, m) = —-—■—-, Dso(/t, m) = ■— ■- ' η + m ' ' (η + ιηψ[η + m — I) Если л, m -> oo так, что m/л = ρ > 0, то ESo(n,m)=T^—+ 0(1), Ds.(/i, m) = "/ a + 0(1). 1 тР И τ Ру 3) Из решения предыдущего пункта следует формула для P(s0(n, m) = ft), приведенная в указании. Пусть ft = (л + 1)р + t л/лР</· Μ < с < оо. Тогда ft(ft;n+liP)==l+£(l)e-"A 2nnpq η — ft = (m — l)p — т/лр</ и поэтому Vp ft (η - ft; m - 1, ρ) = ',+ °(1) г"'/м; наконец, п = (л + m) p, следовательно, О (П П + m, p) = rr—; ; r . 1 'Г' у2л(п 4- m)pq Объединяя эти оценки, получаем P(S.(«, m) = ft) =Vo" + f" ^-''"+pl/(21l + 0(1)). V 2nmnpq ι ι Но это означает, что если & = -г-—^ Ь л: V"P2/([ + ρ)3. 1*1 <| с < °°ι то ' + Р т. е. имеет место локальная (а тем самым и интегральная) нормальная предельная теорема. «+ ι E[s0(n, m) |//,]= Σ Р(я,^0|//,). i = I Рассмотрим сначала слагаемые при / = 2, .... п. Вероятность того, что при фиксированных A"(i_n = jci < -V{i) = хг блок β, пуст, равна [1 — f(Jfa) + Ζ7(*ι)Γ. поэтому по формуле полной вероятности р(* = °i"-) °(i-2)u„-i)i \άχψ - F{x* + + F(*,)]"Y,-2(l - Χ2)"-'(Ιχ'2. Для / = 1 и / = η + 1 эти вероятности будут иными, но если их выражения разделить на η + 1, то в пределе они дадут нулевой вклад в сумму и, следовательно, им можно не уделять внимания. Суммируя эти выражения, в результате получим [ sg{n, m) ι π η (л — I) г rr 205
+ F(A-,)f(l - α·2 + x,Y~4x2 + ε„, где ε„ < 2/(я + 1). Сделап замену переменных xf = t/i, хг = y\ + Уг/п, перепишем эту формулу d виде «о(п, m) ι Ί л — 1 - ι -J' - уг/п x[l ^ — Τ" Λ/2 + ε„ L Уг/п л J Переходя здесь к пределу при л, m-юо, т = рл, и учитывая, что функция F дифференцируема, получим ' so(л, m Г so л. m ι η ,?е-М1 + Р)МЦ2 = Г ^ . 1 + р/W Теперь (см. указание) dx = (ίβι(*)β2(*)ώ)2 < j(l + p/(*))d*JT + p/ Μ (1 + p)(- dx так как \ Ja 1 +Pf(x) ' f (x)dx = 1. Знак равенства имеет место лишь в том случае. когда функции gi(*) и ёг(х) пропорциональны. В данном случае это означает, что 1 + pf (*) = const, т. е. f (χ) == const. Но так как f (х) — функция плотности, то f (х) = I. Таким образом, если справедлива альтернатива, то неравенстпо будет строгим и поэтому при альтернати- пе статистика s0(n, т) асимптотически стремится принимать большие значения, чем при гипотезе Я0. Следовательно, критическую область для гипотезы //о разумно задапать в пиде [s0(n, т) > с). При уровне значимости а критическая граница асимптотически имеет вид с = Са(п) = -г— V" Г+р v" (1 + р);,/'2 " 3.26. Имеем^ = л(у — l) = 8,09 < γ20054 = 9,49. Гппоте- \^ ν,.ν, / за не отвергается. 3.28. 1) См. решение задачи 3.23 п. 1). 2} Пусть (Χ,, Κι) (Х„, Υ„) — иезаписимые наблюдения над (1Ь ξ2) и X = (X Хп), У = (Κι, ..., К„). Тогда выборочные средние X = Vi. - V ι = , К = , выборочные дисперсии η η s? = s>(X) = 1 ς xl - * = ^-(ι - -£-) = 206
наконец, выборочная ковариацня s,2 = 1 ς χίΥι - π = ^ ~^ψ. = Al ί! . _ П П Π Отсюда (см. решение задачи 1.38} Рп = S|2/S|S2 = П Δ] i/Vv 1 -V2-V . iV .2 = II W2Z„. Теоретический коэффициент корреляции Ε(ξ,ξ2) - Εξ,Εξ2 Ρ(ξ, = 1, ξ2 = Ι) - Ρ(ξ, = 1) Ρ(ξ, = 1) Ρ = лЩГОЬ λ/Ρ(5ι = 1) Ρ (ξΓ^~0) Ρ(|2 = 1) Ρ(ξ, = 0) _ Р(Лй)- Ρ(Λ)Ρ(θ) л/РИ) P'fι Р(й) Ρ(β) Но РИА)- Ρ(Λ)Ρ(β) = Р(А)[-РА?-)(Р(А)+Р(£5)_(Р(ЛА) + Р(Л£5)] = P(fl)P(£)[P04fl)/P(fl) - Ρ (Лή/Ρ (11 откуда следует второе представление для р. 3) Рассмотрим случайную величину ς» = Σ(χ· - Ρ(Λ))(Υ, - Ρ(β))Λ/"Ρ(/ΐ)Ρ(λ)Ρ(β)Ρ(ϊ9. Т;и< как при гипотезе Но Ε (Х- - Ρ (-4)) (Yt - Ρ(β)) = Ε (Xi - Я (Л)) Е(К(- - P(S)) = 0, D (X - Ρ (Α)) (Υ, - Ρ(β)) = Ε (Χ - Р(Л))2(У, - Ρ(β))2 = = Е(Х, - P(A)f Ε (Υ,■ - Ρ(β))2 = D Xi D У,- = Ρ (Λ) Ρ (Л) Ρ(β) Ρ (β), το ς„ представляет собой нормированную сумму независимых одинаково распределенных случайных величин. Следовательно, на основании центральной предельной теоремы при η -*■ оо L(i„)^ N(0, 1). Теперь, так как 2 (Xi - л) (У, - Τ) = Σ № - Р(Л)) (У, - Ρ(β)) - '"' -п(7-Р(М))(У-Р(А)), то на основании п. 2 задачи можем записать Zn в виде η я"2 2 (Jfi-^W-F) Ζ. = n,/2S,2/(S,S2) = '-' =— = л!Х(\ -χ) υ{\ - υ) |/2fY_pe/lN V—Pf^ Ί Γ Ρ/Λ\ ΟΙΛ\ D/R\ D/R\l 1/2 _Γ я"'(Л-Р(Л)) У-Р(Д) IJP(A)P(A)P(B)P(B)-l L " νρΗ) р"Й) Vp(b) p(S)J L Xi - x) Щ - Η J мптотической нормальности в L( а»г(Х - КА))\ По теореме об асимптотической нормальности выборочных моментов при η -+ оо 207
а согласно закону больших чисел X -*■ Р(<4), У -»■ Р(В). Следопательно, при /г -»■ оо ΓΡ(Α)Ρ(Α)Ρ(Β)Ρ(Β)γ* t L ί(ΐ -л) Pi - р) J ~* ' „■"(Λ- Ρ(Λ)) (У-Р(Д)) Дп λ/Ρ(/1) Ρ(-4)) P(B) P(S) ~* и поэтому предельные распределения Z„ и ς„ при гипотезе Я0 соппа- дают. Итак, при гипотезе Но имеем ρ = 0, а при альтернатипе //ι ρ > О и, следопательно, Ζ„ -»■ + оо при л->- оо. Поэтому естественно рассматривать слишком большие значения статистики Ζ„ как свидетельство d пользу альтернативы. Другими словами, если проверяется гипотеза Но против альтернативы Ни то критическую область разумно выбрать d dh- де (Z„ > U). Поскольку P(Z„ > /„|//„)« Φ (-/„), при выбранном уровне значимости α критическая граница имеет вид /„ = — ф-'(а) = ф-'(1 - а) = ц,_„. Замечание. Рассуждая аналогично, можно показать, что если «близкая» альтернатива задается услопиями Н\Й): Р{АВ) = Ρ (Α) Ρ (В) + 0(п~"2), ρ = ρ'"' = an-"2, α Φ 0, то L {Ζη\Ηψ>) -»■ Ν (α, 1) и, следовательно, при а > 0 мощность построенного критерия удоплетворяет предельному соотношению Wn(H\n)) -+ Φ (α + u„ *Я^П · Λ9 449 3.29, Здесь (см. задачу 3.28} Z„ ' "' *">-" V360 82/ V 137-305 ~ ~ — 3,86, а Х\ = Ζϋ ~ 14,89. Так как χο.9999. ι = 10,8, то гипотеза о независимости признаков должна быть отвергнута (вероятность совершить при этом ошибку меньше Ю-4). В то же время данные опровергают и гипотезу И], поскольку Ζ„ < 0 (этот факт можно интерпретировать, например, как отсутствие дискриминации в отношении женщин прн приеме в вуз). <,<,„-, , /276 3\_. /749-69-8Ι8 с лс ~ ν2 3.30. Здесь Ζ. = (ш - ёэ)А/ 279.539 s 5'45' а «= Z" s ~ 29,70. Следовательно, на основании полученных данных (см. решение предыдущей задачи) гипотеза Н0 отклоняется. Далее, так как Z„ > > Ф~'(0,9999) ~ 3,72, то (см. решение задачи 3.28 п. 3)) данные подтверждают гипотезу Ну. Вероятность ошибки отклонить Н0 в пользу Н, при Этом меньше \0~\ 3.31. Здесь речь идет о проверке гипотезы случайности //0 [1, с. 133]. В данном случае число инверсий Те = 0 и вероятность получить такое значение при гипотезе Н0 равна (8 !}-' ~ 0,25· Ю-4. Следовательно, при любом разумном уровне значимости гипотеза Н0 отвергается. 3.32. Обозначим Т'п = б(Тп " ^" ~ ' j л_3/2; тогда характеристическая функция *■«-«,{-1%^} *«-■■)- 208
При |/| sj с < оо, л->- оо и любом к = 1 η ехр{6/^л-3/2) - 1 = 6ttkn-3'2 - 18/2А2л-3 + 0{к"п^'г). Отсюда fΣ> - - 1) = -^ _3*V-у-О + о(^). Переходя к логарифмам и используя формулу г-2 получим In (I +*) = *--£-+ О (А * + 0, 3/7 (п — I) 3(7 + -πτΣ('-ν ι/2 л q/2 " - Α- Σ (' - 1) (2л - 1) + * Σ (^ - I)2 + откуда и следует утверждение. 3.39. Здесь статистика отношения прапдоподобия '(*) = [!у а'! = Π d<{\ - 0ι)*~* / Π CWd - Оо)*-*' = _ г 0,(1 —Θ0) η7"/ 1 -0|\»" " _Lo„(i-e,) J V ι -θ0>/ ' £, и при 0, > θο величина —'-j- —-(- > 1. Поэтому неравенстпо /^с θο(1 — οι) эквивалентно неравенству T^t. Пусть α — заданная вероятность ошибки первого рода. Определим целое число /„ из условия kn kn α"= 2 ^ΐΜ{\ -0ο)*"-'"<ο^ Σ C"»0"(1 -Оо)'""" = α'. (*) m—ία + 1 m —/α Если здесь α = α', то искомый критерий является мерандомизированным и имеет вид Х'а = (7" > /а). Так как i^T") = Bi(kn, 0) (см. задачу 1.39 п. 3), то вероятность ошибки первого рода этого критерия равна ЩХ'а; θο) = Р0„(Г > /.) = а' = а, а мощность такопа: Щх]а;01) = Ро,(т>и)= Σ cr„or(i-o,)'"-m. m = la Если же в (*) имеет место строгое неравенство (а < а'), то критерий Неймана — Пирсона является рандомизированным и задается критической функцией 209
1 при Τ ^ ία + 1, - а" —- при Т= t* -а О при Г< ία- 1. α —α Его мощность U%;: 0,)= Ε0ιψ„(η=Ρ0ι(7'^ /а+ 1)+ °, "„ Ро,(Т = Л0 = ее — α = 2 са о г (ι - οί- - - + (а - а") ( -jji) '-( -1=-^) *" - Ч « = („+! \ Uo/ \ 1 — bo/ В этом случае (при α < α') можно также использовать нерандомизи· роваимые наиболее мощные критерии Х]а- = \Т ^ ί„) и ДПа" = \Т > /а+ 1) с уровнями значимости соответственно а' > а и а" < а. 3.40. При л->-оо по теореме Муавра — Лапласа L^T) ~ N{kn0, кпЩ - 0)), поэтому условие (*) для определения критической границы ta можно заменить приближенным условием Роа(Т> Ια) « Φ ( . -) = α. ^ V*:nOo(l — 0ο) Ι Отсюда следует указанный вид критической области. Далее имеем wM"]) = ρ0\·(τ > О = Рос ( iT~kn0f") = > -> л/ fe" "m'"l (11 „ -л/ 0о(1-0~\ = φ(ΡΛ^?^+"» + ο(.)) + 0(ΐ), что эквивалентно второму утверждению. 3.41. Критерий строится по той же схеме, что и в задаче 3.39. Здесь достаточная статистика имеет вид Τ = £ X, и при этом) L,(T) = Π(ηθ), ι = | t(X) = (-^-) е"'°°_0''. Для определения критической границы ία при заданной вероятности ошибки первого рода а имеем условие »ι = /α+| '"· п, = 1а "'■ При а = а' критерий имеет вид Jf"a = (Г ^ /„), а при а < а' он является рандомизированным-н его критическая функция φϋ(Γ) имеет такой же вид, как и в задаче 3.39 (с учетом обозначений, принятых в (*)), В любом случае мощность вычисляется по формуле 11/(0, 1г"^+|"-|Й"·-'"1 Если л-*оо, то i(i(7^~yV(n0, ли), и рассуждая как и при решении задачи 3.40, имеем, что асимптотическая форма критерия имеет вид 210
(7' J5 лОо — иа-\Д10о), а ого мощность при близкой альтернативе Οι = О*,"' = = Оо + β/л/й, β > О, удонлетворяет предельному соотношению lim WMn)) = <b(-^-+"*) л/Go 3.42. Наблюдаемая случайная величина X имеет геометрическое распределение βί(1[ 0) и статистика отношения правдоподобия /(А') = / Θ, \х 1-0, ι = (-д—Ι η д—■ Следовательно, неравенство / J5 с эквивалентно неравенству ЛГ ^ / и для определения критической границы I — 1а имеем соотношение а = 05 = Р0[1(Х > U) = Σ °όΌ - °о) = 0ό·, nr = ta откуда находим la = s. Таким образом, в данном случае критерий Неймана—Пирсона имеет вид Х'а = [Χ Ξ5 s] и его мощность 1_β= 117(0,) =Р0Д^ s)= Σ 0i"(l-0,) = 0I. ΠΙ = S (π \n ~~a~) ^ ХеХР{("07_1г) 4 T = Σ X'< ^(27-/0) = χ2(2η) (см. указание). Если ϋ0<θ,, то неравенство I ^ с эквивалентно неравенству Τ ^ I и для определения критической границы ί = /α при уровне значимости а имеет соотношение α = Ро„(7" > U) = Ро0(2770о > 2/./0») = P(xL ^ 2/а/00). Отсюда 2/α/Οο = χ?_α.2/ι и, следовательно, оптимальный критерий имеет (ft » 7" Ξ5-^-χ?_α.2/ι [■ Его мощность равна 47(0,)=Р0, (7'>-у-Х1_„.2л) = Ро,(27'/в) ^Οοχ?-..2„/β.) = = 1-/Γ2"(ΐ7χ?-°-2")· где F2„{t)— функция распределения закона χ2(2/ι). Аналогично, при 00 ~> 0, оптимальный критерий имеет вид Х'а = Ι?" *ζ -тгх1.2л\ и ег0 мощность 3.44, В данном случае множество критических значений наблюдения определяется из условия которое эквивалентно условию (с— I)*' — 2м + 2с — Ι *ζ 0. Если с = 1, то неравенство выполняется при д; ^ 1/2. Это означает, что критерий Х'а = [Х^ 1/2) имеет уровень значимости 211
a=ρ·(* > τ) = τ Jjt^=у ~ Tarctg τ и его мощность равна Р'(^у) = 1Г|)2ТТ0^ТГ = У+1ГагС,еУ' Положив с = 2, получим неравенство (*2 — 2)2 ;ζ 1 или 1 < д; ;ζ 3. Это означает, что критерий Х\а = (1 *ζ Χ <; 3) имеет уровень значимости з о= Po(l < Л~ < 3) = — \ dx ,=±- (arctg3 - arctgl) π J 1 -f- д; π и мощность Pl(l<^<3)-li-rT*L_5r-i..rctg2. 3.45. Пусть X=z(X, Хп) — выборка из распределения Ц£). Если хотя бы одно |-¥,| > а, то при гипотезе Но это невозможное событие, и поэтому в данном случае Я0 следует отвергнуть. В остальных случаях, т.е. при 71,'' з= max |Χ·| ^ а, решение принимается на основе ι < ι: < η анализа статистики отношения правдоподобия «-H,^-"'"/w-(VFf)"-{-^«l- ft21 = Σ Я. i- I В данном случае неравенство / J5 с эквивалентно неравенству 7V =ζ ί и поэтому искомый критерий имеет вид Х'« = (П'>> a) U (П'> < а, П2) < /„) = (ft" > а либо ft2> =ζ У, где при уровне значимости α граница ta определяется из условия а = Р(*е*и|Яо) = Р(П2><иЯ„)= \.. .\ -%4^ = *\ + ...+xl*il. a> (π/J ,«/2 Γ(|+ΐ)(2αΓ (так как при гипотезе Я0 событие (ft" ;ζ α) достоверно). Для мощности этого критерия имеем, очевидно, оценку W(XL· Я,) > P(ft" > α|/Λ) = 1 - Р"(|*,| < а|Я,) = = ,-[,-2ф(-^)]". Следовательно, при любом значении α вероятность ошибки второго рода β удовлетворяет неравенству Фихтенгольц Г. М. Курс дифференциального и интегрального исчисления. М., 1960. Т. 3., с. 392. 212
Ρ<[ΐ-2Φ(-^]-. Если η->- οο. то на основании центральной предельной теоремы ЦРР\Н0)~ Χ(ημ, nb2), где — а Ь» = D(XhHo) = ± \ x*dx - μ> = Λ-α\ — а Следовательно, для определения 1п можно использопать приближенное ^(( па2 \ L / W \ равенство α~ΦΙ /, ч-J/ V I, откуда 2а' Гп '——+"- 3 V 5- 3.46. Обозначим через Тп число положительных исходов в η испытаниях; тогда Lp{Tn)= Bi(n, ρ) и при гипотезе Н0 событие [Т„ > 0] невозможно. Следопательно, в рассматриваемом случае критерий естественно задать в виде Х{ = [Т„ > 0], т. е. при наблюдении Т„ = 0 принимаем гипотезу Но, а в остальных случаях — гипотезу Н\. Тогда вероятности ошибок соответственно равны α = ψ(Τ„ > 0|Wo) = 0, β = Ρ(Τη = 01 Я,) = 0,99". Для определения п имеем следующее условие: 0,99" ;ζ 0,01, откуда η > 454. 3.47. Здесь статистика отношения правдоподобия приводится к виду 1(Х) = ехр{-^(0, - в0)Х - £j(ei - Θ3)}. поэтому, если 0| > θ0, то критическое множество Jff„ имеет вид Xfa = (* > О» ^uj, а мощность равна Κ7(θ,) = Р0Д ^ θ0 - -^-иа) = фГ^ЧО, - Оо)+ иЛ ·> а. В случае 00 > θ| Xt* = {« < Οο + -^u«|, Ι^(θ,) = φΓ^4θο - θ,) + ιΑ л/я \ σ ' 3.48. Для определения η имеем два уравнения ф(иа) =о, φ^^^θ, - О») + u„] = 1 - β, откуда находим, что η* — минимальное целое число, не меньшее чем а2{иа + «,07(8, - θ„)2. 213
3.49. Поскольку L(T\H„) = N(0, 1), L(T\Ht) = N(A/a, 1), речь идет о различении двух простых гипотез о среднем нормального распределения с единичной дисперсией по одному наблюдению над случайной величиной Т. Из решения задачи 3.47 следует, что искомый критерий имеет вид Jfia = (7" > — иа], при этом β = Ф(— иа — А/а). Отсюда, учитывая, что иа = Ф~'(а), получаем уравнение для определения т: — иа — Δ/σ = Φ"'(β) = ыр или σ2 = Δ2(ϋα + "ρ)-2. Разрешая последнее равенство относительно m, окончательно находим, что т* — минимальное целое число, ие меньшее чем («. + 4)i\^ - 4к + и л L аг θ2 J 3.50. Пусть Я,: θ = θ„ i = 1,2, и θ0 > 0|. Если Χ = (Χ Χ„) — соответствующая выборка, то статистика отношения правдоподобия и при этом Ζ.θ(7"/Θ2) = χ2(η). В данном случае неравенство /> с эквивалентно неравенству Τ ^ t, поэтому при уровне значимости α критическая граница t = ta определяется из условия α = ΡοοίΤ" < Q = Ρο^Τ/βΙ *ξ U/ΘΙ) = Fn(ta/el), где F„(t) — функция распределения закона χ2(η). Отсюда ία/θο = χ2.η и искомый критерий имеет вид Χΐα = [Г < 02οχ2. „). Его мощность У(в,) = Ρ0ι(7- < θ2χ2,„) = Ρο,(7-/θ2 < (γ-)2χΙ«) = -'■((*)*■)· Аналогично находим, что при 00 < θ| критерий имеет вид Χΐ. = [Т^ θ§χ?^α.„) и его мощность 3.51. 1) Утверждение следует из соотношений о(е) = ^ h(x)dx < -7 5 '·(*)<** = Т<1 - Р(с"' РМ = \ i\{*)dx < с ξ l0(x)dx = с(\ - а(с)), Хо(с) = Х,(с). Хо(с) Х„(с) 2) Пусть с > 1, тогда, используя первое из предыдущих соотношений, имеем 214
о(с) + Р(с)<|{1 - p(c)) + р(с)< 1. При с <. 1 используем второе неравенство: о(с) + р(с) < о(с) + с(1 - о(с)) < 1. 3) Пусть с >■ 1. Тогда исходим из соотношения о(с) + β(£) = \ Ux)dx + \ ft(x)dx = 1 - \ (/,(*) - /0(Jc))rfJC. Очевидно, Jfi(c) s Jfi(l) и на множестве Х{(\) функция f{(x) — /о(*) S= Js 0. Следовательно, S (/.М-/оМ)Ле< S (fM-f0(x))dx, ΧΜ *i(D откуда о(с) + β(0 > о(1) + РО)· При с < 1 рассуждаем аналогично, исходя из соотношения о(с) + р(с) =1-5 (/оМ - f>(x))dx. Х0(с) 4) Если верна гипотеза Я0, то на основании закона больших чисел Т„(Х) -*■ Ε ( In Лу\ Wo) = δ, когда η -*■ оо. Поэтому при δ < 0 о„ = Р(Тп(Х) 5г 0| Wo) < Pd^iA-) - δ| ^ ]б| )//о) —*- 0. если л-* оо, На основании симметрии (если поменять ролями Я0 и Wi) также и β„->-0. 3.52. В данном случае Ux) = -whm ехрЬ ус- ^-'(- 4 ■ '■ = °· '· и область Jfi(c) = [χ: ii(x)/fo(x) 5= с] имеет вид Х,(с) = {*: а'х - -^'(μ<°> + μ'") < с, = - Inc], а = Д-'(ц(0>- μ(ι>). Для случайной величины У = а'| —ί'(μ^ + μ01) имеем £.(У|//,-) = *((- 1);|-. р), где ρ = (μ<°> - μ">)'Λ-'(μί0> - μ(ι>) — так называемое расстояние Махаланобиса между распределениями Ν(μ(0\ А) и Λ^μ(ι), А). Отсюда находим вероятности ошибок первого и оторого рода критерия Xi(c): о(с) = P(Y < с,|Я„) = ф( ci —p/2_y р(с)= Р(У> с,|Я,) = ф(- C'+P/2Y \ Ур ' Если задана вероятность ошибки первого рода а, то соответствующий критерий Неймана — Пирсона задается критической областью Xi(c) при с, = р/2 -)- Vpua. и« = Ф_|(о); при этом вероятность ошибки второго рода равна β = Ф(— -Jp — иа). Критерием, минимизирующим сумму вероятностей ошибок, является Х|(1) (см. предыдущую задачу) и эта сумма равна 2Ф(— Vp/2). 215
3.53. Из решения задачи 3.39 следует, что рассматриваемая модель обладает монотонным отношением праводоподобия, поэтому полученный в задаче 3.39 критерий Неймана — Пирсона является одновременно р. м. м. критерием данной задачи. 3.55. Поскольку L0(T,) = B,(r, 0) — распределение экспоненциального типа с монотонно возрастающей функцией Α(β) = Ιηθ, р. и. м. критерий в рассматриваемой задаче существует и задается критической областью вида [Т, > /] (см. п. 5 гл. 3). По центральной предельной теореме Lt,(Τ,) ~ Ν[ — —, -γ -Tj-l при /·->- оо, поэтому для расчета критической границы / = ta при больших г можно воспользо] соотношениями ■-·*■■>« = ·( (-Аг-«■)/■>&). откуда следует указанная формула для ta. 3.57. В данном случае Р„ (Г = χ) = Ι(χ; Θ) = СЪСТЛ/СХ χ = 0, 1 Θ, поэтому функция /(*;θ+1) е+1 Ν—Θ—Π + Χ Цх) = Ι(χ; θ) Λ/-Θ Θ+1-* монотонно возрастает по х. Следовательно, в данной задаче р. н. м. критерий существует и имеет вид [7" !> /], т. е. гипотеза Я0 отвергается, когда Τ чрезмерно велико. 3.60. Рассмотрим класс критериев вида Х,а = {-^-{х - θο) < </„,} U {^-Ос - Оо) > - ««}. где ос, + а2 = а, и рассмотрим также функцию мощности Ц7(Х,0; Θ) = Ф(лМ/о +иа1) + Ф(- лМ/σ + "<*,)■ Δ = θ - θο. Легко убедиться в том, что мощность минимальна при Δ = Δ0 = = a(uai — иа2)/(2л/«). Так как W(X^; θο) = α (т.е. при Δ = 0), το несмещенность имеет место лишь при Δ0 = 0, т. е. при а, = аг = а/2. Таким образом, искомый критерий имеет вид 4п х,а \^-\х- θ„| > - u„/2J. Этот критерий является р. н. м. критерием среди всех несмещенных критериев. 3.61. Здесь функция правдоподобия ч*·θ> = -ns4re~rw/20'· Т^ = 2 (*·■ - ι1'2·и Lt(x, θ) - —δΓ- -[-ψ-- Т)Ц*. θ). Поэтому неравенство Цх; θι) ^ сЦх; θο) + CiLi(x; θο), определяющее наилучшую критическую область, имеет в данном случае вид (см. решение задачи 3.50) „"Г > с' + с', Τ или (7" < /,] U [Т > I 216
Итак, искомый критерий имеет вид *ι = [Т < /.) \]{Т> /2), где границы /ι < /2 определяются при заданном уровне значимости α двумя условиями: W(Q0) = α, W'(Q0) = 0, где W(0) — функция мощности. Но (см. решение задачи 3.50) Щв) = ЫХ е-*,) = Р0(7· < /,) + Р0(7" ^ /2) = f„(/,/02) + 1 - - ^(/2/θ2), откуда имеем два уравнения ^(ί./θ2) + 1 - F„(t2/Ql) = α, /,Λ„(/,/θ20) = /2ί;η(/2/θ2ο), где k„(t) — F'„{t). Полагая /ι = 9oxi,n, /2 = θοχ2_αι. л, получим, что должны выполняться условия Xai, лМхо,. л) = χ2-<χ,. ηΛη(χ2_αι, η), αι + α2 = α. Этими условиями χα,, η и χ2.-<*,, п. а тем самым ί, и (2, однозначно определяются. Следовательно, искомый критерий имеет вид Х>. = [Г < Qlxl,. n)U\T> θ2οχ2_αι, η] и представляет собой объединение двух односторонних р. н. м. критериев задачи 3.59. Значения (χ?,,, „, χί_αι η) для α = 0,05 и η = 2, 5, 10, 20 см. в [1, с. 86]. 3.62. Схемы решения данной и предыдущей задачи аналогичны. Используя решение задачи 3.43 и пведенные там обозначения, найдем, что критерий имеет вид Х\ = [Т <: /,) U (7" ^ /2) и его функция мощности рапна Щ&) = /=·2„(2ί,/θ) + 1 - F2„(2t2/Q). Для определения границ ίι и (2 имеем два уравнения: Ψ(θ0) = а, 47'(θ(ΐ) = 0, которые при замене t\ = -£-%%,,2п, (i = -кЧ(1 -α,, 2л принимают вид ΧΙ,. 2π*2-.(χ2α,. 2.0 = Xi-cij^iifenixi-aj^n), «I + «2 = <*· Определив отсюда χ^,, 2(ι и χ2.-αϊ, 2η, получим, что при уровне значимости α искомый критерий имеет вид Х\с = J Σ Χι < -γ-χΙ,.2η Либо Σ Χ, S= -γχ\-α,,2η\ и представляет собой объединение двух соответствующих односторонних р. н. м. критериев для альтернатив /УГ '· θ < θο и Hf : 0 ~> θο, которые следует из задачи 3.43 и свойств экспоненциальной модели. 3.63. В данном случае (см. решение задачи 3.39) функция правдоподобия Цх;9) = fLCfHl - θ)*-·1·, поэтому /= I _ dln£(x; θ) _ T\x)-knO , . _ А "(Χ·θ) - —аё— - ο(ΐ —θ) · τ{χ) = J, ;· Кроме того (см. задачу 2.43), ί(θ) = k/[Q(\ — θ)], поэтому искомый критерий (см. указание) имеет вид Χι* = (17" - *rtOol/V*«Qo(l - θο) > ~ ua/2). 217
Мощность этого критерия при указанных альтернативах вычисляется так же, как и в задаче 3.40. 3,G4. В данном случае Цх\ 0) = «-""О7"'/^,! ...*„!), 71»= Σ *i. /= I поэтому U(x; Θ) = (Т(х) — η0)/θ, а ί(θ) = 1/0 (см. задачу 2.43). Отсюда искомый критерий имеет вид Х>* = (17" — л001л/«Оо 5= — 1и/2\. Его мощность вычисляется так же, как и в задаче 3.41. 3.65. Используя обозначения, введенные при решении задачи 2.119, находим, что критическая область Χια для сформулированных задач имеет следующий вид: 1) Хы = [х : χ > θ,ο + /[--.„-iSfrWrt - 1): 2) Jfia = [χ : χ ^ θιο — /ι_α, η- iS(x)/^jn — 1) (ср. с решениями для случая известной дисперсии — задачи 3.47 и 3.58); 3) Jfia = (χ : V" — 1 с S= ',_.« _,) (ср. с результатом за- дач 3.60 и 3.73); 4) Х,а = [х : nS2(x) > θ!„χ?-„. „-ι);' 5) Jfi0 = [χ : nS2(x) ^ θ2οχ!η-ι) (ср. с результатом задач 3.50 и 3.59); 6) Хы = [х: nS2(x) sg в2ш\ либо nS2(x) ^ θ™χ' __α _.j (ср. с 2 ■ " 2 ■ " результатом задач 3.61 и 3.74). Действительно, рассмотрим, например, первую задачу (для остальных задач рассуждения аналогичны). Используя для 6. нижний γ-до- верительный интервал G4(X) = (θι : Χ — (-,.„- [S(X)/^fn — 1 < θι < οο), получаем, что область принятие гипотезы Я0 с уровнем значимости α = = 1_— γ имеет вид Х0* = [х : χ — iv.„_|S(x)/V« — 1 < θιο). Но Xla. = = Jfo« имеет вид, указанный в ответе задачи. 3.68. Используя γ-доверительный интервал для отношения τ, построенный в задаче 2.127, находим, что область принятия гипотезы Н0 имеет в данном случае вид Х„. = l(x.y): f-F^2t:2m < 1 < fr^^J. a = 1 - γ. Отсюда искомый критерий задается критической областью X,, = ((,,,,) :^F либо 4->/>,_,. 2jiJJ. у г.сп,-т у !'"'" и его мощность П7(в)=Р„(х-|-<х^211!.1)+Рв(хА^^1_.1я2и) = = ^.2„,2„,·. 2". 2"J) + ' - Ър.-1.,,.^: 2"· 2"0· τ = 17· 3.69, Обращая построенный в задаче 2.128 γ-довернтельный интервал, находим критическое множество для гипотезы Но- Xiv = Ιχ-.Χ(ΐ) < θο либо x(i) ^ Οο — (In α)/η). Так как Ρ/* -> П- V ПРИ ' <θ' 218
(см. решение задачи 2.128), то функция мощности этого критерия равна 47(0) = Ρο№ι, < θ„) + Ро(*(1) ^ 0„ - (1па)/л) = f 1. θ Э= θ0 - (1πα)/η = ae^°-°'\ θ0 < θ < 0ο - (1πο)/η, 1 1 - (1 - α)β"(θ-°«), 0 < θο. Отсюда следует, что 47(θ) Js α при всех θ. 3.70. Искомый критерий имеет вид Xta = [χ: Х(г.) < θ0αι/η либо дг(„) 5= 00), а его функция мощности 47(0) = Р, < Ооа'7") + Ρ(№„) Ϊ& θο) = = mi„(-(-f-)M)+.-m.n((A)".1) = ' 1, θ < θοο'/", = of-^-V. θοο'/"< θ < θ». \ ο / / я \ \\-{\ -a)(f-y. 0> θ„. Отсюда следует, что W(Q) > α -V^· 3.71. Утверждение о выборе границ в несмещенном критерии следует из совпадения распределений соответствующих статистик. 3.72. Обращая построенную в задаче 2.132 доверительную область, находим, что критерий имеет вид Х,а = {х\п{Ху - θ|0, Х2 - θ2θ)'Σ_'(*Ι - θιο, Χ2 - θ20) > XI-α, 2). 3.73. Здесь θ = (0 = (θ,, θ2): —οο <θι < οο, 02 > 0) и (см. решение задачи 2.86) supL(x; θ) = Цх; (χ. s)) = (2nes2)-"/2. s2 = S2(x). Далее, Θ0 = (θ = (0,. θ2): θ, = θ|0, θ2 > 0) и supL(*; θ) = L(x; (0I0, So)) = (2nesl)-"/2, θ« где so = — 2j (* — Οίο)2 — ο. μ. п. для 02 при гипотезе На. Так как sl = _п , = ι = s2 + (х - θ,ο)2, то К = λ„(χ) = (sl/s2)-"'2 = (1 + t2/(n - 1))-"/2, где / = ί(χ) = л]п — \{х — 0|o)/s. Поэтому неравенство 1,< с эквивалентно неравенству |/| Js С. Таким образом, в данном случае к. о. п. имеет вид Χία = {χ: V« - 1|I - θ,οΙ/s > с'). Так как статистика критерия 1(Х) имеет при гипотезе Но распределение Стыодента S(n — 1), то граница с' = t._a_ _. (ср, с задачей 3.65 п, 3), 2'" Распределение S(n — 1) при больших л аппроксимируется распределением N{Q, 1) (см. задачу 1.47), поэтому критическую границу с' можно полагать приближенно равной — ка/2. Отметим также, что и\/2 = χ2_α.ι. Далее, информационная матрица /(Θ) модели Ν(0ι, Θ2) 219
вычислена в задаче 2.44, откуда имеем, что первый главный минор матрицы /~'(θ) есть 02. Согласно асимптотической теории о. м. п., предел мощности при рассматриваемых альтернативах равен 1 — Ρ,{χ2 _„. ,·, λ2), где в данном случае λ2 = β2/02. 3.74. В данном случае (см. решение задачи 3.73) зирЦх; 0) = Цх; (i, s)) = {2nes2)""\ s2 = S2(x), supL(x; 0) = L(x; (x, 020)) = (2π0220) θο откуда λ„(χ) = (/е-'+')"/2, / = s2/9^„. Здесь неравенство λ„ ;ζ с, определяющее критическую область к. о. п., записывается в виде (/ ^ Л] U С > 'а|. Ί < '2, поэтому критерий имеет вид Х\ = [х: S\x)/Q220 < /1 либо S2(x)/9^o > /2). Из указания следует, что функция мощности такого критерия равна U7(0)=F„_,(^,) + l-f._,(^). Отсюда, выбирая Л = χ2α, „_ ι/л, /2 = χί _0,.я- ι/я при α, + α2 = α, получаем, что ΒΡ(θο) = α, Тем самым получаем указанный а формулировке задачи результат (ср. с задачей 3.65 п. 6)). Чтобы получить несмещенный критерий, надо обеспечить пыполне- ние условия 1#"(θ0) = 0, которое сводится к уравнению χοι,η-Ι^ι-ΐ(χο,.η-ΐ) = Xl-a,,n-lfen-!(xi-<x,. п-0· [Замечание. Асимптотический (при η -*■ оо) вариант рассмат- 1 риваемого критерия исследован а [I, с. 175—1761, 3.75. Поскольку оценка максимального правдоподобия параметра θ о модели βι'(Ι,θ) по пыборке объема η равна ?„ = X (см. задачи 2.84 и 2.48), статистика Оо"*(1 - θο)""-*1 Х"\\ — Л)""-* Но в данном случае имеет место полиномиальная модель с N = 2 исходами, поэтому (см. [1, с. 170—171]) при η -*■ оо предельные распределения при гипотезе Но статистик — 2!πλ„ и 2 (Г - ηθ0)2 (η-τ--η(ΐ-θο))2 _ (Г - nOp)2 - ηθ0 + л(1 - θ») ηθ0(1 - θ»)' ' совпадают и есть χ2(1). Это отражает тот факт, что L0,(T) ~ Ν(ηθ0, ηθ0(1 — θ0)) при η -*■ оо. Таким образом, к.о.п. (λ„ ^ с] асимптотически эквивалентен критерию [X2, > /], который совпадает с критерием, построенным в задаче 3.63 (при k = \). 3.76. В данном случае о.м.п. 0„ = X = Т/п (см. задачу 2.109), поэтому статистика λ„ = (пвц/Т)тет~" °. Далее, из решения задачи 3.64 следует, что статистика Qi," = (7" — ηθ0)2/ηθ0, следовательно, асимптотически к.о.п. эквивалентен критерию [\Т — ηθοΙ/VnOo ~> I), исследованному в задаче 3.64. 3.77. Обозначим через Δ;(θ;) = 0"л(\ — θ,)"'"-*'1 функцию правдоподобия для /-й выборки, /= 1 k. Тогда, в силу независимости 220
выборок, функция правдоподобия для всех данных есть L (Οι, ..., Gt) = = Αι(θι) ... Lt(0t)- Далее, так как о.м.п. параметра бернуллиевской модели совпадает со средним арифметическим выборки, то отсюда имеем * * max L(Q θ4) = Дтах Δ, (θ,) = ПЭД О, 0. j=l "ι j=| к <=' max Δ (Οι θ„) = max Δ (0 0) = maxQ"1;i - Ο)"'1-·*1 = θ, = ... = ο, ο ο = Г\\ - Λ)""-* где ~~Χ= —(η,Χι + ... + пьХ/,), η = η\ + ··· + "*· л Таким образом, в данном случае статистика отношения правдоподобия имеет вид _ _ к К...«. = х"\\ - Λ)"(|-*/πλΓλ(ι - Xi)"'(,-Xl) = /-ι д Ф'Шг ,<ι-*,) Наконец, размерность нулевой гипотезы dim Я0 = 1 (одна степень свободы), поэтому асимптотически к.о.п. [1, с. 175] имеет вид ΑΊα = [ 22 η, [х, (!п х, - 1п л:) + (1 - χ,) (In (1 - Xj) - — In (1 — x))]> ϊί-м-,]. Стандартный же критерий однородности χ2 [1, с. 126] имеет вид 3.78. Схема решения такая же, как в предыдущей задаче. Используя те же обозначения и учитывая, что в данном случае A/(e/) = e-"Ao;<yn;fJ,i. ' = ' *· найдем, что '"' и при «ι, .... пк -*■ <х> к.о.п. имеет вид (к 22 η,*; (In *j - In ι) > χ'_0,ι,-ι| . 3.79. Обозначим через ί/(0ι(, Ο2) функцию правдоподобия для у'-й t выборки, j= 1 k, и через Δ (0 Οι*, 02) = ΠΔ;(0ΐ], 02) — для 1-1 221
всех данных. Кроме того, положим л = п\ + ··· + nt, So = — 2 ηι$1· " / = ι 1 ' - ,¥ =— Σ "Λ· пусть S2 — выборочная дисперсия для всех данных. Тогда _(см. задачу 2.11J) о.м.п. параметров (0| , Оц, 02) являются (А",, ..., Xj,, So) и Δ(ΑΊ Ai, So) = (2neS20)-"^. При гипотезе На все данные можно рассматривать как выборку объема л из совокупности Ν(0\,θ\) н поэтому (см. задачу 2.86) max L (0 О,, θ2) = L (X, ..., К S) = (2n<?S2)-"/2. О,. Οι Таким образом, статистика отношения правдоподобия в данном случае равна Число параметров, определяющих модель, равно k + 1, а размерность нулевой гипотезы равна 2, поэтому асимптотическая форма «.о.п. имеет вид Xla = (n(lnS2 - lnS2)> χ2_αι4_,!. Если k = 2, то непосредственно проверяется, что nS2 = n,S? + n2S! + ~- (Χι - &Ϋ. откуда λ».„ = (1 + 7-2/(η- 2))--/2, где 7" задано в формулировке задачи. Здесь неравенство λ„,„, < с эквивалентно неравенству | 7"| > /, откуда и следует указанный в формулировке вид критерия. 3.80. Пусть £;(0|;, 02у) — функция правдоподобия для /-й выборки, к У = 1 A. a i = Π £,(θ|/, 02у) — для всех данных. Тогда, как и в пре- 1= ι дыдущей задаче, имеем, что для общей модели s s max L= f[max /.,(0,,, 02/) = Ц (2ncS2) _,,'/2, /-ι /=' а при гипотезе Я0 max L = maxflMGi/. <Ь) = (2ikS2o)-"/2. /-ι Отсюда *■«,..».= ns/"'/53= n(W'· ι-1 /'-ι В данном случае число параметров, определяющих модель, равно 2k, а размерность нулевой гипотезы равна /г + 1, поэтому асимптотически при «ι, ..., /и ->- оо к.о.п. имеет вид 222
ik Σ η, (}n Si - InS?) Ss χ?_..*_,). /=ι ' При k = 2 имеем λ"'"' U,Sf +n2S2J 'U.Sf + «2S2J - = -^ (JLL^> )"·^../V, + i^l Л-" n, n2" \n2 — \ J \ n2 — 1 У и неравенство λ„,ηι ;ζ с эквивалентно условию [F ιζ. с ι] U [F > ί2], C| < c2. Отсюда, используя указание, получаем приведенный в формулировке задачи вид критерия. 3.81. При гипотезе Н0 все Χι имеют некоторое распределение Λ^θι,θ*). Ηο_ в этом случае (см. решение задачи 1.58) статистика Х{ - X η = /■ _ ι ς имеет распределение, не зависящее от параметров (θ|, θ2), симметричное относительно нуля, и при этом Ρ(η> ») = lfl(i - о»; JLiii, i_), o<v< ι. Следовательно, P(^l.lffo) = fl(l - ι& -^^-, -j) = a, т. е. вероятность ошибочно отвергнуть гипотезу Но при |η[ ^> να равна а. Таким образом, Х\а — подходящая критическая область. 3.82. Так как гипотеза Я0 эквивалентна в данном случае утверждению cov (ii, h) = 0, то согласно задаче 1.59 п. в) L (p. V(n-2)/(l - р»)|//„) = S (« - 2), откуда можем записать (в силу симметрии распределения Стьюдента), что Р(|р„| V(" -2)/(1 -pS) > /,_о/2.„_2|Яо) = а. Но последнее неравенство эквивалентно неравенству, определяющему область Χι», следовательно, Ρ(ΧιΛ\ΙΙ0) = а. Это означает, что вероятность ошибочно отвергнуть гипотезу Я0 при рпе.Х\а равна а, т. е. X,,— искомый критерий. 3.83. Согласно задаче 1.60, L (7"„|#о) -*■ N(0, 1) при п->-оо, поэтому Ρ(Ι7ΊιΙ 5= — "а/г! W0)->-2Ф («α/2) = α, что и утверждается. 3.84. Пусть наблюдаемые случайные величины Χι,...,Χη независимы, имеют одинаковое и известное среднее μ и конечные дисперсии. Если гипотеза Но означает, что все Χι одинаково распределены, то при больших η статистика Т„ имеет при Я0 приближенно нормальное N(0, 1) распределение. Следовательно, статистика 7"„ позволяет построить критерий согласия для Но, который имеет такой же вид, как и в задаче 3.83. 3.85. 1) Так как W„(60) = Ро„(7-„ > γ») = Po.(V" (7\,-μ(θο))/σ(00) ^ -и.). то согласно свойству (a) W„(00) ->- Φ (u„) = α при η ->■ οο, т. е. критерий ΧΙα асимптотически имеет уровень значимости α. 2) Аналогично имеем Ц/„(0<")) = р0„<7\, > γ.) = Р0>»{ЛЪ(Т„ - И(0м))/а(0м > z£"). 223
где - 2о (я) = V" (μ (0'">) - μ (0,))/σ (θ<">) + κ„σ (0„)/σ (0<">). Согласно свойству (б) — ζα (η) ->- βμ'(00)/σ (θ0) + иа при п ->■ со. Из этих соотношений в силу свойства (а) получаем указанное предельное равенство. 3.86. Из предыдущего доказательства имеем Φ(βν?ί + ".)= I'm \Х*»(в0 +-?—) = β = ι^^!(θ0+-|^) = Φ(βν^Α + .α). Отсюда φ'ι = -Jei/λ или λ = εί/εΊ. 3.87. В рассматриваемом случае ί.0(7ΐι'') = Ν_(θ, σ2/η), поэтому (см. задачи 3.86 и 3.85) критерий имеет вид X\i = (λ > G0 — u0o/V«), его эффективность Питмена ei (β, α) = Φ (β/σ -f- и„) и ei = σ-2. Аналогично (см. задачу 1.32) имеем Lu(Tii)) ~ ν(θ, -^—V откуда ХЙ = 2 = j-. Следовательно, λ = й/е1 = 2/л. 3.88. Запишем функцию правдоподобия L (х; 0) в виде /.(*; 0) = ((2лП21Г,/2ехр{-1(зЦ, где квадратичная форма Q0(x) = (χ — 0/)'2_'(* — Щ может быть разложена в сумму Qt(x) = Q.(x) - 20/'J-'* + θ2(?ο(/). Замечая, что t — последний столбец (а значит, и последняя строка) матрицы 2. т· е· (0...01) 2 = ί', находим /'2"' = (0...01). Отсюда f 2-'* = х" и, таким образом, <2о(*)= 0о(х) + θ2/„ - 2вхп. Это означает, что L (χ; Θ) = g (7" (χ); 0) /г(х), где Τ (χ) = х„, a g (7"; θ) = = exp! 07" г- 02/„[ . Отсюда согласно критерию факторизации следует, что Х„ — достаточная статистика; и тем самым все статистические задачи в рассматриваемой модели надо решать, основываясь на этой статистике. Но L0(Xn) — N(Ql„, t„), следовательно, имеет место нормальное распределение, у которого неизвестно среднее. Таким образом, сформулированная задача эквивалентна задаче проверки гипотез о неизвестном среднем одномерного нормального распределения по одному наблюдению, которая решена в задачах 3.47, 3.58 и 3.60. 224
Глава 4 4.1. Имеем О (0,ft)\2 \ -7Σ^Λ = \ς™ Σ 4,2Σ ζ^ - Σ «Ρζί'Σ &Κ Σ 2ίο2Σ 2^ _(Σ 2Ϊ)2ί1)ί V Σ ζΨ Σ № - Σ ^'Σ 4.2. Если не все /, одинаковы, то Ρ, = χ - /Ъ. р2 = Σ ('.- - 0 № - *)/ Σ (ί - <f- ι = ι ; = ι При этом £p, = β,-, i = 1, 2, DP, = — Σ ll Σ U> ~ i)\ Οβ2 = σ2/ Σ (/, - If. П i = I i = I i = I поэтому, если 2 (Ί'—/)2->-°ο при л->-оо, то оценки β,· состоятельны, i = ι η 4.3. Имеем с2 = S(p)/(n - 2), где S(P) = £ (Я, - *)2- — Р? 2 (Ί'~ Ο2. Ρ = (βι. ί^) указаны в предыдущей задаче. Если . = ι DS($) = σ(η2) при п->-оо, то σ2— состоятельная оценка σ2. В частности, для нормальной модели L(S($)/a2) = χ2(η — 2) ([1], с. 193), поэтому DS(P) = 2σ\η - 2) = о(л2). П 4.4. Имееи cov (β,, β2) = - сЧ/ 2 С/ — <f- i = ι 3 3 4.6. Здесь /= 2 β) {Ь ~ a)l , Τ= 2 Ρ/ (& ~ α)' , поэтому Cf= / = ι / = ι ι.ογ= 2 (b ".■?''+ ' c°v(p„ pr). /.' = 1 У 4.9. Доверительные интервалы находятся соответственно по формулам: ( ί±'ц*, „_2vs<0 Σ/■'/["<"-2'.Σ (ί-ό2]) = ι i = I (^'ш... 2V ^){(я - 2). Σ ft - ό2]) 8—190 225
γ-доверительный эллипс имеет вид Gy(X) = ίβ:(β, - β,)2 + 2/"(р, -β,) (β2 - β2) + — Σ 'Kb - β2 ^ ^ ; _ ι ^ 2 F .-.}. где Ρι, β2 и S(P) указаны в задачах 4.2 и 4.3. 4.10. Поскольку \ x(l)dt = 2αβι, речь фактически идет о довери- — а тельном интервале для βι, указанном в предыдущей задаче. 4.11. Здесь теоретическая зависимость имеет вид α(ί) — α(0) -f- ut, поэтому доверительный эллипс строится по правилу, указанному в задаче 4.9. 4.14. Положив в (4.11) m = 3, λ, = (100), λ2 = (010), λ* = (001), получим, что искомая система имеет вид {(Ь±У-^^та"8№у-°*-?) . / = ι. 2. з). 4.15. Обозначив через Х\, ... , Хг результаты указанных последо вательных измерений, получим, что в данном случае речь идет о следующей модели нормальной регрессии: ΛΊ = βι + ε,, Х2 = β2 + ε2, Χι = βι + β2 + ε3, Χ* = 180 - β2 - β3 + ε4, Xs = Рз + е5, А-6 = β4 + eG, Χ, = β3 + β4 + ε7, Χ> = 180 - β, - β4 + ε8. 4.17. Учитывая указание, имеем £S(p) = £S(p) - £(β - ρ)Μ(ρ - β). П Здесь £S(P) = £ 2 ε? = η с2 (см. (4.4)) и i = I £(β - р)'Л(р - Р) = 2 a</c°v (&· &/) = °2 Σ α,,-α" = o2tr (AA~ ') = i./ =1 i.j = 1 = Λσ2. В результате получаем ES($) = (η — /г)о2. Далее, так как X — Ζ'β = ВХ (см. (4.5)) и В2 = В. то S(p) = Я'В2* = Х'ВХ. 4.18. В данном случае матрица А = ZZ' является диагональной, ее диагональные элементы равны zjzy, / = 1, ... , ft, где ζ, — /-й столбец матрицы Ζ', поэтому β, = г\Х/г\ги Dfi; = σ2/ζ/Ζ/, cov ($,-, β,) = 0, j φ г. 4.21. Функция правдоподобия для модели (4.3) имеет (см. (4.4)) вид ЦХ] θ) = (2πσ2)^ 6ХР { ~ i" S(X: P)} ' θ = (Ρ· °2)· и максимизация ее по β эквивалентна минимизации квадратичной формы S(x; ρ); отсюда следует, что о.м.п. р совпадает с о.н.к. О.м.п. σ2 — это то значение σ2, которое минимизирует S(p)/a2-j- nln σ2, откуда σ2 = S(P)/rt. Учитывая задачу 4.17, имеем а2 2 / П — k \ , ft 2 Ее — с = [ 1 Ι σ = с. \ η J η 226
4.22. Решение следует из (4.10) при т=\, Τ = λ'; при этом необходимо учесть, что Ру,\л _* = tf\ +Ί)/ι.η -* (см. задачу 1.50). 4.23. Полагая в предыдущей задаче λ = (1, /) и используя результаты задач 4.2—4.4, получим искомый интервал: ( X + (I - /~)β2 ± ± /(, + т)/2.„_г-у п(п'__2) s®[\ + п{1 - if/Σ С-<)2])■ 4.25. Положить в (4.1 1) λ, = zw, ι = 1, ... , η. 4.27. В данном случае в (4.12) m = 1 и η η sT = min 2 № - β?»'* - βΟ2 = s(p) + ($2 - β20)2 Σ ('<■ - ή2; Ρ' ί = Ι ( = Ι кроме того, F\ _ „, ι, „ _ 2 = /2 „_ _ (см. решение задачи 4.22). Отсюда следует, что F- критерий (4.12) имеет указанный вид. 4.29. Здесь мы имеем модель нормальной регрессии с η = 8, β = = (μι, μ2, μ3, μ<) и соответствующей квадратичной формой Щ = Σ(*ί" - μ<)2 = Σ(*5° - *(")2 + 2Σ (*"> - μ.)2. Отсюда ο.η.κ. μ, = F>, ι = 1 4, min S(P) = Σ (*i° - *<0)2 = S, и '. / α"2 = S,/4. При гипотезе W0 Sr = min Σ (*/'> - μ)2 = min ( Σ (·*5° - *f + n(X - μ)21 = ν- i, ι μ v /. / J = Σ (W ~ X)2 = 5. + 2 Σ (*" - *)2. поэтому /■'•критерий (4.12) при m = 3 имеет вид *. = {Σ (*С'> - Я*/* > у F, - οΛ<) · 4.37. Обозначив Ζ' = ||1α|| матрицу размера η Χ 2, составленную нз вектор-столбцов 1 = (1 1) и α = (oti α„), получим решение в виде β = A-]ZG~lY, где А = ZG~lZ' = II ^', II С-'||1а||. Применяя далее обычные правила· умножения блочных матриц («строку на столбец», если матрицы-блоки рассматривать как элементы), результат можно привести к виду β = (βι, β2) = -г- {a'QY, —\'QY), где A = (l'G-'l)(o'G-'o)-(l'G-'o)2, а матрица Q=G_'(ol'- — lo')G-'. Матрица вторых моментов этих оценок имеет вид D(P) = PU-' = % o'G"'o -o'G-'l -I'G'o I'G-'I 227
Глава 5 5.1. 1) В данном случае выборочное пространство X = (О, 11 состоит из двух точек и для каждого χ ζ X возможны лишь два решения, поэтому всего имеется четыре решающие функции 6, = (6,(0), 6,(1)), i = I, 2, 3, 4:δ, = (d,, d,), 62 = (d,, ώ2), δ3 = (d2, d,), δ, = (d2, d2). Пусть Ri = (R(0\, δ,), /?(02, δ,)) — вектор риска процедуры δ,, где Λ(θ, δ,) = Δ(θ, 6,(0)) (1 - 0) + Цв, 6,(1 ))0. Тогда для рассматриваемой ситуации Л,«=(0.2).Л=(1.4).Л=(-|.-1).Л=(1.0). Процедура 62 предпочтительнее 6з, а процедуры δι, δ2, δ4 между собой несравнимы и, следовательно, образуют совокупность допустимых решающих правил; при этом т(62) < ш(64) < т(6|), τ. е. δ2 — минимаксная процедура. 2) Решение 1. Для байесонских риской г(6,) = aR(0\, δ,) -f- -f (1 — α)/?(02, δ,) имеем следующие значения: /-(δι) =2(1 —α), r(62) = 2 — α 1 = —τ—, г(б4) = α. Отсюда находим, что если α < -^-, то min r(6,) = 1 4 = г(64), т. е. здесь δ* = 64; если -^- < α ^ — , то min r(6,) = г(62), т. е. здесь 6* = δ2; наконец, δ* = δι при α > · Следовательно, график байесовского риска имеет вид, изображенный на рис. 8. Решение 2. Вычислим апостериорное распределение π(θ,|-κ) = f(x; θ,)π(Ο,)//(*), x = 0, 1,1 = 1,2, где в данном случае f(x; θ) = θ*(1 — -О)1"', ί(κ)=ί(χ\ 0,) π (θ,)+/(*; θ2)π(θ2) = θί(1 - θ,)1 " 'α + Θ$(1 - -02)'-'(1-α). Имеем π(θι |0) = ο(1 - 0,)//ιΌ). π(02|0) = (1 - α) (1 - -02)//(0), η(θι|1) = ο0ι//(1), π(θ2|1) = (1 ~")θ2 . Для рассматриваемого случая средняя потеря относительно этого апостериорного распределения при χ = 0 для решения δ(0) = d\ равна 2(1-α) Щ(0, d,)|0) = Α(θι, d,)n(G,|0) + L(02, d2)n(62|0) 2α а для решения δ(0) d2 она равна 1 3/(0) ' Сравнивая эти потери, 3/(0) · получаем, что при а ^я- потеРи Для решения d2 меньше, т.е. δ*(0) = d2, если же α > -г-, то δ*(0) = d\. Аналогичный анализ для случая наблюдения χ = 1 дает, что при а^ 4 4 <— 6*(l) = d2, если же α> —, о о то δ*(1) = d\. Тем самым получены аначения байесовского решения Ь*(х) в каждой точке χ — 0, 1 при любом априорном распределении. 5.2. Для априорного распределения π(θι) = α, as [0,1], искомые средние потери Е(Д0, d)\x), вычисленные по указанным формулам, приведены в следующей таблице. 228
d. d2 d3 (l_„y/(0) 3(l-a)//(l) 3a/4/(0) «/4/(1) (l+2a)/8/(0) (3-2a)/8/(l) x=0 x, = 1 Сравннная указанные значения для определения минимального в каждой строке (а тем самым и отыскание значения &*(х)), получаем следую- 1 1 7 щие результаты: 6*(0) = d2 при а <: — , 6*(0) = d3 при — < a < — , 6*(0) = d, при о>-^-: δ*(1)= d} при α^-^-, δ*(1) = d3 при -| < 21 21 <a^-^7j-, δ*(1) = d, при o>-^j-. Тем самым искомые байесовские решения δ* = (δ*(0), δ*(1)) имеют вид: 6* = (d2, d2) при о <-j- ; δ* = 17 7 3 = (d3, d2) при -j- < a < -ту ; 6* = (d,, d2) при -^- < a < -j- ·, δ* = 3 21 21 = (di, d3) при-^-<a <-22"·, 6* = (d|, d|) при a >—2". Далее, так как ι р(о) = г(б*) = Σ /МВДО. e*W)U). Jt = О то, беря соответствующие значения Ε(Α(Θ, 6*(je))|jt) из таблицы, для функции р(ос) получаем следующее представление: / α при 0<а^ 1/4, (1+4о)/8 при 1/4 <о< 7/10, р(а) = (4 — Зо)/4 при 7/10 <а< 3/4 , (11 — 10о)/8 при 3/4 <а< 21/22, 4(1 — о) при 21/22 <а< 1 График этой функции приведен на рис. 9. 5,3. Для приведенных в решении задачи 5.1 решающих функций векторы риска для первой ситуации имеют вид R\» = (0, Ы М" = (!■ а, |-) , М'> = (у . γ) , «V> = (a, 0), а для второй — следующий вид: *(2> = (о, 6), яР = (|а, 4). Я?> = (-J, |) , ЯР = (а, 0). Отсюда заключаем, что в обоих случаях процедура 6з предпочтительнее &2, следовательно, допустимыми являются процедуры 6|, δ3, δ4; при Рис. 9 229
этом для тех значений α = π(0|), при которых соответствующее байесовское решение 6* = &з, соответствующие риски удовлетворяют неравенству лр>(6*)</'>(6*). 5.4. В данном случае функция риска имеет вид з же, δ) = Σ L(Q· &(k)) £'θ*ο - °)3"' * = о и векторы риска Л, = (Λ(θ|, 6i). Л(вг, δ,·)) для указанных процедур соответственно равны Λι = (5,94· КГ2; 0,792), Лг = (5^96-Ю-"; 0.972), Лз = (2-!0 , 0,999). Отсюда минимаксное решение Ε = δι и т(6) = = 0,792. 5.5. Здесь /(χ·; Θ) = (I — 0)0*. χ = 0, !, 2... , поэтому функция риска оо ί — I Λ(θ. δ,) = 2 *-(θ· «■(*№: ο) = цв, d,) Σ /(*: θ) + χ = 0 χ = 0 + Ц0. 40 Σ Я*: θ) = Д9· <Ί)(Ι - ο') + L(o, ί/2)θ''. Λ = ί Вектор риска для ι·ή процедуры ранен R, = (β(θ,, δ,), β(θ2, δ,)) = (αΟΊ, Ь(! - θϊ)). Здесь с ростом ί первая координата убывает, а вторая возрастает, поэтому при а0|^й(1 —9г) максимальный риск т{Ы) = Ь(\ — Щ и, следовательно, минимаксная процедура δ = δι. Если же а0\ >й(! — Од), то, определив целое k условиями ав[° > Ь(! - θί°), αθί0 + ' < й(! - 0? + '), (αθ\, ί<ίο· U(! -θ'ζ), i>i'o будем иметь т(б,) = { "" 0,s ". ^""' Следовательно, в данном случае § = δ,№ если αθΊ° ;ζ Ь(1 — Ог0 + '), и δ = δίο + , в протинном случае. 5.7. В соответствии с общей теорией (см. п. 4 гл. 5) в данном случае функции h\(x) = йп2/2(д:), hi(x) = an\fi(x), поэтому байесонское решение &*(х) имеет вид ... . ( d, при χ ζ WT, где W\= (f; *·Μ < ^М) = 1 d2 при л: (Ε #f, I * 'WT' Ί^ϊ · а соотнетствующий вектор риска равен (aP0f(X ζ Wf), ЬР„1Х £ Wf)). Следовательно, байесовский риск Γ(δ*) = απ, Ρο,(* ζ Щ + 6π, Μ* £ Щ). Для указанных нормальных распределений при 0, < 02 область Wf = = of _ С + Р/2) , Р0Д е WT) = ф( С~г/2) · we ρ = (0, - 02)2/σ2. 4 Vp ' ч Vp ' Если ΘΓ > 02, то область Щ задается протиноположным неравенством, остальные же выражения остаются прежними. 5.9. В данном случае для решающей функции &(х) = d функция риска /?(0, δ) = Ζ.(θ, d) и поэтому байесовский риск r(d) = oZ.(0, d)+(\ —a)L(\, d) = ad" + (\ -o)(! - d)°. 230
Минимизация этого выражения по d дает искомое решение d*: если а = !, то d* = 0 при а > — ; d* = ! при α < —, а при а = — в качестве d* может быть взято любое d ζ [0, !]; если же а> !, то "-('+(т*тГГ· 5.10. !) Записав Р0Д £ Wf) = \/,(*) d* (в абсолютно иепрерыв- ном случае), нз определения функций h,(x) имеем k k r{&*) = Σ лЛ{&*) = Σ \ hMdx. i = ι /' = ι wf Но из определения областей Wf следует, что последнее выражение можно записать в указанном в формулировке виде. 2) Очевидно, что k k ΐ(Σ xJl*) - */«*)) < h,(x) < /( Σ nlM - «ЛМ) (здесь учтено, что l(j\j) = 0, / = !, ... , k). Отсюда k к ί( Σ π'7'Μ — max V;M) < m«n h,(x) <'( Σ π"7ιΜ — тзх πί/ίΜ) или (см. указание) к к ι Σ m'n ("'AM. max "j/iW) < min Λ/Μ ^ / 2 m'n (π·Ι·(χ)· max "i/iW)· i = 2 / <' / / = 2 / < '' Ho min (π,/ιΜ, max π,/,(χ)) < Σ min (π,/,{^), njfj(x)), I <'' / < i что после интегрирования дает верхнюю оценку для г(6*). Далее имеем, min (nifi(x), max л,/,(л:)) ^ min (л,/,(д:), π,//*)), / = 1 ί — 1, поэтому \ min (nj,(x), max sirfj{x))dx ^ max K,,· / < ί / <' что дает нижнюю оценку для г(6*). Указанные оценки превращаются в точные равенства при k = 2, /(2| !) = /(! 12) = !. 5.11. В данном случае У\2 = π, J/i(x)dx + n2 JM*) dx, χ = (*,, ... , χ,), где Χ] = [χ : πι/|(χ) < π2/2(χ)) и 231
Простые преобразования позволяют записать область Х\ в виде X, ={ж:а'х-уа'(ц0> + ц'2))<1п-^], а = Д-'(ц(,> - μ(2>). Рассмотрим случайную величину Υ = а'Х——α' (μ(ι> + μ(2>)· Если £(Х) = Λ/(μ">, Д), то ЦУ) = Л/(|-, ρ) и )/■(«№ = я(г<.„^.)=ф((,„^-|.)/^. Аналогично имеем 5/2(^=ι-φ((ΐπ^+|)/ν?)=φ((ΐπ^-|)/4 Из этих формул следует указанный результат. В частности, /|2 = = Ф( гН при π, = π2 = —. В случае пуассоновских распределений Π(λι) и Π(λ2) при λι > λ2 /12 = л,е-^2 4f + ^-*24· <·6Χ, <■€*,· где |дс π2 λ2 [■]— целая часть.- Таким образом, 1,2 = π,Π(Γ0; λι) + π2(1 - П(г0; λ2)), где П(г; λ) = 2 е~Ц-· / = о '· 5.12. Используя решение предыдущей задачи и введенные там обозначения, находим, что область наилучшей классификации Wf имеет вид т = [х: А,(х) = /(Ι |2)π2/2(χ) < й2(х) = /(2| 1)π,/,(χ)) = -(■•■'«-y«w"+^>'--lSn11· Таким образом, байесовское решение 6* состоит в том, что при наблюдении χ ζ Wf истинным считается распределение Л/(ц(1>> Л), в противном же случае (т.е. при χ ζ Щ = Щ) — распределение ΛΓ(μ(2), А). Соответствующий вектор риска равен (см. предыдущее решение) Λ(β·) = (/(2| !) \ U{x)dx, /(1|2) \ h(x)dx) = = (,(2Ι„Φ(^Ζ1),,(,Ι21Φ(_^Ζ1)). 232
следовательно, байесовский риск с + р/2' г(в·) = π,/(2| 1)ф(С Р/2) + π2/(1 |2)ф( Vp ' ^ Vp ' При- /(2|!) = /(!|2) = !, π, = π2 = — величина с = 0 и г(6*) Vp~^ ·(-*)· Для получения минимаксного решения δ имеем следующее уравнение для определения константы с (а тем самым и наименее благоприятного априорного распределения (πι, π$ /(2|,)φ/^Ζ2_) = /(Ι|2)φ/_^±£Ζ1). \ л/р / \ Vp ' В частности, при /(2| !) = /(! |2) = ! решением является с = 0 (т.е. π, = πι = -=-] и в этом случае максимальный риск т(8) = Φι —1 ; само же минимаксное правило задается указанной выше областью V/f при с = 0. 5.13. !) Если £.(£) = Si(m. θ), то [(χ; θ) ~ ΘΣ х'(\ - Θ)Σ"" ""'' = = Gr(! — θ)""1 r, априорная же плотность распределения параметра π(θ) au Θ" ~ '(! — θ)4 ~ '. Следовательно, для апостериорной плотности имеем π(θ|χ) ζ=ζ π(θ)/(χ; θ) ss Ва + '-'(! - θ)" + "m " х~ ', т.е. π(θ Ι χ) — плотность бета-распределения β(α + л:, й + nm — л:). 2) Здесь f(x; θ) ^ θ '(! —θ)"', π(θ) указано выше, поэтому π(θ|χ) is Ва+'-'(! -Β)"-*-"'-'. 3) Здесь /(χ; Θ) ^ е""^'1, π(θ) s θ'·- 'е-0-"1, следовательно, π(θ|χ) s ol+r-,e-"<«',+ n/n _ плотность гамма-распределения Pf г , \па+ ] 4) Здесь /(χ; θ) = G"e , π(θ) указано выше, поэтому π(θ|χ) = ^ 0λ + л- !β-θ(ώ+ Ι)/0< 5) Используя функцию Хевисайда е(л:), запишем плотность выборки /(χ ; Θ) в виде {(х; В) = 0_"e(G — *(„)), *(*) = птах (л х„). Аналогично, π(Β) ^ 0"°_|е(е — α), откуда -π(θ|χ) is θ-"-"- 'е(В — max (α, х(л))). 6) Если /г /ΐ/ν — целые неотрицательные чи^сла, удовлетворяющие условию 1ц + ... +ΙιΝ = η, то /(Λ; θ) 3ί θί4 ... Θ/Λ Отсюда π(θ|Λ) is θί' +"'"'...6^ + ^-', т. е. снова получаем плотность распределения Дирихле D(oc + Λ). 7) В данном случае плотность выборки /(*; е)-ехр{-^,-_2 (*.-e)2} = exp{-^(e-i)2- 233
а априорная плотность π(θ) = expi —7τχ(®~ μ)2} · Отсюда *(θ|*) а π(θ)/(χ·; θ) s exp { - -^г (θ - μ)2 - -^ (0 - xf} г -»'{-τ(7+-ρ-)·'+·(ΐ+7-)}· здесь опущены выражения в показателях экспонент, не зависящие от Θ. Последнее выражение пропорционально ехр ·! тг~г(® ~ Μ·0( ■· мто и доказывает утверждение. 5.14. Пусть X = *£(!, ..., п— !). В данном случае апостериорное распределение π(θ|jk) 3ϊ Θγ(! — О)"- * и средняя потеря для решения &(х) = d относительно этого распределения пропорциональна ι \(ά-θ)4χ-'(\ -ΘΓ-'-'ίίθ = duB(x, n-x)-2dB(x + !, η - χ) + a + Β{χ + 2, η - χ) = с Υ d - -£\ + c2. Минимум этого выражения достигается при d = —. Если же χ = η = ο(η), то лишь при d = 0 (соответственно d = !) указанный интеграл конечен. Таким образом, &*(х) = — при любом х. Далее, Λ(θ, δ*) = £,/Α_θ) /θ(! -θ) = Do^)/6(! -θ) = -i- = const, следовательно, δ* также и минимаксное решение и его риск г(б*) = !/п. 5.15. В силу задачи 5.13 п. !) апостериорная плотность n(BU) = as θ° + *_'(! —θ)' + «-*-' и средняя потеря относительно этого распределения пропорциональна ((d-efO"+ '-'(! — θ)* + " -' - ' ίίθ = cYd £±£_У + С2. о Vn + a + u/ Отсюда следует, что 6*(х) = ——— . Вычислим функцию риска: / ne + Q Л2 _ (д-е(д + б))' + ηθ(ΐ-θ) "Чп+а + й J (n + a + bf ' Условие /?(θ, δ*) = const выполняется при а = Ь = V"/2. следователь- но, минимаксное решение о(д:) = ——, а его риск л + л/я m(g) = β(ο, δ) = ■ 4(1+ V"T 234
5.16. Запишем среднюю потерю для решения 6(х) = d относительно апостериорного распределения п(6|л:) в виде £[(θ - df\x] = £[(θ - Ь*{х) + 6*(х) - df\x] = D(Q\x) + (6* (χ) - df > >D(Q\x). Равенство здесь имеет место при d = δ*(л:), следовательно, 6*(х) — искомое решение и его условный (при условии X = х) риск равен D(GIjc). Отсюда байесовский риск имеет указанный вид. В задаче 5.15 апостериорное среднее параметра равно первому моменту распределения (3(а + х, Ь + η — χ), τ. е. 6·(χ) = Е(0\х) = ° + *h ■ 5.17. Здесь ЦХ) = Bi(r, θ) и в силу задачи 5.13 п. 2) апостериорное распределение ί(θ Ι χ) = β(α + χ, b + г). Используя формулы для моментов бета-распределения (см. введение к гл. !}, отсюда (см. задачу 5.16} находим, что искомая байесовская оценка имеет вид δ*(*) = £(θ|*) = α + Χ а + Ь + г + χ 5.18. В силу задачи 5.13 п. 3) апостериорное распределение £.(θ|χ)= г( паа | , λ + *), χ = (х х„), χ = 2 Xi. I Используя формулы для моментов гамма-распределения (см. введение к гл. !), отсюда на основании задачи 5.16 находим, что в«(х) = Е(Ш) = α(λ + "', D(0U) = α2(λ + Х) ■ па+ I ' ' (па+ \γ ' при этом, поскольку L((X) = П(я0) (см. задачу 1.39 п. 4), по формуле полного математического ожидания ЕХ = Е(Е0(А')) = Ε(ηΟ) = ηαλ и поэтому „2 г(&*) = ED(0U)= , , ы (λ + EX) = λα2 (па+ \f ч ' па+ ! ' Наконец, минимизируя величину =-+ся по я, получаем опти- па + 1 мальное число наблюдений Замечание. Число я* должно быть целым неотрицательным. Поэтому, если получаем отрицательное значение, то полагаем п* = 0 (т. е. наблюдений делать не нужно), в остальных случаях в качестве искомого числа наблюдений берется наименьшее целое, большее или равное получаемого по указанной формуле значения. Это замечание относится и к последующим аналогичным задачам. 5.20. В силу задачи 5.13 п. 4) значение d* байесовской оценки при X = χ находится минимизацией по d °dO. 235
Решая относительно d соотношение -т-г Е(Ш, d)\x) = 0, получаем да указанный вид 6*. Далее, так как L0( 2 хч = М7Г' п) (см- заДачУ f-39 п. 2), то ρ ., в + ап ., ва(к+п- !) ' " θ2(λ + η- Отсюда находим функцию риска Л(0. 6*) = Е.( β· - 1)' = Drf· + ( Ε„δ* - 1)2 = _ дгя + (θ — α(λ— Ι))2 ~ βν(λ + η- !}2 ' Наконец, г(6*) = Е/?(0, 6*) и используя формулы для моментов гамма- распределения, приходим к указанному результату. Число п* находится минимизацией по η величины г(6*) + сп. 5.2!. Среднее и дисперсия распределения Парето с параметрами аа ааг _ а и α > 2 равны соответственно .- и -; т-л- —. Отсюда α — 1 (α — 1) (о — 2) (см. задачу 5.13 п. 5} на основании задачи 5.16 имеем указанный вид оценки 6*. Кроме того, получаем также, что Для нахождения риска r(5*) = ED(0|X) достаточно вычислить Е(тах(а, А'(л)))2 = Е[Е„(тах (а, Х(п)))2]. Запишем (max (α, Χ(η))γ = {al{XM < a) + *(„,/(*(„> > a)f = a2I(X(n) < a) + + ХУ(Х{П) > а), где /(Л) — индикатор события А. Тогда, поскольку плотность распределения .fy,,) при заданном 0 равна η/"_ι/°". 0<ί<θ (см. задачу 1.35), имеем --^iV^+4^=-^+2αη+2 E^max (α. XW)Y = _j,- -Λ + _)/« + -Λ = __ О' + ^^^т . Далее находим Ee2 = D9 + (E0}2= -Ϊ5-. Εθ-" = αα»ί—-^—, = " , . следовательно, ere/ /ν »2ι αα2(η + «—2) E[£„(max(a. ЗД] = 7__J.. Окончательно получаем, что риск байесовской оценки равен П ; (o-2)(n+o- \f 236
Наконец, минимизируя величину г(6*) + сп по л, находим, что оптимальное число наблюдений 1/з \ с(о - 2) / ^ 5.23. Искомое решение — это значение d, минимизирующее следующее математическое ожидание: л/ л/ Ε(Ζ.(Θ, d)\h) = Σ Ц(в,--Й,-)21А]= Σ Ε[(Θ,-Ε(Θ,|Λ))2|Λ] + i = I i = I Λ/ Ν Ν + Σ (ε(θ,ιλ)-4)2= 2 ο(θ,ιλ)+ 2 (ε(θ,ιλ)-4)2. i=I i=I i=l Минимум этого выражения достигается при d, = Е(0,|Л), i = !, ..., /V, л/ и равен 2 D(6i|A). Здесь (см. задачу 5.13 п. 6} и указание) i = ι Е(0.|А) = ^±HL , D(0,| Λ) = (a, + yVrUnfc) · a + n (a + nf(a + η + !) Первая формула определяет вид байесовской оценки 6,*(Л), а вторая л/ позволяет вычислить соответствующий риск: г(6*) = 2 ED(6(|n), если ί = 1 при этом воспользоваться формулами (см. задачу 1.52 и указание) EAj = Е(ЕД) = Εηθ, = noi/o, ЕА,-(А,- — !) = E(E0ft,(n,- !)) = Ел(л - !)θ? = "(" "^ ' ^ + '} ■ 5.24. Используя обозначения задачи 5.13 п. 7), на основании задачи 5.16 имеем, что искомая оценка β·(χ)= Ε(θ|*) = μ, = 0f(-£-+ -£-); при этом D(G|x) = а] = const, следовательно, риск г(6*) = (σ-2+ + п6_2)_|. Минимизируя по л общие затраты г(6*) + сп, находим оптимальный объем выборки -»Gf-*)- 5.25. !) Пусть d>d*. Тогда id* — d при θ 5г d, |θ —d| — |θ —d*| ={ d + d* — 2Θ при ίί* <θ < d, 1 d — d* при θ «ς d*. Так как d + d* — 2Θ > d* — d при d* < θ < d, то Ε[(|θ-«ί| - |θ — d*|)U]Xrf* —ίί)Ρ(θ> ti)jK) + + (d *- d)P(d* < θ < d\x) +(d- d*)P(Q ^ d*U) = = {d- d*) [Ρ(θ < d*|*) - Ρ(θ > d*U)]. Поскольку d* — медиана, последняя разность больше или равна 0. Аналогично рассматривается случай d < d*. Таким образом, решение 237
d* минимизирует среднюю условную (при X = х) потерю, т. е. это — байесовское решение. 2) В силу задачи 5.13 п. 7) медианой апостериорного распределения Ц0\х) является точка d* = μ,, следовательно, байесовская оценка по выборке X = (ΑΊ, ... , Х„) имеет вид ™-<*($ + %-) и ее риск равен г(6*) = Ε|θ- δ*(Χ)| = Е[Е(|0- δ*(Χ)| \Χ)]. Но так как условное (при X = х) распределение случайной величины 0-&*(Х) есть N(0, σ2) и для ЦГ) = Щ, σ?) Е|У| ="у_σ,, то * π ' π Если цена за одно наблюдение ранца с > 0, то, минимизируя общие затраты r(fi*) + сп по п, находим оптимальное число наблюдений п* = b\-^.cb2f'3 - σ-2}. Глава 6 6.1. Для дисперсии X справедливо представление D^ = -r 2 cov (Л*. Л,) =-V Σ Rk-,= k,s = 1 П A.s = 1 из которого при условии (6.!) следует, что DX — 0[—) при п- т. е. X — состоятельная сщенка. 6.3. Для среднего ЕСь(п) можно получить представление ЕС(п) = R„ - , ' ,. Σ Σ (Ri-s + Rl + *-s)+4r Σ Ri-·. ПУП R) ι = I j = I " l,s = I в котором добавок к Ri, при условии (6.!) имеет порядок 01 —) , когда 6.4. Здесь ЕХ, = 0 и cov {Хк + ι, */) = Е(** + ,, X,) = o2(cos λ(Α + /)cos kt + + sin Цк + /)sin λ/) = o2cos λ£ г 6.5. Здесь ΕΛΊ = m J α( и / = ° г-1*1 σ2 Σ αίαί +ι*ι cov(*t + i, Χι) = Σ α,ο,ΐον^+,-ί, ξ,-j) ;./ = о / - о при \k\ ^ л, О при \k\ >г, 238
поскольку cov (ξ* + ι —,-, ξ,_() = σ2 при i — j = k и равна нулю при 1-\фк. 6.8. Значения коэффициентов оптимального предиктора определя- о ются по формулам βί, = £ R''Ri-и где ||Λ''||- = ||/?,_,1Г'(/, /'= i = ~п = 0, —1, .... — п). Для величины а[п) справедливо представление о2(«) '"' |/?(л + 2)|/|/?(л + *(« + О = где (см. [!], с. 218} матрица Rofli R, R,Ro Rn- ]R„R»-i...Ro 6.9. Поскольку ||p,v(/)|| = ||ρ,,(!)||\ достаточно проверить равенство llPij(' + l)ll = llPi/(0ll llPiiOll· Так как матрица ||р,;(1)[[ дважды стохастическая, то стационарное распределение является равномерным. 6.10. Пусть U, Uo, U\, ... — последовательность независимых равномерно распределенных на отрезке [0, !] случайных величии. Определим случайные величины: So(iV) ι! при "Ь при при U < ^и> —\2 при 6/^ I ■ 2 · 2 ς ^) -\2 при iV^ct. α; Тогда реализация цепи Маркова определяется формулами vo = ς0((Λ), ν/ = 6.11. Имеем Εη, = 0, Rt = Εηκ, + ,η, = Ρ(η* + 1 ς'"'-'![{//). 1>\. η/)-Ρ(η*- η/] = = γ(Ριι(*) + Ρ22(*) — Pu(ft) -Рг.(*)) = (Ι -2α)'. 6.13. Здесь Εη, = Ε[Ε(ξνχί}Ί^ί>] = 0, так как ЕЦ1) = 0. Далее (см. задачу 6.9) имеем Εη* + ,η, = Е[Щ1„, + /Λ + /)£ν,(ί)Ιν* + ,, ν,)] = P(vt + , = ν, = 1)Й" + + Ρ(ν* 2)Й2' =-- -ί- (1 + (1 - 2α)4) (Й" + ΛΪ>).
ПРИЛОЖЕНИЯ 1. Нормальное распределение Квантили распределения: ρ - V2n _ г.- ">Чх Ρ 0,50 0,51 0,52 0,53 0,54 0,55 0,56 0,57 0,58 0,59 0,60 0,61 0,62 0,63 0,64 0,65 0,66 0,67 11, 0,000 0,025 0,050 0,075 0,100 0,126 0,151 0,176 0,202 0,228 0,253 0,279 0,305 0,332 0,358 0,385 0,412 0,440 Ρ 0,68 0,69 0,70 0,71 0,72 0,73 0,74 0,75 0,76 0,77 0,78 0,79 0,80 0,81 0,82 0,83 0,84 0,85 u, 0,468 0,496 0,524 0,553 0,583 0,613 0,643 0,674 0,706 0,739 0,772 0,806 0,842 0,878 0,915 0,954 0,994 1,036 Ρ 0,86 0,87 0,88 0,89 0,90 0,91 0,92 0,93 0,94 0,95 0,96 0,97 0,98 0,99 0,999 0,9999 0,99999 Up 1,080 1,126 1,175 1,227 1,282 1,341 1,405 1,476 1,555 1,645 1,751 1,881 2,054 2,326 3,090 3,720 4,265 2. Распределение Пуассона Значения функции £ k=x λ* ο,ι 0,2 0.3 0,4 0.5 0,6 0 I 2 3 4 1,000 0,095 005 1,000 0,181 018 001 ι,οοο 0,259 037 003 1,000 0,330 062 008 001 1,000 0,394 090 014 002 1,000 0,451 122 023 003 240
0,7 0,9 1,0 2,0 3,0 0 1 2 3 4 5 6 7 8 1,000 0,503 156 034 006 001 1,000 0,551 191 047 009 001 1,000 0,593 228 063 014 002 1,000 0,632 264 080 019 004 001 1,000 0,865 594 323 143 053 018 005 001 1,000 0,950 801 577 353 185 084 034 012 4,0 5,0 6,0 7,0 9,0 0 1 2 3 4 5 6 7 8 9 10 II 12 13 14 15 16 17 18 19 20 1,000 0,982 908 762 567 371 215 III 051 021 008 003 ooi ι,οοο 0,993 960 875 735 560 384 238 138 068 032 014 005 002 001 1,000 0,998 983 938 849 715 554 394 256 153 084 043 020 008 004 001 001 1,000 0,999 924 970 918 827 699 550 401 271 170 099 053 027 013 006 002 001 1,000 1,000 0,997 986 958 900 809 687 547 408 283 184 112 068 034 017 008 004 002 001 1,000 1,000 0,999 994 979 945 884 793 676 544 413 294 197 124 074 042 022 Oil 005 002 001
χ ^ .ο" α ο. ο ja φ £ν ΐ© ON Ю 0,27 0,00 0,29 0,00 — ο со о ο ο ■ί· ο со о ο ο г» ο со о 0,36 0,00 0,39 0,00 — ο ττ ο ο ο ю ο 4- Ο ο ο СО О *^ ο 0,43 0,02 Ю CM ■Ί· Ο ο" ο" 00 CM ■Ί· Ο ο ο CM CM ю о о о ш со ю о 0,48 0,04 — ю ю о о'о" ю о о о ю о о о — ш (О о 0,52 0,07 ю с» ю о о'о" 00 00 ю о о о (£> О о о ю о 0,56 0,10 σ> — ю — о'о" см см (О _ о о 1Я Μ (О _ о о (О _ 0,59 0,13 0,52 0,14 (О — о о 00 Ш (О — о о ОО о о о о о о о о — о -tj- О о о со о ю о о о О СО СО О о о LO Г*- (О о о о ю — о о см ю о о о о <£> О -tj- О о о оо о ю О о о Ю СО UD О о о О оо г» о о о ^ см о о г» г» о о σ> — h- см о о см о ю о -tj- О ЦЭ О — -Ί· г-- о ш <л г» о σι -з· h- — — <л CO — СО СО СО СМ о о о о 00 о см — г^- О о о со *& г- о о о см о 00 — о о 00 — о о 00 СМ о о 00 <£> 00 СМ о о — о г» о о о со о о о Ю 1П 00 О о о со см 00 — о о О 00 о о см ю σι см о о со о σι со о о *ί> О 00 О о о <л о о о со г*- σ> о о о о о <£> см σι см о о <£> СП σι см о о <Л СО о о ОО О <л о о о о~> о о о сп σι СП О о о сп о σ> — о о о σι о <м — о о ш О СО — о О СО о о — о" о см О *j> —Ό 242
см to to ~- to ~- <X> CM <X> CM to см СЛ h- tD CM ~ СЛ h- см о о о о Ο* Ο h- 00 to — r^ ο tD CM о" о" СЛ CM чэ см ел со to см о*о" h- см о <х> г*-, см о" о СО h- h- СМ «М 00 ■h- см о'о" ^Г О h- со со — о" о <Х> CM h- СО о о о о о о о о CN СО г*- см о'о" о о о о СО — h-_CN о'о оо оо оо оо h- СО г-, см ел h- г-. см о о 00 СО см со 00 СО СО <Х> 00 СО -=J- 00 00 СО о о о о о о о о 00 СМ о о см ел 00 СМ о о -=j- см 00 СО о о 00 СО о о to оо 00 СО о о 00 -=J- о о Ю 00 00 СМ о о to см 00 СО о о оо со о о 00 00 ОСЮО о о ел ~ ОО ^Г о о о ^ СЛ ^г о о σι — со со о о о ю σι со о о <Л СО о о см см σι -ί* о о см ю σι -з· о о со со σι "^ о о СО Ю σ> со о о ■ί· σι σι со о о ю со σι -ί* о о σι -з· о о σι -ί* о о <£> СМ О О г» о σι **■ о о со ^ σι -з· о о со со σι -ί* о о СО CN О О со г*- о о со г*- о о о г» о -* — о О CN 3 Ю — о О ί£> о ю — о о σ> о ю — о О CN о ш — о о ^ о ш — о 243
4. Распределение χ5(η) Квантили распределения: р= \ k„(x)dx = —-, - \ д;"/2 'е x/2dx о 2"^Γ(η/2) £ \ р η \ 0,1 0,3 0,5 0,7 0,9 0,95 0,999 0,9999 1 2 3 4 5 6 7 8 9 Ю Π 12 13 14 15 16 17 18 19 20 21 22 23 24 25 26 27 28 29 30 0,016 0,211 0,584 1,06 1,61 .2,20 2,83 3,49 4,17 4,87 5,58 6,30 7,04 7,79 8,55 9,31 10,09 10,9 11,7 12,4 13,2 14,0 14,8 15,7 16,5 17,3 18,1 18,9 19,8 20,6 0,148 0,713 1,42 2,20 3,00 3,83 4,67 5,53 6,39 7,27 8,15 9,03 9,93 10,08 11,7 12,6 13,5 14,4 15,4 16,3 17,2 18,1 19,0 19,9 20,9 21,8 22,7 23,6 24,6 25,5 0,455 1,39 2,37 3,36 4,35 5,35 6,35 7,34 8,34 9,34 10,3 11,3 12,3 13,3 14,3 15,3 16,3 17,3 18,3 19,3 20,3 21,3 22,3 23,3 24,3 25,3 26,3 27,3 28,3 29,3 1,07 2,41 3,67 4,88 6,06 7,23 8,38 9,52 10,7 11,8 12,9 14,0 15,1 16,2 17,3 18,4 19,5 20,6 21,7 22,8 23,9 24,9 26,0 27,1 28,2 29,2 30,3 31,4 32,5 33,5 2,71 4,61 6,25 7,78 9,24 10,6 12,0 13,4 14,7 16,0 17,3 18,5 19,8 21,1 22,3 23,5 24,8 26,0 27,2 28,4 29,6 30,8 32,0 33,2 34,3 35,6 36,7 37,9 39,1 40,3 3,84 5,99 7,82 9,49 II,I 12,6 Η,Ι 15,5 16,9 18,3 19,7 21,0 22,4 23,7 25,0 26,3 27,6 28,9 30,1 31,4 32,7 33,9 35,2 36,4 37,7 38,9 40,1 41,3 42,6 43,8 6,63 9,21 П,3 13,3 15,1 16,8 18,5 20,1 21,7 23,2 24,7 26,2 25,7 29,1 30,6 32,0 33,4 34,8 36,2 37,6 38,9 40,3 41,6 43,0 44,3 45,6 47,0 48,3 49,6 50,9 10,8 13,8 16,3 18,5 20,5 22,5 24,3 26,1 27,9 29,6 31,3 32,9 34,5 36,1 37,7 39,3 40,8 42,3 43,8 45,3 46,8 48,3 49,7 51,2 52,6 54,1 55,5 56,9 58,3 59,7 244
5. Распределение Стьюдента S(n) Значение функции t't.„ Ύ» ■ " у( "+' \ 'ϊ.'ι 11 V 0,9 0,95 0,98 0,99 I 2 3 4 5 6 7 8 9 10 12 14 16 18 20 22 24 26 28 30 6,314 2,920 2,353 2,132 2,015 1,943 1,895 1,860 1,833 1,812 1,782 1,761 1,746 1,734 1,725 1,717 1,711 1,706 1,701 1,697 1,645 12,706 4,303 3,182 2,776 2,571 2,447 2,365 2,306 2,262 2,228 2,179 2,145 2,120 2,101 2,086 2,074 2,064 2,056 2,048 2,042 1,960 31,821 6,965 4,541 3,747 3,365 3,143 2,998 2,896 2,821 2,764 2,681 2,625 2,584 2,552 2,528 2,508 2,492 2,479 2,467 2,457 2,326 63,657 9 925 5.841 4,604 4,032 3,707 3,499 3,355 3,250 3,169 3,055 2,977 2,921 2,878 2,845 2,819 2,797 2,779 2,763 2,750 2,576
43 + ю о" с[г' + ■θ- та ч. 3 со -tf ю to см со см см ю о см со со со -tj- О CM СЧ **■ о СЧ — СЧ Ш СЧ О СП — со со СЧ СП ■ί· σι СО Ю СЧ СО ю ю ю СМ СЧ СЧ (£> ю — СЧ — о (О СО (£> h- О СО Ю СЧ_ (£>_ Ю_ ^_ — οοΌ" юм -^ σι" со ю Ю_сО. СО~(£>" σι~σί — σι *& {J> СО Г^ г^ σι со со — СМ СП СП ~ СП г^ о 00 h- см сп со Ю ^Г 00 СП <Х> 00 ^Г СП о см о г^ ■ч}" h- СП О со ^г СП СП ~ СП 00 СО h- см 00 h- СМ СП Ю Ю ^Г Г- О ^Г О О 00 ^Г h- 19,37 99,36 19,33 99,33 8,84 27,49 8,94 27,91 6,04 14,80 6,16 15,21 4,82 10,27 4,95 10,67 4,15 8,10 4,28 8,47 СЧ СЧ СЧ ~ σι со СЧ <л со СО <Л ш ю — СО ю — СО Ю ■ί· σι 216 5403 200 4999 19,16 99,17 19,00 99,01 9,28 29,46 9,55 30,81 6,59 16,69 6,94 18,00 5,41 12,06 5,79 13,27 4,76 9,78 5,14 10,92 — СП 00 00 — СП со см О ^г — со — о to — см — to <хэ см to to СП ^Г СП h- Ю СО 246
σ> со <Ό — см см ю оо о <л СО Ю см СО 00 **■ ш 00 см со СО <£> о о СО Ю ■3· СМ см ^ о ш см со Ш СО — см Г-. CM ~ CM о *r <X> CD — — 00 CO —. — CM CM — — CO Ю —. — ю to ~ rt_ со'ю" h- — r-._ ^r_ CM -=J-" -=J- <X> Ю00 CM*c0~ CM -=J- ~_сл см~см" CO Ю qiq ~ cm" 00 t£> Г-.СМ ~*oi 00 tD tD СЭ ~ см" CM h- tqo> ~S ~ 00 СЛ Ю 00 — S 00 Г-- CM <X> CO Ю СЛ Г-- CM -=J- CM -=J- 00 CO CM CM CM CO О 00 CM CM — CM 00 CO — CM о со 00 CM — CM ш о h- см — CM -=J- CM CO 00 CO Ю <J> 00 CM -=J- iO О Г-. CO CM -=J- CO CO CM CO to oo CM CM CM О о t^- CM CN CM ~ — CM 00 -=J- ~ CM 00 CO ~ CM h- h- CM ~ CO OO ~ oo CO <J> о to 00 О <J> (Ω Ю CO <J> Ю CO to СОЮ 00 Г-- Ю CO CM <J> CM CO о см P.» О Г- tD 00 CM Г-. ^r ^r σ> oo CM — <J> <J> ~ <J> ~ Oi о см ~ 00 CM CO CM CO ^J* '"""' oo o_ со' t*- oo σ> *& <j> CO Ю t£t ^- CM -=J- ^f Ю t^. CO 00 -=J- CM -=J- <J> CM <D О CM -=J- to см ю r-. CM CO t£t ~, ^f Ю CM CO _· — ^ ^ см" со" 00 -=J- CO CO CM CO 3.2 Г-. СЛ 4,46 8,65 5,32 11,26 — ю г- ю о ш 4,96 10,04 3,49 5,95 00 CO 00 <J> co~<rT 4,75 9,33 Ο ^ϊ- со~^" 3,49 5,85 4,35 8,10 см — см"^-" 3,32 5,39 4,17 7,56 <л о г-._см_ см~ч-~ 00 Ю со'ю" 4,03 7,17 О 00 г-._с^ см" со" σι см о_оо_ СО ^Г 3,94 6,90 Ю 00 см"со~ со ^г- 3,89 6,76 2,61 3,80 о см СО **■ 3,85 6,66 r S с X .. cz . Значения Персии; пг - н и я: 1 ,шей дис о о о 247
7. Критерий Колмогорова Значения функции λρ: ρ = Р(£>„ = supl/7,,^) — F{x)\ > λρ Χ I 2 3 4 5 6 7 8 9 10 π 12 13 14 15 16 17 18 0,10 0,950 776 636 565 509 468 436 410 387 369 352 338 325 314 304 295 286 279 0,05 0,975 842 708 624 563 519 483 454 430 409 391 375 361 349 338 327 318 309 0,01 0,995 929 829 734 669 617 576 542 513 489 468 449 432 418 404 392 381 371 >ч Ρ 11 >ν 19 20 25 30 35 40 45 50 55 60 65 70 75 80 85 90 95 100 0,10 0,271 265 238 218 202 189 179 170 162 155 149 144 139 135 131 127 124 121 0,05 0,301 294 264 242 224 210 198 188 180 172 166 160 154 150 145 141 137 134 0,01 0,361 352 317 290 269 252 238 226 216 207 199 192 185 179 174 169 165 161 248
ооооооо>о>спа>оосомо ΟΟΟΟΟΟΟ^Ο^ΟϊΟϊΟϊΟϊΟϊΟϊ ΟΟΟΟΟΟΟϊΟϊΟϊΟϊΟϊΟϊΟϊΟϊ ΟΟΟΟΟΟθσ)ΟΟΝ(ΟιΟΓΟ-ΌΟ 0000000(^0)0)0)010102!» ΟΟΟΟΟΟΟ0)0)0>0)0>0)0>0) ООООООЮОМОСО-ООЧ-ОЮО οοοοοσίΦΦσισισιοοοοοοΝΝ ΟΟΟΟΟΟϊΟϊΟϊΟϊΟϊΟϊΟϊΟϊΟϊΟϊΟϊ 1 4. >Ооа>а>Г^ЮСча>-Фа>С0(£>а>~СЧСЧ — — — о OOOiCOOOOoor^iOiD^i'CO^-OCTir^iO οοσϊσισισισιοσισιαισισιοοιοοοοοο -Γ -Γ ο" ι осэоос^оооэоэ^о — юг^оооо^ю^с^оэг^. OtJiOiCTiOOh-tDiO^fCJOOOtD^fCJOOOiOn οσϊΟϊσισϊΟϊΟϊΟϊΟϊΟϊΟϊοοοοοοοοοο^- г*- г*- —"о ν/ га О. 00 *. OC44'NcC^iM<j5,tONTj.^40N(Na)NCOO*t θ№Ν ^I,^-^-c^oo^I^OLO^-^-fOOϊ^Dc^^Oϊ^D^I, — οσισ)θ)<ο)οοοοΝ^Ν(θΐοΐοΐοτί'4'4':οσ3η:ο § — — ν oo t^ σο iM — ^fCJiocj^fOJOicjoot^-oocjoo t^CJiOOO — TfOOCJtD— tOiN 00 4*— Οϊ tO **" C^ ~ 0> OOJOJ00r^-(^.t£>mm,«i''«i'CT3CT3CgCJCg~~~~~O §O^CO(DiOOO^S^iNO)(NOOM»(NNnOCOlD ONtWOJt^.— (DC*3 0^.tOrfCOC^iN — _ _ о О OOir^-tDiO-rfCOiMCM^-^-^-OOOOOOOOOOO ONOOlSON O — tDCT3CJ~00000000000 О(ООС^1МЮС0 1М — OOOOOOOOOOOOOOO OtD-^fCM — OOOOOOOOOOOOOOOOOOO 249
9. Равномерно распределенные на [0, 1| случайные числа 0,5916 6562 3127 0690 3617 7128 6635 9162 9313 8216 9470 3361 5303 1039 9031 9481 8922 5725 4398 3652 9612 6836 4601 9154 2212 7158 9192 3072 2380 3290 4448 7042 5978 7147 4386 3406 7463 1413 0120 6700 6396 5477 3901 5889 8851 2099 6387 5501 9430 4215 4474 6145 6542 7198 5889 0448 ΟΙΟΙ 8247 7397 8036 2036 9019 2267 4955 8562 1790 4161 5412 7403 9362 6079 8203 971 I 3993 5940 6787 9121 7480 0399 4184 1992 3374 4237 6838 1197 8315 5759 2141 5643 8865 9632 8861 6883 3584 6484 5270 7880 6512 7803 2558 1932 9279 2154 5033 6122 4101 1643 6253 3136 9629 3147 4513 6319 2226 0471 0836 4963 5307 4188 8764 1752 5489 7449 3687 2378 3741 2786 2196 2139 9953 7441 4728 5673 1907 5986 0833 4049 9202 8549 0193 5314 5825 4135 3821 1094 2625 6213 2645 7919 9664 5050 1255 8954 2383 8382 8546 1479 1653 2311 2198 4776 5132 6570 1019 8382 4387 0115 2943 5803 1904 7005 1693 7586 6005 1987 10. -0,486 -0,256 0,065 1,147 -0,199 1,237 - 1,384 -0,959 0,731 0,717 Нормально 0,856 -0,212 0,415 — 0,121 -0,246 1,046 0,360 0,424 1,377 -0,873 распределенные — 0,491 0,219 — 0,169 1,096 1,239 -0,508 -0,992 0,969 0,983 — 1,096 N(0, 1) -1,983 0,779 0,313 0,181 -2,574 - 1,630 — 0,116 — 1,141 - 1,330 — 1,396 случайные — 1,787 -0,105 -1,339 1,041 0,279 —0,146 — 1,698 -1,041 1,620 1,047 числа — 0,261 -0,357 1,827 0,535 - 2,056 -0,392 -2,832 0,362 - 1,040 0,089 250
-1,805 - 1,186 0,658 -0,439 -1,399 0,032 0,151 0,290 0,873 - 0,289 -0,291 - 2,828 0,247 -0,584 0,446 0,034 0,234 -0,736 - 1,206 -0,491 -1,334 -0,287 0,161 - 1,346 1,250 2,923 - 1,190 0,192 0,942 1,216 0,499 0,665 0,754 0,298 1,456 0,593 - 1,127 -0,142 -0,023 0,777 0,241 0,022 -0,853 - 0,501 0,439 — 2,008 1,180 — 1,141 0,358 -0,230 0,079 -0,376 -0,902 -0,437 0,513 1,221 -0,439 1,291 0,541 -1,661 — 2,127 -0,656 1,041 -0,899 — 1,114 1,278 -0,144 -0,886 0,193 -0,199 0,500 -0,318 -0,432 1,045 0,733 — 0,431 -0,135 -0,732 1,049 2,040 0,993 -1,407 -0,504 -0,463 0,833 -0,957 0,525 — 1,860 -0,273 -0,035 -1,633 1,114 1,151 - 1,939 0,385 0,199 0,159 2,273 0,041 -1,132 1,119 -0,792 0,063 0,484 1,045 0,665 0,340 0,008 0,110 1,297 -0,568 -0,254 -0,921 -1,202 -0,288 0,630 0,375 - 1,420 -0,151 -0,309 1,705 -0,145 -0,066 1,810 -0,124 -0,106 -1,579 0,532 -0,899 0,410 - 1,885 -0,255 -0,423 0,857 -0,260 0,542 0,882 -1,210 0,891 -0,649 0,208 0,272 0,606 -0,307 — 2,098 0,004 — 1,275 — 1,793 -0,986 -1,363 0,084 -0,086 0,427 — 0,528 -1,433 -0,109 0,574 -0,509 0,394 1,810 -0,537 — 1,941 0,489 -0,243 0,531 1,164 -0,498 1,006 2,885 0,196 0,116 -1,616 1,381 -0,394 -0,349 0,371 -0,702 -0,973 -0,465 0,120 0,250 1,265 -0,927 -0,227 -0,577 -1,083 -0,313 0,606 0,121 0,921 0,768 0,375 -0,513 0,292 1,026 -0,880 -0,158 -0,831 -0,813 -1,345 -0,515 -0,451 1,410 -1,045 1,378 0,782 0,247 — 1,711 -0,430 0,416 0,424 0,593 0,862 0,235 -0,853 0,484 1,458 0,022 -0,538 - 1,094 -2,830 0,953 -1,016 -1,691 -9,558 -0,166 -0,202 0,425 0,602 0,237 -0,219 0,084 -0,747 0,790 0,145 0,079 - 1,656 -0,344 -0,521 2,990 -1,473 -0,851 0,210 1,266 -0,574 - 0,566 -1,181 -0,518 0,843 0,584 0,060 -0,491 -1,186 -0,762 - 1,541 - 0,444 0,658 -0,885 -0,628 0,402 - 1,272 1,262 -0,281 1,707 0,580 - 0,238 -0,869 - 1,726 0,417 0,056 251
II. Д-l. Датчик случайных чисел на автокоде ЭВМ серии ЕС RAN START USING STM L L SR Μ 0 A N ST SRL A ST LE LM BR DS DS END ι *,I5 I, 15, SA 9,0(1) 3, 0(9) 2, 2 2,= F*8433H86l* 3,=XT80000000* 3, = F*4538I6693T 3,=XT7FFFFFFF* 3,0(9) 3, 7 3, = XT40000000* 3, X 0, X I, 15, SA 14 I5F F 12. Д-2. Датчик случайных чисел иа автокоде БЭСМ-6 RAN ,ΝΑΜΕ, ,ΑΤΙ,9 ,NTR,3 9.XTA, ,E + N,24 ,A*X,= 1431777206549 ,ΥΤΑ, AAX, = 00I7 77777777 7777 ,AOX,= :642 ,A+X,= I 232354146751 ,AAX, = 7757 7777 7777 7777 9, ATX, ,Ε+Λ/,24 NTR, 6 I3.UJ, ,END, 13. Подпрограмма «ВП» (вариационный ряд) SUBROUTINE RANK (X,Ν) DIMENSION X(N) Μ = Ν—I DO 2 [=1,Μ K=l I J=K+I IF(X(K)„LE.X(J)) GOTO 2 R = X(K) X(K) =X(J) X(J) = R IF(K.GE.l) GOTO 1 252
2 CONTINUE RETURN END Подпрограмма располагает в неубывающем порядке числа массива X(N). 14. Подпрограмма «НЧ» (нормальные числа) REAL FUNCTION RNORM (К) S=0 DO I 1 = 1, 12 I S=S + RAN (K) PNORM = S-6. RETURN END Подпрограмма вызывается оператором Y= RNORM (К) УКАЗАТЕЛЬ РАСПРЕДЕЛЕНИИ I Нормальное I: 9, Ю, I I, 27, 29, 32, 39, 41, 43, 46, 47, Μ(μ,σ2) 56, 57, 58, 60, 61. 2: 13, 14, 15, 16, 17, 18, 19, 20, 43, 48, 50, 52, 53, 64, 65, 66, 67, 68, 70, 71, 72, 84, 85, 86, 87, 88, 89,94, 106, 109, ПО, 114, 115, 116, 117, 118, 119, 120, 121, 122, 123, 124, 125, 126, 131, 142, 143, 145, 146, 148. 3: 35, 45, 47, 48, 49, 50, 58, 59, 60, 61, 65, 66, 67, 73, 74, 79, 80, 84, 87, 4: 8, 9, 10, 11, 13, 14, 15, 19, 20, 21, 22, 23, 24, 25, 27, 28, 29, 34, 36. 5: 7, 8, 13, 24, 25. 2. Многомерное нормаль- I: 28, 40, 53, 59, 62, 63. 2: 39, 44, 90, ное Ν(μ, Σ) 91, 92, 93, 132. 3: 52, 72, 88. 5: 11, 12. 3. Биномиальное I: 1,2, 12, 13, 14, 15, 16, 17, 18, 39, 52, Bi{n, ρ) 54. 2: 5, 6, 7, 8, 43, 48, 57, 84, 109, ПО, 133, 134, 135, 136, 148. 3: 1, 2, 5, 17, 18, 39, 40, 46, 53, 63, 75, 77. 5: 1,2, 3, 4, 13, 14, 15, 16. 4. Полиномиальное 1:3, 19, 20, 26, 39, 52, 53, 54. 2: 29, 38, М(п; р\...ры) 45, 63, 144, 3: 3, 4, 7, 8, 9, 12, 13, 20, 24, 34, 37, 38. 5: 13, 23. 5. Пуассона I: 39, 54, 55, 60, 64, 2: 9, 10, 31, 43, Π (λ) 48, 58, 59, 61, 84, 97, 108, 109, 137, 138, 139. 3: 14, 15, 16, 41, 54, 64, 76, 78, 5: 6, 11, 13, 18, 19. 6. Отрицательное биноми- 1: 39, 55, 2: 11, 12, 43, 48, 62, 84, 96, альное Bi(r, p) 140. 3: 42, 55. 5: 5, 13, 17. 7. Гамма Γ(α, λ) 1: 6, 7, 8, 21, 34, 39, 42, 44, 51, 55. 2: 21, 22, 30, 43, 48, 51, 73, 74, 75, 84, 104, 109, 127, 128, 130, 141. 3: 10, Π, 43, 56, 62, 68, 69. 5: 13, 18, 20. 8. Равномерное Ι: 5, 19, 35, 36, 43, 46. 2: 23, 24, 25, 32, R(a,b) 79, 80, 81, ι00, 101, 107, 110, 129, 148. 253
3: 25, 36, 45, 70. 4: 7, 12, 33, 35. 5: 13, 21, 22. 9, Вейбулла Ща,а,Ь) 10, Коши К(а) 11, Гииергеометрическое H(r,N,n) 12, хи-квадрат χ'(η) 13, Бета β(α,β) 14, Стьюдента S(n) 15, Снедекора S(ni,n2) 16, Логистическое 17, Степенного ряда 18, Парето 19, Дирихле 20, Лапласа 21, Кептайна 22, Обратное гауссовское 23, Конечная совокупность I: 3: I: 2: I: I: I: I: 2: 2: 5: 5: 2: 2: 2: 2: 37. 2: 26, 76, 77, 71. 46. 2: 28, 43, 99. 33, ИЗ. 3: 57. 40, 45, 47, 51, 57 44, 47, 48, 49. 2: 47, 50, 59 48, 49, 50, 51 27, 49 60, 96, 140 13, 21 13, 23 105 95 54, 34, 35, 36, 37, 83, 102, 103, 107, 3: 44. , 59, 147. 3: 19 48. 5: 13, 17. 111, П2
ЛИТЕРАТУРА 1, Ивченко Г. И., Медведев Ю. И. Математическая статистика. М., 1984. 2, Чистяков В. П. Курс теории вероятностей. Μ., Ι982. 3, Кендалл М., Стьюарт А. Теория распределений. Μ., Ι966. 4, Крамер Г. Математические методы статистики. Μ., Ι975. 5, Неман Э. Проверка статистических гипотез. М„ 1964. 6, Бикел П., Доксам К. Математическая статистика. Μ., Ι983. Т. 2. 7, Феллер В. Введение а теорию вероятностей и ее применения. М., 1984. Т. I. 8, Ивченко Г. И., Глибоченко А. Ф., Иванов В. А,, Медведев Ю. И. и др. Статистический анализ дискретных случайных последовательностей. МИЭМ. — Μ.: Ι984. 9, Кнут Д. Е. Искусство программирования (т. 2. Получисленные алгоритмы). Μ., Ι977. 10, Ермаков С. М., Михайлов Г. А. Статистическое моделирование. — Μ., Ι982.
Учебное издание Ивченко Григорий Иванович, Медведев Юрий Иванович, Чистяков Александр Владимирович СБОРНИК ЗАДАЧ ПО МАТЕМАТИЧЕСКОЙ СТАТИСТИКЕ Зав. редакцией учебно-методической литературы по физике и математике Е, С. Гридасова Редактор Ж. И. Яковлева Мл. редакторы Г. В. Вятоха, Н. П. Майкова Оформление художника И. Д. Блынского Художественный редактор В. И. Пономареико Технический редактор В. М. Романова Корректор Г. Н. Буханова ИБ № 8011 Изд. № ФМ-942. Сдано в набор 10.03.89. Подп. в печать 19.10.89. Формат 84Х108'/з2. Бум. тип. №2, Гарнитура литературная. Печать высокая. Объем 13,44 усл. печ. л. 13,65 усл. κρ.·οττ. 14,44 уч..нзд. л. Тираж 25 000 экз. Зак. № 190. Цена 55 коп. Издательство «Высшая школа», 101430, Москва, ГСП-4, Неглннная ул., Д. 29/14. Ярославский полиграфкомбинат Госкомпечати СССР, 150014, Ярославль, ул. Свободы, 97.